Bài tập đọc hiểu tiếng anh luyện thi thpt quốc gia có đáp án-tập 1

Bài tập đọc hiểu tiếng anh luyện thi thpt quốc gia có đáp án-tập 1

4.5/5

Tác giả: Thầy Tùng

Đăng ngày: 22 Aug 2022

Lưu về Facebook:
Hình minh họa Bài tập đọc hiểu tiếng anh luyện thi thpt quốc gia có đáp án-tập 1

Công thức toán học không thể tải, để xem trọn bộ tài liệu hoặc in ra làm bài tập, hãy tải file word về máy bạn nhé

BÀI TẬP ĐỌC HIỂU TẬP 1

EXERCISE 1: Read the following passage and mark the letter A, B, C or D on your answer sheet to indicate the correct answer to each of the questions.

Do you feel like your teenager is spending most of the day glued to a phone screen?

You're not too far off. A new survey from the Pew Research Center reveals the surprising ways that technology intersects with teen friendships — and the results show that 57 percent of teens have made at least one new friend online. Even more surprisingly, only 20 percent of those digital friends ever meet in person.

While teens do connect with their friends face-to-face outside of school, they spend 55 percent of their day texting with friends, and only 25 percent of teens are spending actual time with their friends on a daily basis (outside of school hallways). These new forms of communication are key in maintaining friendships day-to-day — 27 percent of teens instant message their friends every day, 23 percent connect through social media every day, and 7 percent even video chat daily. Text messaging remains the main form of communication — almost half of survey respondents say it's their chosen method of communication with their closest friend.

While girls are more likely to text with their close friends, boys are meeting new friends (and maintaining friendships) in the gaming world-89 percent play with friends they know, and 54 percent play with online-only friends. Whether they're close with their teammates or not, online garners say that playing makes them feel "more connected" to friends they know, or garners they've never met.

When making new friends, social media has also become a major part of the teenage identity- 62 percent of teens are quick to share their social media usernames when connecting with a new friend (although 80 percent still consider their phone number the best method of contact). Despite the negative consequences-21 percent of teenage users feel worse about their lives because of posts they see on social media — teens also have found support and connection through various platforms. In fact, 68 percent of teens received support during a challenging time in their lives via social media platforms.

Just as technology has become a gateway for new friendships, or a channel to stay connected with current friends, it can also make a friendship breakup more public. The study reveals that girls are more likely to block or unfriend former allies, and 68 percent of all teenage users report experiencing "drama among their friends on social media."

Question 1: What is the main idea of the passage?

  1. Social media affects friendship too much.
  2. Teenagers are making and keeping friends in a surprising way.
  3. The difference of making new friends between girls and boys.
  4. Social media connects friendship.

Question 2: The word "digital" in the first paragraph is closest in meaning to .

A. analogue B. numeracy C. numerous D. online

Question 3: According to the passage, what percentage of teens spend actual time with their friends?

A. 25% B. 55% C.27% D. 23%

Question 4: The following sentences aretrue, EXCEPT .

  1. According to the survey, more than half of teens have ever made new friends online.
  2. Teens only meet face-to-face one fifth of online friends they have made.
  3. Most teenagers use video chat to maintain relationship with friends.
  4. New forms of communication play an important role in keeping friendships.

Question 5: The word "they" in paragraph 3 refers to .

A. friends B. online garners C. their teammates D. online-only friends

Question 6: What can be inferred from the passage?

  1. Boys are more likely to meet new friends than girls.
  2. Most teens are not easy to give others their usernames when making new friends.
  3. The majority of teenage users agree that social media has negative consequences in their lives.
  4. Thanks to social media, more than two thirds of teens are supported when they face with challenges in their lives.

Question 7: What is the synonym of the word "breakup" in the last paragraph?

A. termination B. divorce C. commencing D. popularity Question 8: What does the writer mean when saying "68 percent of all teenage users report experiencing "drama among their friends on social media""?

  1. Most teenagers take part in drama on social media.
  2. Most friends on social media of teens are reported in drama.
  3. Most teenagers use their experience in drama with their friends on social media.
  4. Most teenagers have ever had conflicts with friends on social media.

Exercise 2:Read the following passage and mark the letter A, B, C, or D on your answer sheet to indicate the correct answer to each of the questions.

The difference between the nuclear family and the extended family is that a nuclear family refers to a single basic family unit of parents and their children, whereas the extended family refers to their

relatives such as grandparents, in-laws, aunts and uncles, etc. In many cultures, and particularly indigenous societies, the latter is the most common basic form of social organization.

A nuclear family is limited, according to Kristy Jackson of Colorado State University, to one or two parents (e.g. a father and mother) and their own child, or children, living together in a single house or other dwellings. In anthropology, they only must be related in this fashion; there is no upper or lower limit on the number of children in a nuclear family.

The extended family is a much more nebulous term, but in essence refers to kin or relations not covered by the above definition. In historical Europe and Asia as well as in Middle Eastern, African, and South American Aboriginal cultures, extended family groups were typically the most basic unit of social organization, The term can differ in specific cultural settings, but generally includes people related in age or by lineage.

Anthropologically, the term "extended family" refers to such a group living together in a household, often with three generations living together (grandparents, parents, and children) and headed in patriarchal societies by the eldest man or by some other chosen leadership figure. However, in common parlance, the term "extended family" is often used by people simply to refer to their cousins, aunts, uncles, and so on, even though they are not living together in a single group.

Question 9: What is the passage mainly about?

  1. The dominance of nuclear families over extended ones
  2. The dominance of extended families over nuclear ones
  3. A distinction between nuclear families and extended ones
  4. The changes of family types over times

Question 10: The word "the latter" in paragraph 1 refers to .

A. family unit B. relatives C. the nuclear family D. the extended family

Question 11: The word "nebulous" in passage 3 is closest in meaning to .

A. ambiguous B. featured C. difficult D.incomprehensive Question 12: Historically, extended families were the most basic unit of social organization in all of the following places EXCEPT .

A. The Middle East B. Asia C. North America D. Europe

Question 13: The word "patriarchal" in paragraph 4 is closest in meaning to .

A. ruled or controlledby men B. equal for both men and women

C. simple with no rules and laws D. modern with advanced facilities

Question 14: Which of the following is TRUE according to the passage?

  1. Since the 20th century, more and more American couples have lived in extended families because of the financial burdens.
  2. Nuclear families are the most basic form of social organization all over the world.
  3. The popularity of nuclear families in western countries helps to stabilize family arrangement
  4. Traditional nuclear families have changed a lot over times.

Question 15: What can be inferred from the reading passage?

    1. Indigenous communities have been completely eradicated all over the world.
    2. In the future, all extended families will be replaced by nuclear ones.
    3. Anthropology is a science concerning human race and its development.
    4. Couples with no children can't be defined as families.

Exercise 3: Read the following passage and mark the letter A, B, C, or D on your answer sheet to indicate the correct answer to each of the questions.

For hundreds of years, giving flowers have been a social means of communication. In the United States, flowers are often given during rites of passage, for commemorating special occasions or as a heartfelt gift between loved ones and friends. Flower gifting also occurs in most countries around the world. However, the meanings and traditions often vary.

While students traditionally gave their favorite teacher an apple in past years, in China, teachers are given flowers. Peonies are by far the flower most often given in China. They are also quite popularly used for weddings. Strangely, potted plants are not considered a pleasant gift among Asian cultures. The people believe that like a plant confined by a pot, the gift symbolizes a binding or restriction.

In Russia, in lieu of giving birthday presents, the guest of honor receives a single flower or an unwrapped bouquet. Floral arrangements or baskets are not given. Russians celebrate a holiday known as Woman's Day. Traditional gifts include red roses, hyacinths or tulips. When there is a funeral or other occasion where someone wishes to express sympathy, carnations, lilies or roses are given in circular configurations, which signify the transition of birth, life and death to rebirth. In this instance, the color of choice is commonly yellow. For joyous occasions, arrangements and bouquets generally contain an odd number of flowers.

In the times of ancient Rome, brides carried flowers to scare away evil spirits and encourage fertility. The Dutch believed that flowers were food for the soul. When invited to someone's home in Great Britain, it is tradition to bring a gift of flowers. All types are acceptable except white lilies, which are usually seen at funerals. Unlike the United States, red roses are a symbol of love.

Flowers are generally gifted in odd numbered increments regardless of the occasion. However, the Brits also have superstitions regarding the number 13, so the number is avoided.

In the southern region of the continent, flowers are traditionally given during Christmas. Egyptians are much more conservative and restrict flower gifting to funerals and weddings. While certain flowers may have significant meanings for some, flowers in Las Vegas and across the United States flowers are an accepted gift for any reason desired.

Question 16: What does the topic mainly discuss?

  1. The fascinating tradition of giving flowers.
  2. The different meanings of flowers in different cultures.
  3. The comparison of giving flowers between Asian and European cultures.
  4. The kinds of flowers people often give each other in different cultures.

Question 17: What does the word "They" in paragraph 2 refer to?

    1. Students B. Teachers C. Flowers D. Peonies

Question 18: Why should not you give a potted plant to an Asian?

  1. Because the Asian prefer to be given flowers.
  2. As this gift is often given at weddings in Asia.
  3. Since this gift is believed to symbolize an astriction and limitation in Asia.
  4. Because Asian students like to give an apple or flowers to others.

Question 19: According to the passage, the following flowers are given at Woman's Day in Russia,

EXCEPT .

    1. red roses B. hyacinths C. tulips D. yellow roses

Question 20: What could the word "fertility" in paragraph 4 best be replaced by?

A. fecundity B. good spirit C. happiness D. loyalty

Question 21: The word "superstitions" in paragraph 4 is closest in meaning to

A. deep-seated belief B. unfounded belief

C. religious belief D. traditional belief

Question 22: In which country should not people bring white lilies to other houses?

A. China B. Russia C. Great Britain D. United States

Question23: It can be inferred from the passage that

  1. People can give flowers to the American in any occasion.
  2. Egyptians are rather comfortable when receiving flowers at funerals and weddings.
  3. Flowers given in Britain are in even numbers in any case.
  4. At the funerals in any cultures, flowers are gifted in circular configurations.

Exercise 4:Read the following passage and mark the letter A, B, C, or D on your answer sheet to indicate the correct answer to each of the questions.

FIRST TIME IN THE AIR

When John Mills was going to fly in an airplane for the first time, he was frightened. He did not like the idea of being thousands of feet up in the air. "I also didn't like the fact that I wouldn't be in control," says John.

"I'm a terrible passenger in the car. When somebody else is driving, I tell them what to so. It drives everybody crazy."

However John couldn't avoid flying any longer. It was the only way he could visit his grandchildren in Canada.

"I had made up my mind that I was going to do it, I couldn't let my son, his wife and their three children travel all the way here to visit me. It would be so expensive for them and I know Tom's business isn't doing so well at the moment - it would also be tiring for the children - it's a nine-hour flight!" he says.

To get ready for the flight John did lots of reading about airplanes. When he booked his seat, he was told that he would be flying on a Boeing 747, which is better known as a jumbo jet. "I needed to know as much as possible before getting in that plane. I suppose it was a way of making myself feel better. The Boeing 747 is the largest passenger aircraft in the world at the moment.

The first one flew on February 9th 1969 in the USA. It can carry up to 524 passengers and 3.400 pieces of luggage. The fuel for airplanes is kept in the wings and the 747's wings are so big that they can carry enough fuel for an average car to be able to travel 16,000 kilometers a year for 70 years. Isn't that unbelievable? Even though I had discovered all this very interesting information about the jumbo, when I saw it for the first time, just before I was going to travel to Canada, I still couldn't believe that something so enormous was going to get up in the air and fly. I was even more impressed when I saw how big it was inside with hundreds of people!"

The biggest surprise of all for John was the flight itself. "The take-off itself was much smoother than I expected although I was still quite scared until we were in the air. In the end, I managed to relax, enjoy the food and watch one of the movies and the view from the window was spectacular. I even managed to sleep for a while!

"Of course," continues John, "the best reward of all was when I arrived in Canada and saw my son and his family, particularly my beautiful grandchildren. Suddenly, I felt so silly about all the years when I couldn't even think of getting on a plane. I had let my fear of living stop me from seeing the people I love most in the world. I can visit my son and family as often as I like now!" Question 24: Why did John Mills fly in an airplane?

    1. He wanted to go on holiday B. He wanted to try it.

C. He wanted to see his family D. He had to travel on business.

Question 25: Why did John read about airplane?

A. He wanted to know how they work. B. It was his hobby.

C. It made him feel safer. D. He had found a book on them.

Question 26: What does the word "which" in the paragraph refer to?

A. reading about airplanes B. booking his seat

C. flying on a Boeing 747 D. a Boeing 747

Question 27: What happened when he saw the jumbo jet for the first time?

A. He felt much safer. B. He liked the shape of it.

C. He couldn't believe how big it was. D. He thought the wings were very small.

Question 28: How did John feel when the airplane was taking off?

A. excited B. happy C. sad D. frightened

Question 29: What surprised John most about the flight?

A. that he liked the food. B. that he was able to sleep

C. that there was a movie being shown D. that the view was good

Question 30: How did John feel about his fears in the end?

  1. He thought he had wasted time being afraid.
  2. He realized it was okay to be afraid.
  3. He hoped his grandchildren weren't afraid of flying.
  4. He realized that being afraid kept him safe.

Exercise 5: Read the following passage and mark the letter A, B, C, or D on your answer sheet to indicate the correct answer to each of the questions

In America, when dining, people consider it rude for a guest or dining partner to belch or burp, eat with an open mouth, smack, or lick your fingers. Napkins, generally provided are available at every meal and should be placed in one's lap and then used throughout the meal to clean one's fingers and mouth.

It is acceptable to refuse additional servings of food by saying "No, thank you" and the host or hostess will not be insulted if you do so. Similarly, if you leave a small amount of uneaten food on your plate at a restaurant or in a home, it is not considered an insult. If you eat everything on

the plate, a host or hostess may possibly feel that they have not prepared enough food and might be embarrassed. People in the United States serve and eat food with either hand, but never take food from a communal serving dish with their hands. Generally, a serving utensil is used.

Americans typically use forks, spoons and knives to eat, but there are some types of foods that are acceptable to eat with one's fingers, like sandwiches or pizza. When in doubt, look to see what others are doing. In formal dining situations, if you wonder whether or not it is acceptable to begin eating, you should wait until the oldest woman (or oldest man if no women are present) begins to eat. When eating, do not pick up the bowl or plate from the table to hold underneath your mouth, Even noodles, soup, and rice are eaten with the plate or bowl remaining on the table. When consuming soup and hot liquids, it is considered impolite to slurp - do not do this. When consuming noodles, twirl them around your fork and then put it in your mouth.

If you are a man taking out a woman for dinner, you are almost always expected to pay. This is for the woman to gauge your intentions and interest with her. For example, taking a woman for coffee, versus tacos, versus a fancy dinner, versus for drinks at 11:30pm, all signal many different things to them. So, the date is a -test- of many. Paying is just as important as where you take her, and how late. So, don't assume she is just trying to get a "free meal". Most girls aren't. Also, if you are going out with a friend to eat, almost always, the bill is expected to be split in half, or each person pays for themselves.

If you are eating in a restaurant, you will be expected to add a 15 to 20 % tip for the server to your bill. In America, wait staff might occasionally stop by your table to ask how your meal is, which is considered good service. They will also bring you your check when it seems reasonable that you are finished with your meal, however this is not necessarily an indication that you must leave right away (Do not be too embarrassed to ask for the check either waiters and waitresses cannot read minds.) Take your time to finish your meal, and unless there is a line of people waiting at the door, it is not considered rude to linger at your table for as long as you like.

(Source:https://www.tripadvisor.com/)

Question 31: Which of the following could be the best title of the passage?

    1. What should we do when being invited to the American's dinner?
    2. Table etiquette in America.
    3. Polite behavior at American's restaurants.
    4. What is acceptable in dining etiquette in America?

Question 32: According to the passage, what action may make the American unpleasant?

  1. Refusing the supplementary food they serve.
  2. Leaving the left-overs on the plate.
  3. Eating food with the guests'hands.
  4. Cleaning the food on the fingers by the tongue.

Question 33: What sentence is NOT stated in the passage?

  1. In formal dinners, the eldest often eat first.
  2. In America, the dishes are expected to remain on the table.
  3. Food is sometimes delivered from the communal serving dish with people's hands.
  4. It is not courteous to slurp when eating soup.

Question 34: The word "it" in paragraph 3 refers to .

A. your fork B. noodles C. your mouth D. soup

Question 35: What could the word "gauge" in paragraph 4 best be replaced by?

A. determine B. impress C. express D. estimate

Question 36: When will the bill be divided for the people having the meal?

  1. When a man is having a date with a woman.
  2. When people are having meals with their friends.
  3. When people are eating with the elderly.
  4. When a girl is testing a boy.

Question 37: The word "linger" in the last paragraph is closest in meaning to _.

A. appear B. arrange C. reserve D. remain

Question 38: Which of the following can he inferred from the passage?

  1. It is impolite if you give extra money for the waiter.
  2. Whenever you receive your bill, you should think of leaving soon.
  3. Paying meals for another may insult them.
  4. Imitating others if you are not sure what to do at the meal is a good idea.

Exercise 6:Read the following passage and mark the letter A, 11, C, or D on your answer sheet to indicate the correct answer to each of the questions.

Christina and James met in college and have been dating for more than five years. For the past two years, they have been living together in a condo they purchased jointly. While Christina and James were confident in their decision to enter into a commitment like a 20-year mortgage, they are unsure if they want to enter into marriage. The couple had many discussions about marriage and decided that it just did not seem necessary. Wasn't it only a piece of paper? And didn't half of all marriages end in divorce?

Neither Christina nor James had seen much success with marriage while growing up. Christina was raised by a single mother. Her parents never married, and her father has had little contact with the family since she was a toddler, Christina and her mother lived with her maternal

grandmother, who often served as a surrogate parent. James grew up in a two-parent household until age seven, when his parents divorced. He lived with his mother for a few years, and then later with his mother and her boyfriend until he left for college. James remained close with his father who remarried and had a baby with his new wife.

Recently, Christina and James have been thinking about having children and the subject of marriage has resurfaced. Christina likes the idea of her children growing up in a traditional family; while James is concerned about possible marital problems down the road and negative consequences for the children should that occur. When they shared these concerns with their parents, James's mom was adamant that the couple should get married. Despite having been divorced and having a live-in boyfriend of 15 years, she believes that children are better off when their parents are married. Christina's mom believes that the couple should do whatever they want but adds that it would "be nice" if they wed. Christina and James's friends told them, married or not married; they would still be a family.

(https://goo.gl/phi6w2)

Question 39: Why did Christina and James suppose that marriage was unnecessary? Because

_

  1. It was only a piece of paper.
  2. Half of all marriages ended in divorce.
  3. Neither of them had seen much success with marriage while growing up.
  4. They led to an independent life.

Question 40: Which can be used as an antonym of the word "maternal"?

A. relative B. bloody C. close D. paternal

Question 41: According to the paragraph 2, which of the following statements is TRUE?

  1. Christina lived with her parents until she left for college.
  2. James was brought up by his father and his new wife.
  3. Both James and Christina grew up in broken families.
  4. James lived with his mother and her boyfriend for a year.

Question 42: Which of the following could best replace the word "traditional" ?

A. old-fashioned B. customary C. antique D. ancient

Question 43: Why have Christina and James mentioned the subject of marriage again?

  1. They have been thinking about having children.
  2. Christina likes the idea of her children growing up in a traditional family.
  3. They have been living together for a long time.
  4. James is concerned about possible marital problems down the road and negative consequences for the children should that occur.

Question 44: What is the advice of the couple's parents?

  1. Married or not married, they would still be a family.
  2. They should get married so that children are better off.
  3. The couple should do whatever they want.
  4. They can have children without entering into a commitment of marriage.

Question 45: What does the passage mainly discuss?

  1. Marital problems
  2. Negative consequences of a broken family
  3. Changes in young people's attitude to marriage
  4. Arguments about marriage

Exercise 7: Read the following passage and mark the letter A, B, C, or D on your answer sheet to indicate the correct answer to each of the questions.

People have been donating blood since the early twentieth century to help accident victims and

patients undergoing surgical procedures. Usually a pint of whole blood is donated, and it is then divided into platelets, white blood cells, and red blood cells. People can donate blood (for red blood cells] about once every two months. Transfusing the blood from the donor to the recipient is straightforward. It involves taking the blood from a donor's arm vein by means of a hypodermic syringe. The blood flows through a plastic tube to a collection bag or bottle that contains sodium citrate, which prevents the blood from clotting. When the blood is given to a patient, a plastic tube and hypodermic needle are connected to the recipient's arm. The blood flows down from the container by gravity. This is a slow process and

may last as long as 2 hours to complete the infusion of blood into the recipient. The patient is protected from being infected during the transfusion. Only sterile containers, tubing, and needles are used, and this helps ensure that transfused or stored blood is not exposed to disease causing bacteria.

Negative reactions to transfusions are not unusual. The recipient may suffer an allergic reaction or be sensitive to donor leukocytes. Some may suffer from an undetected red cell incompatibility. Unexplained reactions are also fairly common, Although they are rare, other causes of such negative reactions include contaminated blood, air bubbles in the blood, overloading of the circulatory system through administration of excess blood, or sensitivity to donor plasma or platelets. Today, hospitals and blood banks go to great lengths to screen all blood donors and their blood. All donated blood is routinely and rigorously tested for diseases, such as HIV, hepatitis B, and syphilis. When the recipient is a newborn or

an infant, the blood is usually irradiated to eliminate harmful elements. Donated blood is washed, and the white blood cells and platelets removed. Storing the blood sometimes requires a freezing process. To freeze the red blood cells, a glycerol solution is added. To unfreeze, the glycerolis removed. The ability to store blood for long periods has been a boon to human health.

Question 46: The word "it" refers to .

A. surgical procedures B. accident victims

C. a pint of whole blood D. surgery patients

Question 47: According to the passage, how often can people donate blood for red blood cells?

A. Everyfour months B. Every three month

C. Everytwo months D. Every month

Question 48: All of the following are mentioned as potential negative reactions to transfusion

EXCEPT _.

A. Sensitivity to donor leukocytes B. Air bubbles in the blood

C. Allergies D. Red-cell incompatibility

Question 49: What answer choice is closest in meaning to the word "undetected"?

A. not illustrated B. not captured C. not found D. not wanted

Question 50: What can the phrase "go to great length" best replaced by?

A. irradiate B. test rigorously C. monitor routinely D. removed Question 51: Based on the information in the passage, what can be inferred about blood transfused to infants and newborns?

  1. It is treated with radiant energy.
  2. It is not treated differently from adults.
  3. It is not dangerous for children.
  4. It is rigorously tested as blood for adults.

Question 52: What does the author imply in the passage?

  1. Donating blood benefits mankind.
  2. Clotting cannot be prevented.
  3. Freezing blood destroys platelets.
  4. Transfusing blood is a dangerous process.

Exercise 8:Read the following passage and mark the letter A, B, C, or 0 on your answer sheet to indicate the correct answer to each of the questions.

Every summer, when the results of university entrance exam come out, many newspaper stories are published about students who are top-scorers across the country. Most portray students as hard- working, studious, smart and, generally, from low-income families. They are often considered heroes or heroines by their families, communes, villages and communities, And they symbolise the efforts made to lift them, and their relatives, out of poverty. The students are often too poor to attend any extra-classes, which make their achievements more illustrious and more newsworthy. While everyone should applaud the students for their admirable efforts, putting too much emphasis on success generates some difficult questions.

If other students look up to them as models, of course it's great. However, in a way, it contributes to society's attitude that getting into university is the only way to succeed. For those who fail, their lives are over. It should be noted that about 1.3 million high school students take part in the annual university entrance exams and only about 300,000 of them pass. What's about the hundreds of thousands who fail? Should we demand more stories about those who fail the exam but succeed in life or about those who quit university education at some level and do something else unconventional?

"I personally think that it's not about you scoring top in an entrance exam or get even into Harvard. It's about what you do for the rest of your life," said Tran Nguyen Le Van, 29. He is the founder of a website, vexere.com, that passengers can use to book bus tickets online and receive tickets via SMS. His business also arranges online tickets via mobile phones and email. Van dropped out of his MBA at the Thunderbird School of Global Management in Arizona in the United States. His story has caught the attention of many newspapers and he believes more coverage should be given to the youngsters who can be role-models in the start-up community. Getting into university, even with honours, is just the beginning. We applaud them and their efforts and obviously that can give them motivation to do better in life. However, success requires more than just scores," Van said. Van once told a newspaper that his inspiration also came from among the world's most famous drop-outs, such as Mark Zuckerberg of Facebook or Bill Gates who also dropped out of Harvard University.

Alarming statistics about unemployment continues to plague us. As many as 162,000 people with some kind of degree cannot find work, according to Labour Ministry's statistics this month. An emphasis on getting into university does not inspire students who want to try alternative options. At the same time, the Ministry of Education and Training is still pondering on how to reform our exam system, which emphasises theories, but offers little to develop critical thinking or practice. Vu Thi Phuong Anh, former head of the Centre for Education Testing and Quality Assessment at Vietnam National University in Ho Chi Minh City said the media should also monitor student successes after

graduation. She agreed there were many success stories about young people, but added that it was imbalanced if students taking unconventional paths were not also encouraged.

Vietnam is, more than ever, in desperate need of those who think outside the box. Time for us to recognise talent, no matter where it comes from or how.

Question 53: Which of the following best describes the main idea of this passage?

  1. Many students consider universities as their only way after school.
  2. A good model of being successful in the real life out of school,
  3. Not many students are successful after graduating from universities.
  4. University is not the only way to success.

Question 54: What is NOT stated in the passage about the top-scorers in the entrance exam?

  1. The majority of them are poor but intelligent and eager to learn.
  2. They are hoped to find the way to better their families' lives.
  3. Their success is more glorious because they attend more classes than others.
  4. The students are admired for the great efforts.

Question 55: The word "unconventional" in paragraph 2 could best be replaced by ______ _.

A. common B. unusual C. well-known D. infamous

Question 56: The author described Tran Nguyen Le Van in the third passage as _.

  1. a good example to achieve success although he didn't finish his education.
  2. a businessman who gains money by selling mobile phones online.
  3. a founder whose website was inspired from social networks like Facebook.
  4. a top-scorer who books online tickets and confirms through messages.

Question 57: The word "them" in paragraph 3 refers to ________ _.

A. honours B. role-models C. the youngsters D. newspapers

Question 58: The word "plague" in paragraph 4 is closest in meaning to ______ _.

A. conflict B. afflict C. remind D. bother

Question 59: According to the fourth paragraph, what is TRUE about the modern exam system?

  1. It puts too much pressure on students who must get a place in a university.
  2. Students are not encouraged to do something different.
  3. The government is trying to change the theories of exam.
  4. Many stories about successful students cannot inspire those who attend universities.

Question 60: What can be inferred from the passage?

  1. It's high time for the young to change their ways of thinking about success.
  2. High scores are the first step to attain achievements in the future.
  3. Failing the entrance exam will not determine that these students are doing unconventional things.
  4. The most concerned thing for each student is unemployment.

Exercise 9: Read the following passage and mark the letter A, B, c, or D on your answer sheet to indicate the correct answer to each of the questions.

If you ask any Vietnamese girl right now who she has a crush on, chances are you will encounter

the name of the 20-year-old goalkeeper who plays for Vietnam national team. Bui Tien Dung made a name for himself in the AFC U23 Championship where he accurately blocked opponents' shots time after time and saved Vietnam’s chance at the championship in the process. Bui Tien Dung was born on February 28th, 1997 in a poor farming family in Thanh Hoa Province. Dung has loved soccer since he was a kid, but his family was so poor that they could not even

afford a plastic soccer ball for him and his brother. The Bui brothers had to resort to playing with balls made from scrap papers and old grapefruits.

Dung's parents recognized the brothers' passion for football and used the little money they had to support their sons' early training. When a big football club in the city announced a recruitment event, Dung convinced his younger brother to go to the tryout. He passed on the opportunity knowing that his parents could not afford to have both their sons gone to city.

While his younger brother training in the city, Dung's local training center went bankrupted and abruptly ended Dung's football career. For a year, instead of kicking balls, the young man worked as a construction worker to support his parents. He seemingly gave up on football.

Fate finally smiled on Dung when a football coach rediscovered him and sent him to train with a local football team. Reentering the football field, Dung wanted to play in a defense position but he was pushed to goalkeeping because of his height. At first, Dung was unhappy about his new position, but his parents advised him to follow and make the best out of it. And Dung did.

The young goalkeeper trained hard for his new position and patiently climbed up the ranks the following years. Through hard work, he was named the best goalkeeper of Vietnam's U19 division. Those who have worked with Dung described him as a likable young man. He is very respectful to others and speaks very little. However, when the gloves are on, the nice young man transforms into a fierce competitor and an unyielding goalkeeper.

At AFC U23 Championship, Dung finally reunited with his brother on the field. They fought battle after battle together with other amazing players on the team and helped Vietnam made history for Southeast Asian football.

Even though Vietnam came short of winning the championship, Dung and his teammates are already heroes and legends in the heart of Vietnamese people. They played fairly and lost with glory and grace.

Question 61. What is the passage mainly about?

(Source: https://goo.gl/499LtH)

A. Bui Tien Dung's ambitions B. Bui Tien Dung and his brother

C. Bui Tien Dung's biography D. Bui Tien Dung’s successes

Question 62. What made a name for himself in the AFC U23 Championship?

  1. He accurately blocked opponents' shots time after time.
  2. He saved Vietnam’s chance at the championship in the process.
  3. Many Vietnamese girl have crush on him.
  4. Both A and B are correct.

Question 63. Which of the following could best replace the word "afford" ?

A. buy B. sell C. throw D. give

Question 64. Which can be used as a synonym of the word "recruitment” ?

A. participation B. employment C. selection D. graduation

Question 65. According to the passage, what interrupted Dung's soccer career for a year?

A. His poor family B. The bankruptcy of his local training center

C. The oppositions from his parents D. His younger brother training in the city

Question 66. According to the passage, these following sentences are true EXCEPT .

  1. Dung had to work as a construction worker to support his parents.
  2. Dung liked playing in a goalkeeping position.
  3. Dung was named the best goalkeeper of Vietnam’s U19 division.
  4. The Bui brothers played at AFC U23 Championship.

Question 67. How is Bui Tien Dung in the field described?

  1. He is a likable young man who is very respectful to others and speaks very little.
  2. He is a handsome but a fierce man.
  3. He is a nice young man with many girl fans.
  4. He is a fierce competitor and an unyielding goalkeeper

ĐỀ THI THỬ THPTQG 2019 – CÔ TRANG ANH – ĐỀ 5 – TẬP 1

Exercise 10:Read the following passage and mark the letter A, B, c, or D on your answer sheet to indicate the correct answer to each of the questions.

Successful students often do the followings while studying. First, they have an overview before

reading. Next, they look for important information and pay greater attention to it [which often needs

jumping forward or backward to process information). They also relate important points to one another. Also, they activate and use their prior knowledge. When they realize that their understanding is not good, they do not wait to change strategies. Last, they can monitor understanding and take action to correct or "fix up" mistakes in comprehension.

Conversely, students with low academic achievement often demonstrate ineffective study skills. They tend to assume a passive role, in learning and rely on others (e.g., teachers, parents) to monitor their studying, for example, low-achieving students often do not monitor their understanding of content; they may not be aware of the purpose of studying; and they show little evidence of looking back, or employing "fix-up” strategies to fix understanding problems. Students who struggle with learning new information seem to be unaware that they must extent effort beyond simply reading the content to understand and remember it Children with learning disabilities do not plan and judge the quality of their studying. Their studying may be disorganized. Students with learning problems face challenges with personal organization as well. They often have difficulty keeping track of materials and assignments, following directions, and completing work on time. Unlike good studiers who employ a variety of study skills in a flexible yet purposeful manner, low-achieving students use a restricted range of study skills. They cannot explain why good study strategies are important for learning; and they tend to use the same, often ineffective study approach for all learning tasks, ignoring task content, structure or difficulty.

(Source: Adapted from Study Skills: Managing Your Learning NUl Galway)

Question 68. What is the topic of the passage?

  1. Successful and low-academic achieving students
  2. Successful learners and their learning strategies
  3. Study skills for high school students
  4. Effective and ineffective ways of learning

Question 69. The word “prior" in the first paragraph is closest meaning to ?

A. important B. earlier C. forward D. good

Question 70. Which of the following could best replace the word "Conversely" in paragraph 2?

A. On the contrary B. In contrast C. On the other hand D. all are correct

Question 71: According to the passage, what can be learnt about passive students?

  1. They depend on other people to organize their learning
  2. They are slow in their studying
  3. They monitor their understanding
  4. They know the purpose of studying

Question 72: Which of the followings is NOT an evidence of monitoring studying?

A. Being aware of the purpose of studying B. Monitoring their understanding of content

C. Fixing up mistakes in understanding D. Looking at their backs

Question 73: According to the passage, to learn new information, low-achieving students do NOT

.

A. just understand it B. relate it to what they have known

C. simply remember it D. read it

Question 74: In compared with low-achieving students, successful students use .

A. aimless study techniques B. various study skills

C. restricted strategies D. inflexible study ways

Question 75: The underlined pronoun "They" in the last sentence refers to .

A. study strategies B. study skills

C. low-achieving students D. good studiers

ĐỀ THI THỬ THPTQG 2019 – CÔ TRANG ANH – ĐỀ 5 – TẬP 1

Exercise 11: Read the following passage and mark the fetter A, B, C, or D on your answer sheet to indicate the correct answer to each of the questions.

Facebook users spend an average of more than 15 hours a month on the social networking site. While there are plenty who caution against such intensive use — and there are a number of studies detailing the harm Facebook could potentially cause — there also are lots of reports extolling the site's virtues. As the social media giant prepares for its upcoming initial public offering, here are some ways Facebook just might be good for you.

Spending time on Facebook can help people relax, slow down their heart rate and decrease stress levels, according to researchers from the Massachusetts Institute of Technology and the University of Milan. In a study published earlier this year, researchers studied 30 students and found that a natural high was sparked when they were on the social media network that led to the relaxed heart rates and lower levels of stress and tension. In the study, the students were monitored in three situations: looking at panoramic landscapes, performing complicated mathematical equations and using Facebook. While the first situation was the most relaxing to students and the math problems were the most stressful, the time on Facebook uncovered high levels of attractiveness and arousal. The findings support the researchers' hypothesis that Facebook's success, as well as that of other social media networks, correlates to the specific positive mental and physical state users experience.

While many may argue that social media networks only distract employees, research shows the opposite may be true. Research from Keas.com found that a 10-minute Facebook break makes employees happier, healthier and more productive. The study examined workers in three groups:

one that was allowed no breaks, one that was allowed to do anything but use the Internet and one that was allowed 10 minutes to use the Internet and Facebook. The Facebook group was found to be 16 percent more productive than the group that was not allowed to use the Internet and nearly 40 percent more productive than the group that was allowed no breaks. "Short and unobtrusive breaks, such as a quick surf on the Internet, enables the mind to rest itself, leading to a higher net total concentration for a day's work, and as a result, increased productivity," said Brent Coker of the department of management and marketing at the University of Melbourne in Australia.

Facebook is also in the business of matchmaking. Research shows that nearly 60 percent of singles will friend someone new on Facebook after meeting them in person. If they like what they see, 25 percent are likely to contact their new love interest via Facebook. Once the courting is over, nearly 40 percent of those social networking adults will update their relationship status on Facebook, with just 24 percent telling their friends first. Facebook use between couples will continue through the dating process, the research shows. Throughout the day, 79 percent of couples said they send partners Facebook messages or chat on the social network. In addition, more than 60 percent would post romantic messages on their significant other's Facebook wall. When the relationship ends, more than half of those surveyed immediately update their status to single, which automatically sends out a notification to their friend list to start the dating cycle over again.

(Source: https://www.businessnewsdaily.com/)

Question 76: Which of the following could be the main idea of the passage?

  1. The benefits that Facebook might bring you.
  2. The disadvantages that Facebook causes in the modern life.
  3. The classes are most influenced by Facebook.
  4. The functions that are used most on Facebook.

Question 77: The word "sparked" in the second paragraph probably means ____ _.

A. produced B. reduced C. prevented D. controlled

Question 78: According to paragraph 2, what do the scientists discover in their study?

  1. Spending too much time on Facebook may increase levels of pressure and heart disease,
  2. Students often suffer from stress when they are working with something related to mathematics.
  3. Facebook can make its users more optimistic and aroused.
  4. The success of Facebook may motivate the development of other social networks.

Question 79: The word "one" in paragraph 3 refers to _.

A. the study B. a group C. the Internet D. a research

Question 80: Why does Facebook motivate the productivity of employees?

  1. Because the Internet distracts users to take breaks out of work.
  2. Because it helps people refresh their mind and then enhance the concentration on work.
  3. Because employees are happy with what they read on the Face book.
  4. Because they are allowed to relax after a hard work.

Question 81: The word "courting" in the last paragraph is closest in meaning to _.

A. dating B. marriage C. divorce D.making acquaintance Question 82: Which of the following is NOT correct about the impact of Facebook on users' relationship?

  1. More than half of single people tend to make new friends on Facebook after a face-to-face contact
  2. Married people often use Facebook to keep in touch with their spouse.
  3. Not many people change the relationship status to single after their breakdown.
  4. Adults are more likely to update their relationship status on Facebook than to tell their friends about that.

Question 83: Which of the following could best describe the tone of the passage?

    1. Informative B. Argumentative C. Sarcastic D. Ironic

ĐỀ THI THỬ THPTQG 2019 – CÔ TRANG ANH – ĐỀ 6 – TẬP 1

Exercise 12:Read the following passage and mark the letter A, B, C, or D on your answer sheet to indicate the correct answer to each of the questions.

Environmental pollution is one of the impacts of human activities on the Earth. Pollution occurs when pollutants contaminate the natural surroundings; which brings about changes that affect our normal lifestyles adversely. Pollution disturbs our ecosystem and the balance in the environment. Pollution occurs in different forms; air, water, soil, radioactive, noise, heat/ thermal and light Let us discuss the causes and effects of air pollution on mankind and the environment as a whole.

Air pollution is the most prominent and dangerous form of pollution. It occurs due to many reasons. Excessive burning of fuel which is a necessity of our daily lives for cooking, driving and other industrial activities; releases a huge amount of chemical substances in the air every day; these pollute the air. Smoke from chimneys, factories, vehicles or burning of wood basically occurs due to coal burning; this releases sulphur dioxide into the air making it toxic.

The effects of air pollution are evident too. The increase in the amount of CO2 in the atmosphere leads to smog which can restrict sunlight from reaching the earth. Thus, preventing plants in the process of photosynthesis. Gases like sulfur dioxide (SO2) and nitrogen oxide (NO) can cause acid rain.

The decrease in quality of air leads to several respiratory problems including asthma or lung cancer. Chest pain, congestion, throat inflammation, cardiovascular disease, respiratory disease are some of diseases that can be caused by air pollution.

The emission of greenhouse gases particularly CO2 is leading to global warming. Every other day new industries are being set up, new vehicles come on roads and trees are cut to make way for new homes. All of them, in direct or indirect way lead to increase in CO2 in the environment. The increase in CO2 leads to melting of polar ice caps which increases the sea level and pose danger for the people living near coastal areas.

Ozone layer is the thin shield high up in the sky that stops ultra violet rays from reaching the earth. As a result of human activities, chemicals, such as chlorofluorocarbons (CFCs), were released into the atmosphere which contributed to the depletion of ozone layer.

Question 84: What does the passage mainly discuss?

  1. How pollutants contaminate the natural surroundings
  2. The causes and effects of air pollution.
  3. Our ecosystem and the balance in the environment
  4. Some solutions to air pollution.

Question 85: The word "these" in paragraph 2 refers to

(Source: https://goo.gl/RpdPw3)

A. chemical substances B. industrial activities

C. our daily lives D. excessive burning of fuel for cooking, driving

Question 86: How many diseases are mentioned in the passage?

A. 5 B. 6 C. 7 D. 8

Question 87: The word "respiratory" in paragraph 4 is closest in meaning to

A. connected with the digestion of food B. connected with breathing

C. connected with the skeleton of a person D. connected with the body's brain

Question 88: According to paragraph 5, air pollution contributes to the greenhouse effect which can lead to

  1. global warming
  2. polar ice melting
  3. increase in CO2 in the environment
  4. global warming, polar ice melting, rise of sea levels and danger for people.

Question 89: According to the passage, which of the following sentences is NOT true?

  1. Ozone layer can't stop ultra violet rays from reaching the earth anymore.
  2. The toxic air is caused by sulphur dioxide which is released from coal burning.
  3. When the amount of CO2 in the air increases, it prevents plants in the process of photosynthesis.
  4. There are other different types of pollution besides air pollution.

Question 90: Which of the following can be inferred from the passage?

    1. The more modern our lives become, the more serious the pollution is.
    2. People can control air pollution soon in the future.
    3. Only air pollution is dangerous and has a negative impact on humans and the environment.
    4. CO2 is the cause of the depletion of ozone layer.

ĐỀ THI THỬ THPTQG 2019 – CÔ TRANG ANH – ĐỀ 6 – TẬP 1

Exercise 13: Read the following passage and mark the letter A, B, C, or D on your answer sheet to indicate the correct answer to each of the questions.

Translators and interpreters for tech jobs of the future are expected to be one of the fastest growing occupations in the nation, according to a just released survey by Vietnamworks. Almost all positions for programmers, application developers, database and network administrators, engineers, designers, architects, scientists, technicians, and tech support will require bilingual or multilingual fluency. In just the last two years the demand for tech professionals with foreign language skills has increased more than two and one-half fold, said the survey, and the uptick shows no signs of abating anytime soon. Roughly 400,000 jobs are expected to open for interpreters (who focus on spoken language) and translators (who focus on written language) in the tech segment, between 2017 and 2020, says Tran Anh Tuan. Tuan, who works for the Centre for Forecasting Manpower Needs and Labour Market Information in Ho Chi Minh City doesn't include other industries in his prediction,

which are also recruiting ferociously for more people with these same language skills.

While that claim might seem a bit overblown (and amounts to little more than a guess by Tuan), it is clear that innovative technologies like robotics, 3D printing, drones, artificial intelligence and virtual reality will create major upheavals in all sorts of labor markets, not just technology over the next few years. In the last month alone, most every job posted on employment websites throughout Vietnam included the word bilingual. Far higher salaries go to people who work in high tech positions and can speak a foreign language such as English in addition to Vietnamese, says Tran Quang Anh from the Posts and Telecommunications Institute of Technology.

Unfortunately, the surveys show that most graduating Vietnamese students are unable to do more than understand a few basic phrases of foreign languages, and practically none of them can speak any foreign language coherently, The good paying jobs with high salaries and benefits are only available to translators and interpreters who specialize in high tech jobs, says Anh. But it's not just English— graduates are needed with fluency in middle eastern languages like Arabic, Farsi and Pashto (Afghani) as well as German, Japanese and Korean to name just a few. Spanish is also in high demand in Vietnam, primarily because it is the second most common language in the US after English.

A recent tech expo in Hanoi sponsored by Vietnamworks and the Navigos Group attracted nearly 4,000 young tech graduates and recruiters from 14 leading companies looking to fill job vacancies with skilled bilingual workers. The job applicants were young and industrious, said the recruiters. However, missing were candidates with the requisite language skills and most lacked basic 'soft skills' such as written and verbal communication abilities to effectively communicate even in their native Vietnamese language.

Notably, the recruiters said they considered language abilities and soft skills just as, if not more important, than academic ability. Yet virtually all the prospective academically qualified employees lacked even the most basic of interpersonal communication abilities.

(Source: http://engiiskvov.vni)

Question 91: Which of the following could be the main idea of the passage?

  1. The most popular jobs in Vietnam's job markets.
  2. The necessity of foreign languages in most tech jobs.
  3. The skills needed in tech jobs nowadays.
  4. Vietnamese students are not aware of the importance of learning foreign languages.

Question 92: According to the passage, interpreters and translators are described as the jobs that

.

  1. are decreasing dramatically in the number of employees.
  2. there are not enough employees for technology companies to recruit.
  3. the requirements have risen considerably and steadily.
  4. are expected to experience a downward trend in the near future.

Question 93: The word "upheavals" in paragraph 3 is closest in meaning to _______.

A. upward displacements B. upward disruptions

C. downward changes D. downward problems

Question 94: Which of the following is TRUE about employment in Vietnam according to Tran Quang Anh from the Posts and Telecommunications Institute of Technology?

  1. People whose major in foreign languages is high tech often earn high salaries.
  2. The demand of interpreters and translators in the workforce is not as much as what people believe.
  3. Not only technology but also other jobs are putting more pressure on language skills.
  4. Foreign languages in Vietnam are only needed in the posts on websites.

Question 95: What does the word "them" in paragraph 4 refer to?

A. foreign languages B. a few basic phrases

C. Vietnamese students D. the surveys

Question 96: The word "requisite" in paragraph 5 could best be replaced by _.

A. shortage B. necessity C. specification D. measurement

Question 97: What can be inferred from the passage?

  1. Academic ability is much more important than foreign languages and soft skills,
  2. To get a good job nowadays, applicants must know more than a language and be good at other soft skills.
  3. Written and verbal communication abilities determine the opportunity of applicants to get a desirable job.
  4. Translators and interpreters are suitable for any position in the work markets. Question 98: Which of the following most accurately reflects the attitude of the author towards improving foreign language skills?

A. neutral B. skeptical C. supportive D. sensitive

ĐỀ THI THỬ THPTQG 2019 – CÔ TRANG ANH – ĐỀ 7– TẬP 1

Exercise 14:Read the following passage and mark the letter A, B, C, or D on your answer sheet to indicate the correct answer to each of the questions.

On March 15, Dunes View Middle School held a contest for school bands. Student bands tried

out for the opportunity to perform at the school picnic, which will be held at the end of June. The winner of the contest was the band called Four Square. "We're very proud that we won the contest and are excited to perform at the picnic," says Peter Zandt, who plays the guitar in the band. "And since we hope to perform someday at other local places, like restaurants and parks, this will be a great first step."

The contest was the creation of music teacher Mr. Lopez and drama teacher Ms. Cho. The two thought of the idea while discussing recent years' school picnics. "The picnic is one of the biggest events of the year, but it has become a bit formulaic ," said Ms. Cho. "The activities are the same every year. We thought that a performance by a student band would make the school picnic more interesting and fun." Mr. Lopez, Ms. Cho, and three other teachers judged the contest, which took place in the gym. Eight student bands signed up to audition. The bands varied in their musical forms: there were several rock bands, a folk band, and even a jazz band. "I'm disappointed that my band didn't win, but I think the judges made the right choice," says student Marisol Varga, a member of the folk trio called The Bell Girls. "Four Square is really excellent."

To see if the bands could present a wide range of musical skills, the teachers asked them each to prepare two songs: one song with original words, and another in which students played instrumental music only. The judges finally chose the band Four Square as the winner of the contest. Four Square is a rock band with an unusual twist: it includes a violin player! The

members of Four Square write their own songs and practice three times a week after school. Students and teachers agreed that the band competition was a big success. All are looking forward to the school picnic in June.

(Source: https://goo.g1/SZazhm)

Question 99: Which would be the most appropriate headline for the article?

A. Famous Band Visits School B. Teachers Give Music Lessons

C. Students Have Fun at School Picnic D. School Holds Student Band Contest

Question 100: What is probably true about school picnics in recent years?

A. They were heldin March. B. They were not very exciting.

C. They were very hard to organize. D. They included musical performances

Question 101: The word formulaic is closest in meaning to _.

A. expensive B. uncertain C.long D. dull

Question 102: What type of band is NOT mentioned in the article?

A. Pop B. jazz C.Folk D. Rock

Question 103: Why did the teachers ask each band to play two songs?

  1. To decide which songs to play at the picnic
  2. To see if the bands could play a variety of songs
  3. To make sure the contest did not go on too long
  4. To make the concert more enjoyable for students

Question 104: According to the article, what is unusual about the band Four Square?

    1. It is named after a popular children's game.
    2. Its members practice several times a week.
    3. It is a rock band with a violinist.
    4. Its members all play guitar.

Question 105: According to the article, what does the band Four Square hope to do in the future?

      1. Perform at the school picnicevery year B. Perform in many different places

C. Buy new instruments D. Record an album

ĐỀ THI THỬ THPTQG 2019 – CÔ TRANG ANH – ĐỀ 7– TẬP 1

Exercise 15: Read the following passage and mark the letter A, B, C, or D on your answer sheet to indicate the correct answer to each of the questions.

China - Missing Women

In China the growing difference between the genders is giving signals of alarm to Government authorities. According to the latest census figures, 119 boys are born for every 100 girls. [A■]

This striking difference is expected to shoot up by the year 2020 with almost 40 million unsettled bachelors. This distribution of the social ecology would create havoc in the future. The social leaders are trying to pressurize the masses into producing more females. The Government has embarked on policies extending innumerable incentives to the families bearing girls.

Monetary support, free education, guaranteed employment is being gifted to parents who gift the country with a girl child. The Government is trying to persuade people to suppress their personal preferences and regulate their community behavior according to the new blueprint to stimulate the girl ratio. [B■] Sometimes the Government tries to woo them and sometimes it uses stein policies to force them into it

Consequent to the population explosion, the Government introduced, in the 80's, one child policy in China. Any additional pregnancy had to be terminated. This was aimed to put a check on the teeming millions. [C■] The policy had no relation to extermination of girl child in the womb. But the policymakers had no idea about its long term impact. People, with a patriarchic mindset, came up with their preference for a single male child. The idea of a happy family became 'parents with a single male child'.

The Chinese culture has always promoted sons over daughters because the society has been dominated by males. In villages, where hard work is needed to sustain the agriculture, a boy is always preferable due to his superior physical strength compared to that of a girl. [D■] In such circumstances, looking forward to a male baby seems justified. If people have to limit their families, it is obvious they would prefer a boy over a girl child. This problem has been accentuated by the use of ultrasound scanning which helps determine the sex of the fetus. This technology has played a crucial role in creating gender imbalance.

(Source: haps://goo.gilryW765)

Question 106: The word "havoc" in paragraph 1 is closest in meaning to ?

A. ordinary problem B. great destruction and trouble

C. serious dispute D. an infectious disease

Question 107: What is the main first of paragraph 1?

  1. Government is providing incentives to girls.
  2. Government is trying to decipher implications of the census records.
  3. Government is devising polices to undermine the position of boys in the society.
  4. Government is extending incentives to encourage parents with single girls

Question 108: What is the "blueprint" as discussed in paragraph 1?

  1. It is a list of rules for the Chinese people to follow to maintain population control.
  2. It is a rough plan for regulating community behavior.
  3. It is the Government's plan containing details of regulations and incentives to improve the girl child ratio.
  4. It is a printed guide for conduct rules governing sex ratio.

Question 109: The word "teeming" in paragraph 2 is closest in meaning to

A. overflowing B. female population C. couples D. general population

Question 110: What was the vision behind the government's policy discussed in paragraph 2?

  1. The vision about China with women at top positions in the government
  2. The vision for China to control its burgeoning population in near future
  3. The vision about a female dominated society
  4. The vision that Chinese young men and women would find life partners among themselves

Question 111: Which of the following is NOT correct about the one child policy in China?

    1. Each Chinese family was allowed to have only one child,
    2. Although the additional pregnancy was a girl child, it also had to be terminated.
    3. The one child policy was introduced by the Chinese government in the 1980s.
    4. With the purpose of controlling the population explosion, the Government introduced one child policy in China.

Question 112: Look at the four squares [■] that indicate where the following sentence can be added to the passage:

“Another reason is that daughters have to leave their parents after marriage to enrich their husband's family.”

Where would the sentence best fit?

[A■] [B■] [C■] [D■]

ĐỀ THI THỬ THPTQG 2019 – CÔ TRANG ANH – ĐỀ 8– TẬP 1

Exercise 16:Read the following passage and mark the letter A, B, C, or D on your answer sheet to indicate the correct answer to each of the questions.

Overpopulation, the situation of having large numbers of people with too few resources and too little space, is closely associated with poverty. It can result from high population density, or from low amounts of resources, or from both. Excessively high population densities put stress on available resources. Only a certain number of people can be supported on a given area of land, and that number depends on how much food and other resources the land can provide. In countries where people live primarily by means of simple farming, gardening, herding, hunting, and gathering, even large areas of land can support only small numbers of people because these labour intensive subsistence activities produce only small amounts of food.

In developed countries such as the United States, Japan, and the countries of Western Europe, overpopulation generally is not considered a major cause of poverty. These countries produce large quantitics of food through mechanized farming, which depends on commercial fertilizers, large-scale irrigation, and agricultural machinery. This form of production provides enough food to support the high densities of people in metropolitan areas.

A country's level of poverty can depend greatly on its mix of population density and agricultural productivity. Bangladesh, for example, has one of the world's highest population densities, with 1,147 persons per sq km. A large majority of the people of Bangladesh engage in low

  • productivity manual farming, which contributes to the country's extremely high level of poverty. Some of the smaller countries in Western Europe, such as the Netherlands and Belgium, have high population densities as well. These countries practise mechanized farming and are involved in high- tech industries, however, and therefore have high standards of living.

At the other end of the spectrum, many countries in sub-Saharan Africa have population densities of less than 30 persons per sq km. Many people in these countries practise manual subsistence farming; these countries also have infertile land, and lack the economic resources and technology to boost productivity. As a consequence, these nations are very poor. The United Slates has both relatively low population density and high agricultural productivity; it is one of the world's wealthiest nations.

High birth rates contribute to overpopulation in many developing countries. Children are assets to many poor families because the provide labour, usually for farming. Cultural norms in traditionally rural societies commonly sanction the value of large families. Also, the governments of developing countries often provide little or no support, financial or political, for family planning; even people who wish to keep their families small have difficulty doing so. For all these reasons, developing countries lend to have high rates of population growth.

(Trich dg thi THPTQGWarn 2015)

Question 113: Which of the following is given a definition in paragraph 1?

A. Overpopulation B. Population density C. Simple farming D. Poverty

Question 114: What will suffer when there are excessively high population densities?

A. Available resources B. Skilled labor

C. Farming methods D. Land area

Question 115: The phrase "that number" in paragraph 1 refers to the number of _.

A. people B. densities C. resources D. countries

Question 116: In certain countries, large areas of land can only yield small amounts of food because .

A. there is a lack of mechanization B. there are small numbers of labourers

C. there is on abundance of resources D. there is no shortage of skilled labour

Question 117: Bangladesh is a country where the level of poverty depends greatly on ______ _.

  1. its population density only
  2. both population density and agricultural productivity
  3. population density in metropolitan areas
  4. its high agricultural productivity

Question 118: The word "infertile" in paragraph 4 probably means _____ _.

A. disused B. impossible C. unproductive D. inaccessible Question 119: Which of the following is a contributor to overpopulation in many developing countries?

A. High-tech facilities B. Economic resources

C. Sufficient financial support D. High birth rate

Question 120: Which of the following could be the best title for the passage?

A. High Birth Rate and its Consequences B. Overpopulation; A Cause of Poverty

C. Overpopulation: A Worldwide Problem D. Poverty in Developing Countries

ĐỀ THI THỬ THPTQG 2019 – CÔ TRANG ANH – ĐỀ 8– TẬP 1

Exercise 17: Read the following passage and mark the letter A, B, C, or D on your answer sheet to indicate the correct answer to each of the questions.

If you could travel back in time five centuries, you'd encounter a thriving Aztec empire in Central Mexico, a freshly painted "Mona Lisa" in Renaissance Europe and cooler temperatures across the Northern Hemisphere. This was a world in the midst of the Little Ice Age (A.D. 1300 to 1850) and a period of vast European exploration now known as the Age of Discovery. But what if we could look 500 years into the future and glimpse the Earth of the 26th century? Would the world seem as different to us as the 21st century would have seemed to residents of the 16th century? For starters, what will the weather be like?

Depending on whom you ask, the 26th century will either be a little chilly or infernally hot.

Some solar output models suggest that by the 2500s, Earth's climate will have cooled back down

to near Little Ice Age conditions. Other studies predict that ongoing climate change and fossil fuel use will render much of the planet too hot for human life by 2300.

Some experts date the beginning of human climate change back to the Industrial Revolution in the 1800s, others to slash-and-burn agricultural practices in prehistoric times. Either way, tool- wielding humans alter their environment -- and our 26th century tools might be quite impressive indeed.

Theoretical physicist and futurist Michio Kaku predicts that in a mere 100 years, humanity will make the leap from a type zero civilization to a type I civilization on the Kardashev Scale. In other words, we'll become a species that can harness the entire sum of a planet's energy. Wielding such power, 26th-century humans will be masters of clean energy technologies such as fusion and solar power. Furthermore, they'll be able to manipulate planetary energy in order to control global climate. Physicist Freeman Dyson, on the other hand, estimates the leap to a type I civilization would occur within roughly 200 years.

Technology has improved exponentially since the 1500s, and this pace will likely continue in the centuries to come. Physicist Stephen Hawking proposes that by the year 2600, this growth would see 10 new theoretical physics papers published every 10 seconds. If Moore's Law holds true and both computer speed and complexity double every 18 months, then some of these studies may be the work of highly intelligent machines.

What other technologies will shape the world of the 26th century? Futurist and author Adrian Berry believes the average human life span will reach 140 years and that the digital storage of human personalities will enable a kind of computerized immortality. Humans will farm the oceans, travel in starships and reside in both lunar and Martian colonies while robots explore the outer cosmos.

(Source: haps://science,howstuffworks.com)

Question 121: Which of the following could be the main idea of the passage?

  1. How would the world change in the next 500 years?
  2. What would we do in the next five centuries?
  3. What problems would happen in the 26th century?
  4. How would technology improve the life in the far future?

Question 122: The word “infernally” in paragraph 2 is closest in meaning to _.

A. differently B. extremely

C. permanently D. contemporaneously

Question 123: According to the passage, what do experts predict about the weather in the future?

  1. The temperature will decline as much as possible because of the excessive fossil fuel use.
  2. The Little Ice Age will return due to the influence of solar energy.
  3. The Earth will have to face with extreme weather like frost or scorching period.
  4. The climate change will happen more quickly because of deforestation.

Question 124: The word "harness" in paragraph 4 could best be replaced by _____ _.

A. renew B. adjust C. exploit D. discover

Question 125: What does the word "they" in paragraph 4 refer to?

A. fusion and solar energy B. clean energy technologies

C. masters D. 26th - century humans

Question 126: Which of the following is TRUE about the future predictions?

  1. Michio Kaku believes that the progress from type 0 to type I civilization will take about two centuries.
  2. People in the 2500s are likely to control the energy of the Earth to limit the global climate.
  3. The speed of technology improvement will remain changeable in the far future.
  4. Solar energy will be the main power for the 26th -century citizens.

Question 127: According to Adrian Berry, the following are what future humans can do, EXCEPT

.

A. cultivating in the oceans B. traveling between the stars

C. exploring the universe by robots D. living as long as they want

Question 128: It can be inferred from the passage that ________ _.

  1. no one could be sure what the life would be like in the 2500s.
  2. what we imagine about the life in the 26th century may be the same what the 16th- ceritury people thought about the current life.
  3. the predictions of different scientists may draw the same world in the next 500 years.
  4. technology will affect most of the fields in the future life.

ĐỀ THI THỬ THPTQG 2019 – CÔ TRANG ANH – ĐỀ 9– TẬP 1

Exercise 18:Read the following passage and mark the letter A, B, C, or D on your answer sheet to indicate the correct answer to each of the questions

In Africa, people celebrate with joy the birth of a new baby. The Pygmies would sing a birth-

song to the child. In Kenya, the mother takes the baby strapped to her back into the thorn enclosure where the cattle is kept. There, her husband and the village elders wait to give the child his or her name.

In West Africa, after the baby is eight days old, the mother takes the baby for it's first walk in the big, wide world, and friends and family are invited to meet the new baby. In various African nations, they hold initiation ceremonies for groups of children instead of birthdays. When children reach a certain designated age, they learn the laws, beliefs, customs, songs and dances of their tribes, Some African tribes consider that children from nine to twelve are ready to be initiated into the grown up world. They may have to carry out several tests.

Masai boys around thirteen years old to seventeen undergo a two stage initiation. The first stage lasts about three months. The boys leave their parents' homes, paint their bodies white, and

are taught how to become young warriors. At the end of this stage, they have their heads shaved and they are also circumcised. The second stage, the young warriors grow their hair long and live in a camp called a manyatta where they practice hunting the wild animals that might attack the Masai herds. This stage may last a few years. When they are ready, they will marry and become owners of large cattle herds like their fathers.

The girls are initiated when they are fourteen or fifteen. They are taught by the older women about the duties of marriage and how to care for babies. Soon after that they are married and lead a life similar to that of their mothers.

Question 129: What does the passage mainly discuss?

  1. Celebrations for a new child in Africa.
  2. Birthday ceremonies for children in Africa.

(Source: http://www.birthdayceiebrations.ni)

  1. Traditions of Masai people when having a new baby.
  2. Activities in a birth celebration.

Question 130: Where do the father and older villagers in Kenya give the name to the child?

A. in the village B. at the place to keep the cattle

C. at their house D. at the fence of the cattle's cages.

Question 131: Which of the following is NOT mentioned in paragraph 2?

  1. An eight-day-old child will be taken for the first walk by his or her mother.
  2. Children have to learn their tribes' cultures and traditions when they are old enough.
  3. Children are prepared for a mature life when being met by friends and family.
  4. Children are likely to overcome some tests to enter the grown-up world.

Question 132: The word "undergo" in the third paragraph is closest in meaning to ______ _.

A. commence B. experience C. participate in D. explore

Question 133: The word "shaved" in paragraph 3 mostly means ____________ _.

A. cut the hair off with a razor B. tie the hair with a hairpin

C. dye the hair with another color D. cover the hair with a hat

Question 134: What does the word "they" in paragraph 3 refer to?

A. the Masai herds B. the wild animals C. the young warriors D. their heads

Question 135: What do the girls have to learn in their initiation?

  1. How to take care of her husband and family in the future.
  2. The responsibilities of marriage and the way to look after their children.
  3. Homemaking and childbearing.
  4. How to lead a life after marriage and giving birth to a child.

130

ĐỀ THI THỬ THPTQG 2019 – CÔ TRANG ANH – ĐỀ 9– TẬP 1

Exercise 19: Read the following passage and mark the letter A, B, C, or D on your answer sheet to indicate the correct answer to each of the questions.

EDUCATION IN THE FUTURE

Education is another area of social life in which information technology is changing the way we communicate. Today's college students may not simply sit in a lecture or a library to learn about their field. Through their computers and the wonders of virtual reality they can participate

in lifelike simulated experiences. Consider the following scenario of the future of education made possible through developments in information technology.

For children over the age of 10, daily attendance at schools is not compulsory. Some of the older children attend school only once or twice weekly to get tutorial support or instruction from a teacher. For the most part, pupils are encouraged to work online from home. Students must complete a minimum number of study hours per year; however, they may make up these hours by studying at home at times that suit their family schedule. They can log on early or late in the day and even join live classes in other countries. In order to ensure that each student is learning adequately, computer software will automatically monitor the number of hours a week each student studies online as well as that students' learning materials and assessment

activities. Reports will

be available for parents and

teachers. The software can then identify the best

learning activities and condition for each individual student and generate similar activities. It can also identify areas of weak achievement and produce special programs adjusted to the students' needs.

(Source: http://www.edudernic.corn/technology-pros-cons/)

Question 136. What is the topic of the passage?

  1. The effect of information technology on education.
  2. Students don't have to go to school any more.
  3. Computer software will make sure students learn at home.
  4. Students can know about their weak aspects to focus.

Question 137. How many times are children who are older than 10 required to go to school weekly?

A. No time B. Onceor twice C. Three D. Four

Question 138. Which of the following could best replace the word "encouraged"?

A. discouraged B. emboldened C. allowed D. banned

Question 139. The word "they" in paragraph 2 refers to .

A. students B. study hours C. children D. Older children

Question 140. Who/What counts the number of hours per week that students spend learning?

A. Virtual reality B. Teacher C. Parents D. Computer software

Question 141. What CAN'T the software do?

  1. Monitor the time the students learn.
  2. Design materials for the students.
  3. Find out the best activities for the students.
  4. Identify weaknesses of the students.

Question 142. What is NOT mentioned as a benefit of information technology to the students?

  1. Students can stay at home to learn.
  2. Students can learn at times that suit their schedule.
  3. Students' learning time won't be monitored.
  4. Students' weak achievement can be identified.

ĐỀ THI THỬ THPTQG 2019 – CÔ TRANG ANH – ĐỀ 10– TẬP 1

Exercise 20:Read the following passage and mark the letter A, 13, C, or D on your answer sheet to indicate the correct answer to each of the questions.

The Rise of Robots

As kids, our grandparents frequently scared us by talking about how difficult life was when they were growing up. They mentioned walking miles to school in the snow, or doing hours of manual labour for little pay. Life has changed greatly since then, and it seems to get easier year by year. In fact, with the help of robots we soon might have to do much at all. But is this a good thing?

By 2030, ifs estimated that robotics will be a $10 billion business worldwide. Companies are already starting to integrate them into the workforce. The electronics manufacturer Foxconn is drawing up plans to launch a factory within the next 10 years that's completely staffed by robot workers. Meanwhile, an American company Briggo has invented a robot that serves gourmet-quality coffee to customers. With the push of a button it will crush coffee beans, measure exact quantities of water, and even wave a steam wand to ensure customers get the perfect cream on top. Unlike human baristas, it can serve multiple drinks at once and work all day and night without a break.

Robots are also invading our homes. The Rooma is a mini-robot that vacuums rooms automatically according to a schedule. The Robomow is a device that will cut the grass for you while you sit in the shade. Then there's the Nanda clocky, an alarm clock that makes sure that even the deepest sleepers get up on time. The clock is attached to a pair of wheels, and it will randomly move around the room. Once you finally catch it, you're probably too awake to hit the snooze button. Although these early home robots are somewhat basic, they will likely become more capable as times goes on.

Although robots certainly help us to eliminate tedious tasks, many people are concerned about a future filled with robots. Some fear that humanity will start to decline if machines do everything for

us. Others have even warned about the robot rebellion, in which robots become so smart that they may decide to turn on their masters. These ideas may seem a bit far-fetched, but there are certainly lots of questions that need to be answered before everyone opens up to the idea of a robotic future.

(Source: https://goagi/ZAhTBI)

Question 143. How is Briggo's invention superior to human workers?

A. It is more knowledgeable about coffee-making. B. It is better at conversation.

C. It never has to stop. D. It can operate machinery.

Question 144. What would happen if you pressed the snooze button?

A. You would have to wake up immediately B. The alarm clock would turn off forever

C. The alarm would stop, but go off again soon D. Nothing would happen at all

Question 145. Which of the following is NOT true about the concerns over robots?

  1. It takes some time for people to accept robots.
  2. Robots may do some harm to humans.
  3. The idea of robotics may seem far-fetched.
  4. Robots may be dominant at the workplace.

Question 146. The word "tedious" in the last paragraph mostly means ______ _.

    1. boring B. difficult C. intelligent D. expensive

Question 147. What is the last paragraph mainly about?

  1. Some reasons why people don't accept robots yet.
  2. A very amusing science-fiction story about robots.
  3. How people in local communities are supporting robots.
  4. A few of the lastest robots on the market today.

Question 148. According to the first paragraph, how is the life changed since our parents time?

  1. The education system has got much worse
  2. Things aren't as hard as they once were
  3. Children have to walk longer distance to school
  4. It hasn't changed much at all

Question 149. All of the following are true about robots EXCEPT that _____ _.

  1. they can work all day and night without break
  2. they have greater capabilities
  3. they can draw up plans to launch a story
  4. they can do boring tasks for human

Question 150. We can infer from the passage that _____________.

  1. present domestic robots can work without any programs
  2. the communication will decline with the use of robots
  3. a robot rebellion can happen daily
  4. robotics will have been a major business by 2030

ĐỀ THI THỬ THPTQG 2019 – CÔ TRANG ANH – ĐỀ 10– TẬP 1

Exercise 21: Read the following passage and mark the letter A, B, C, or D on your answer sheet to indicate the correct answer to each of the questions.

The blue whale is the largest animal ever known to have existed. During the 20th century, the species was almost exterminated due to commercial whaling. The species has slowly recovered following the global whaling ban but it remains endangered and faces a number of serious threats including ship strikes and the impact of climate change.

Blue whales are simply enormous with most ranging in length from 24-30 m. The largest ever recorded was a gargantuan 33.5 m long. Females are up to 10 m longer than males. And they can weigh up to 200 tonnes. Just to put that in perspective: an adult male African elephant weighs 6 tonnes. The blue whale's heart is the size of a small car and its beat can be detected two miles away. But that's nothing compared to their calls. Blue whales are the loudest animals on earth and their calls are louder than a jet engine: reaching 188 decibels, while a jet's engine hit 'just' 140 decibels. Apart from their gigantic size, blue whales can be identified by their relatively small dorsal fin, a fairly rounded rostrum (anterior part of the skull), and approximately 90 ventral grooves, which reach the navel. They also have row of 300-400 baleen plates on each side of the mouth, which are black in color and range in length from 50 cm in front to 100 cm in back.

Blue whales mostly travel alone or in groups of 2-3. Larger groups of up to 60 whales have been reported and are probably associated with feeding grounds. However, the blue whale has the most powerful voice in the animal kingdom and its low-frequency sounds can travel in deep water over hundreds, or even thousands, of miles. Under these circumstances, animals which may appear to us to be traveling alone may actually be in constant contact with one another.

At birth, a blue whale calf is the largest baby on earth: approximately 8m long and weighing about 4 tonnes. They grow at a rate of 90 kg per day and wean after 7-8 months, once they have reached about 15 m in length, and are able to follow the normal migration pattern alone. They reach sexual maturity at 5-10 years. This growth rate is astonishing and is probably the fastest in the animal kingdom. From conception to weaning, it represents a several billion-fold increase in tissue in just over a year and a half. Like other baleen whales, the blue whale has no teeth so it is hard to tell its age but scientists believe they live until at least 50.

Like other large whales, blue whales are threatened by chemical and sound pollution, habitat loss, overfishing of krill, ship strikes and becoming entangled in fishing gear. Climate change could also have a

major impact on its food supply, since global warming and associated ocean acidification may impact krill populations. In addition, frontal zones - critical whale habitats - are projected to move further south due to climate change. Frontal zones are boundaries between different water masses, where water can rise from the depths, bringing with it large amounts of nutrients that stimulate the growth of phytoplankton and support substantial populations of prey species for whales. Blue whales would have to migrate further (perhaps 200-500 km more) to reach and feed at these food-rich areas where they build up reserves to sustain themselves for the rest of the year. These longer migration paths could increase the energy costs of migration and reduce the duration of the main feeding season. As frontal zones move southward, they also move closer together, reducing the overall area of foraging habitat available.

(Source: http://wwfpanda.orgl)

Question 151: What is the purpose of the author in the passage?

  1. To provide basic information about the blue whale and its major threats.
  2. To prove that the blue whale is the biggest mammal on the Earth.
  3. To give evidence of how the blue whale changes the habit and feeding grounds.
  4. To raise people's awareness of the extinction of the blue whale,

Question 152: Which of the following is NOT correct about the blue whale's physical description?

  1. It is very easy to discover the appearance of a blue whale via its heart beat
  2. Male blue whales are the longest in their species.
  3. The calls of the blue whale are louder than those of any animals on the Earth.
  4. A small dorsal fin is also the identity of the blue whale.

Question 153: According to the passage, why isn't a blue whale lonely when it travels alone?

  1. Because most of other blue whales also travel alone.
  2. Because it can contact with others through their powerful voice.
  3. Because it shares the same feeding grounds with others.
  4. Because other blue whales will appear when it reaches the destination.

Question 154: What does the word "wean" in paragraph 4 mean?

A. stop feeding a calf with its mother's milk B. stop growing as fast as before

C. begin living independently D. begin trying to find food

Question 155: The word "entangled" in paragraph 5 is closest in meaning to _.

A. endangered B. disappeared C. ensnared D. attracted

Question 156: What does the word "it" in the last paragraph refer to?

A. the depth B. water C. water mass D. climate change

Question 157: According to the last paragraph, how does climate change affect the blue whale?

  1. It increases the acidity in the ocean, which makes the water polluted.
  2. It impacts on the tide which contains a lot of nutrients to feed the blue whale's prey.
  3. It makes the blue whale travel farther south to find feeding grounds.
  4. It encourages frontal zones so that the blue whale cannot move around.

Question 158: It can be inferred from the passage that ____ _.

  1. Human activities are one of the primary reasons for the decline of the blue whale's population.
  2. The blue whale is the animal that has the longest life expectancy.
  3. Despite being the largest animals, blue whales are the most vulnerable on the Earth.
  4. The blue whale lives safely in the ocean thanks to their solidarity.

ĐỀ THI THỬ THPTQG 2019 – CÔ TRANG ANH – ĐỀ 11– TẬP 1

Exercise 22:Read the following passage and mark the letter A, B, C, or D on your answer sheet to indicate the correct answer to each of the questions.

Scientists have uncovered a new threat to the world's endangered coral reefs. They have found that

most are incapable of growing quickly enough to compensate for rising sea levels triggered by global warming. The study suggests that reefs - which are already suffering serious degradation because the world's seas are warming and becoming more acidic - could also become overwhelmed by rising oceans.

The research - led by scientists at Exeter University and published in Nature this week - involved studying growth rates for more than 200 tropical western Atlantic and Indian Ocean reefs. It was found only 9% of these reefs had the ability to keep up with even the most optimistic rates of sea-level rises forecast by the Intergovernmental Panel on Climate Change. "For many reefs across the Caribbean and Indian Ocean regions, where the study focused, rates of growth are slowing due to coral reef degradation," said Professor Chris Perry, of Exeter University. “Meanwhile, rates of sea-level rise are increasing - and our results suggest reefs will be unable to keep up. As a result, water depths above most reefs will increase rapidly through this century.”

Sea levels rose by several inches over the past century and measurements indicate the speed of this increase is now rising significantly. Two key factors are involved: climate change is making ocean water warmer and so it expands. And as ice sheets and glaciers melt, they increase amounts of water in the oceans.

At the same time, reefs are being weakened by ocean warming and also by ocean acidification, triggered as the seas absorb more and more carbon dioxide. These effects lead to bleaching events that kill off vast stretches of coral and limits their ability to grow.

“Our predictions, even under the best case scenarios, suggest that by 2100, the inundation of reefs will expose coastal communities to significant threats of shoreline change,” said co-author Prof Peter Mumby of Queensland University. This point was backed by US marine scientist Ilsa

Ruffner writing in a separate comment piece for Nature. “The implications of the study are dire. Many island nations and territories are set to quickly lose crucial natural resources.”

(Source: https://www.theguardiamcomi)

Question 159: Which of the following does the passage mainly discuss?

  1. Sea levels will become the main factor of bleaching.
  2. Coral reefs will be overwhelmed by rising oceans.
  3. Coral reefs may escape from extinction because of the increase in sea levels.
  4. Global warming will cause the rise of sea levels.

Question 160: What does the word "compensate" in the first paragraph probably mean?

A. recompense B. keep up with C. develop D.benefit Question 161: What did scientists at Exeter University find in their research?

  1. Tropical coral reefs are increasing more quickly than others in Pacific Ocean.
  2. The majority of tropical coral reefs cannot keep pace with the increasing rate of sea levels.
  3. Many coral reefs are developing in spite of their degradation.
  4. The rapid rise in sea levels does not affect the density of coral reefs.

Question 162: According to the passage, the two factors causing the rise of sea levels are _____ _.

A. global warming and freeze B. severe weather and climate change

C. climate change and ice expansion D. climate change and ice melt

Question 163: What does the phrase "these effects" in paragraph 4 refer to?

A. ocean warming and ocean acidification B. reef weakening and ocean warming

C. reef weakening and ocean acidification D. ocean warming and CO2 absorb

Question 164: The word "inundation" is closest in meaning to _.

A. drought B. extinction C. flood D. tsunami

Question 165: The author implies in the last paragraph that _______.

  1. even in the most optimistic prediction, coral reefs will experience their extinction.
  2. the results of the study are more serious than what scientists have predicted.
  3. human activities will not only affect marine life but also put themselves at risk.
  4. people often exploit natural resources in island nations and territories.

ĐỀ THI THỬ THPTQG 2019 – CÔ TRANG ANH – ĐỀ 11– TẬP 1

Exercise 23: Read the following passage and mark the letter A, B, C, or D on your answer sheet to indicate the correct answer to each of the questions.

Plastic bags are used by everybody. From a vegetable vendor to a designer store, everyone seems to use them. Even though they are one of the modern conveniences that we seem to be unable to do without, they are responsible for causing pollution, killing wildlife, and using up the precious resources of the Earth. But, most of us are blissfully unaware of the repercussions that are occurring and will take place in the future because of the plastic bags.

Every once in a while, the government passes out an order banning store owners from providing plastic bags to customers for carrying their purchases, with little lasting effect. Plastic bags are very popular with both retailers as well as consumers because they are cheap, strong, lightweight, functional, as well as a hygienic means of carrying food as well as other goods. About a hundred billion plastic bags are used every year in the U.S. alone. And then, when one considers the huge economies and populations of India, China, Europe, and other parts of the world, the numbers can be staggering. The problem is further exacerbated by the developed countries shipping off their plastic waste to developing countries like India.

Once they are used, most bags go into landfills. Each year, more and more bags are ending up littering the environment. Once they become litter, plastic bags find their way into our waterways, parks, beaches, and streets. And, if they are burned, they infuse the air with toxic fumes. About 100,000 animals, such as dolphins, turtles, whales, penguins are killed every year due to these bags. Many animals ingest plastic bags, mistaking them for food, and therefore, die. And worse, the ingested plastic bag remains intact even after the death and decomposition of the animal. Thus, it lies around in the landscape where another victim may ingest it. One of the worst environmental effects is that they are non- biodegradable. The decomposition takes about 400 years. No one will live so long to witness the decomposition of plastic! Thus, save the environment for the future generations as well as animals.

Petroleum products are diminishing and getting more expensive by the day, since we have used this non-renewable resource increasingly. And to make plastic, about 60-100 million barrels of oil are needed every year around the world. Surely, this precious resource should not be wasted on producing plastic bags, should it? Petroleum is vital for our modern way of life. It is necessary for our energy requirements - for our factories, transportation, heating, lighting, and so on. Without viable alternative sources of energy yet on the horizon, if the supply of petroleum were to be turned off, it would lead to practically the entire world grinding to a halt.

So, what can be done? A tote bag can make a good substitute for carrying groceries and the shopping. You can keep the bag with the cashier, and then put your purchases into it instead of the usual plastic bag. Recycling the bags you already have is another good idea. These can come into use for various purposes, like holding your garbage, instead of purchasing new ones. While governments may be working out ways to lessen the impact of plastic bags on the environment; however, each of us should

shoulder some of the responsibility for this problem, which ultimately harms us. Plastics are not only non- biodegradable, but are one of the major pollutants of the sea. For a clean and green environment, try to use alternatives to plastic whenever and wherever possible. Cut down your use of plastic, and do your bit to save our planet.

(Source: https://helpsa.venature.com/)

Question 166: Which of the following could be the best tittle of the passage?

  1. Plastic pollution - Problems and solutions.
  2. Harmful effects of plastic bags on the environment.
  3. Plastic pollution - What should we do?
  4. Plastic bags - New threat for the environment,

Question 167: What is the synonym of the word "repercussions" in the first paragraph?

A. situation B. interference C. contamination D. consequence

Question 168: According to the second paragraph, what is NOT true about the reality of plastic bags?

  1. The more population a country has, the more complicated the plastic pollution is.
  2. Plastic bags are often used because of their convenience.
  3. Importing plastic waste from developed countries makes the problem more sophisticated.
  4. The governments do not prohibit the use of plastic bags at the stores. Question 169: The following are the negative effects of plastic bags on the environment, EXCEPT .
  5. They make water, soil and air polluted.
  6. Marine animals may die because of eating plastic bags.
  7. It takes plastic bags a very long time to be biodegraded.
  8. People do not live long enough to check whether plastic bags are decomposed or not.

Question 170: The word "intact" in paragraph 3 is closest in meaning to _______ _.

A. undamaged B. broken C. harmful D. dangerous

Question 171: According to the passage, petroleum is described as a resource that _____ _.

  1. increases the cost of products.
  2. is wasted on many energy demands of life.
  3. is indispensable in almost all activities of the modern world.
  4. is replaced by other alternatives in producing plastic bags.

Question 172: What does the word "it" in the last paragraph refer to?

A. the cashier B. the bag C. the shopping D. a good substitute

Question 173: What does the author suggest in the last paragraph?

175

  1. Plastic bags should be limited as much as possible to save the Earth from their harmful impacts.
  2. Each individual should reuse their plastic bags to save money for other purchases.
  3. The governments should cooperate with every citizen in cleaning the plastic bags in the polluted oceans.
  4. It is easier to keep the environment clean at first than to save it after it is polluted.

Exercise 24:Read the following passage and mark the letter A, 8, C, or D on your answer sheet to indicate the correct answer to each of the questions.

Many ants forage across the countryside in large numbers and undertake mass migrations; these

activities proceed because one ant lays a trail on the ground for the others to follow. As a worker ant returns home after finding a source of food, it marks the route by intermittently touching its stinger to the ground and depositing a tiny amount of trail pheromone - a mixture of chemicals that delivers diverse messages as the context changes. These trails incorporate no directional information and may be followed by other ants in either direction.

Unlike some other messages, such as the one arising from a dead ant, a food trail has to be kept secret from members of other species. It is not surprising then that ant species use a wide variety of compounds as trail pheromones. Ants can be extremely sensitive to these signals. Investigators working with the trail pheromone of the leafcutter ant Atta texana calculated that one milligram of this substance would suffice to lead a column of ants three times around Earth.

The vapor of the evaporating pheromone over the trail guides an ant along the way, and the ant detects this signal with receptors in its antennae. A trail pheromone will evaporate to furnish the highest concentration of vapor right over the trail, in what is called a vapor space. In following the trail, the ant moves to the right and left, oscillating from side to side across the line of the trail itself, bringing first one and then the other antenna into the vapor space. As the ant moves to the right, its left antenna arrives in the vapor space.

The signal it receives causes it to swing to the left, and the ant then pursues this new course until its right antenna reaches the vapor space. It then swings back to the right, and so weaves back and forth down the trail.

Question 174: What does the passage mainly discuss?

  1. The information contained in pheromones
  2. The mass migration of ants
  3. How ants mark and follow a chemical trail

(Source: https://goo.gl/G1HjCF)

  1. Different species of ants around the world

Question 175: The word "intermittently" in paragraph 1 is closest in meaning to _.

A. roughly B. incorrectly C. rapidly D. periodically

Question 176: The phrase "the one" in paragraph 2 refers to a single _.

A. message B. food trail C. dead ant D. species

Question 177: According to the passage, why do ants use different compounds a trail pheromones?

  1. To reduce their sensitivity to some chemicals
  2. To indicate how far away the food is
  3. To attract different types of ants
  4. To protect their trail from other species

Question 178: The author mentions the trail pheromone of the leafcutter ant in paragraph 2 to point out .

  1. a type of ant that is common in many parts of the world
  2. how little pheromone is needed to mark a trail
  3. the different types of pheromones ants can produce
  4. that certain ants can produce up to one milligram of pheromone

Question 179: According to the passage, how are ants guided by trail pheromones?

  1. They sense the vapor through their antennae.
  2. They avoid the vapor spaces by moving in a straight line.
  3. They concentrate on the smell of food.
  4. They follow an ant who is familiar with the trail

Question 180: According to the passage, the highest amount of pheromone vapor is found

A. in the source of food B. in the receptors of the ants

C. under the soil along the trail D. just above the trail

Exercise 25: Read the following passage and mark the letter A, B, C, or D on your answer sheet to indicate the correct answer to each of the questions.

Deforestation is the clearing, destroying, or otherwise removal of trees through deliberate,

natural or accidental means. It can occur in any area densely populated by trees and other plant life, but the majority of it is currently happening in the Amazon rainforest. The loss of trees and other vegetation can cause climate change, desertification, soil erosion, fewer crops, flooding, increased greenhouse gases in the atmosphere, and a host of problems for indigenous people.

Deforestation occurs for a number of reasons, including: farming, mostly cattle due to its quick turn around; and logging for materials and development. It has been happening for thousands of years, arguably since man began converting from hunter/gatherer to agricultural based societies, and required larger, unobstructed tracks of land to accommodate cattle, crops, and housing. It was only after the onset of the modern era that it became an epidemic.

One of the most dangerous and unsettling effects of deforestation is the loss of animal and plant species due to their loss of habitat; not only do we lose those known to us, but also those unknown, potentially an even greater loss. Seventy percent of Earth's land animals and plants live in forests, and many cannot survive the deforestation that destroys their homes. The trees of the rainforest that provide shelter for some species also provide the canopy that regulates the temperature, a necessity for many others. Its removal through deforestation would allow a more drastic temperature variation from day to night, much like a desert, which could prove fatal for current inhabitants.

In addition to the loss of habitat, the lack of trees also allows a greater amount of greenhouse gases to be released into the atmosphere. Presently, the tropical rainforests of South America are responsible for 20% of Earth's oxygen and they are disappearing at a rate of 4 hectares a decade. If these rates are not stopped and reversed, the consequences will become even more severe.

The trees also help control the level of water in the atmosphere by helping to regulate the water cycle. With fewer trees left, due to deforestation, there is less water in the air to be returned to the soil. In turn, this causes dryer soil and the inability to grow crops, an ironic twist when considered against the fact that 80% of deforestation comes from small-scale agriculture and cattle ranching.

Further effects of deforestation include soil erosion and coastal flooding, In addition to their previously mentioned roles, trees also function to retain water and topsoil, which provides the rich nutrients to sustain additional forest life. Without them, the soil erodes and washes away, causing farmers to move on and perpetuate the cycle. The barren land which is left behind in the wake of these unsustainable agricultural practices is then more susceptible to flooding, specifically in coastal regions. Coastal vegetation lessens the impact of waves and winds associated with a storm surge. Without this vegetation, coastal villages are susceptible to damaging floods.

(Source: haps://www.pacharnarna.ory/)

Question 181: What does the passage mainly discuss?

  1. The definition of deforestation.
  2. The negative impacts of deforestation.
  3. The reasons why deforestation occurs frequently.
  4. The effective solutions to solve the deforestation all over the world.

Question182: The word "indigenous" in the first paragraph probably means .

A. foreign B. agrarian C. native D. mountainous

Question 183: Which of the following is NOT stated as the reason of deforestation in paragraph 2?

A. cutting trees for wood B. ranching cattle

C. cultivating D. hunting for food.

Question 184: What does the word "others" in the third paragraph refer?

A. canopy B. species C. rainforest D. trees

Question 185: The following are negative effects of deforestation, EXCEPT .

  1. Myriad flora and fauna species have been eradicated before we discover them.
  2. The alteration of temperature in a complete day is becoming more radical due to the loss of trees.
  3. The soil is infertile to grow crops because of the shortage of rain and erosion.
  4. More and more animals have to find a place of refuge under the canopy of leaves.

Question 186: The word "perpetuate" in the last paragraph is closest in meaning to _.

A. maintain B. stop C. prevent D. start

Question 187: Coastal regions are mentioned in the last paragraph as an example of regions that .

  1. are easy to be vulnerable in a storm surge.
  2. are left behind because of the reduction of forests.
  3. are not much influenced by flood.
  4. often experience droughts or erosion.

Question 188: The paragraph following the last paragraph in the passage may probably discuss .

  1. a number of disadvantages that deforestation puts local people at.
  2. some drastic measures that should be taken to deal with deforestation.
  3. the reality of deforestation in some specific countries.
  4. some scientists' perspectives about deforestation.

ĐỀ THI THỬ THPTQG 2019 – CÔ TRANG ANH – ĐỀ 13– TẬP 1

Exercise 26:Read the following passage and mark the letter A, B, C, or D on your answer sheet to indicate the correct answer to each of the questions.

Thanks to electricity, candles are no longer a necessity in our lives, but they are still a pleasure. The warm flame of candlelight can quickly alter the mood and atmosphere of a room, often creating a peaceful scene that electric light just cannot match.

Candles are an important part of many cultural and religious festival and have been burnt in various forms for centuries. The ancient Romans and Egyptians made candles from a type of fiber coated with wax. However, up until the nineteenth century, most candles were made from a substance called tallow,

obtained from beef fat. Tallow candles were very smoky and, understandably, had an unpleasant odor.

In the 19th century, candle makers learned to separate stearin, the solid form fat used as a type of glue, from tallow and used it to harden other fats. Candles that contained stearin would burn longer than previous ones and had a better smell. Stearin is still one of the main components of modern candle- making, and the techniques used to create candles today are much the same as they have been for many years. These days, the increase range of wax dyes, perfumes and other additives that are now available make candle making a very exciting and rewarding hobby.

Sales of candles have increased greatly over the last few years, showing that they have become part of our lives again, not through necessity, but because of the magical atmosphere they create. In our increasingly stressful lives, the calming quality of candlelight has a relaxing effect that many enjoy.

For those would like to learn to make candles, finding and buying candle-making kits is easy. Candle- making is definitely enhanced by the exciting possibilities of experimentation with various materials. Be brave and try out different effects - some of the most wonderful creations can happen by accident. With a bit of practice, you will be amazed at the very professional finish that can be achieved.

Despite their delicate beauty, candles can, of course, be highly dangerous. One should never leave lit candles unattended, even for a few moments. Always make sure candles are securely placed within candleholders.

Teach your children to respect a burning candle, and of course keep burning candles away from flammable materials. Keep your home safe by remembering that a candle is magical, but fire can be very destructive. Be careful, and enjoy the beauty of your candles!

Question 189: According to the text, what is the importance of candles?

  1. We need them in case there is an electric backout.
  2. We gain a feeling of calm when they are lit in a room.
  3. We light them to create a more romantic atmosphere.
  4. We cannot do without them in our daily lives.

(Source: https://goo.gl/5jMKjN)

Question 190: Why dose the writer mention ancient Romans and Egyptians in the text?

  1. to show that before making candles, we should learn their history
  2. to show one of the ways candles were made in the past
  3. to show how candles were invented
  4. to show that candles have always been used

Question 191: Why was stearin used in candles?

  1. It create a more colorful flame than tallow.
  2. It covered up unpleasant smells in the air.
  3. It was used to get rid of fat in the candles.
  4. It helped make candles burn longer.

Question 192: According to the text, what does the increase of candle sales illustrate?

  1. People enjoy the calming effect that candles produce.
  2. The variety of candles to choose from makes them more attractive.
  3. People buy candles as gifts more often than they used to.
  4. Candles are cheaper to buy now than they were in the past.

Question 193: According to the text, what is needed to succeed at candle-making?

A. a few years of practice B. a lot of time and money

C. willingness to take risks D. many expensive materials

Question 194: What dose the passage warn readers not to do?

A. burn candle without adult supervision B. leave a room where a candle is burning

C. use unapproved ingredients in candles D. burn candles for only a few minutes

Question 195: Which of the following best expresses the main idea of this passage?

  1. Candle-making can be a successful business.
  2. Candles are made from different types of material.
  3. Candles continue to be a part of people's lives.
  4. Candle-making has changed little over the years.

ĐỀ THI THỬ THPTQG 2019 – CÔ TRANG ANH – ĐỀ 13– TẬP 1

Exercise 27: Read the following passage and mark the letter A, B, C, or D on your answer sheet to indicate the correct answer to each of the questions.

Garbage cans are not magical portals. Trash does not disappear when you toss it in a can. Yet, the

average American throws away an estimated 1,600 pounds of waste each year. If there are no magic garbage fairies, where does all that trash go? There are four methods to managing waste:

Recycling, landfilling, composting, and incinerating. Each method has its strengths and weaknesses. Let’s take a quick look at each.

Recycling is the process of turning waste into new materials. For example, used paper can be turned into paperboard, which can be used to make book covers. Recycling can reduce pollution, save materials, and lower energy use. Yet, some argue that recycling wastes energy. They believe that collecting, processing, and converti g waste uses more energy than it saves. Still, most people agree that recycling is better for the planet

than landfilling.

Landfilling is the oldest method of managing waste. In its simplest form, landfilling is when people bury garbage in a hole. Over time the practice of landfilling has advanced. Garbage is compacted before it is thrown into the hole. In this way more garbage can fit in each landfill. Large liners are placed in the bottom of landfills so that toxic garbage juice doesn't get into the ground water. Sadly, these liners don't always work. Landfills may pollute the local water supply. Not to mention that all of that garbage stinks. Nobody wants to live next to a landfill. This makes it hard to find new locations for landfills.

Compositing is when people pile up organic matter, such as food waste, and allow it to decompose. The product of this decomposition is compost. Compost can be added to the soil to make the soil richer and better for growing crops. While composting is easy to do onsite somewhere, like home or school, it's hard to do after the garbage gets all mixed up. This is because plastic and other inorganic materials must be removed from the compost pile or they will pollute the soil. There's a lot of plastic in garbage, which makes it hard to compost on a large scale.

One thing that is easier to do is burning garbage. There are two main ways to incinerate waste. The first is to create or harvest a fuel from the waste, such as methane gas, and burn the fuel. The second is to burn the waste directly. The heat from the incineration process can boil water, which can power steam generators. Unfortunately, burning garbage pollutes the air. Also, some critics worry that incinerators destroy valuable resources that could be recycled.

Usually, the community in which you live manages waste. Once you put your garbage in that can, what happens to fit is beyond your control. But you can make choices while it is still in your possession. You can choose to recycle, you can choose to compost, or you can choose to let someone else deal with it. The choice is yours.

Question 196: Which of the following serves as the best title for this passage?

  1. The Magic of Recycling: Bringing Back What Was Once Lost
  2. Recycling, Landfilling or Composing: Which is Best for You?
  3. Do Your Part How to Save the Earth by Recycling and Composting.
  4. Methods of Waste Management: Advantages and Disadvantages

Question 197: According to the passage all of the following are mentioned as an issue with landfilling EXCEPT that

A. landfills are smelly B. landfills may pollute the water supply

C. it is difficult to find locationsfor landfills D. usable materials are wasted in landfills

Question 198: What does the word "stinks" in paragraph 3 mean?

A. smells unpleasant B. seems dirty C. looks attractive D. feels soft

Question 199: Which of the following best explains why composting is not feasible on a large scale?

  1. People wouldn't want to touch all of that gross rotting food.
  2. Plastic would get into the compost and turn it into a pollutant.
  3. It would smell too bad in densely populated cities.
  4. It would attract rodents that would spread disease.

Question 200: According to the passage which of the following best defines "fincineTaittion"?

  1. buying waste materials in a large hole
  2. allowing waste products to decompose and become fertilizer
  3. turning waste materials into products like book covers
  4. burning waste materials and harvesting the energy

Question 201: The word "it" in the last paragraph refers to

A. community B. garbage C. waste D. possession

Question 202: According to the passage how many main ways to incinerate waste?

A. one B. three C. two D. four

Question 203: Which conclusion could be drawn from the passage?

  1. Recycling is without a doubt the best way to handle waste.
  2. Each method of waste management has its drawbacks.
  3. Incineration is the best way to process waste.
  4. All large cities should create massive compost piles.

Exercise 28:Read the following passage and mark the letter A, B, C, or D on your answer sheet to indicate the correct answer to each of the questions.

One of the most interesting parts of the earth for many people was and continues to be the

beautiful world of the sea. People were always attracted to the mysteries of the deep waters when they saw a little part of the under water world on television or in photographs. Due to this intense attraction caused by the mystery of the waters, more and more people choose to practice scuba diving, which became in time one of the most popular sports around the world. Scuba diving is a complex sport that requires some research before being understood at its real value. In order for someone to practice it, it is required for them to take some courses so accidents can be avoided.

Scuba diving is not only about diving and observing the amazing environment that exists under the waters. This sport can be transformed into a successful career by those who are truly passionate about it. At the moment, scuba diving has four main areas of interest: recreation purposes, commercial purposes, scientific research and military activity as well. The most

frequent reason why people go for scuba diving is because they want to relax in a special way, by having contact with a less known world.

A great thing about this sport is that it can be practiced anytime around the year; however, you should choose the summer if you have problems with low temperatures. Once you take the lessons, you get the basic knowledge before diving, you should talk to a specialist and obtain quality equipment for this activity. For the beginning, you can only rent scuba diving equipment, but if you get attached to this sport, then you will have to purchase one of your own at some moment. Some of the most important parts of the scuba gear are the mask, the snorkel, the diving suit, the gloves, the boots and the diving regulator.

If you finally decided to try scuba diving and you already took some scuba courses then you must be thinking about what location to choose. This is one of the most difficult decisions to make as our world is filled with numerous destinations, equally beautiful and interesting. If this passion catches you then you will surely get to visit more and more amazing locations each year. There is no such thing as the most beautiful waters to scuba diving in as each part has something to offer and shelters special creatures that should be observed.

Question 204: Which of the following could be the best tittle of the passage?

A. The big attraction to scuba diving. B. The new interest on the Earth.

C. The requirements for scuba diving. D. The place to practice scuba diving.

Question 205: The word "intense" in the first paragraph is closest in meaning to

A. moderate B. special C. strong D. obtuse

Question 206: What does the word "it" in paragraph 1 refer to?

A. its real value B. research C. scuba diving D. the world

Question 207: According to the passage, which of the interest of scuba diving is NOT mentioned?

A. relaxing B. trading

C. activities for the armed forces D. adventure

Question 208: The word "attached" in paragraph 3 probably means _

A. joined B. enclosed C. suitable D. attacked

Question 209: Which of the following is TRUE about scuba diving?

  1. Although you are really interested in scuba diving, you cannot have a successful career with it.
  2. Many people want to try scuba diving in order to explore the new world.
  3. To practice scuba diving, people need plenty of indispensable equipment.
  4. Buying suitable scuba diving equipment is a good idea for those who go for this sport.

Question 210: What benefit of scuba diving does the author mention in the last paragraph?

  1. Exploring more wonderful destinations in the world.
  2. Observing more locations to fulfill your passion.
  3. Taking more scuba courses to understand this sport.
  4. Choosing a location to visit the most beautiful water.

Exercise 29: Read the following passage and mark the letter A, B, C, or D on your answer sheet to indicate the correct answer to each of the questions.

Children all around the world love to read comic books. Maybe they like to read the kind of

comic books that come out once a month, like Spiderman or Batman, or maybe they like mangastyle comic books. That means that publishers who make comic books need to keep in touch with the things young people like to do. One thing that many young people today are interested in is technology, so of course kids want to read digital comic books on electronic devices.

Let's take Japanese rnanga as an example. There are lots of people around the world who enjoy reading manga. However, in the past, it was difficult to find translated versions of manga in comic bookstores. But that did not stop manga fans. Some fans who could translate Japanese into their own language started translating manga themselves. Then they scanned the pages of manga books and uploaded the scanned pages to the Internet with their translation. This practice has become so popular. People in the comic book industry made up a name for it. By combining the words scan and translation, they call it "scanlation", and it is a big problem today. The people who do scanlation usually share their manga for free, so readers do not buy manga books. Writers, artists, and publishers all end up losing money because of scanlation. Another problem is piracy in the comic book industry. illegal copies of old and brand new comics alike have been hurting comic book sales.

Despite the progress made in converting comics to digital formats, hardcopy comic books are still by far the most popular format among fans. In the meantime, comic book publishers and stores will just have to keep an eye on their buyers' habits. As the popularity of digital books for e- readers and tablets grows, the popularity of digital comic books will grow as well.

Question 211. What is the passage mainly about?

A. The need for digital comic books B. Children's most favourite type of books

C. Problems in the book industry D. The manga industry in Japan

Question 212. The word "they" in paragraph 1 refers to_____

A. children B. comic books C. publishers D. things

Question 213. According to paragraph 1, Spiderman is the name of .

A. a TV programme B. a comic book C. an electronic device D. a child

Question 214. According to paragraph 2, the practice of "scanlation" mostly involves .

A. selling translated manga books B. scanning and translating manga books

C. scanning uploaded manga books D. translating uploaded manga books Question 215. According to the passage, in the past, some manga fans who knew Japanese translated manga because

  1. it was enjoyable to translate Japanese books
  2. they wanted to make a name for themselves
  3. it was difficult to buy translated manga books
  4. they wanted to read manga for free

Question 216. According to paragraph 2, who could benefit from scanlation?

A. Readers B. Publishers C. Writers D. Artists

Question 217. The word "piracy" in paragraph 2 probably means the act of

A. controlling the production of a book B. hurting the sale of illegal comic books

C. copying and using abook illegally D. advertising a book without permission

ĐỀ THI THỬ THPTQG 2019 – CÔ TRANG ANH – ĐỀ 15– TẬP 1

Exercise 30:Read the following passage and mark the letter A, B, C, or D on your answer sheet to indicate the correct answer to each of the questions

The American type of football was developed in the 19th century from soccer and rugby football. Played by professionals, amateurs, college and high school students, or young children, football in American is one of the most popular sports besides basketball and baseball. It attracts millions of fans each fall and people are very supportive of their favourite teams. The football playing field of today is rectangular in shape and measures 100 yards long and 53.5 yards wide. White lines are painted on the playing field to mark off the distances to the end zone. The games is divided into four quarters, each fifteen minutes long. The first two quarters are known as the first half. There is a rest period between two halves which usually last about fifteen minutes. Each team has eleven players. Each team has offensive players who play when the team has possession of the ball and defensive players who play when the other team has the possession of the ball. Because of the body contact players have during the game, helmets are worn to protect their head and face area, whereas pads are worn to protect the shoulders, arms, and legs. Also, there are officials carrying whistles and flags to make certain that the rules of the game are followed during the game. The football is made of leather and is brown in colour. It is shaped much like an oval and thrown or someone is running with it. The eight stitches on the top of the football help the players to grip ball when throwing or passing. The most famous game of the year is Super Bowl that is played in January or February. It is televised around the world and is watched by millions of people each year.

Question 218: What do officials do during the game of football?

A. build up excitement among the fans B. supervise the game

C. takeup tickets D. spectate the game

Question 219: As mentioned in the text, who are the most active when their team has the ball?

A. offensive players B. defensive players C. the officials D. the fans

Question 220: Playing American football is the most similar to playing .

A. rugby football B. basketball C. baseball D. volleyball

Question 221: What do pads help the players to protect?

A. their legs and arms B. their heads C. thewhole body D. their faces

Question 222: Which sport is played most in the US?

A. Rugby football B. Soccer C. American football D. It's not mentioned

Question 223: When is the most famous football game held annually?

A. inthe fall B. January C. February D. January or February

Question 224: Why are there white rings on each end of the football?

A. to mark off the distances to the end zone B. to help players run

C. to helpplayers score D. to help players see the ball

Question 225: The word "grip" in the passage means to .

A. avoid something B. take something away

C. old something tightly D. detect something

ĐỀ THI THỬ THPTQG 2019 – CÔ TRANG ANH – ĐỀ 15– TẬP 1

Exercise 31: Read the following passage and mark the letter A, B, C, or D on your answer sheet to indicate the correct answer to each of the questions.

How is the news different from entertainment? Most people would answer that news is real but entertainment is fiction. However, if we think more carefully about the news, it becomes clear that the news is not always real. The news does not show us all the events of the day, but stories from a small number of chosen events. The creation of news stories is subject to specific constraints, much like the creation of works of fiction. There are many constraints, but three of the most important ones are: commercialism, story formulas, and sources.

Newspapers, radio, and TV stations are businesses, all of which are rivals for audiences and advertising revenue. The amount of time that the average TV station spends on news broadcasts has grown steadily over the last fifty years - largely because news is relatively cheap to produce, yet sells plenty of advertising. Some news broadcasts are themselves becoming advertisements. For example, during one week in 1996 when the American CBS network was airing a movie about the

sinking of the Titanic, CBS news ran nine stories about that event (which had happened 84 years before). The ABC network is owned by Disney Studios, and frequently runs news stories about Mickey Mouse. Furthermore, the profit motive drives news organizations to pay more attention to stories likely to generate a large audience, and to shy away from stories that may be important but dull. This pressure to be entertaining has produced shorter, simpler stories: more focus on celebrities than people of substance, more focus on gossip than on news, and more focus on dramatic events than on nuanced issues.

As busy people under relentless pressure to produce, journalists cannot spend days agonizing over the best way to present stories. Instead, they depend upon certain story formulas, which they can reuse again and again. One example is known as the inverted pyramid. In this formula, the journalist puts the most important information at the beginning of the story, than adds the next most important, and so on. The inverted pyramid originates from the age of the telegraph, the idea being that if the line went dead halfway through the story, the journalist would know that the most crucial information had at least been relayed. Modern journalists still value the formula for a similar reason. Their editors will cut stories if they are too long. Another formula involves reducing a complicated story into a simple conflict. The best example is "horse race" election coverage. Thorough explication of the issues and the candidates' views is forbiddingly complex. Journalists therefore concentrate more on who is winning in the opinion polls, and whether the underdog can catch up in the numbers than on politicians' campaign goals.

Sources are another constraint on what journalists cover and how they cover it. The dominant sources for news are public information officers in businesses and government offices. The majority of such officers try to establish themselves as experts who are qualified to feed information to journalists. How do journalists know who is an expert? In general, they don't. They use sources not on the basis of actual expertise, but on the appearance of expertise and the willingness to share it. All the major news organizations use some of the same sources (many of them anonymous), so the same types of stories always receive attention. Over time, the journalists may even become close friends with their sources, and they stop searching for alternative points of view. The result tends to be narrow, homogenized coverage of the same kind.

Question 226: It can be inferred from paragraph 1 that the author of the passage thinks____ _.

  1. that watching or reading the news is extremely boring
  2. that most news stories are false
  3. that most people don't realize how different news is from reality
  4. that most people don't pay enough attention to the news

Question 227: According to paragraph 2, which of the following is true?

  1. One effect of commercialism is news stories with more complex content.
  2. The ABC network owns Disney Studios.
  3. Some news broadcasts are shown without advertisements.
  4. More time is devoted to news on TV now than 50 years ago.

Question 228: Why does the author mention Mickey Mouse in paragraph 2?

  1. To indicate that ABC shows entertaining news stories
  2. To give an example of news stories that are also advertisements
  3. To contrast ABC's style with that of CBS
  4. To give an example of news content that is not serious

Question 229: According to paragraph 3, an advantage of the inverted pyramid formula for journalists is that ___ _.

  1. if a story is cut by the editor, only the less crucial information will be lost
  2. it makes a story more likely to be cut by the editor
  3. it makes a story more likely to attract the attention of the audience
  4. it makes a story simpler and easier to understand

Question 230: The word relayed in paragraph 3 is closest in meaning to ______.

A. chosen B. Known C. gathered D. sent

Question 231: According to the passage, which of the following tends to lead to homogenized coverage?

  1. Journalists' use of experts as sources
  2. Journalists' becoming friends with their sources
  3. Journalists' search for alternative points of view
  4. Journalists' using government officials as sources

Question 232: The word "them" in paragraph 4 refers to _.

A. journalists B. organizations C. experts D. sources Question 233: Which of the following best expresses the essential information in the highlighted sentences "Thorough explication of the issues......than on politicians' campaign goals. " in the passage?

  1. Journalists focus on poll numbers instead of campaign issues because it is easier.
  2. journalists are more interested in issues and candidates' views, but viewers are more interested in who is winning.
  3. During an election campaign, journalists mainly concentrate on "horse race" coverage.
  4. Candidates' views and how they are explained by journalists can have a big effect on poll numbers.

ĐỀ THI THỬ THPTQG 2019 – CÔ TRANG ANH – ĐỀ 16– TẬP 1

Exercise 32:Read the following passage and mark the letter A, B, C, or D on your answer sheet to indicate the correct answer to each of the questions.

The Internet is very much like television, in which it takes time away from other pursuits, provides entertainment and information, but in no way can compare with the warm, personal experience of reading a good book. This is not the only reason why the Internet will never replace books, for books provide the in-depth knowledge of a subject that sitting in front of a computer monitor cannot provide. We can download text from an Internet source, but the aesthetic quality of sheets of downloaded text leave much to be desired. A well-designed book enhances the reading experience.

The book is still the most compact and inexpensive means of conveying a dense amount of knowledge in a convenient package. The easy portability of the book is what makes it the most user- friendly format for knowledge ever invented. The idea that one can carry in one's pocket a play by Shakespeare, a novel by Charles Dickens or Tom Clancy, Plato's Dialogues, or the Bible in a small paperback edition is mind-boggling. We take such uncommon convenience for granted, not realizing that the book itself has undergone quite an evolution since the production of the Gutenberg Bible in 1455 and Shakespeare's First Folio in 1623, just three years after the Pilgrims landed at Plymouth to colonize the New World.

Not only has the art and craft of printing and book manufacturing been greatly improved over the centuries, but the great variety of subject matter now available in books is astounding, to say the least. In fact, the Internet requires the constant input of authors and their books to provide it with the information that makes it a useful tool for exploration and learning.

Another important reason why the Internet will never replace books is because those who wish to become writers want to see their works permanently published as books - something you can hold, see, feel, skim through, and read at one's leisure without the need for an electric current apart from a lamp. The writer may use a word processor instead of a typewriter or a pen and pad, but the finished product must eventually end up as a book if it is to have value to the reading public. The writer may use the Internet in the course of researching a subject just as he may use a library for that purpose, but the end product will still be a book.

Question 234: What is the author's main idea in the passage?

  1. The Internet distracts people from other pursuits.
  2. Books have been improved in both appearance and content.
  3. The Internet and books will replace each other.
  4. Books will not be replaced by the Internet.

Queston 235: What does the word "this" in the first paragraph refer to?

  1. a good book
  2. the Internet
  3. the warm, personal experience of reading a good book
  4. entertainment and information

Queston 236: The word “aesthete” is closest in meaning to

A. artistic B. usual C. print D. invisible

Question237: Which of the following is mentioned as the advantage of books in paragraph 2?

  1. expensive, moderate and portable
  2. compact, cheap and convenient
  3. luxurious, enormous and flexible
  4. uncommon, inexpensive and knowledgeable

Question 238: The word "astounding" in paragraph 3 could be best replaced by____ _.

A. astonishing B. unsurprising C. astringent D. accelerating

Question 239: The author mentioned the Internet in the last paragraph as a tool that

  1. will replace books if the writers don't want to publish their works.
  2. requires more equipment to use than books.
  3. provides the writers information about the subject they are finding for their books.
  4. cannot be held, seen or felt in our hands.

Question 240: According to the passage, which sentence is NOT true about books?

  1. Whenever books are still useful for our society, they cannot be replaced.
  2. A well-designed book is more effective for reading than a download text.
  3. There has been no book evolution because of its uncommon convenience.
  4. Over many centuries, the appearance of books has been upgraded.

ĐỀ THI THỬ THPTQG 2019 – CÔ TRANG ANH – ĐỀ 16– TẬP 1

Exercise 33: Read the following passage and mark the letter A, B, C, or D on your answer sheet to indicate the correct answer to each of the questions.

WWF, in full World Wide Fund for Nature, international organization committed to conservation of the environment. In North America, it is called the World Wildlife Fund.

In 1960, a group of British naturalists - most notably biologist Sir Julian Huxley, artist and conservationist Peter Scott, and ornithologists Guy Montfort and Max Nicholson - led an effort to establish an organization that protected endangered species and their habitats. The following year the World Wildlife Fund was founded; the international name was subsequently changed to World Wide Fund for Nature in 1989, although in the United States and Canada it retained the founding name. The

organization's distinctive panda logo was created by Scott. In the face of growing environmental threats over the ensuing years, the WWF's activities expanded in scope. Today its mission statement is threefold: to conserve the world's biological diversity, to ensure that the use of renewable natural resources is sustainable, and to promote the reduction of pollution and of wasteful consumption. The organization has long included both conservationists and businesspeople with the intention of combining solid scientific data with well-managed action. It also seeks cooperation between nongovernmental organizations, local governments, and local populations. The WWF works closely with the World Conservation Union and has formed partnerships with the United Nations, the World Bank, and the European Union.

The WWF provides money for conservation initiatives around the world. These include programs focused on individual species, forests, and freshwater and marine issues as well as climate change and responsible international trade. The group has also been involved in efforts to provide a safe and sustainable habitat for the world's peoples, both urban and rural, including clean water, clean air, healthful food, and rewarding recreation areas. Among the WWF's notable achievements is its use of debt-for-nature swaps, in which an organization buys some of a country's foreign debt at a discount, converts the money to local currency, and then uses it to finance conservation efforts. The WWF's first successful debt-for-nature swap took place in 1987 in Ecuador.

At the beginning of the 21st century, the WWF was active in more than 100 countries and had more than five million supporters. The organization's international headquarters are in Gland, Switz., and it has more than 90 offices around the world.

Question 241: What does the passage mainly discuss?

(Source: https://www.britannicaconlitopic/WWF)

A. The history of WWF B. What WWF has done to conserve the nature

C. ThefoundationofVVWF D. The missions of WWF

Question 242: When was VVWF established?

A. In 1960 B. In 1961 C. In 1989 D. In 1990

Question243: The word "ensuing" in the first paragraph is closest in meaning to .

A. bygone B. present C. following D. current

Question 244: According to the passage, what contemporary mission of WWF is NOT stated?

  1. to protect the biological variety of the world.
  2. to reduce pollution and prodigal consumption.
  3. to make sure that renewable natural resources are used sustainably.
  4. to protect endangered species and their habitats.

Question 245: The word "It" in paragraph 1 refers to

A. The organization B. The attention

C. Solidscientific data D. Well-managed action

Question 246: It can be inferred from the passage that .

  1. WWF was founded by a group of American biologists.
  2. WWF has cooperated with many organizations to change its name.
  3. WWF has made an effort to provide people with a sustainable world.
  4. WWF uses debt-for-nature swap to lend money to foreign countries.

Question 247: The phrase "focused on" in paragraph 2 could best be replaced by ________.

A. concentrated on B. depended on C. relied on D. based on

Question 248: Which sentence is CORRECT about WWF?

  1. Its panda logo was created by 2 men, an artist and a conservationist
  2. It has millions of supporters in about a hundred countries,
  3. Its headquarters are in Sweden.
  4. Its programs covered the issues of terrestrial and marine areas.

ĐỀ THI THỬ THPTQG 2019 – CÔ TRANG ANH – ĐỀ 17– TẬP 1

Exercise 34:Read the following passage and mark the letter A, B, C, or D on your answer sheet to indicate the correct answer to each of the questions.

In the 1960s, The Beatles were probably the most famous pop group in the whole world.

Since then, there have been a great many groups that have achieved enormous fame, so it is perhaps difficult now to imagine how sensational The Beatles were at that time. They were four boys from the north of England and none of them had any training in music, They started by performing and recording songs by black Americans and they had some success with these songs, Then they started writing their own songs and that was when they became really popular. The Beatles changed pop music. They were the first pop group to achieve great success from songs they had written themselves. After that it became common for groups and singers to write their own songs. The Beatles did not have a long career. Their first hit record was in 1963 and they split up in 1970. They stopped doing live performances in 1966 because it had become too dangerous for them - their fans were so excited that they surrounded them and tried to take their clothes as souvenirs! However, today some of their songs remain as famous as they were when they first came out Throughout the world many people can sing part of a Beatles song if you ask them.

(https://goo.g.1/NsFNPQ)

Question 249: The passage is mainly about _ _.

  1. How the Beatles became more successful than other groups
  2. Why the Beatles split up after 7 years
  3. The Beatles's fame and success
  4. Many people's ability to sing a Beatles song

Question 250: The four boys of the Beatles .

    1. Came from the same family
    2. Came from a town in the north of England
    3. Were at the same age
    4. Received good training in music

Question 251: The word "sensational" is closest in meaning to

A. Notorious B. Bad C. Shocking D.Popular Question 252: What is not true about the Beatles?

  1. The members had no training in music
  2. They had a long stable career
  3. They became famous when they wrote their own songs
  4. They were afraid of being hurt by fans

Question 253: The Beatles stopped their live performances because

  1. They had earned enough money
  2. They did not want to work with each other
  3. They spent more time writing their own songs
  4. They were afraid of being hurt by fans.

Question 254: The word "they" is closest in meaning to

A. The Beatles B. the fans C. the songs D. the performances

Question 255: What word might describes the attitudes of the writer?

A. admiration B. criticism C. neutral D. sarcasm

ĐỀ THI THỬ THPTQG 2019 – CÔ TRANG ANH – ĐỀ 17– TẬP 1

Exercise 35: Read the following passage and mark the letter A, B, C, or D on your answer sheet to indicate

the correct answer to each of the questions.

It is an undeniable fact that a woman's place was once in the home. In the past, women were merely required to fulfill the role of mother and housewife. Today, this situation has changed tremendously. If a woman possesses the attributes and qualities of her male counterpart, she will definitely be given equal opportunities in the career world without much bias. Therefore, women began to make the scene and, eventually, there was a steady flow of women leaving their homes. A certain vacuum or emptiness was thus created in the households. No more could the husbands turn

to their wives after a hard day's work. Society marveled at the ability of women, but it also suffered at the realization of the important role that women play in their homes. Should women be allowed to work after marriage then? The answer is undoubtedly positive although this issue is highly debatable in terms of the nature of the professions involved. If a woman pursues her career but is at the same time able to care for her home and children, one simply cannot find any reason why she should not be allowed to do so.

A woman's influence is greatly needed in the home, on the children. What a child needs most is his mother's care because how the child is molded depends greatly on her. It is a real pity that women who leave their homes solely in search of a career seldom give a thought to this. The children, being helpless and dependent creatures, may have nobody to turn to at home, except servants or relatives. With the mother's back only after a hard day's toil, the children surely do not get much attention.

Whether a woman should continue to work after marriage would depend on the nature of her profession. It is a waste of resources if women, after seeking higher education, immediately abandon their careers after matrimony. A woman's effort can also contribute to the well-being and development of society. In the Malaysian context, a teacher is only required to work a five-day week with term holidays every now and then. Moreover, she is in school for only half of the day. The other half of the day can be devoted to her home. A teacher, besides educating the society, can fulfill the role of both mother and housewife. There are many other careers like those of nurses, clerks and typists where women can fulfill the double role.

Nevertheless, there are many professions which would not be advisable for women to indulge in after marriage. A public relations officer spends almost three-quarters of her time in her career. She has heavy tasks to shoulder which might require her to entertain others till odd hours at night. Women who venture into the business world should think twice before plunging into it. It would be beyond their means to fulfill two demanding roles at the same time.

As it is, a woman's most important responsibility still lies in her home. Without her around in the house, one just cannot bear to think of the consequences. Unless and until she can fulfill the basic role of a housewife and mother, she should not make a career her sole responsibility.

Question 256: What is the best title of the passage?

  1. Should women be allowed to work after marriage?
  2. The role of women in society.
  3. What jobs are suitable for women after marriage?
  4. The necessary characteristics for women to work after marriage.

Question 257: Which of the following about women is NOT true in paragraph 1?

  1. The main role of women in bygone days was childbearing and homemaking.
  2. Women will have equality of job opportunities if they are as qualified as men.
  3. Women's going out to work didn't change anything in the life of their families.
  4. Women play an important role in their homes.

Question 258: The word "mearvelled" in paragraph 1 could be best replaced by _____ _.

A. was amazed B. was excited

C. was fascinated D. was frustrated

Question 259: The word “this” in paragraph 2 refers to .

  1. the child of the woman
  2. the woman’s influence at her home
  3. the mother’s care which is the most necessary for a child
  4. the career that the woman wants to do

Question 260: The word "matrimony" in paragraph 3 is closest in meaning to _____.

A. divorce B. remarriage C. celibate D. marriage Question 261: According to the passage, what job is NOT suitable for women after getting married?

A. nurses B. clerks

C. typists D. public relations officers

Question 262: Why should a businesswoman have the second thought before starting her career?

  1. Because she has heavy tasks to shoulder till very late at night.
  2. As it would be hard for her to fulfill the double role at the same time.
  3. Since she has to spend three quarters of her time on business.
  4. Because she has to plunge into this job.

Question 263: Which can be inferred from the last paragraph?

  1. Women should stay at home to fulfill their responsibilities.
  2. Women should think of the consequences in their houses.
  3. Women should not have their own careers.
  4. Women should fulfill their primary roles at home before making their own careers.

ĐỀ THI THỬ THPTQG 2019 – CÔ TRANG ANH – ĐỀ 18– TẬP 1

Exercise 36:Read the following passage and mark the letter A, B, C, or D on your answer sheet to indicate the correct answer to each of the questions

Jupiter is the largest and most massive planet and is fifth in order of distance from the sun. It is well placed for observation for several months in every year and on average is the brightest of the planets apart from Venus, though for relatively brief periods Mars may outshine it. Jupiter's

less than 10 hour rotation period gives it the shortest day in the solar system in so far as the principal planets are concerned. There are no true seasons on Jupiter because the axial inclination to the perpendicular of the orbital plane is only just over 3°-less than that for any other planet.

The most famous mark on Jupiter is the Great Red Spot. It has shown variations in both intensity and color, and at times it has been invisible, but it always returns after a few years. At its greatest extent it may be 40,000 kilometers long and 14,000 kilometers wide, so its surface area is greater than that of Earth. Though the latitude of the Red Spot varies little, it drifts about in longitude. Over the past century the total longitudinal drift has amounted to approximately 1200°. The latitude is generally very close to -22°. It was once thought that the Red Spot might be a solid or semisolid body floating in Jupiter's outer gas. However, the Pioneer and Voyager results have refuted that idea and proven the Red Spot to be a phenomenon of Jovian meteorology. Its longevity may well due to its exceptional size, but there are signs that it is decreasing in size, and it may not be permanent. Several smaller red spots have been seen occasionally but have not lasted. Question 264: It can be inferred from the passage that .

  1. a day on Earth is shorter than a day on Jupiter
  2. there are other structures on Jupiter that has the same size as the Great Red Spot
  3. there are times when Great Red Spot cannot be observed from the earth
  4. the Great Red Spot is the only structure on Jupiter

Question 265: According to the passage, which planet typically shines the most brightly?

A. Earth B. Jupiter C. Venus D. Mars

Question 266: According to the passage, the Great Red Spot .

A. has different colors B. is as big as the earth

C. is a solid structure floating in the air D. has increased its size over the years

Question 267: The word "it" in paragraph 2 refers to _.

A. Jupiter B. The Great Red Spot C. intensity D. color

Question 268: The word "exceptional" in paragraph 2 mostly means _.

A. extreme B. sustainable C. temporary D. infrequent

Question 269: According to the passage, which of the following is NOT true?

  1. Jupiter is bigger than all the other planest in the solar system.
  2. A day in Jupiter is nearly 10 hours long.
  3. The Red Great Spot moves more vertically than horizontally.
  4. Scientists have proof showing that smaller red spots are increasing their size to become other Great Red Spots.

Question 270: The passage was probably taken from _____ _.

A. an art journal B. a geology magazine

C. a high school textbook D. an archaeology book

ĐỀ THI THỬ THPTQG 2019 – CÔ TRANG ANH – ĐỀ 18– TẬP 1

Exercise 37: Read the following passage and mark the letter A, B, C, or D on your answer sheet to indicate the correct answer to each of the questions.

A generation gap in the workplace can make workers both young and old feel inferior, as well as hamper productivity and teamwork. Differences between generations can be seen in work ethics, habits and communication styles. Younger workers might fear not being taken seriously by their older colleagues, while older workers might fear that their experience is not valued but replaced by workers with knowledge of more current technology. However, members of each generation can close the gap between them if they're willing to meet one another halfway.

Older workers can show respect to the younger set by asking for their opinions and recognizing their contributions to the workplace as valid, or complimenting them on a job well done. Younger workers can show their elders respect by asking for advice on how to manage a situation with work, based on the older worker's many years of experience. It's important for both entry- and senior-level workers to see each other as equals, regardless of the type of position in which they work. No one wants to feel inferior or irrelevant just because of their age. Rather, a generation gap at work can be a learning opportunity.

Workers can also put themselves in their colleagues' shoes to determine what might be bothering them about their generational age difference. If a person is much older than another, perhaps it is bitterness about fewer job opportunities, or fear that a younger worker might seem more relevant and edge him out of his job. If workers open their minds to understand where co-workers are coming from, it can help ease any tension between them and appreciate each other's

work contributions.

If age seems to be a problem for someone at the workplace, it can be helpful to do the very opposite of what a co-worker might expect from someone of a different age set due to stereotypes. For example, if a worker is considerably younger such as right out of college, she can share researched information to indicate that she knows what she's doing, or show curiosity instead of upset to indicate emotional maturity if the person makes a disparaging remark about her youth. Older workers can maintain an enthusiastic attitude about work instead of showing boredom or bitterness from past experiences.

Workers can, moreover, directly address the concern of age differences at work with the colleague at odds with them by asking the person for constructive advice on how to handle the issue. For example,

older workers who are unfamiliar with new software that younger colleagues understand might acknowledge to them that they did the same tasks differently in years past but show interest in learning the program to keep up with modern technology. Learning to speak their technological language can make them feel more connected. Likewise, a younger worker can admit to being green on the work scene, but eager to gain experience by learning from senior colleagues.

Question 271: What is the purpose of writer in the passage?

  1. To describe the status of generation gap in the workplace.
  2. To suggest the solutions to bridge the generation gap between the older and younger workers.
  3. To determine which generation will have more influence on the workforce.
  4. To show the differences between the older and younger at their work.

Question 272: Each generation should respect the other generation at work to .

  1. affirm their ability or personal experience they contribute to work.
  2. express the recognition to the other’s position in the workplace.
  3. make the other feel that they are necessary or useful in the workplace.
  4. treat the other equally at work.

Question 273: What does the writer mean by stating “put themselves in their colleagues' shoes” in paragraph 3?

  1. Workers should try on their colleagues' shoes to master the difference of their generation.
  2. Workers should buy their colleagues' shoes not to bother them about generational age difference.
  3. Workers should determine the differences between their generational ages so that they can understand their colleagues’ situation.
  4. Workers should imagine that they are in their colleagues’ situation to understand and sympathy the difficulties they meet.

Question 274: The word "him" in paragraph 3 refers to .

A. an older worker B. a younger worker C. a colleague D. a person

Question 275: What is the synonym of the word “stereotypes” in paragraph 4?

A. achievements B. failures C. prejudices D. jealousness Question 276: According to the passage, the following are measures to overcome the generation gap, EXCEPT .

A. Keeping an open mind B. Doing the opposite

C. Requesting feedback D. Appreciating the opportunity

Question 277: The word “acknowledge” is closest in meaning to .

A. recognize B. enhance C. acquire D. distinguish

Question 278: It can be inferred from the passage that .

  1. The younger and older workers should meet one another on the way to reduce generation gap in the workplace.
  2. The main principle to bridge the generation gap is to balance the experience and technology.
  3. Positive attitude will help to decrease the conflicts between the older and younger.
  4. The competitive environment in the workplace might make the generation gap wider.

ĐỀ THI THỬ THPTQG 2019 – CÔ TRANG ANH – ĐỀ 19– TẬP 1

Exercise 38:Read the following passage and mark the letter A, B, C, or D on your answer sheet to indicate the correct answer to each of the questions.

One of the seven wonders of the ancient world, the Great Pyramid of Giza was a monument of

wisdom and prophecy built as a tomb for Pharaoh Cheops in 2720 B.C. Despite its antiquity, certain aspects of its construction makes it one of the truly wonders of the world. The thirteen- acre structure near the Nile river is a solid mass of stone blocks covered with limestone. Inside are the number of hidden passageways and the burial chamber of the Pharaoh. It is the largest single structure in the world. The four sides of the pyramid are aligned almost exactly on true north, south, east and west-an incredible engineering feat. The ancient Egyptians were sun worshippers and great astronomers, so computations for the Great Pyramid were based on astronomical observations.

Explorations and detailed examinations of the base of the structure reveal many intersecting lines. Further scientific study indicates that these represent a type of timeline of events - past, present and future. Many of the events have been interpreted and found to coincide with known facts of the past. Others are prophesied for future generations and are currently under investigation. Many believe that pyramids have supernatural powers and this one is no exception. Some researchers even associate it with extraterrestrial beings of ancient past.

Was this superstructure made by ordinary beings, or one built by a race far superior to any known today?

Question 279: What is the best title for the passage?

  1. Problems with the Construction of the Great Pyramid.
  2. Exploration of the Burial Chamber of Cheops.
  3. Symbolism of the Great Pyramid.
  4. Wonders of the Great Pyramid of Giza.

Question 280: Why is the Great Pyramid of Giza considered one of the seven wonders of the world?

  1. It was built by a super race.
  2. It is perfectly aligned with the four cardinal points of the compass and contains many prophecies.
  3. It was selected of the tomb of Pharaoh Cheops.
  4. It was very old.

Question 281: What was the most probable reason for providing so many hidden passages?

  1. To allow the weight of the pyramid to settle evenly.
  2. To permit the high priests to pray at night.
  3. To keep grave robbers from finding the tomb and the treasure buried with the Pharaoh.
  4. To enable the Pharaoh's family to bring food for his journey to the afterlife

Question 282: The word 'feat’ in the first paragraph is closet in meaning to .

A. accomplishment B. festivity C. appendage D. structure

Question 283: On what did the ancient Egyptians base their calculations?

A. Advanced tools of measurement B. Knowledge of the earth’s surface

C. Advanced technology D. Observation of the celestial bodies

Question284: What has research of the base revealed?

  1. There are cracks in the foundation
  2. Tomb robbers have stolen the Pharaoh's body
  3. A superior race of people built in
  4. The lines represent important events

Question 285: In the second passage, the word ‘prophesied’ is closest in meaning to .

A. foretold B. terminated C. precipitated D. affiliated

ĐỀ THI THỬ THPTQG 2019 – CÔ TRANG ANH – ĐỀ 19– TẬP 1

Exercise 39: Read the following passage and mark the letter A, B, C, or D on your answer sheet to indicate the correct answer to each of the questions.

WATER SPORTS

Recreational diving or sport diving is a type of diving that uses scuba equipment for the purpose of leisure and enjoyment. In some diving circles, the term "recreational diving" is used in contradistinction to "technical diving", a more demanding aspect of the sport which requires greater levels of training, experience and equipment.

Recreational scuba diving grew out of related activities such as snorkeling and underwater hunting. For a long time, recreational underwater excursions were limited by the amount of breath that could be held. However, the invention of the aqualung in 1943 by Jacques-Yves Cousteau and its development over subsequent years led to a revolution in recreational diving. However, for much of the 1950s and early1960s, recreational scuba diving was a sport limited to those who were able to afford or make their own kit, and prepared to undergo intensive training to

use it. As the sport became more popular, manufacturers became aware of the potential market, and equipment began to appear that was easy to use, affordable and reliable. Continued advances in' SCUBA technology, such as buoyancy compensators, modern diving regulators, wet or dry suits, and dive computers, increased the safety, comfort and convenience of the gear encouraging more people to train and use it.

Until the early 1950s, navies and other organizations performing professional diving were the only providers of diver training, but only for their own personnel and only using their own types of equipment. There were no training courses available to civilians who bought the first scuba equipment. Professional instruction started in 1959 when the non-profit National Association of Underwater Instructors was formed.

Further developments in technology have reduced the cost of training and diving. Scuba- diving has become a popular leisure activity, and many diving locations have some form of dive shop presence that can offer air fills, equipment and training. In tropical and sub-tropical parts of the world, there is a large market in holiday divers, who train and dive while on holiday, but rarely dive close to home. Generally, recreational diving depths are limited to a maximum of between 30 and 40 meters (100 and 130 feet), beyond which a variety of safety issues make it unsafe to dive using recreation diving equipment and practices, and specialized training and equipment for technical diving are needed.

Question 286: What is the topic of the passage?

  1. The reasons why people like scuba-diving
  2. The history of scuba-diving
  3. The equipment for scuba-diving
  4. The reasons why not many people take up scuba-diving

Question 287: Recreational diving ____ _.

  1. requires more equipment than technical diving ,
  2. are taken up by many people for leisure and entertainment
  3. needs no equipment
  4. requires more experience than technical diving

(Source: https://goaglinbYTDC)

Question 288. Recreational underwater excursions used to be limited _____ _.

  1. as underwater hunting was banned
  2. because the necessary amount of breath was too expensive to afford
  3. because divers could not take enough amount of breath with them
  4. because the necessary amount of breath was too heavy to bring

Question 289: Which of the following could best replace the word "subsequent" ?

A. successive B. following C. ancient D. succeeding Question 290. According to the second paragraph, in the 1950s and early 1960s, recreational scuba diving was a sport limited because _____ _.

  1. divers did not like to take part in any intensive training courses
  2. there were not any intensive training courses for divers
  3. there were not enough kit for many divers
  4. kit and intensive training were too expensive for many people to afford

Question 291. These following sentences are true EXCEPT _.

  1. In the early 1950s anyone who wanted to dive could be professionally trained.
  2. In the early 1950s there were no training courses available to civilians who bought the first scuba equipment.
  3. As recreational diving became more popular, manufacturers have made more and more diving equipment.
  4. Advances in scuba technology encourage more and more people to train and use it.

Question 292. Holiday divers _______ _.

  1. do not like to dive in tropical and sub-tropical parts
  2. can dive as deep as they like because of safety
  3. are those who go away from home to dive
  4. are limited in tropical and sub-tropical parts

ĐỀ THI THỬ THPTQG 2019 – CÔ TRANG ANH – ĐỀ 20– TẬP 1

Exercise 40:Read the following passage and mark the letter A, B, C, or D on your answer sheet to indicate the correct answer to each of the questions

Pollution emitted in industrial areas represents a threat to human health and the surrounding natural resources. We have a tendency to believe that the production processes are the only source of environmental damage, and often forget about the possible long-term effects of harmful production practices. We may think that the closure of these huge industrial areas would improve the quality of the environment. Unfortunately, this ignores the threat of the remaining waste, which is abandoned and poorly stored. It represents an even bigger danger because it stands neglected as it degrades and leaks into the earth without any control at all.

Changes in the water chemistry due to surface water contamination can affect all levels of an ecosystem. It can affect the health of lower food chain organisms and, consequently, the availability of food up through the food chain. It can damage the health of wetlands and damage their ability to support healthy ecosystems, control flooding, and filter pollutants from storm

water runoff. The health of animals and humans are affected when they drink or bathe in contaminated water. In addition water-based organisms, like fish and shellfish, can pile up and concentrate contaminants in their bodies. When other animals or humans eat these organisms, they receive a much higher dose of contaminant than they would have if they had been directly exposed to the original contamination.

Contaminated groundwater can badly affect animals, plants and humans if it is removed from the ground by manmade or natural processes. Depending on the study of rocks of the area, groundwater may rise to the surface through springs or seeps, flow sideways into nearby rivers, streams, or ponds, or sink deeper into the earth. In many parts of fhe world, groundwater is pumped out of the ground to be used for drinking, bathing, other household uses, agriculture, and industry.

Contaminants in the soil can harm plants when they take up the contamination through their roots. Eating, breathing in, or touching contaminated soil, as well as eating plants or animals that have piled up soil contaminants can badly affect the health of humans and animals.

Air pollution can cause breathing-related problems and other bad health effects as contaminants are absorbed from the lungs into other parts of the hody. Certain air contaminants can also harm animals and humans when they contact the skin. Plants rely on breathing for their growth and can also be affected by exposure to contaminants moved in the air.

(Source: Adapted from http://www.grid.unep.ch/waste/clownload/waste1213.pdj)

Question 293: What is the topic of the passage?

A. Sources of environmental damage B. The pollution from the city

C. Bad effects of industrial waste D. The quality of the environment

Question 294: According to the passage, the industry is likely to be thought as ____.

A. a danger to the environment B. the only source of pollution

C. the utmost harmful activity D. a threat to human health

Question 295: The word "it" in the first paragraph refers to____ _.

A. the remaining waste B. a danger

C. the environment D. the threat of the remaining waste

Question296: Which of the followings affect an ecosystem as the whole?

A. Surface water contamination B. Soil contamination

C. Groundwater contamination D. Air contamination

Question 297: According to the passage, which of the followings supports healthy ecosystems?

A. Lower food chain organisms B. Animals C. Water-based organisms D. Wetlands

Question 298: Which of the followings is NOT badly affected by contaminated groundwater?

A. Human B. Plants C. Rocks D.Animals Question 299: Which of the followings is the flow of water from the ground to the surface?

A. Streams B. Ponds C. Rivers D. Springs Question 300: Whichof the followings has the closest meaning to the word "absorbed" in the last paragraph?

A. Consumed B. Taken in C. Swallowed D. Piled up

ĐỀ THI THỬ THPTQG 2019 – CÔ TRANG ANH – ĐỀ 20– TẬP 1

ĐÁP ÁN BÀI TẬP ĐỌC HIỂU

Câu 1: Đáp án B

CHỦ ĐỀ FRIENDSHIP

Ý chính của đoạn văn này là gì?

  1. Phương tiện truyền thông mạng xã hội ảnh hưởng quá nhiều đến tình bạn.
  2. Giới trẻ đang kết bạn và giữ tình bạn theo một cách rất đáng ngạc nhiên.
  3. Sự khác nhau về cách kết bạn giữa nam và nữ.
  4. Phương tiện truyền thông mạng xã hội kết nối tình bạn.

Căn cứ vào thông tin đoạn đầu tiên:

A new survey

from the Pew Research Center reveals the surprising

ways that technology

intersects with teen friendships – and the results show that 57 percent of teens have made at least one new friend online. Even more surprisingly, only 20 percent of those digital friends ever meet in person.

(Một cuộc khảo sát mới do trung tâm nghiên cứu Pew thực hiện cho thấy những cách đáng ngạc nhiên mà công nghệ can thiệp vào tình bạn và kết quả chỉ ra rằng 57% thanh thiếu niên đã kết bạn với ít nhất một người bạn trực tuyến. Thậm chí đáng ngạc nhiên hơn, chỉ có 20% số người bạn trực tuyến đó đã từng gặp mặt ngoài đời)

Câu 2: Đáp án D

Từ “digital” trong đoạn 1 gần nhất với từ .

A. vật tương tự B. sự giỏi toán C. nhiều D. trực tuyến

Xét nghĩa trong bài: digital friends (những người bạn kĩ thuật số) = online friends (những người bạn trực tuyến

Câu 3: Đáp án A

Theo bài đọc, tỉ lệ thanh thiếu niên thực sự dành thời gian cho bạn bè mình là bao nhiêu? A. 25% B. 55% C. 27% D. 23%

Căn cứ thông tin đoạn 2:

While teens do connet with their friends face-to-face outside of school, they spend 55 percent of their day texting with friends, and only 25 percent of teens are spending actual time with their friends on a daily basis (outside of school hallways). (Mặc dù giới trẻ thực sự có kết nối trực tuyến với bạn bè của họ bên ngoài trường học, nhưng họ dành 55% thời gian trong ngày để nhắn tin với bạn bè và chỉ có 25% thanh thiếu niên thực sự dành thời gian cho bạn bè mình hàng ngày (ngoài hành lang trường học)).

Câu 4: Đáp án C

Những câu sau đây là đúng, ngoại trừ .

  1. Theo khảo sát, hơn một nửa thanh thiếu niên đã từng kết bạn trực tuyến.
  2. Giới trẻ chỉ gặp mặt 1/5 người bạn trực tuyến mà họ đã làm quen.
  3. Hầu hết thanh thiếu niên sử dụng video chat để duy trì tình bạn.
  4. Những loại hình giao tiếp mới đóng một vai trò quan trọng trong việc giữ gìn tình bạn.

Căn cứ thông tin đoạn 2:

(…) and 7 percent even video chat daily. (và 7% giới trẻ sử dụng video chat hằng ngày)

  • Câu A đúng vì căn cứ

thông tin: the results show that 57 percent of teens have made at least one new

friend online. (kết quả chỉ ra rằng 57% thanh thiếu niên đã kết bạn với ít nhất một người bạn trực tuyến)

  • Câu B đúng vì căn cứ thông tin: only 20 percent of those digital friends ever meet in person. (chỉ có 20% số người bạn trực tuyến đó đã từng gặp mặt ngoài đời)
  • Câu D đúng vì căn cứ thông tin: These new forms of communication are key in maintaining friendships day-to-day (những loạihình giao tiếp mới này là chìa khóa để duy trì tình bạn hằng ngày)

Câu 5: Đáp án B

Từ “they” trong đoạn 3 đề cập đến .

  1. những người bạn
  2. những game thủ trực tuyến
  3. thành viên cùng đội của họ D. chỉ những người bạn trực tuyến Từ “they” thay thế cho cụm từ “những game thủ trực tuyến” trong câu phía trước.

Whether they’re close with their teammates or not, online gamers say that playing makes them feel “more connected” to friends they know, or gamers they’ve never met. (Cho dù họ có gần gũi với đồng đội hay không, thì các game thủ trực tuyến cũng nói rằng việc chơi game khiến họ cảm thấy “kết nối nhiều hơn” với bạn bè họ biết hay những game thủ mà họ chưa từng gặp.)

Câu 6: Đáp án D

Có thể suy ra điều gì từ đoạn văn?

  1. Con trai thì dễ kết bạn mới hơn con gái.
  2. Hầu hết thanh thiếu niên không dễ dàng cho người khác tên người dùng của họ khi kết bạn mới.
  3. Đa sổ người dùng thanh thiếu niên đồng ý rằng phương tiện truyền thông mạng xã hội có ảnh hưởng tiêu cực đến cuộc sống của họ.
  4. Nhờ vào phương tiện truyền thông mạng xã hội, có hơn 2/3 giới trẻ được hỗ trợ khi họ gặp khó khăn trong cuộc sống.

Căn cứ vào thông tin đoạn 4:

In fact, 68 percent of teens received support during a challenging time in their lives via social media platforms. (5ự thật, 68% giới trẻ được nhận hỗ trợ trong thời gian khó khăn của cuộc sống thông qua các phương tiện truyền thông mạng xã hội.)

Câu 7: Đáp án A

Từ đồng nghĩa của "breakup" trong đoạn cuối là gì?

A. sự đổ vỡ, chấm dứt B. sự chia ly

C. sự bắt đầu D. tính phổ biến

Từ đồng nghĩa: breakup (sự tan vỡ) = termination

Câu 8: Đáp án D

Tác giả có ý gì khi nói

"68% những người dùng thanh thiếu niên cho biết từng

có kinh nghiệm kịch

tính với bạn bè trên phương tiện truyền thông mạng xã hội"?

  1. Hầu hết thanh thiếu niên tham gia vào các vở kịch trên phương tiện truyền thông mạng xã hội.
  2. Hầu hết bạn bè trên

phương tiện truyền thông mạng xã hội được báo cáo trong

vở kịch, c. Hầu hết

thanh thiếu niên sử dụng mạng xã hội.

kinh nghiệm của họ trong các vở kịch với bạn bè trên phương tiện truyền thông

D. Hầu hết thanh thiếu niên từng có mâu thuẫn với bạn bè trên phương tiện truyền thông mạng xã hội.

Căn cứ vào thông tin đoạn cuối:

Just as technology has become a gateway for new friendships, or a channel

to stay connected with

current friends, it can also make a friendship breakup more public. The study reveals

that girls are more

likely to block or

unfriend former allies, and 68 percent of all teenage users report

experiencing "drama

among their friends on social media. (Công nghệ cũng đã trở thành 1 cánh cổng

cho những tình bạn mới

hoặc là 1 kênh để kết nối bạn bè hiện tại, nó cũng có thể làm cho việc chia tay tình bạn trở nên công khai hơn. Nghiên cứu chỉ ra rằng các cô gái thường dễ chặn hay huỷ kết bạn với những người bạn cũ hơn và 68% những người dùng thanh thiếu niên cho biết từng trải qua xích mích với bạn bè trên phương tiện truyền thông mạng xã hộí)

Dịch bài

Bạn có cảm thấy thanh thiếu niên hiện nay đang sử dụng hầu hết thòi gian trong ngày dán mắt vào màn hình điện thoại? Bạn không quá xa xôi. Một cuộc khảo sát mói do trung tắm nghiên cứu Pew thực hiện cho thấy những điều đáng ngạc nhiên mà công nghệ can thiệp vào tình bạn và kết quả chỉ ra rằng 57% thanh thiếu niên đã kết bạn vói ít nhất 1 người bạn trực tuyến. Thậm chí đáng ngạc nhiên hơn, chỉ có 20% số người bạn trực tuyến đó đã từng gặp mặt ngoài đòi.

Mặc dù giới trẻ thực sự có kết nối trực tiếp với bạn bè của họ bên ngoài trường học, nhưng họ dành 55% thời gian trong ngày để nhắn tin với bạn bè và chỉ có 25% thanh thiếu niên thực sự dành thời gian cho bạn bè mình hàng ngày (ngoài hành lang trường học). Những loại hình giao tiếp mới này đóng một vai trò quan trọng trong việc giữ gìn tình bạn. 27% thanh thiếu niên nhắn tin qua lại cho bạn bè của họ mỗi ngày, 23% kết nối thông qua phương tiện truyền thông xã hội mỗi ngày, và 7% giới trẻ sử dụng video chat hằng ngày. Nhắn tin văn bản vẫn là hình thức giao tiếp chính - hơn một nửa số người được hỏi cho biết đó là phương thức giao tiếp được họ lựa chọn với người bạn thân nhất của họ.

Trong khi các cô gái thường hay nhắn tin với bạn bè thân thiết hơn thì các chàng trai lại thường gặp gỡ những người bạn mới (và duy trì tình bạn) trong thê giới game - 89% trong số họ chơi với bạn bè họ biết và 54% chỉ chơi với bạn bè trực tuyến. Cho dù họ có gần gũi với đông đội

hay không, các game thủ trực tuyến đêu nói rằng việc chơi game khiến họ cảm hơn” với bạn bè họ biết hoặc những game thủ mà họ chưa từng gặp.

thấy "kết nối nhiêu

Khi kết bạn mới, mạng xã hội cũng trở thành một phân quan trọng trong

bản sắc tuổi teen -

62% thanh thiếu niên

nhanh chóng chia sẻ tên người dùng của họ khi kết

nối với một người bạn

mới (mặc dù 80% vẫn xem số điện thoại của họ là phương thức liên lạc tốt nhất).

Mặc dù có những

hậu quả tiêu cực - đăng họ thấy trên

21% người dùng tuổi teen cảm thấy tồi tệ hơn về cuộc mạng xã hội — thanh thiếu niên cũng tìm thấy sự hỗ trợ

sống của họ vì các bài và kết nối thông qua

nhiều nền tảng khác

nhau. Trong thực tế, 68% thanh thiếu niên nhận được

hỗ trợ khi gặp khó

khăn trong cuộc sổng của họ thông qua các nền tảng truyền thông xã hội.

Công nghệ cũng đã trở thành một cánh cổng cho những tình bạn mới

hoặc là một kênh đê’

kết nối bạn bè hiện tại,nó cũng có thể làm cho việc chia tay tình bạn trở nên

công khai hơn. Nghiên

cứu chỉ ra rằng các cô gái thường dễ chặn hay huỷ kết bạn với những ngườbiạn cũ hơn và 68% bè

những người dùng

thanh thiếu niên cho biết từng trải qua xích mích với bạn

trên phương tiện

truyền thông mạng xã hội. Câu 9: Đáp án C

CHỦ ĐỀ FAMILY

Ý chính của bài là gì?

  1. Sự vượt trội của các gia đình hạt nhân so với các gia đình mở rộng.
  2. Sự vượt trội của các gia đình mở rộng đối với các gia đình hạt nhân.
  3. Sự khác biệt giữa các gia đình hạt nhân và các gia đình mở rộng.
  4. Sự thay đổi của các loại gia đình theo thời gian.

Thông tin:

Đoạn 1: The difference between the nuclear family and the extended family is that...

Đoạn 2: A nuclear family is limited... Đoạn 3: The extended family is...

Đoạn 4: Historically, most people in the world have lived in extended family groupings rather than in nuclear families.

Câu 10: Đáp án D

Cum từ "ithe latter" chỉ

A. đơn vị gia đình B. họ hàng

C. gia đình hạt nhân D. gia đình mở rộng Giải thích: the latter: cái thứ hai, cái được nhắc đến sau

Thông tin: The difference between the nuclear family and the extended family is that a nuclear familv refers to a sinale basic familv unit of parents and their children, whereas the extended familv refers to their relatives such as arandparents. in-laws, aunts and uncles."

Câu 11: Đáp án A

Từ "nehuỉaus" trong đoạn văn 3 gần nghĩa nhất với

  1. ambiguous /aem'bigjuas/(a): mơ hồ, không rõ ràng
  2. featured/'fì:tjbd/(a): có đường nét
  3. difficult /’difikəlt/ (a): khó khăn
  4. incomprehensive /in,kɔmpri’hensiv/ (a): chậm hiểu

“The extended family is a much more nebulous term, but in essence refers

to kin or relations not

covered by the above

definition.” (Gia đình mở rộng là một thuật ngữ mơ hồ hơn

nhiều, nhưng về bản

chất đề cập đến thân nhân hoặc quan hệ không bao gồm định nghĩa ở trên)

Câu 12: Đáp án C

Về lịch sử, những gia đình mở rộng là đơn vị cơ bản nhất cấu thành nên tổ chức xã hội ở tất cả các

nơi nêu dưới đây trừ .

A. Trung Đông B. Châu Á

C. Bắc Mỹ D. Châu Âu

Thông tin: In historical Europe and Asia as well as in Middle Eastern, African, and South American Aboriginal cultures, extended family groups were typically the most basic unit of social organization. Câu 13: Đáp án A

Từ “patriarchal” trong đoạn văn 4 gần nghĩa nhất với . Patriarchal (a): (thuộc về) gia trưởng

A. cai trị hoặc kiểm soát bởi nam giới B. bình đẳng cho cả nam và nữ

C. đơn giản mà không có quy tắc và luật lệ D. hiện đại với tiện nghi cao cấp

Câu 14: Đáp án D

Câu nào dưới đây là Đúng theo đoạn văn?

  1. Kể từ thế kỷ 20, ngày càng có nhiều cặp vợ chồng Mỹ sống trong các gia đình mở rộng vì gánh nặng tài chính.
  2. Gia đình hạt nhân là hình thức cơ bản nhất của tổ chức xã hội trên toàn thế giới.
  3. Sự phổ biến của các gia đình hạt nhân ở các nước phương Tây giúp ổn định sự sắp xếp gia đình.
  4. Các gia đình hạt nhân truyền thống đã thay đổi rất nhiều theo thời gian.

Thông tin: The rapid growth in single-parent households, for instance, also represents a substantial change in the traditional nuclear family.

Câu 15: Đáp án C

Có thể suy ra điều gì từ đoạn văn trên?

  1. Các cộng đồng thổ dân đã được loại trừ hoàn toàn trên toàn thế giới.
  2. Trong tương lai, tất cả các gia đình mở rộng sẽ được thay thế bằng các gia đình hạt nhân.
  3. Nhân chủng học là một khoa học liên quan đến nhân loại và sự phát triển của nó.
  4. Không thể định nghĩa cặp vợ chồng không có con là gia đình.

Thông tin: Anthropologically, the term “extended family” refers to such a group living together in

a household, often with

three generations living together (grandparents, parents,

and children) and

headed in patriarchal societies by the eldest man or by some other chosen leadership figure.

Dịch bài

Sự khác biệt giữa gia đình hạt nhân và gia đình mở rộng là gia đình hạt nhân chỉ một đơn vị

gia đình cơ bản gồm

cha mẹ và con cái của họ, trong khi gia đình mở rộng bao gồm cả nguwoif

thân của họ như ông

bà, bố mẹ, cô dì chú bác…Trong nhiều nền văn hóa, và

đặc biệt là các xã hội

bản địa, loại hình thứ hai là hình thức phổ biến nhất của tổ chức xã hội.

Theo Kristy Jackson thuộc Đại học Bang Colorado, gia đình hạt nhân

có giới hạn, cho một

hoặc cả hai cha mẹ (ví dụ như cha và mẹ) và một đứa con của họ, hoặc nhiều đứa, sống chung trong một ngôi nhà hoặc nhiều ngôi nhà khác. Trong nhân chủng học, họ chỉ có liên quan theo cách này; không có giới hạn trên hoặc dưới đối với số con trong một gia đình hạt nhân.

Gia đình mở rộng là một thuật ngữ mơ hồ hơn nhiều, nhưng về bản chất đề cập đến thân nhân hoặc quan hệ không được định nghĩa ở trên. Trong lịch sử châu Âu và châu Á cũng như ở các nền văn minh thổ dân ở Trung Đông, châu Phi và Nam Mỹ, các nhóm gia đình mở rộng thường là đơn vị cơ bản nhất của tổ chức xã hội. Thuật ngữ có thể khác nhau trong các bối cảnh văn hóa cụ thể, nhưng nói chung bao gồm những người liên quan đến tuổi tác hoặc huyết thống.

Về mặt nhân chủng học, thuật ngữ “gia đình mở rộng” dùng để chỉ một nhóm người sống chung trong một hộ gia đình, thường có ba thế hệ cùng sống chung với nhau (ông bà, cha mẹ và con cái) và đứng đầu trong các xã hội gia trưởng bởi người đàn ông nhiều tuổi nhất hoặc bởi người lãnh

đạo được lựa chọn. Tuy nhiên, theo cách hiểu chung, thuật ngữ “gia đình mở rộng” thường được mọi người sử dụng để chỉ họ hàng, cô dì, chú bác,…mặc dù họ không sống cùng nhau trong một nhóm.

    1. A CHỦ ĐỀ CULTURALDIVERSITY

Đoạnvănnàychủyếuthảoluậnvềchủđềgì?

A. Truyềnthôngthúvịvềviệctặnghoa.

B. Nghĩakhácnhaucủacácloàihoatrongcácnềnvănhóakhácnhau.

C. SựsosánhviệctặnghoagiữavănhóachâuÁvàchâuÂu.

D. Nhữngloạihoamọingườithườngtặngnhautrongcácvănhóakhácnhau.

Căncứvàothôngtinđoạn 1:

Flower gifting also occurs in most countries around the world. However, the meanings and traditions often vary. (Tặng hoa cũng xuất hiện ở hầu hết các nước trên thế giới. Tuy nhiên, ý nghĩa và truyền thống thường khác nhau)

    1. D Từ "They" trong đoạn 2 đề cập đến từ nào?
      1. Học sinh B. Giáo viên C.Hoa D. Hoa mẫu đơn. “They” là đại từ thay thế cho danh từ hoa mẫu đơn trong câu trước.

Peonies are by far the flower most often given in Chinna. They are also quite popularly used for weddings. (Hoa mẫu đơn là loài hoa từ lâu vẫn thường được tặng nhất ở Trung Quốc. Chúng cũng được sử dụng khá phổ biến trong các đám cưới)

C Tại sao bạn không nên tặng một chậu cây cho một người Châu Á?

      1. Tại vì người Châu Á thích được tặng hoa hơn.
      2. Bởi vì món quà này thường được tặng vào đám cưới ở Châu Á.
      3. Bởi vì món quà này được cho là biểu tượng của sự bó buộc và sự giới hạn ở Châu Á.
      4. Bởi vì sinh viên Châu Á thích tặng táo hay hoa cho người khác.

Căn cứ thông tin đoạn 2:

Strangely, potted plants are not considered a pleasant gift among Asian cultures. The people believe that like a plant confined by a pot, the gift symbolizes

a binding or restriction. (Lạ lùng là những chậu cây không được coi là một món quà dễ chịu ở các nền văn hóa ở châu Á. Người ta tin rằng nó giống như một cái cây bị giam giữ bởi cái chậu, món quà này tượng trưng cho sự bó buộc và giới hạn)

D Theo đoạn văn, những loại hoa sau thường được tặng vào ngày Quốc tế Phụ nữ ở Nga, ngoại trừ .

      1. hoa hồng đỏ B. hoa lan dạ hương

C. hoa tulip D. hoa hồng vàng

Căn cứ thông tin đoạn 3:

Russians celebrate a holiday known as Woman's Day. Traditional gifts include red roses, hyacinths or tulips. When there is a funeral or other occasion where someone wishes to express sympathy, carnations, lilies or roses are given in circular configurations, which signify the transition of birth, life and death to rebirth. In this instance, the color of choice is commonly yellow. (Người Nga tổ chức một ngày lễ gọi là ngày Quốc tế Phụ nữ. Những món quà truyền thống bao gồm hoa hồng đỏ, hoa lan dạ hương hay hoa tulip. Khi có đám tang hay các dịp khác mà người ta thường muốn bày tỏ sự cảm thông thì hoa cẩm chướng, hoa huệ tây hay hoa hồng được kết theo hình tròn, có ý nghĩa là sự chuyển giao giữa sự sinh ra, lớn lên và chết đi để tái sinh. Trong dịp này, màu sắc thường được chọn là màu vàng)

    1. A Từ "fertility" trong đoạn 4 có thể được thay thế tốt nhất bằng từ nào?
      1. sự mắn đẻ B. tinh thần tốt C. hạnh phúc D. sự thủy chung Từ đồng nghĩa: fertility (khả năng sinh sản) = fecundity

In the times of ancient Rome, brides carried flowers to scare away evil spirits and encourage fertility. (Trong thời La Mã cổ đại, cô dâu thường mang hoa để xua đuổi các linh hồn ma quỷ và khuyến khích khả năng sinh sản)

    1. B Từ "superstitions" trong đoạn 4 gần nghĩa nhất với từ_ .
      1. niềm tinlâu đời B. niềm tin vô căn cứ

C. niềm tin tôn giáo D. niềm tin truyền thống Từ đồng nghĩa: superstitions (mê tín) = unfounded belief.

Flowers are generally gifted in odd numbered increments regardless of the occasion. However, the Brits also have superstitions regarding the number 13, so the number is avoided. (Hoa thường được tặng theo số lẻ bất kể vào dịp nào. Tuy nhiên, người Anh cũng có mê tín về số 13, vì vậy số này được tránh tặng)

C Ở đất nước nào người ta không nên mang hoa huệ tây trắng đến nhà người khác?

      1. Trung Quốc B. Nga C. Anh D. Hoa Kỳ

Căn cứ thông tin đoạn 4:

When invited to someone's home in Great Britain, it is tradition to bring a gift of flowers. All types are acceptable except white lilies, which are usually seen at funerals. (Khi được mời tới nhà ai đó ở Anh, mang quà là hoa là nét truyền thống. Tất cả các loại hoa đều được chấp nhận ngoại trừ hoa huệ tây trắng, loài hoa

thường được thấy ở các đám tang)

A Có thể suy ra từ đoạn văn rằng .

      1. Người ta có thể tặng hoa cho người Mỹ vào bất kì dịp nào.
      2. Người Ai Cập khá thoải mái trong việc nhận hoa ở đám tang và đám cưới.
      3. Hoa ở nước Anh được tặng theo số chẵn ở mọi trường hợp.
      4. Ở đám tang ở bất kì văn hóa nào, hoa được tặng theo hình vòng tròn.

Căn cứ thông tin đoạn cuối:

In the southern region of the continent, flowers are traditionally given during Christmas. Egyptians are much more conservative and restrict flower gifting to funerals and weddings. While certain flowers may have significant meanings for some, flowers in Las Vegas and across the United States flowers are an accepted gift for any reason desired. (Ở khu vực phía nam của châu lục, hoa thường được tặng theo truyền thống vào dịp Giáng sinh. Người Ai Cập thận trọng hơn nhiều và giới hạn việc tặng hoa vào đám tang hay đám cưới. Trong khi một số loài hoa thường có những ý nghĩa khác nhau, hoa ở Las Vegas và xuyên suốt nước Mỹ là hai món quà được chấp nhận cho bất kì lí do nào)

Dịch bài Trong hàng trăm năm, việc tặng hoa là một phương tiện giao tiếp xã hội.

Tại Hoa Kỳ, hoa thường được tặng trong các nghi thức, để kỷ niệm những dịp đặc biệt hoặc là một món quà chân thành giữa những người thân và bạn bè. Việc tặng hoa cũng xuất hiện ở hầu hết các nước trên thế giới. Tuy nhiên, ý nghĩa và truyền thống thường khác nhau.

Ngày xưa, trong khi theo truyền thống sinh viên thường tặng cho giáo viên yêu quý của họ một quả táo, ở Trung Quốc, giáo viên được tặng hoa. Hoa mẫu đoen là hoa lâu đời hất thường được tặng ở Trung Quốc. Chúng cũng được sử dụng khá phổ biến cho đám cưới. Kỳ lạ thay, cây trồng trong chậu không được coi là món quà dễ chịu trong nền văn hóa châu Á. Mọi người tin rằng nó giống như một cái cây bị giam giữ bởi một cái chậu, món quà tượng trưng cho sự ràng buộc hoặc hạn chế.

Ở Nga, thay vì tặng quà sinh nhật, khách mời danh dự nhận được một bông hoa duy nhất hoặc một bó hoa chưa được gói. Hoa được gói sẵn hoặc giỏ hoa không được tặng. Người Nga ăn mừng một ngày lễ được gọi là Ngày Quốc tế phụ nữ. Quà tặng truyền thống bao gồm hoa hồng đỏ, lan dạ hương hoặc hoa tulip. Khi có một đám tang hoặc dịp khác, nơi ai đó muốn thể hiện sự cảm thông, hoa cẩm chướng, hoa huệ tây hay hồng được gói tặng theo hình tròn, biểu thị sự chuyển tiếp

của sinh ra, sống và chết để tái sinh. Trong trường hợp này, màu sắc được lựa chọn thường là màu vàng. Đối với những dịp vui vẻ, việc sắp xếp và bó hoa thường theo số lẻ hoa.

Trong thời La Mã cổ đại, các cô dâu mang hoa để xua đuổi linh hồn ma quỷ và khuyến khích khả năng sinh sản. Người Hà Lan tin rằng hoa là thức ăn cho linh hồn. Khi được mời đến nhà của một ai đó ở Vương Quốc Anh, mang quà là hoa là nét truyền thống. Tất cả các loại hoa đều được chấp nhận ngoại trừ hoa huệ tây trắng, loài hoa thường thấy ở các đám tang. Không giống như Hoa Kỳ, hoa hồng đỏ là biểu tượng của tình yêu. Hoa thường được tặng theo số lẻ bất kể dịp nào. Tuy nhiên, người Anh cũng có mê tín dị đoan về số 13, vì vậy con số này được tránh tặng.

Ở khu vực phía nam của châu lục, hoa thường được tặng theo truyền thống vào dịp Giáng sinh. Người Ai Cập thận trọng hơn nhiều và giới hạn việc tặng hoa vào đám tang hay đám cưới. Trong khi một số loài hoa thường có những ý nghĩa khác nhau, hoa ở Las Vegas và xuyên suốt nước Mỹ là hai món quà được chấp nhận cho bất kì lí do nào.

    1. C CHỦ ĐỀ PERSONAL EXPERIENCES

Tại sao ông John Mill đi máy bay?

A. Ôngấymuốnđinghỉ. B.Ôngấymuốnthử.

C. Ôngấymuốnthămgiađình D.Ôngấyphảiđicôngtác.

Dẫnchứng: It was the only way he could visit his grandchildren in Canada. "I had made up my mind that I was going to do it, I couldn't let my son, his wife and their three children travel all the way here to visit me. It would be so expensive for

them and I know Tom's business isn't doing so well at the moment - it would also be tiring for the children - it's a nine-hour flight!" he says." (Đó là cách duy nhất ông có thể tới thăng các cháu của mình ở Canada. “Tôi đã quyết định sẽ thực hiện điều này, tôi không thể để con trai tôi, vợ và ba đứa cháu bay cả chuyến đường dài để tới thăm tôi được. Sẽ rất tốn kém và tôi biết việc kinh doanh của Tom lúc này không tốt – bọn trẻ cũng sẽ thấy mệt nữa – chuyến bay dài 9 tiếng!” ông nói)

D Tại sao ông John đọc về máy bay?

      1. Ông muốn biết nó hoạt động như nào?
      2. Sở thích của ông.
      3. Để đảm bảo an toàn cho ông.
      4. Ông ấy tìm được một quyển sách về máy bay.

Dẫn chứng: To get ready for the flight John did lots of reading about airplanes. (Để sẵn sàng cho chuyến bay John đọc rất nhiều về máy bay.)

    1. D Từ "which" trong đoạn văn ám chỉ _ .
      1. đọc về máy bay B. đặt vé

C. bay trên Boeing 747 D. một chiếc Boeing 747

Dẫn chứng: When he booked his seat, he was told that he would be flying on a Boeing 747, which is better known as a jumbo jet. (Khi ông đặt vé, ông được thông báo rằng ông sẽ bay trên một chiếc Boeing 747, tốt hơn thì được gọi là một máy bay phản lực lớn)

C Điều gì đã diễn ra khi ông ấy nhìn thấy chiếc máy bay phản lực đầu tiên?

      1. Ông ấy cảm thấy an toàn hơn B. Ông ấy thích hình dáng của nó

C. Ông ấy ngạc nhiên về độ lớn của nó D. Ông ấy nghĩ cánh của nó rất nhỏ Dẫn chứng: Even though I had discovered all this very interesting information about the jumbo, when I saw it for the first time, just before I was going to travel to Canada, I still couldn't believe that something so enormous was going to get up in the air and fly. I was even more impressed when I saw how big it was inside with hundreds of people!"

(Mặc dù tôi đã phát hiện ra tất cả các thông tin rất thú vị về chiếc máy bay, khi tôi thấy nó lần đầu tiên, ngay trước khi tôi đến Canada, tôi vẫn không thể tin rằng có thứ lại lớn như vậy được đưa lên không trung và bay. Tôi đã thậm chí còn ấn tượng

hơn nhiều khi thấy nó có thể chứa hàng trăm người!)

D Ông John cảm thấy thế nào khi máy bay cất cánh?

      1. thú vị B. vui C. buồn D. sợ hãi

Dẫn chứng: "The take-off itself was much smoother than I expected although I was still quite scared until we were in the air. (Nó cất cánh êm hơn tôi nghĩ nhiều, mặc dù tôi vẫn còn khá sợ hãi cho đến khi chúng tôi lên không trung)

B Điều gì khiến ông John ngạc nhiên nhất về chuyến bay?

      1. Ông ấy thích đồ ăn. B. Ông ấy có thể ngủ.

C. Có chiếu phim. D. Cảnh rất đẹp.

Dẫn chứng: "In the end, I managed to relax, enjoy the food and watch one of the movies and the view from the window was spectacular. I even managed to sleep for a

while! Of course," (Cuối cùng, tôi đã thư giãn, thưởng thức các món ăn, xem một trong những bộ phim và cảnh từ cửa sổ thật hùng vĩ. Tôi thậm chí còn có thể ngủ được một lát!)

A Cuối cùng thì ông John đã cảm thấy thế nào về những nỗi sợ?

      1. Ông nghĩ mình đã phí thời gian để sợ hãi
      2. Ông nhận ra khá ổn khi sợ hãi
      3. Ông hy vọng cháu mình sẽ không sợ bay
      4. Ông nhận ra việc sợ hãi giúp ông được an toàn

Dẫn chứng: Suddenly, I felt so silly about all the years when I couldn't even think of getting on a plane. I had let my fear of living stop me from seeing the people I love most in the world. I can visit my son and family as often as I like now! (Đột nhiên, tôi cảm thấy rất ngớ ngẩn về tất cả những năm tháng khi tôi thậm chí không dám nghĩ về việc bước lên máy bay. Nỗi sợ hãi đã ngăn tôi được gặp những người tôi yêu thương nhất trên đời. Bây giờ tôi có thể thăm con trai và gia đình của tôi thường xuyên!)

Dịch bài Khi John Mills lần đầu tiên đi máy bay, ông đã rất sợ hãi. Ông không thích cảm giác cách xa mặt đất cả ngàn feet trong không trung. “Tôi cũng không thích thực tế là tôi không kiểm soát được”, John nói. “Tôi là một hành khách khá khó chịu trong ô tô. Khi người khác lái xe, tôi sẽ nói với họ phải lái thế nào. Điều này khiến họ phát điên lên.”

Tuy nhiên John không thể trốn tránh việc đi máy bay mãi được. Đó là cách duy nhất ông có thể tới thăm các cháu của mình tại Canada. “Tôi đã quyết định sẽ thực hiện điều này, tôi không thể để con trai tôi, vợ ba đứa cháu bay cả chuyến đường dài để tới thăm tôi được. Sẽ rất tốn kém và tôi biết việc kinh doanh của Tom lúc này không tốt – bọn trẻ cũng sẽ thấy mệt nữa – chuyến bay dài 9 tiếng!” ông nói.

Để sẵn sàng cho chuyến bay John đọc rất nhiều về máy bay. Khi ông đặt vé, ông được thông báo rằng ông sẽ bay trên một chiếc Boeing 747, tốt hơn thì được gọi là một máy bay phản lực. “Tôi cần phải biết càng nhiều càng tốt trước khi đặt vé bay chiếc máy bay đó. Tôi cho rằng đó là một cách để làm cho bản thân mình cảm thấy tốt hơn. Boeing 747 là máy bay chở khách lớn nhất thế giới ở thời điểm này. Chiếc đầu tiên bay vào ngày 9 tháng 2 năm 1969 tại Hoa Kỳ. Nó có thể chở 524 hành khách và 3.400 hành lý. Nhiên liệu cho máy bay được giữ ở cánh và đôi cánh của chiếc 747 lớn đến mức có thể mang đủ nhiên liệu

cho một chiếc xe trung bình để có thể đi du lịch 16.000 kilômét một năm trong vòng 70 năm. Không thể tin được đúng không? Mặc dù tôi đã phát hiện ra tất

cả các thông tin này rất thú vị về chiếc máy bay, khi tôi thấy nó lần đầu tiên, ngay trước khi tôi đến Canada, tôi vẫn không thể tin rằng có thứ lại lớn như vậy được đưa lên không trung và bay. Tôi đã thậm chí còn ấn tượng hơn nhiều khi thấy nó có thể chứa hàng trăm người!”

Bất ngờ lớn nhất của John chính là chuyến bay. “Nó cất cánh êm hơn tôi nghĩ nhiều, mặc dù tôi vẫn còn khá sợ hãi cho đến khi chúng tôi lên không

trung. Cuối cùng, tôi đã thư giãn, thưởng thức các món ăn, xem một trong những bộ phim và cảnh từ cửa sổ thật hùng vĩ. Tôi thậm chí còn có thể ngủ được một lát!”

“Tất nhiên là,” John tiếp tục “phần thưởng lớn nhất là khi tôi đến Canada và thấy con trai cùng gia đình, đặc biệt là những đứa cháu xinh xắn của mình. Đột nhiên, tôi cảm thấy rất ngớ ngẩn về tất cả những năm tháng khi tôi thậm chí không dám nghĩ về việc bước lên máy bay. Nỗi sợ hãi đã ngăn tôi được gặp những người tôi yêu thương nhất trên đời. Bây giờ tôi có thể thăm con trai và gia đình của tôi thường xuyên!”

Câu 31: Đáp án B

CHỦ DỀ WAYS OF SOCIALIZING

Câu nào sau đây có thể là tiêu đề phù hợp nhất cho đoạn văn?

  1. Chúng ta nên làm gì khi được mời đến ăn bữa tối ở nhà người Mỹ?
  2. Quy tắc ứng xử trong bàn ăn ở Mỹ.
  3. Cách cư xử lịch sự ở nhà hàng của Mỹ.
  4. Điều gì là có thế chấp nhận được trong quy tắc bữa tối ở Mỹ.

Căn cứ vào những ý chính của các đoạn văn: Các đoạn văn đầu nói về các quy tắc ứng xử trong bàn ăn ở Mỹ, cả ở nhà và ở nhà hàng.

Câu 32: Đáp án D

Theo đoạn văn, hành động nào sau đây có thể khiến người Mỹ cảm thấy khó chịu?

A. Từ chối ăn thêm thức ăn họ mời. B. Để thức ăn dư lại trên đĩa.

C. Dùng tay để ăn các món. D. Dùng lưỡi liếm thức ăn trên ngón tay.

Căn cứ vào thông tin đoạn 1:

In America, when dining, people consider it rude for a guest or a dining partner to belch or burp, eat with an open mouth, smack, or lick your fingers. (Ở Mỹ, khi ăn tối, người ta thường coi là thô lỗ nếu một vị khách hay một người bạn dùng chung bữa tối ợ hơi, nhai mở miệng, chép môi hay liếm các ngón tay của họ)

Các hành động ở câu A, B và C là chấp nhận được, căn cứ vào thông tin đoạn 2 và 3:

  • It is acceptable to refuse additional servings of food by saying “No, thank you” and the host or hostess will not be insulted if you do so. (Việc từ chối các phần ăn bổ sung bằng cách nói “Không, cảm ơn” là có thể chấp nhận được và chủ nhà sẽ không cảm thấy khó chịu nếu bạn làm vậy)
  • Similarly, if you leave a small amount of uneaten food on your plate at a restaurant or in a home, it is not considered an insult. (Tương tự, nếu bạn để lại một phần nhỏ thức ăn thừa trên đĩa của bạn ở nhà hàng hay ở nhà, thì điều này cũng không bị coi là khó chịu)
  • Americans typically use forks, spoons and knives to eat, but there are some types of foods that are acceptable to eat with one's fingers, like sandwiches or pizza. (Người Mỹ thường sử dụng nĩa, thìa và dao để ăn, nhưng có một vài món ăn thì việc ăn bằng tay là có thể chấp nhận được, như ăn bánh sandwich hay pizza)

Câu 33: Đáp án C

Câu nào sau đây không được để cập trong đoạn văn?

  1. Trong các bữa ăn trang trọng, người lớn tuổi thường ăn trước.
  2. Ở Mỹ, các đĩa thức ăn thường được đặt nguyên vị trí trên bàn.
  3. Đôi khi người ta dùng tay để lấy thức ăn từ đĩa ăn chung.
  4. Việc húp sùm sụp khi ăn súp là không lịch sự.

Căn cứ vào thông tin đoạn 2:

People in the United States serve and eat food with either hand, but never take food from a communal serving dish with their hands. Generally, a serving utensil is used. (Người dân Hoa Kỳ thường phục vụ và ăn các món ăn bằng một trong hai tay, nhưng không bao giờ dùng tay lấy thức ăn từ đĩa phục vụ chung. Thông thường, một dụng cụ phục vụ sẽ được sử dụng)

Câu 34: Đáp án A

Từ “it” trong đoạn 3 đề cập đến từ nào sau đây?

A. cái nĩa B. C. miệng D. món súp

Căn cứ vào thông tin đoạn 3:

Từ “it” thay thế cho danh từ “your fork" trong câu trước.

When consuming noodles, twirl them around your fork and then put it in your mouth. (Khi ăn món mì, hãy xoắn các sợi mì quanh cái nĩa của bạn và sau đó cho nó vào miệng của bạn)

It là đại từ số ít nên không thể thay thế cho danh từ noodles ở số nhiều.

Câu 35: Đáp án A

Từ “gauge” trong đoạn 4 co thể được thay thế bởi từ nào sau đây?

A. đánh giá B. gây ấn tượng C. thể hiện D. ước tính Từ đồng nghĩa: gauge (xác định, đánh giá) = determine

If you are a man taking out a woman for dinner, you are almost always expected to pay. This is for the woman to gauge your intentions and interest with her. (Nếu bạn là một người đàn ông mời một người phụ nữ ra ngoài ăn tối, bạn gần như luôn luôn dự kiến sẽ trả tiền. Điều này là để cho người phụ nữ ấy đánh giá ý định về sự quan tâm của bạn với cô ấy)

Câu 36: Đáp án B

Trong trường hợp nào thì hoá đơn thường được chia cho những người dùng bữa?

  1. Khi một người đàn ông đang hẹn hò với một người phụ nữ.
  2. Khi một người đang dùng bữa cùng bạn bè.
  3. Khi một người đang ăn cùng với người lớn tuổi.
  4. Khi một cô gái đang thử lòng một chàng trai.

Căn cứ vào thông tin đoạn 4:

Also, if you are going out with a friend to eat, almost always, the bill is expected to be split in half, or each person pays for themselves. (Thêm vào đó, nếu bạn đang ra ngoài ăn uống với bạn bè, gần như luôn luôn rằng hoá đơn sẽ được chia đôi, hoặc mỗi người sẽ tự trả tiền cho chính mình.)

Câu 37: Đáp án D

Từ “linger” trong đoạn cuối có nghĩa gần nhất với từ nào?

A. xuất hiện B. sắp xếp C. đặt chỗ D. ở lại Từ đồng nghĩa: linger (ở lại) = remain

Take your time to finish your meal, and unless there is a line of people waiting at the door, it is not considered rude to linger at your table for as long as you like.

(Hãy dành thời gian để hoàn thành bữa ăn của bạn, và trừ khi có một dòng người đang chờ đợi ở cửa, việc bạn nán lại ở bàn mình bao lâu đi nữa cũng không bị coi là thô lỗ)

Câu 38: Đáp án D

Câu nào trong các câu sau có thể suy ra từ đoạn văn?

  1. Thật là bất lịch sự nếu bạn đưa thêm tiền cho phục vụ.
  2. Bất kì khi nào bạn nhận hoá đơn, bạn nên nghĩ đến việc ra về sớm.
  3. Trả tiền cho người khác có thể khiến họ khó chịu.
  4. Bắt chước theo người khác khi bạn không chắc nên làm gì ở bữa ăn là một ý kiến hay.

Căn cứ thông tin đoạn 3:

When in doubt, look to see what others are doing. (Khi không chắc chắn, hãy quan sát những gì người khác đang làm)

Dịch bài

Ở Mỹ, khi ăn tối, người ta thường coi là thô lỗ nếu một vị khách hay một người bạn dùng chung bữa tối ợ hơi, nhai mở miệng, chép môi hay liếm các ngón tay của họ. Khăn ăn, thường được

cung cấp sẵn trong mỗi bữa ăn và nên được đặt trong lòng của mỗi người và sau đó được sử dụng trong suốt

bữa ăn để lau sạch các ngón tay và miệng của họ.

Việc từ chối các phần ăn bổ sung bằng cách nói “Không, cảm ơn" là có thể chấp nhận được và chủ nhà sẽ không cảm thấy khó chịu nếu bạn làm vậy. Tương tự như vậy, nếu bạn để lại một lượng nhỏ thức ăn thừa trên đĩa của bạn tại một nhà hàng hoặc trong một ngôi nhà, nó không được coi là xúc phạm. Nếu bạn ăn tất cả mọi thứ trên đĩa, vị chủ nhà có thể cảm thấy rằng họ đã không chuẩn bị đủ thức ăn và có thể xấu hổ. Người dân ở Hoa Kỳ phục vụ và ăn thức ăn bằng một trong hai tay, nhưng không bao giờ lấy thức ăn từ đĩa phục vụ chung với bàn tay của họ. Thông thường, một dụng cụ phục vụ được sử dụng.

Người Mỹ thường sử dụng nĩa, thìa và dao để ăn, nhưng có một vài món ăn thì việc ăn bằng tay là có thể chấp nhận được, như ăn bánh sandwich hay pizza. Khi không chắc, hãy quan sát những gì người khác đang làm. Trong các bữa ăn trang trọng, nếu bạn thắc mắc là có nên bắt đầu ăn hay chưa, bạn nên đợi cho đến khi người phụ nữ lớn tuổi nhất (hoặc người đàn ông lớn tuổi nhất nếu không có phụ nữ) bắt đầu ăn. Khi ăn, không lấy bát hoặc đĩa từ bàn để giữ bên dưới miệng của bạn. Ngay cả mì, súp và cơm đều được ăn bằng đĩa hoặc bát ở nguyên trên bàn. Khi ăn súp và các món lỏng nóng, việc húp sùm sụp được coi là bất lịch sự - đừng làm điều này. Khi ăn món mì, hãy xoắn các sợi mì quanh cái nĩạ của bạn và sau đó cho nó vào miệng của bạn.

Nếu bạn là một người đàn ông đưa một người phụ nữ ra ngoài ăn tối, bạn gần như luôn luôn dự kiến sẽ trả tiền. Điều này là dành cho người phụ nữ để đánh giá ý định và sự quan tâm của bạn với cô ấy. Ví dụ, mời một người phụ nữ đi uống cà phê, so với món bánh thịt chiên giòn, so với một bữa ăn tối ưa thích, so với đồ uống lúc 11:30 tối, tất cả chuyền tải tín hiệu về nhiều điều khác nhau cho họ. Vì vậy, hẹn hò là một "cuộc kiểm tra" nhiều thứ. Thanh toán tiền cũng quan trọng như nơi bạn đưa cô ấy đến và việc bạn đến trễ bao lâu. Vì vậy, đừng cho rằng cô ấy chỉ đang cố gắng để có được một “bữa ăn miễn phí". Hầu hết các cô gái thì không. Ngoài ra, nếu bạn đang đi ăn cùng với một người hạn, hầu như luôn luôn, hoá đơn sẽ được chia làm đôi, hoặc mỗi người tự trả tiền cho mình.

Nếu bạn đang ăn trong một nhà hàng, bạn sẽ phải trả thêm 15 đến 20% tiền tip (boa) cho người phục vụ vào hóa đơn của bạn. Ở Mỹ, nhân viên phục vụ đôi khi có thể ghé qua bàn của bạn để hỏi bữa ăn của bạn như thế nào, đây được coi là dịch vụ tốt. Họ cũng sẽ mang đến cho bạn hoá đơn khi có vẻ như là

bạn đã ăn xong, tuy nhiên điều này không nhất thiết phải là dấu hiệu cho thấy bạn phải rời đi ngay lập tức. (Đừng quá ngại khi hỏi lại để chắc chắn: người phục vụ và bồi bàn không thể đọc được suy

nghĩ của bạn). Hãy dành thời gian để hoàn thành bữa ăn của bạn, và trừ khi có một dòng người chờ đợi ở cửa, việc bạn nán lại ở bản mình bao lâu đi nữa cũng không bị coi là thô lỗ.

Câu 39: Đáp án C Chủ đề MARRIAGE

Tại sao Christina và James lại cho rằng kết hôn là không cần thiết? Bởi vì .

  1. Nó chỉ là một mảnh giấy.
  2. Một nửa những cuộc hôn nhân đều kết thúc bằng ly hôn.
  3. Không ai trong số họ lúc lớn lên được chứng kiến nhiều cuộc hôn nhân thành công.
  4. Họ thích cuộc sống tự do.

Căn cứ vào phần thông tin đầu đoạn văn:

“The couple had many discussions about marriage and decided that it just did not seem necessary. Wasn't it only a piece of paper? And didn't half of all marriages end in divorce? Neither Christina nor James had seen much success with marriage while growing up.”

(Cặp đôi đó đã có nhiều cuộc thảo luận về việc kết hôn và quyết định rằng điều đó không thực sự cần thiết. Không phải nó chỉ là một mảnh giấy thôi sao? Và không phải một nửa những cuộc hôn nhân đó đều kết thúc bằng ly hôn sao? Cả Christina và James khi lớn lên đều không được chứng kiến nhiều cuộc hôn nhân thành công.)

Câu 40: Đáp án D

Từ nào có thể được dùng như một từ trái nghĩa của từ “maternal"?

A. relative (a): có liên quan tới, tương đối B. bloody (a): chảy máu, dính máu

C. close (a): gần gũi D. paternal (a): về đằng nội

Căn cứ vào câu sau: “Christina and her mother lived with her maternal grandmother, who often served as a surrogate parent.” (Christina và mẹ của Cô ấy đã sống với bà ngoại, người đã từng nhận làm việc đẻ hộ.)

maternal (a): về đằng ngoại ><paternal (a): về đằng nội

Câu 41: Đáp án C

Theo như đoạn văn số 2, câu nào dưới đây là đúng?

  1. Christina sống với bố mẹ của cô ấy cho đến khi cô ấy học xong đại học.  sai
  • Căn cứ vào thông tin sau: "Her parents never married, and her father has had little contact with the family since she was a toddler. Christina and her mother lived with her maternal grandmother" (Bố mẹ cô ấy chưa bao giờ kết hôn và bố của cô ấy gần như cũng không liên lạc gì với gia đình từ khi cô ấy còn là một đứa trẻ chập chững biết đi. Christina và mẹ của cô ấy đã sống với bà ngoại)
  1. James được nuôi nấng bởi bố và vợ mới của bố.  sai
  • Căn cứ vào thông tin sau: “James grew up in a twowparent household until age seven, when his parents divorced. He lived with his mother for a few years, and then later with his mother and her boyfriend until he left for college." (James lớn lên trong một gia đình có đủ cả bố lẫn mẹ cho đến khi anh ấy 7 tuổi thì bố mẹ anh ấy li dị. Anh ấy sống với mẹ khoảng một năm, rồi sau đó sống với mẹ và bạn trai của mẹ cho đến khi anh ấy học xong đại học)
  1. Cả James và Christina đều sinh ra trong gia đinh đổ vỡ.  đúng

Câu 42: Đáp án B

Từ nào dưới đây có thế được dùng để thay thế cho từ “traditional”?

A. old-fashioned (a): cũ, không hợp thời trang B. customary (a): truyền thống

C. antique (a): cổ, đồ cổ D. ancient (a): cổ xưa

Căn cứ vào thông tin ở câu sau: "Christina likes the idea of her children growing up in a traditional family" (Christina thích cái ý tưởng con cái lớn lên trong một gia đình truyền thống)

Traditional (a) = customary (a): truyền thống

Câu 43: Đáp án A

Tại sao Christina và James lại đề cập lại chủ đề kết hôn?

  1. Họ đang nghĩ đến việc có con.
  2. Christina thích cái ý tưởng về việc con cái lớn lên trong một gia đình truyền thống
  3. Họ đã sống cùng nhau rất lâu rồi.
  4. James đang lo lắng về vấn đề hôn nhân đổ vỡ và những hệ lụy của nó có thể xảy tới con cái.

Căn cứ vào thông tin của câu đầu tiên của đoạn 3: “Recently, Christina and James have been thinking about having children and the subject of marriage has resurfaced.”(Gần đây, Christina và James đã nghĩ về việc sinh con và chủ để kết hôn lại được gợi lại)

Câu 44: Đáp án B

Lời khuyên của bố mẹ cho cặp đôi này là gì?

  1. Dù có kết hôn hay không thì họ vẫn là một gia đình.
  2. Họ nên kết hôn để con cái được chăm sóc tốt hơn.
  3. Cặp đôi này nên làm bất cứ cái gì họ muốn.
  4. Họ có thể có con mà không cần phải trói buộc bằng hôn nhân.

Căn cứ vào thông tin sau: “When they shared these concerns with their parents, Iames’s mom was adamant that the couple should get married. Despite having been divorced and having a live-in boyfriend of 15 years, she believes that children are better off when their parents are married. Christina’s mom believes that the couple should do whatever they want but adds that it would "be nice” if they wed."

(Khi họ chia sẻ vấn đề này với bố mẹ mình, mẹ của James cương quyết rằng họ nên kết hôn. Mặc dù đã li dị và sống với bạn trai trong khoảng 15 năm, bà vẫn tin rằng trẻ con sẽ được chăm sóc tốt hơn nếu bố mẹ

chúng kết hôn. Mẹ của Christina tin rằng họ có thể làm bất cứ điều gì họ muốn nhưng nhấn mạnh thêm rằng sẽ là tốt hơn nếu họ kết hôn)

Câu 45: Đáp án A

Đoạn văn này chủ yếu thảo luận về điều gì?

    1. Vấn đề hôn nhân
    2. Ảnh hưởng tiêu cực của một gia đình đổ vỡ
    3. Những thay đổi trong thái độ của con người về vấn đề kết hôn
    4. Những tranh luận về vấn đề hôn nhân

Dịch bài

Christina và James gặp nhau trong trường đại học và đã hẹn họ nhau được hơn 5 năm. Trong suốt hai năm qua, họ đã sống cùng nhau trong một căn hộ cao cấp mà họ đã góp tiền mua. Trong khi cả Christina và James đều tự tin với quyết định của mình là bước vào một mối quan hệ ràng buộc như một bản thế chấp có giá trị 20 năm nhưng họ lại không chắc là họ có muốn bước vào cuộc sống hôn nhân hay không. Cặp đôi đó đã có nhiều cuộc thảo luận về việc kết hôn và quyết định rằng nó là không thực sự cần thiết. Không phải nó chỉ là một mảnh giấy thôi sao? Và không phải một nửa những cuộc hôn nhân đó đều kết thúc bằng ly hôn sao?

Cả Christina và James khi lớn lên đều không được chứng kiến nhiều cuộc hôn nhân thành công. Christina được nuôi dưỡng bởi một bà mẹ đơn thân. Bố mẹ cô ấy chưa bao giờ kết hôn và bố của cô ấy gần như cũng không liên lạc gì với gia đình từ khi cô ấy còn là một đứa trẻ chập chững biết đi. Christina và mẹ của cô ấy đã sống với bà ngoại, người đã từng nhân làm việc đẻ hộ. James lớn lên trong một gia đình có đủ cả bố lẫn mẹ cho đến khi anh ấy 7 tuổi thì bố mẹ anh ấy li dị. Anh ấy sống với mẹ khoảng một năm, rồi sau đó sống với mẹ và bạn trai của mẹ cho đến khi anh ấy học xong đại học. James vẫn rất gần gũi với bố mình, người mà đã tái hôn và có một em bé với vợ mới.

Gần đây, Christina và lames đã nghĩ về việc sinh con và chủ đề kết hôn lại được gợi lại. Christina thích cái ý tưởng con cái lớn lên trong một gia đình truyền thống trong khi lames thì lại quan tâm tới những hệ lụy của hôn nhân và hậu quả tiêu cực cho con cái có thể xảy ra. Khi họ chia sẻ vấn đề này với bố mẹ mình, mẹ của lames cương quyết rằng họ nên kết hôn. Mặc dù đã li dị và sống với bạn trai trong khoảng 15 năm, bà vẫn tin rằng trẻ con sẽ được chăm sóc tốt hơn nếu bố mẹ chúng kết hôn. Mẹ của Christina tin rằng họ có thế làm bất cứ điều gì họ muốn nhưng nhấn mạnh thêm rằng sẽ là tốt hơn nếu họ kết hôn. Bạn bè của Christina và lames thì cho rằng dù có kết hôn hay không thì họ vẫn là một gia đình.

46

C

CHỦ ĐỀ VOLUNTEER WORK

Từ “it” dùng để chỉ .

      1. thủ tục phẫu thuật B. người bị tai nạn

C. một lượng máu D. bệnh nhân phẫu thuật

Thông tin: Usually a pint of whole blood is donated, and it is then divided into platelets, white blood cells, and red blood cells. (Thông thường, một lượng máu được hiến tặng, sau đó chia thành tiểu cầu, bạch cầu và hồng cầu.)

=> "it" chỉ "a pint of whole blood"

C Theo đoạn văn, người ta thường hiến máu cho hồng cầu như thế nào?

    1. bốn tháng một lần B. ba tháng một lần

C. hai tháng một lần D. mỗi tháng một lần

Thông tin: People can donate blood (for red blood cells) about once every two months.

(Mọi người có thể hiến máu (đối với hồng cầu) khoảng hai tháng một lần.)

B Tất cả những điều sau đây được đề cập như là những phản ứng phụ có thể xảy ra đối với truyền máu TRỪ .

    1. Nhạy cảm với bạch cầu B. Bong bóng khí trong máu

C. Dị ứng D. Sự không tương thích của hồng cầu Thông tin: Negative reactions to transfusions are not unusual. The recipient may suffer an allergic reaction or be sensitive to donor leukocytes. Some may suffer from an undetected red cell incompatibility. (Phản ứng phụ của truyền máu không phải là bất thường. Người nhận có thể bị phản ứng dị ứng hoặc nhạy cảm với bạch cầu. Một số có thể bị ảnh hưởng bởi sự không tương thích của các tế bào hồng cầu không bị phát hiện.)

=> Chỉ có phương án B không được đề cập

C Từ nào gần nghĩa nhất với từ "undetected"?

    1. không minh họa B. không bị bắt

C. không tìm thấy D. không muốn

Thông tin: Some may suffer from an undetected red cell incompatibility. Unexplained reactions are also fairly common. (Một số có thể bị ảnh hưởng bởi sự không tương thích của các tế bào hồng cầu không bị phát hiện. Các phản ứng không giải thích được cũng khá phổ biến.)

=> "undetected": không bị phát hiện

B Cụm từ "go to great length" có thể được thay thế tốt nhất bằng?

    1. chiếu xạ B. theo dõi một cách nghiêm ngặt

C. theo dõi thường xuyên D. loại bỏ

Thông tin: Today, hospitals and blood banks go to great lengths to screen all blood donors and their blood. (Ngày nay, các bệnh viện và ngân hàng máu nghiêm ngặt theo dõi

những người hiến máu và máu của họ.)

A Dựa vào những thông tin trong đoạn văn, có thể suy ra những gì về truyền máu cho trẻ sơ sinh và trẻ nhỏ?

    1. Nó được xử lý bằng năng lượng bức xạ.
    2. Nó không được xử lý khác với người lớn.
    3. Nó không gây nguy hiểm cho trẻ em.
    4. Nó được kiểm tra nghiêm ngặt như máu cho người lớn.

Thông tin: When the recipient is a newborn or an infant, the blood is usually irradiated to eliminate harmful elements. (Khi người nhận là trẻ sơ sinh hoặc trẻ nhỏ, máu thường được chiếu xạ để loại bỏ các yếu tố gây hại.)

A Tác giả có ý gì trong đoạn văn?

    1. Hiến máu có lợi cho nhân loại.
    2. Không thể ngăn ngừa đông máu.
    3. Làm đông máu sẽ phá hủy tiểu cầu.
    4. Truyền máu là một quá trình nguy hiểm.

Thông tin: The ability to store blood for long periods has been a boon to human health.

(Khả năng lưu trữ máu trong thời gian dài là một lợi ích cho sức khỏe con người.)

Dịch bài Mọi người đã hiến máu từ đầu thế kỷ XX để giúp người bị tai nạn và bệnh nhân trải qua các thủ tục phẫu thuật. Thông thường, một lượng máu được hiến tặng, sau đó chia thành tiểu cầu, bạch cầu và hồng cầu. Mọi người có thể hiến máu (đối với hồng cầu) khoảng hai tháng một lần. Truyền máu từ người hiến tặng cho người nhận là điều đơn giản. Nó bao gồm lấy máu từ tĩnh mạch cánh tay của người hiến tặng bằng ống tiêm dưới da. Máu chảy qua ống nhựa vào túi hoặc chai chứa natri citrat, giúp ngăn máu đông. Khi máu được truyền cho bệnh nhân, một ống nhựa và kim dưới da được nối với cánh tay của người nhận. Máu chảy xuống từ bình chứa bằng trọng lực. Đây là một quá trình chậm và có thể kéo dài đến 2 giờ

để hoàn thành truyền máu vào cho người nhận. Bệnh nhân được bảo vệ khỏi bị nhiễm bệnh trong quá trình truyền máu. Chỉ sử dụng các bình chứa, ống và kim vô trùng, điều này giúp đảm bảo rằng máu truyền hoặc lưu trữ không bị nhiễm vi khuẩn gây bệnh. Phản ứng phụ của việc truyền máu không phải là bất thường. Người nhận có thể bị dị ứng hoặc nhạy cảm với bạch cầu. Một số có thể bị ảnh hưởng bởi sự không tương thích của các tế bào hồng cầu không bị phát hiện. Các phản ứng không giải thích được cũng khá phổ biến. Mặc dù hiếm gặp, các nguyên

nhân khác của phản ứng phụ như máu bị nhiễm trùng, bong bóng không khí trong máu, quá tải hệ thống tuần hoàn thông qua việc dùng quá mức máu, hoặc nhạy cảm với huyết tương hoặc tiểu cầu. Ngày nay, các bệnh viện và ngân hàng máu theo dõi nghiêm ngặt những người hiến máu và máu của họ. Tất cả máu hiến tặng đều được kiểm tra nghiêm ngặt và nghiêm túc để kiểm tra các bệnh như HIV, viêm gan B và giang mai. Khi người nhận là trẻ sơ sinh hoặc trẻ nhỏ, máu thường được chiếu xạ để loại bỏ các yếu tố gây hại. Máu đã được hiến được làm sạch, và lấy ra bạch cầu và tiểu cầu. Lưu trữ máu đôi khi đòi hỏi quá trình đông lạnh. Để làm đông các hồng cầu, một dung dịch glycerol được thêm vào. Để giải đông, glycerolis được loại bỏ. Khả năng lưu trữ máu trong thời gian dài là một lợi ích cho sức khỏe con người.

  1. D CHỦ ĐỀ VỀ EDUCATION

Câu nào trong các câu sau mô tả chính xác nhất ý chính của đoạn văn?

    1. Nhiều học sinh coi đại học là con đường duy nhất sau khi học xong cấp THPT.
    2. Một tấm gương sáng về thành công trong cuộc sống thực sau khi tốt nghiệp cấp THPT.
    3. Không nhiều sinh viên thành công sau khi tốt nghiệp đại học.
    4. Đại học không phải là con đường duy nhất tới thành công.

Căn cứ vào nội dung bài đọc:

Tác giả đã chỉ ra rằng có những người không hoàn thành chương trình đại học nhưng vẫn thành công, trong khi hàng ngàn sinh viên ra trường không tìm được việc làm. Vì vậy, đại học không phải là con đường duy nhất đến thành công.

C Thông tin nào về những thủ khoa kì thi tuyển sinh đại học không được đề cập đến trong bài?

    1. Đa số những người này là con nhà nghèo nhưng thông minh và hiếu học.
    2. Họ được hi vọng là sẽ tìm ra cách để cải thiện cuộc sống của gia đình mình.
    3. Thành công của họ huy hoàng hơn vì họ tham dự nhiều lớp học hơn những người khác.
    4. Những học sinh này được khâm phụ bởi nghị lực phi thường.

Căn cứ vào thông tin đoạn 1:

Every summer, when the results of university entrance exam come out, many newspaper stories are published about students who are top-scorers across the country. Most portray students as hard-working, studious, smart and, generally, from low-income

families. They are often considered heroes or heroines by their families, communes,

villages and communities. And they symbolise the efforts made to lift them and their relatives, out of poverty. The students are often too poor to attend any extra-classes, which make their achievements more illustrious and more newsworthy. (Mỗi mùa hè,

khi có kết quả kì thi tuyển sinh đại học, nhiều câu chuyện trên báo chí được xuất bản về những học sinh là những thủ khoa trên toàn quốc. Hầu hết chân dung các sinh viên đều chăm chỉ, hiếu học, thông minh và nói chung, xuất thân từ các gia đình có thu nhập thấp. Họ thường được coi là anh hùng hay nữ anh hùng bởi gia đình, xã, làng và cộng đồng của họ. Và họ tượng trưng cho những nỗ lực được thực hiện để đưa họ và người thân của họ thoát khỏi đói nghèo. Các học sinh thường quá nghèo để tham dự bất kỳ lớp học thêm nào, điều này làm cho thành tích của họ thêm lừng lẫy và đáng chú ý hơn.)

55

B

Từ "unconventional" trong đoạn 2 có thể được thaythế bởi .

A. phổ biến B. bất thường C. nổi tiếng D. nổi tiếng xấu Từ đồng nghĩa: unconventional (bất thường, khác thường) = unusual

Căn cứ vào thông tin: Should we demand more stories about those who fail the exam but succeed in life or about those who quit university education at some level and do something else unconventional? (Chúng ta có nên yêu cầu nhiều câu chuyện hơn về những người thất bại trong kỳ thi nhưng thành công trong cuộc sống hay về những người bỏ học đại học ở một mức độ nào đó và làm điều gì đó khác thường không?)

56

A

Tác giả miêu tả Trần Nguyễn Lê Vân trong đoạn 3 như là .

  1. Một tấm gương sáng đạt được thành công mặc dù anh ấy không hoàn thành chương trình giáo dục của mình.
  2. Một doanh nhân kiếm tiền bằng cách bán điện thoại trực tuyến.
  3. Một người sáng lập mà website của anh ta có cảm hứng từ mạng xã hội như Facebook.
  4. Một thủ khoa đặt vé trực tuyến và xác nhận qua tin nhắn.

Căn cứ thông tin đoạn 3:

He is the founder of a website, vexere.com, that passengers can use to book bus tickets online and receive tickets via SMS. His business also arranges online tickets via mobile phones and email. Van dropped out of his MBA at the Thunderbird School

of Global Management in Arizona in the United States. (Anh là người sáng lập ra trang web, vexere.com, mà hành khách có thể sử dụng để đặt vé xe buýt trực tuyến và nhận vé qua tin nhắn điện thoại. Doanh nghiệp của anh cũng hỗ trợ đặt vé trực tuyến qua điện thoại di động

và email. Vân bỏ học cử nhân Quản trị Kinh doanh (MBA) tại Trường Quản lý Toàn cầu Thunderbird ở Arizona, Hoa Kỳ.)

  1. C Từ "them" trong đoạn 3 đề cập đến .
    1. sự vinh dự B. tấm gương C. những bạn trẻ D. báo chí Từ “them” thay thế cho những thanh niên trẻ tuổi trong câu trước.

His story has caught the attention of many newspapers and he believes more coverage should be given to the youngsters who can be role-models in the startup community. Getting into university, even with honours, is just the beginning. 'We applaud them and their efforts and obviously that can give them motivation to do better in life. (Câu chuyện của ông đã thu hút được sự chú ý của nhiều tờ báo và ông tin rằng cần phải có nhiều sự dũng cảm hơn cho các bạn trẻ, những người có thể là tấm gương trong cộng đồng khởi nghiệp. Vào được đại học, ngay cả với niềm vinh dự, chỉ là khởi đầu. “Chúng ta hoan nghênh họ và những nỗ lực của họ và rõ ràng là điều này có thể tiếp thêm cho họ động lực để làm tốt hơn trong cuộc sống.)

  1. B Từ "plague" trong đoạn 4 gần nghĩa nhất với từ .
    1. mâu thuẫn B. làm ưu phiền C. nhắc nhở D. làm phiền Từ đồng nghĩa: plague (làm buồn lòng, gây rối không yên) = afflict

Alarming statistics about unemployment continues to plague us. As many as 162,000 people with some kind of degree cannot find work, according to Labour Ministry's statistics this month. (Số liệu thống kê báo động về tình trạng thất nghiệp tiếp tục làm cho chúng ta lo lắng. Theo thống kê của Bộ Lao động trong tháng này, có tới 162.000 người có bằng đại học không thể tìm được việc làm.)

B Theo đoạn 4, câu nào là đúng về hệ thống thi cử hiện nay?

    1. Nó đặt quá nhiều áp lực lên những học sinh phải thi đậu đại học.
    2. Học sinh không được khuyến khích làm những điều khác biệt.
    3. Chính phủ đang nỗ lực thay đổi lý thuyết của kì thi.
    4. Nhiều câu chuyện về những học sinh thành công không truyền cảm hứng cho những sinh viên Đại học.

Căn cứ vào thông tin đoạn 4:

She agreed there were many success stories about young people, but added that it was imbalanced if students taking unconventional paths were not also encouraged. (Cô đồng ý rằng có rất nhiều câu chuyện thành công về những người trẻ tuổi, nhưng nói thêm rằng thật là không cân bằng nếu các sinh viên đi theo con đường khác biệt cũng không

được khuyến khích)

A Điều gì có thể suy ra từ đoạn văn?

    1. Đã đến lúc giới trẻ nên thay đổi cách suy nghĩ về thành công.
    2. Điểm cao là bước đầu tiên để đạt được thành công trong tương lai.
    3. Thi rớt đại học sẽ không quyết định được là những học sinh này đang làm những việc khác thường.
    4. Điều đáng quan tâm nhất cho mỗi sinh viên là tình trạng thất nghiệp.

Căn cứ vào thông tin cả bài và đoạn cuối:

Vietnam is, more than ever, in desperate need of those who think outside the box. Time for us to recognise talent, no matter where it comes from or how. (Việt Nam, hơn bao giờ hết, đang rất cần những người suy nghĩ vượt ra khỏi khuôn khổ. Đã đến lúc để chúng ta công nhận tài năng, bất kể nó đến từ đâu hay đến như thế nào.)

Dịch bài Mỗi mùa hè, khi có kết quả kì thi tuyển sinh đại học, nhiều câu chuyện trên báo chí được xuất bản về những học sinh là những thủ khoa trên toàn quốc. Hầu hết chân dung các sinh viên đều chăm chỉ, hiếu học, thông minh và nói chung, xuất thân từ các gia đình có thu nhập thấp. Họ thường được coi là anh hùng hay nữ anh hùng bởi gia đình, xã, làng và cộng đồng của họ. Và họ tượng trưng cho những nỗ lực được thực hiện để đưa họ và người thân của họ thoát khỏi đói nghèo. Các học sinh thường quá nghèo để tham dự bất kỳ lớp học thêm nào, điều này làm cho thành tích của họ

thêm lừng lẫy và đáng chú ý hơn. Trong khi tất cả mọi người nên hoan nghênh các học sinh này vì những nỗ lực đáng ngưỡng mộ của họ, nhấn mạnh quá nhiều vào thành công sẽ dẫn đến một vài câu hỏikhó.

Nếu các học sinh ngưỡng mộ họ như là những tấm gương, tất nhiên điều này là rất tuyệt. Tuy nhiên, theo một cách nào đó, nó góp phần vào thái độ của xã hội rằng vào được đại học là cách duy nhất để thành công. Đối với những người thất bại, cuộc sống của họ đã kết thúc. Cần lưu ý rằng khoảng 1,3 triệu học sinh trung học tham gia kỳ thi tuyển sinh đại học hàng năm và chỉ có khoảng 300.000 em đậu. Thế còn hàng trăm ngàn người thất bại? Chúng ta có nên yêu cầu nhiều

câu chuyện hơn về những người bỏ học đại học ở một mức độ nào đó và làm điều gì đó khác thường không?

“Cá nhân tôi nghĩ rằng thành công không phải là việc bạn đạt điểm cao nhất trong kỳ thi tuyển sinh hoặc thậm chí vào được đại học Harvard. Đó là về những gì bạn làm trong suốt cuộc đời”, Trần Nguyễn Lê Vân, 29 tuổi, cho biết.

Anh là người sáng lập ra trang web, vexere.com, mà hành khách có thể sử dụng để đặt vé xe buýt trực tuyến và nhận vé qua tin nhắn điện thoại. Doanh nghiệp của anh cũng hỗ trợ đặt vé trực tuyến qua điện thoại di động và email. Vân bỏ học cử nhân Quản trị Kinh doanh (MBA) tại Trường Quản lý Toàn cầu Thunderbird ở Arizona, Hoa Kỳ. Câu chuyện của ông đã thu hút được sự chú ý của nhiều tờ báo và ông tin rằng cần phải có nhiều sự dũng cảm hơn cho các bạn trẻ, những người có thể là tấm gương trong cộng đồng khởi nghiệp. Vào được đại học, ngay cả với niềm vinh dự, chỉ là khởi đầu. “Chúng ta hoan nghênh họ và những nỗ lực của họ và rõ ràng là điều này có thể tiếp thêm cho họ động lực để làm tốt hơn trong cuộc sống. Tuy nhiên, thành công đòi hỏi nhiều hơn chỉ là điểm số”, Vân nói. Vân từng phát biểu với một tờ báo rằng nguồn cảm hứng của anh cũng xuất phát từ một trong những người nổi tiếng nhất thế giới, như Mark Zuckerberg của Facebook hay Bill Gates, người cũng bỏ học tại Đại học Harvard.

Số liệu thống kê báo động về tình trạng thất nghiệp tiếp tục làm cho chúng ta lo lắng. Theo thống kê của Bộ Lao động trong tháng này, có tới 162.000 người có bằng đại học không thể tìm được việc làm. Nhấn mạnh vào việc đi học đại học không truyền cảm hứng cho những sinh viên muốn thử các lựa chọn thay thế. Đồng thời, Bộ Giáo dục và Đào tạo vẫn còn cân nhắc về cải cách hệ thống thi cử của chúng ta, trong đó chú trọng lý thuyết, nhưng ít quan tâm phát triển tư duy hoặc chú trọng thực hành. Vũ Thị Phương Anh, nguyên Giám đốc Trung tâm Kiểm định và Đánh giá Chất lượng Giáo dục tại Đại học Quốc gia Hồ Chí Minh cho biết các phương tiện truyền thông cũng nên theo dõi thành công của học sinh sau khi tốt nghiệp. Cô đồng ý rằng có rất nhiều câu chuyện thành công về những

người trẻ tuổi, nhưng nói thêm rằng thật là không cân bằng nếu các sinh viên đi theo con đường khác biệt cũng không được khuyến khích.

Việt Nam, hơn bao giờ hết, đang rất cần những người suy nghĩ vượt ra khỏi khuôn khổ. Đã đến lúc để chúng ta công nhận tài năng, bất kể nó đến từ đâu hay đến như thế nào.

Câu 61: Đáp án C

Chủ đề PEOPLE’S BACKGROUND

Đoạn văn chủ yếu nói về điều gì?

A. Tham vọng của Bùi Tiến Dũng B. Bùi Tiến Dũng và em trai mình

C. Tiểu sử của Bùi Tiến Dũng D. Thành công của Bùi Tiến Dũng

Câu 62: Đáp án D

Điều gì đã làm nên tên tuổi của anh ấy tại giải AFC U23?

  1. Anh ấy từng lần một cản chính xác việc ghi bàn của đối phương.
  2. Anh ấy giúp cho Việt Nam có cơ hội tại giải vô địch này.
  3. Rất nhiều cô gái việt cảm nắng anh ấy.
  4. Cả A và B đều đúng.

Thông tin ở đoạn 1: "Bui Tien Dung made a name for himself in the AFC U23 Championship where he accurately blocked opponents’ shots time after time and saved Vietnam's chance at the championship in the process."

Câu 63: Đáp án A

Từ nào dưới đây có thể được dùng để thay thế cho từ “afford”?

A. mua B. bán C. vứt đi D. đưacho

Căn cứ vào câu: “Dung has loved soccer since he was a kid, but his family was so poor that they could not even afford a plastic soccer ball for him and his brother.” (Dũng đã yêu bóng đá từ khi anh ấy còn là một đứa trẻ nhưng gia đình anh ấy nghèo đến nỗi mà họ thậm chí không thể có đủ tiền để mua cho anh em anh ấy một quả bóng nhựa.)

Câu 64: Đáp án C

Từ nào sau đây có thể được dùng đồng nghĩa với từ "recruitment"?

A. participation (n): sự tham gia B. employment (n): sự thuê mướn

C. selection (n): sự lựa chọn D. graduation (n): sự tốt nghiệp

Căn cứ vào câu sau: "When a big football club in the city announced a recruitment event, Dung convinced his younger brother to go to the tryout." (Khi có một câu lạc bộ bóng đá lớn ở thành phố thông báo một sự kiện tuyển mộ, Dũng đã thuyết phục em trai mình cùng tham gia.)

recruitment = selection: sự lựa chọn, tuyến chọn

Câu 65: Đáp án B

Theo như đoạn văn này, điều gì đã làm gián đoạn sự nghiệp bóng đá của Dũng trong khoảng một năm?

A. Giađình nghèo B. Sự phá sản của trung tâm huấn luyện địa phương

C. Sự phản đối của bố mẹ D. Việc tập luyện của em trai tại thành phố

Căn cứ vào thông tin sau trong đoạn 4: "While his younger brother training in the city, Dung's local training center went bankrupted and abruptly ended Dung’s football career.” (Trong khi em trai của anh ấy tham gia chương trình huấn luyện ở thành phố, trung tâm đào tạo địa phương của anh ấy bị phá sản và bất ngờ chấm dứt sự nghiệp bóng đá của anh ấy.)

Câu 66: Đáp án B

Theo như đoạn văn này, Tất cả những câu dưới đây đều đúng NGOẠI TRỪ:

  1. Dũng phải làm việc như một công nhân xây dựng để phụ giúp gia đình
  2. Dũng thích chơi ở vị trí thủ môn.
  3. Dũng được ghi danh là thủ môn xuất sắc nhất lứa tuổi U19 Việt Nam.
  4. Cả hai anh em họ Bùi đều thi đấu tại giải vô địch AFC U23.

Căn cứ vào thông tin sau: "Reentering the football field, Dung wanted to play in a defense position but he was pushed to goalkeeping because of his height. At first, Dung was unhappy about his new position." (Quay trở lại sân cỏ, Dũng muốn chơi ở vị trí hậu vệ nhưng anh ấy bị bắt buộc chơi ở vị trí thủ môn bởi vì chiều cao của mình. Ban đầu, anh ấy không vui về vị trí mới này.)

Câu 67: Đáp án D

Khi ờ trên sân cỏ, Bùi Tiến Dũng được miêu tả như nào?

  1. Là một chàng trai trẻ dễ mến, người mà luôn tôn trọng người khác và trầm tính.
  2. Là một anh chàng đẹp trai và mãnh mẽ.
  3. Là một chàng trai tốt với rất nhiều fan nữ
  4. Là một người thi đấu quyết liệt và một thủ môn bất bại

Căn cứ vào thông tin ở trong đoạn văn: "Those who have worked with Dung described him as a likable young man. He is very respectful to others and speaks very little. However, when the gloves are on, the nice young man transforms into a fierce competitor and an unyielding goalkeeper." (Những người mà làm việc với Dũng miêu tả anh ấy là một chàng trai trẻ dễ mến. Anh ấy luôn tôn trọng người khác và rất ít nói. Tuy nhiên, khi đã đeo bao tay vào, chàng trai trẻ ấy biến thành một người thi đấu quyết liệt và một thủ môn bất bại.)

Câu 68 Đáp án D

Chủ đề EDUCATION

Chủ đề của đoạn văn là gì?

  1. Học sinh giỏi và học sinh kém
  2. Những người học giỏi và các chiến lược học tập của họ
  3. Kỹ năng học tập dành cho học sinh trung học
  4. Cách học hiệu quả và không hiệu quả Căn cứ vào câu đầu tiên của mỗi đoạn văn :

+ Successful students often do the followings while studying. First ... Next

+ Conversely, students with low academic achievement often demonstrate ineffective study skills.

Câu 69: Đáp án B

Từ "prior" ở đoan đầu tiên có V nahĩa aân nhất với?

A. quan trọng B. trước đó C. chuyển tiếp D. tốt

=> prior = earlier: trước đó

Câu 70: Đáp án D

Cụm từ nào dưới đây có thể thay thế tốt nhất cho từ “Conversely” trong đoạn văn số 2?

On the contrary = In contrast = On the other hand = Conversely: trái lại, trái với

Câu 71: Đáp án A

Theo đoạn văn, điều gì có thể rút ra về những học sinh thụ động?

  1. Họ phụ thuộc vào người khác để tổ chức học tập
  2. Họ chậm trong việc học tập
  3. Họ theo dõi hiểu biết của mình
  4. Họ biết mục tiêu học tập

Thông tin: They tend to assume a passive role, in learning and rely on others (e.g., teachers, parents) to monitor their studying. (Họ thường thụ động trong học tập và dựa vào người khác (ví dụ: giáo viên, phụ huynh) để theo dõi việc học tập của họ.)

Câu 72: Đáp án D

Điều nào sau đây KHÔNG phải là bằng chứng của việc theo dõi học tập?

A. Nhận thức được mục đích học tập B. Theo dõi sự hiểu biết của họ về bài học

C. Xử lý sai lầm trong hiểu biết D. Nhìn vào lưng của họ

Căn cứ vào thông tin sau:

“Low-achieving students often do not monitor their understanding of content; they may not be aware of the purpose of studying; and they show little evidence of looking back, or employing “fix-up” strategies to fix understanding problems.”

(Học sinh yếu thường không theo dõi hiểu biết của mình về bài học; họ có thể không nhận thức được mục tiêu học tập; và họ thường không đọc lại hoặc sử dụng các chiến lược “sửa chữa” để khắc phục các vấn đề về hiểu biết.)

Câu 73: Đáp án B

Theo đoạn văn, để tìm hiểu thông tin mới, học sinh kém KHÔNG .

A. chỉ hiểu nó B. liên hệ nó với những gì họ đã biết

C. chỉ đơn giản là nhớ nó D. đọc nó

Thông tin: “Students who struggle with learning new information seem to be unaware that they must extent effort beyond simply reading the content to understand and remember it.” (Các học sinh gặp khó khăn trong việc tìm hiểu thông tin mới dường như không biết rằng họ phải nỗ lực vượt ra ngoài việc chỉ đọc nội dung để hiểu và nhớ nó.)

Câu 74: Đáp án B

So với các học sinh kém, học sinh giỏi sử dụng .

A. các phương pháp học tập vô nghĩa B. các kỹ năng học tập khác nhau

C. các chiến lược hạn chế D. cách học tập không linh hoạt

Căn cứ vào thông tin sau: “Unlike good studiers who employ a variety of study skills in a flexible yet purposeful manner, low-achieving students use a restricted range of study skills.” (Không giống những

học sinh gỏi, những người sử dụng nhiều kỹ năng học tập một cách linh hoạt nhưng có mục đích, những học sinh kém sử dụng các kỹ năng học tập rất hạn chế.)

Câu 75: Đáp án C

Đại từ được gạch dưới “They” trong câu cuối cùng đề cập đến .

A. các chiến lược học tập B. kỹ năng học tập

C. học sinh yếu D. người học giỏi

“they” được thay thế cho “low-achieving students”

Câu 76: Đáp án A

CHỦ ĐỀ TECHNOLOGY AND YOU

Câu nào trong các câu sau có thể là ý chính của đoạn văn?

  1. Những lợi ích mà Facebook có thể mang lại cho bạn.
  2. Những bất lợi mà Facebook gây ra trong cuộc sống hiện đại.
  3. Những tầng lớp bị ảnh hưởng nhiều nhất bởi Facebook.
  4. Những chức năng được sử dụng nhiều nhất trên Facebook.

Căn cứ vào thông tin đoạn 1:

There also are lots of reports extolling the site's virtues. As the social media giant prepares for its upcoming initial public offering, here are some ways Facebook just

might be good for you. (Có rất nhiều báo cáo đã làm nổi bật được các lợi ích của trang web này. Vì mạng xã hội khổng lồ này chuẩn bị cho đợt chào bán công khai ban đầu sắp tới, dưới đây 1 số lợi ích mà Facebook có thể mang lại cho bạn.)

Câu 77: Đáp án A

Từ "sparked" trong đoạn 2 có thể có nghĩa là .

A. tạo ra B. giảm xuống C. ngăn cản D. kiểm soát Từ đồng nghĩa: sparked (sinh ra, tạo ra) = produce

In a study published earlier this year, researchers studied 30 students and found that a natural high was sparked when they were on the social media network that led to the relaxed heart rates and lower levels of stress and tension. (Trong một nghiên cứu được công bố đầu năm nay, các nhà nghiên cứu đã nghiên cứu 30 sinh viên và thấy rằng một sự thích thú tự nhiên được tạo ra khi họ ở trên mạng truyền thông xã hội dẫn đến nhịp tim thoải mái và mức độ căng thẳng thấp hơn.)

Câu 78: Đáp án C

Theo đoạn 2, các nhà khoa học đã phát hiện ra điều gì trong nghiên cứu của họ?

  1. Sử dụng quá nhiều thời gian trên Facebook có thể làm tăng mức độ áp lực và bệnh tim mạch.
  2. Học sinh thường chịu áp lực khi họ làm việc với những thứ có liên quan tới Toán.
  3. Facebook có thể khiến người dùng lạc quan và kích thích hơn.
  4. Thành công của Facebook có thể thúc đẩy sự phát triển của các mạng xã hội khác.

Căn cứ vào thông tin đoạn 2:

In the study, the students were monitored in three situations: looking at panoramic landscapes, performing complicated mathematical equations and using Facebook. While the first situation was the most relaxing to students and the math problems were the most stressful. the time on Facebook uncovered high levels of

attractiveness and arousal. The findings support the researchers' hypothesis that Facebook's success, as well as that of other social media networks, correlates to the specific positive mental and physical state users experience.

(Trong nghiên cứu, các sinh viên được theo dõi trong ba tình huống: nhìn ngắm toàn cảnh phong cảnh, thực hiện các phương trình toán học phức tạp và sử dụng Facebook. Trong khi tình huống đầu tiên là thư giãn nhất cho sinh viên và các vấn đề toán học là căng thẳng nhất, thời gian trên Facebook lại tạo ra mức độ hấp dẫn và sự kích thích cao. Những phát hiện này ủng hộ giả thuyết của các nhà nghiên cứu rằng sự thành công của Facebook, cũng như của các mạng truyền thông xã hội khác, tương quan với trải nghiệm tích cực cụ thể về tâm thần và thể chất của người dùng.)

Câu 79: Đáp án B

Từ “one” trong đoạn 3 đề cập đến từ nào?

A. nghiên cứu B. một nhóm C. Internet D. nghiên cứu Từ "one” thay thế cho 1 nhóm trong câu phía trước.

The study examined workers in three groups: one that was allowed no breaks, one that was allowed to do anything but use the Internet and one that was allowed 10 minutes to use the Internet and Facebook. (Nghiên của này khảo sát công nhân trong ba nhóm: một nhóm không được phép nghỉ, một nhóm được phép làm bất cứ điều gì trừ sử dụng internet và một nhóm được phép sử dụng internet và Facebook trong 10 phút.)

Câu 80: Đáp án B

Tại sao Facebook lại thúc đẩy năng suất của các công nhân?

  1. Bởi vì Internet làm người dùng xao nhãng để nghỉ giải lao.
  2. Bởi vì nó giúp mọi người tỉnh táo đầu óc và sau đó nâng cao sự tập trung vào công việc.
  3. Bởi vì các công nhân hạnh phúc với những gì họ đọc được trên Facebook.
  4. Bởi vì họ được phép thư giãn sau 1 ngày làm việc vất vả.

Căn cứ thông tin đoạn 3:

The Facebook group was found to be 16 percent more productive than the group that was not allowed to use the Internet and nearly 40 percent more productive than the group that was allowed no

breaks. "Short and unobtrusive breaks, such as a quick surf on the Internet, enables the mind to rest itself, leading to a higher net total concentration for a day‘s work, and as a result, increased productivity."

(Nhóm sử dụng Facebook được nhận thấy có hiệu suất cao hơn 16% so với nhóm không được phép sử dụng internet và gần 40% hiện quả hơn nhóm không được phép nghỉ giải lao. “Nghỉ giải lao ngắn và nhẹ nhàng, chẳng hạn như Emit web nhanh trên internet cho phép tâm trí tự nghỉ ngơi, dẫn đến sự tập trung cao hơn cho công việc trong một ngày, và kết quả là, tăng năng suất”.)

Câu 81: Đáp án A

Từ "courting” trong đoạn cuối gần nghĩa nhất với từ

A. hẹn hò, tán tỉnh B. hôn nhân C. li hôn D. làm quen Từ đồng nghĩa: courting [tan tỉnh] = dating

Once the courting is over, nearly 40 percent of those social networking adults will update their relationship status on Facebook, with just 24 percent telling their friends first. (Khi sự tán tỉnh đã kết thúc, gần 40% những người lớn sử dụng mạng xã hội sẽ cập nhật trạng thái mối quan hệ của họ trên Facebook, chỉ với 24% người sử dụng sẽ nói với bạn bè của họ trước tiên.)

Câu 82: Đáp án C

Câu nào trong các câu sau đây là không đúng về sự tác động của Facebook lên mối quan hệ của những người sử dụng?

  1. Hơn một nửa những người độc thân có khuynh hướng kết bạn mới sau khi gặp mặt trực tiếp.
  2. Những người đã kết hôn thường sử dụng Facebook để giữ liên lạc với vợ chồng mình.
  3. Không nhiều người thay đổi trạng thái mối quan hệ của họ sang độc thân sau khi đổ vỡ mối quan hệ.
  4. Những người trưởng thành thường cập nhật trạng thái mối quan hệ trên Facebook hơn là kể với bạn bè của họ về mối quan hệ đó.

Căn cứ vào thông tin đoạn cuối:

Facebook is also in the business of matchmaking. Research shows that nearly 60 percent of singles will friend someone new on Facebook after meeting them in person. If they like what they see, 25 percent are likely to contact their new love interest via Facebook. Once the courting is over, nearly 40 percent of those social networking adults will update their relationship status on Facebook, with just 24 percent telling their friends first. Facebook use between couples will continue through the dating process, the research shows. Throughout the day, 79 percent of couples said they send partners Facebook messages or chat on the social network. In addition, more than 60 percent would post romantic messages on their significant other’s Facebook wall. When the relationship ends, more than half of those surveyed immediately update their status to single, which automatically sends out a notification to their friend list to start the dating cycle over again.

(Facebook cũng kì một lĩnh vực mai mối. Nghiên cứu cho thấy gần 60% người độc thân sẽ kết bạn với một người mới trên Facebook sau khi gặp họ trực tiếp. Nếu họ thích những gì họ thấy, 25% có khả năng liên hệ với mối quan tâm tình yêu mới của họ thông qua Facebook. Khi sự tán tỉnh đã kết thúc, gần 40% những người lớn sử dụng mạng xã hội sẽ cập nhật trạng thái mối quan hệ của họ trên Facebook, chỉ với 24% người sử dụng sẽ nói với bạn bè của họ trước tiên. Các nghiên cứu cho thấy việc sử dụng Facebook giữa các cặp vợ chồng sẽ tiếp tục thông qua quá trình hẹn hò. Trong suốt cả ngày, 79% các cặp vợ chồng cho biết họ gửi tin nhắn Facebook cho người bạn đời hoặc trò chuyện trên mạng xã hội. Ngoài ra, hơn 60 phần trăm sẽ đăng thông điệp lãng mạn trên tường Facebook của người quan trọng của họ. Khi mối quan hệ kết thúc, hơn một nửa số người được khảo sát ngay lập tức cập nhật trạng thái của họ thành độc thân, tự động gửi thông báo đến danh sách bạn bè của họ để bắt đầu lại cho kỳ hẹn hò.) Câu 83: Đáp án A

Câu nào trong các câu sau miêu tả chính xác nhất giọng văn của bài đọc?

A. cung cấp thông tin B. tranh luận C. chế nhạo D. mỉa mai

Căn cứ vào nội dung cả bài:

Tác giả cung cấp những thông tin lí giải vì sao Facebook lại rất có ích cho chúng ta. Vì vậy, giọng điệu của bài đọc là cung cấp thông tin.

Dịch bài

Người dùng Facebook dành trung bình hơn 15 giờ một tháng trên trang mạng xã hội. Trong khi có rất nhiều người phản đối việc sử dụng quá nhiều như vậy - và có một số nghiên cứu chi tiết về sự tổn hại mà Facebook có thể gây ra - có rất nhiều báo cáo đã làm nổi bật được các lợi ích của trang web này. Vì mạng xã hội khổng lồ này chuẩn bị cho đợt chào bán công khai ban đầu sắp tới, dưới đây là một số lợi ích mà Facebook có thể mang lại cho bạn, Dành thời gian trên Facebook có thể giúp mọi người thư giãn, làm chậm nhịp tim và giảm mức độ căng thẳng. Theo các nhà nghiên cứu từ Viện Công nghệ Massachusetts và Đại học Milan. Trong một nghiên cứu được công bố đầu năm nay, các nhà nghiên cứu đã nghiên cứu 30 sinh viên và thấy rằng một sự thích thủ tự nhiên đã được tạo ra khi họ ở trên mạng truyền thông xã hội dẫn đến nhịp tim thoải mái và mức độ căng thẳng thấp hơn. Trong nghiên cứu, các sinh viên được theo dõi trong ba tình huống: nhìn ngắm toàn cảnh phong cảnh, thực hiện các phương trình toán học phức tạp và sử dụng Facebook. Trong khi tình huống đầu tiên là thư giãn nhất cho sinh viên và các vấn đề toán học là căng thắng nhất, thời gian trên Facebook lại tạo ra mức độ hấp dẫn và sự kích thích cao. Những phát hiện này ủng hộ giả thuyết của các nhà nghiên cứu rằng sự thành công của Facebook, cũng như của các mạng truyền thông xã hội khác, tương quan với trải nghiệm tích cực cụ thể về tâm thần và thể chất của người dùng.

Trong khi nhiều người cho rằng các mạng truyền thông xã hội chỉ làm sao nhãng nhân viên, thì nghiên cứu cho thấy điều ngược lại có thể đúng. Nghiên cứu từ Keas.com phát hiện ra rằng 10 phút nghỉ ngơi trên Facebook khiến cho nhân viên hạnh phúc hơn, khỏe mạnh và hiện quả hơn. Nghiên cứu này khảo sát công nhân trong ba nhóm: một nhóm không được phép nghỉ, một nhóm được phép làm bất cứ điều gì trừ sử dụng Internet và một nhóm được phép sử dụng Internet và Facebook trong 10 phút. Nhóm sử dụng Facebook được nhận thấy có hiệu suất cao hơn 16% so với nhóm không được phép sử dụng Internet và gần 40% hiện quả hơn nhóm không được phép nghỉ giải lao. "Nghỉ giải lao ngắn và nhẹ nhàng, chẳng hạn như lướt web nhanh trên internet, cho phép tâm trí tự nghỉ ngơi, dẫn đến sự tập trung cao hơn cho công việc trong một ngày, và kết quả là, tăng năng suất”, Brent Coker thuộc khoa quản lý và tiếp thị tại Đại học Melbourne ở Úc cho biết.

Facebook cũng là một lĩnh vực mai mối. Nghiên cứu cho thấy gần 60 % người độc thân sẽ kết bạn với một người mới trên Facebook sau khi gặp họ trực tiếp. Nếu họ thích những gì họ thấy, 25 % có khả năng liên hệ với mối quan tâm tình yêu mới của họ thông qua Facebook. Khi sự tán tỉnh đã kết thúc, gần 40% những người lớn sử dụng mạng xã hội sẽ cập nhật trạng thái mối quan hệ của họ trên Facebook, chỉ với 24% người sử dụng sẽ nói với bạn bè của họ trước tiên. Các nghiên cứu cho thấy việc sử dụng Facebook giữa các cặp vợ chồng sẽ tiếp tục thông qua quá trình hẹn hò. Trong suốt cả ngày, 79% các cặp vợ chồng cho biết họ gửi tin nhắn Facebook cho người bạn đời hoặc trò chuyện trên mạng xã hội. Ngoài ra. hơn 60% sẽ đăng thông điệp lãng mạn trên tường Facebook của người quan trọng của họ. Khi mối quan hệ kết thúc, hơn một nửa số người

được khảo sát ngay lập tức cập nhật trạng thái của họ thành độc thân, tự động gửi thông báo đến danh sách bạn bè của họ để bắt đầu lại chu kỳ hẹn hò.

Câu 84: Đáp án B

Đoạn văn chủ yếu thảo luận về vấn đã gì?

  1. Các chất ô nhiễm gây ô nhiễm môi trường tự nhiên như thế nào.
  2. Nguyên nhân và ảnh hưởng của ô nhiễm không khí.
  3. Hệ sinh thái của chúng ta và sự cân bằng trong môi trường.
  4. Một số giải pháp cho vấn đề ô nhiễm môi trường.

Căn cứ vào câu: Let us discuss the causes and effects of air pollution on mankind and the environment as a whole.

(Hãy để chúng tôi thảo luận về nguyên nhân và ảnh hưởng của ô nhiễm không khí đối với nhân loại và môi trường nói chung)

Câu 85: Đáp án A

Từ “these" trong đoạn văn thứ hai ám chỉ .

A. các chất hóa học B. các hoạt động công nghiệp

  1. cuộc sống hàng ngày của chúng ta
  2. việc đốt cháy nhiên liệu quá mức cần thiết để nấu ăn, lái xe

Căn cứ vào câu: Excessive burning of fuel which is a necessity of our daily lives for cooking, driving and other industrial activities; releases a huge amount of chemical substances in the air every day; these pollute the air.

(Việc đốt cháy nhiên liệu quá mức cần thiết cho cuộc sống hàng ngày của chúng ta để nấu ăn, lái xe và các hoạt động công nghiệp khác; giải phóng một lượng lớn chất hóa học trong không khí hàng ngày; những chất này làm ô nhiễm không khí.)

Câu 86: Đáp án C

Có bao nhiêu loại bệnh được đề cập trong đoạn văn?

A. 5 B. 6 C. 7 D. 8

Căn cứ vào đoạn văn thứ 4: The decrease in quality

of air leads to several respiratory problems

including asthma or lung cancer. Chest pain, congestion, throat inflammation, cardiovascular disease, respiratory disease are some of diseases that can be caused by air pollution.

(Việc giảm chất lượng không khí dẫn đến một số vấn đề hô hấp bao gồm hen suyễn hoặc ung thư phổi. Đau ngực, nghẹt mũi, viêm cổ họng, bệnh tim mạch, bệnh hô hấp là một số bệnh có thể có nguyên nhân do ô nhiễm không khí.)

Câu 87: Đáp án B

Từ “respiratory” trong đoạn 4 gần nghĩa nhất với từ nào?

A. có liên quan đến việc tiêu hóa thức ăn B. có liên quan đến việc hít thở

  1. có liên quan đến bộ xương của con người
  2. có liên quan đến não của con người

Căn cứ vào câu: The decrease in quality of air leads to several respiratory problems including asthma or lung cancer.

(Việc giảm chất lượng không khí dẫn đến một số vấn đề hô hấp bao gồm hen suyễn hoặc ung thư phổi.)

Câu 88: Đáp án D

Theo như đoạn văn thứ 5,6 ô nhiễm môi trường góp phần gây ra hiệu ứng nhà kính mà có thể dẫn tới .

A. việc trái đất nóng lên B. tan băng ở cực

  1. sự gia tăng khí CO2 trong môi trường
  2. việc trái đất nóng lên, tan bang ở cực, mực nước biển dâng và nguy hiểm cho con người.

Căn cứ vào các câu: The emission of greenhouse gases particularly CO2 is leading to global warming.

The increase in CO2 leads to melting of polar ice caps which increases the sea level and pose danger for the people living near coastal areas.

(Sự phát thải khí nhà kính đặc biệt khí CO2 đang dẫn đến sự nóng lên toàn cầu.

Sự gia tăng CO2 dẫn đến sự tan chảy của các chỏm băng cực làm tăng mực nước biển và gây nguy hiểm cho những người sống gần các khu vực ven biển)

Câu 89: Đáp án A

Theo như đoạn văn câu nào sau đây không đúng?

  1. Tầng Ozone không thể ngăn tia cực tím chiếu vào trái đất được nữa.
  2. Không khí độc hại bị gây ra bởi khi SO2, khí mà được thải ra từ việc đốt than.
  3. Khi lượng CO2 trong không khí tăng lên, nó ngăn cản thực vật trong quá trình quang hợp.
  4. Ngoài ô nhiễm không khí, vẫn còn những loại ô nhiễm khác nhau khác.

Căn cứ vào câu: Ozone layer is the thin shield high up in the sky that stops ultra violet rays from reaching the earth. As a result of human activities. chemicals, such as chlorofluorocarbons (CFCS), were released into the atmosphere which contributed to the depletion of ozone layer.

(Lớp ôzôn là lá chắn mỏng coo lên trên bầu trời ngăn tia cực tím chiếu vào trái đất. Bởi vì các hoạt động của con người, các hóa chất, chẳng hạn như chlorofluorocorbons (CFCS), được giải phóng vào khí quyển góp phần làm suy giảm tầng Ôzôn)

Câu 90: Đáp án A.

Có thể suy ra điều gì từ đoạn văn?

  1. Đời sống của chúng ta càng trở nên hiện đại, vấn đề ô nhiễm càng trở nên nghiêm trọng.
  2. Con người có thể sớm kiểm soát vấn đề ô nhiễm không khí trong tương lai.
  3. Chỉ có ô nhiễm không khi mới nguy hiểm và có những ảnh hưởng tiêu cực đến con người và môi trường.
  4. Khí CO2 là nguyên nhân của việc suy giảm tầng ozone.

Tạm dịch

Ô nhiễm môi trường là một trong những tác động của hoạt động con người vào trái đất. Ô nhiễm xảy ra khi các chất ô nhiễm gây ô nhiễm môi trường tự nhiên; mang lại những thay đổi ảnh hướng đến lối sống bình thường của chúng ta. Ô nhiễm gây rối loạn hệ sinh thái của chúng ta và sự cân bằng trong môi trường. Ô nhiễm xảy ra ở các dạng khác nhau; không khí, nước, đất, phóng xạ, tiếng ồn, sức nóng/ nhiệt và ánh sáng. Hãy để chúng tôi thảo luận về nguyên nhân và ảnh hướng của ô nhiễm không khí đối với nhân loại và môi trường nói chung.

Ô nhiễm không khí là dạng ô nhiễm nguy hiểm và nổi bật nhất. Nó xảy ra do nhiều lý do. Việc đốt cháy nhiên liệu quá mức cần thiết cho cuộc sống hàng ngày của chúng ta để nấu ăn, lái xe và các hoạt động công nghiệp khác; giải phóng một lượng lớn chất hóa học trong không khí hàng ngày; những chất này làm ô nhiễm không khí. Khói từ các ống khói, nhà máy, xe cộ hoặc đốt gỗ về cơ bản xảy ra do

đốt than; điều này giải phóng sulfur dioxide vào không khí làm cho nó độc hại.

Các tác động của ô nhiễm không khí cũng rất hiển nhiên. Sự gia tăng lượng CO2 trong khí quyển dẫn đến khói bụi có thế hạn chế ánh sáng mặt trời chiếu vào trái đất. Do đó, ngăn chặn thực vật trong quá trình quang hợp. Các loại khí như sulphur dioxide và nitơ oxit có thể gây ra mưa axit.

Việc giảm chất lượng không khí dẫn đến một số vấn đề hô hấp bao gồm hen suyễn hoặc ung thư phổi. Đau ngực, nghẹt mũi, viêm cổ họng, bệnh tim mạch, bệnh hô hấp là một số bệnh có thể là nguyên nhân gây ô nhiễm không khí.

Sự phát thải khí nhà kính đặc biệt là CO2 đang dẫn đến sự nóng lên toàn cầu. Mỗi ngày các ngành công nghiệp mới đang được thiết lập, xe mới đi trên đường và cây cối bị chặt để nhường chỗ cho những ngôi nhà mới. Tất cả chúng, theo cách trực tiếp hoặc gián tiếp đều dẫn đến gia tăng CO2 trong môi trường. Sự gia tăng CO2 dẫn đến sự tan chảy của các chỏm băng cực làm tăng mực nước biển và gây nguy hiểm cho những người sống gần các khu vực ven biển.

Lớp ôzôn là lá chắn mỏng trên bầu trời cao ngăn tia cực tím chiếu vào trái đất. Bởi vì các hoạt động của con người, các hóa chất, chẳng hạn như chlorot1uorocarhons (CFCs), được giải phóng vào khí quyển góp phần làm suy giảm tầng ôzôn.

Câu 91: Đáp án B CHỦ ĐỀ về JOBS

Câu nào trong các câu sau có thể là nội dung chính của đoạn văn?

  1. Những ngành nghề phổ biến nhất trong thị trường việc làm ở Việt Nam.
  2. Sự cần thiết của ngoại ngữ trong hầu hết các công việc kĩ thuật.
  3. Những kĩ năng cần trong các công việc kĩ thuật ngày nay.
  4. Sinh viên Việt Nam không nhận thức được tầm quan trọng của việc học ngoại ngữ.

Căn cứ vào đoạn đầu và nội dung xuyên suốt bài đọc:

Tác giả đề cập đến sự cần thiết của kĩ năng sử dụng ngoại ngữ trong hầu hết các công việc kĩ thuật.

Câu 92: Đáp án C

Theo đoạn văn, phiên dịch và biên dịch viên được mô tả là những công việc mà .

    1. đang giảm mạnh về số lượng nhân viên.
    2. không đủ nhân viên cho các công ty công nghệ tuyển dụng.
    3. nhu cầu đang tăng lên đáng kể và khá ổn định.
    4. được dự kiến là sẽ có xu hướng giảm dần trong tương lai gần.

Căn cứ thông tin đoạn 2:

In just the last two years the demand for tech professionals with foreign language skills has increased more than two and one-half fold, said the survey, and the uptick shows no signs of abating anytime soon. (Cuộc khảo sát cho biết rằng chỉ trong vòng hai năm qua, nhu cầu về các chuyên gia công

nghệ có kỹ năng ngoại ngữ đã tăng hơn hai lần rưỡi, và sự tăng vọt vẫn chưa có dấu hiệu giảm xuống sớm.)

Câu 93: Đáp án A

Từ “upheavals” trong đoạn 3 gần nghĩa nhất với từ

  1. sự chuyển đổi theo hướng tăng lên
  2. sự gián đoạn theo hướng tăng lên
  3. những sự thay đổi theo hướng giảm xuống
  4. những vấn đề theo hướng giảm xuống

Từ đồng nghĩa: upheavals (bước dịch chuyển, bước đột biến) = upward displacements

While that claim might seem a bit overblown (and amounts to little more than a guess by Tuan], it is clear that innovative technologies like robotics, 3D printing. drones, artificial intelligence and virtual reality will create major upheavals in all sorts of labor markets, not just technology over the next few years. (Trong khi tuyên bố đó có vẻ hơi bị thổi phồng (và có giá trị hơn một chút so với sự suy đoán của Tuấn), rõ ràng là các công nghệ tiên tiến như robot, in 3D, máy bay, trí thông minh nhân tạo và thực tế ảo sẽ tạo ra những biến động khi trong tất cả các loại thị trường lao động, không chỉ riêng công nghệ trong vài năm tới.)

Câu 94: Đáp án A

Câu nào là đúng về tình trạng việc làm ở Việt Nam theo Trần Quang Anh, công tác tại Viện Công nghệ Bưu chính Viễn thông?

  1. Những người mà có chuyên ngành ngoại ngữ là về công nghệ cao thường được trả lương cao.
  2. Nhu cầu của nghề phiên dịch và biên dịch viên trong lực lượng lao động thì không nhiều như những gì người ta thường nghĩ.
  3. Không chỉ công nghệ mà các nghề khác cũng đang đặt áp lực về kĩ năng ngôn ngữ.
  4. Ngoại ngữ ở Việt Nam chỉ cần trong các bài viết đăng trên websites.

Căn cứ vào thông tin đoạn 4:

The good paying jobs with high salaries and benefits are only available to

translators and interpreters who specialize in high tech jobs, says Anh. (Các công việc tốt với mức lương cao và nhiều chế độ đãi ngộ chỉ có sẵn cho những biên dịch và thông dịch viên chuyên về các công việc công nghệ cao, Anh nói.)

Câu 95: Đáp án C

Từ "them ” trong đoạn 4 để cập tới

A. các ngoại ngữ B. một vài cụm từ cơ bản

C. các sinh viên Việt Nam D. các cuộc khảo sát Từ "them” trong đoạn 4 đang nhắc tới các sinh viên Việt Nam.

Unfortunately, the surveys Show that most graduating Vietnamese students are unable to do more than understand a few basic phrases of foreign languages, and practically none of them can speak any foreign language coherently. (Thật không may, các cuộc khảo sát cho thấy hầu hết sinh viên Việt Nam tốt nghiệp chỉ có thể hiểu một vài cụm từ ngoại ngữ cơ bản, và thực tế không ai trong sống có thể nói bất kỳ ngoại ngữ nào một cách mạch lạc.)

Câu 96: Đáp án B

Từ “requisite” trong đoạn 5 có thể được thay thể tốt nhất bằng từ .

A. thiếu B. sự cần thiết

C. đặc điểm kĩ thuật D. sự đo lường Từ đồng nghĩa: requisite (điều kiện cần thiết) = necessity

The job applicants were young and industrioas, said the recruiters. However, missing were candidates with the requisite language skills and most lacked basic ‘soft skills’ such as written and verbal communication abilities to effectively communicate even in their native Vietnamese language. (Các nhà tuyển dụng cho biết những người nộp đơn xin việc đều trẻ và siêng năng. Tuy nhiên, lại thiếu các ứng cử viên với các kỹ năng ngôn ngữ cần thiết và hầu hết thiếu các kỹ năng mềm cơ bản như khả năng giao tiếp bằng lời nói và văn bản để giao tiếp hiệu quả ngay cả trong tiếng mẹ đẻ của họ.)

Câu 97: Đáp án B

Điều gì có thể suy ra từ đoạn văn?

  1. Khả năng học tập thì quan trọng hơn rất nhiều so với kĩ năng ngoại ngữ và kĩ năng mềm.
  2. Để có được một công việc tốt ngày nay, những người xin việc cần phải biết nhiều hơn một ngôn ngữ và giỏi các kĩ năng mềm khác.
  3. Khả năng giao tiếp bằng ngôn ngữ viết và nói xác định cơ hội của các ứng cử viên để có được công việc mơ ước.
  4. Biên dịch và phiên dịch viên thì phù hợp cho bất kì vị trí nào trong thị trường việc làm.

Căn cứ vào thông tin cả bài:

Những người thông dịch và biên dịch có chuyên ngành kĩ thuật cao thường được trả lương cao và nhiều chế độ đãi ngộ, trong khi không chỉ công nghệ mà hầu hết các ngành khác đều yêu cầu khả năng ngoại ngữ và các kĩ năng mềm khác.

Câu 98: Đáp án C

Từ nào sau đây thể hiện đúng nhất thái độ của tác giả về việc cải thiện khả năng ngoại ngữ?

A. trung lập B. nghi ngờ C. ủng hộ D. nhạy cảm

Căn cứ theo thông tin cả bài:

Tác giả đề cập đến rất nhiều nhu cầu tuyển dụng cho các ứng cử viên có khả năng về ngoại ngữ, đồng thời cũng chỉ ra rằng những sinh viên Việt Nam ra trường rất yếu về các kĩ năng này. Do đó, tác giả rất ủng hộ việc cải thiện khả năng ngoại ngữ.

Dịch bài

Các nhà dịch thuật và phiên dịch cho các công việc công nghệ cao trong tương lai dự kiến sẽ là một trong những ngành nghề phát triển nhanh nhất trên toàn quốc, theo một khảo sát vừa được công bố bởi Vietnamworks. Hầu như tất cả các vị trí cho lập trình viên, nhà phát triển ứng dụng, quản trị viên mạng và cơ sở dữ liệu, kỹ sư, nhà thiết kế, kiến trúc sư, nhà khoa học, kỹ thuật viên và hỗ trợ kỹ thuật sẽ yêu cầu thông thạo song ngữ hoặc đa ngôn ngữ.

Cuộc khảo sát cho biết rằng chỉ trong vòng hai năm qua, nhu cầu về các chuyên gia công nghệ có kỹ năng ngoại ngữ đã tăng hơn hai lần rưỡi, và sự tăng vọt vẫn chưa có dấu hiệu giảm xuống sớm. Khoảng 400.000 việc làm được dự kiến sẽ mở cho phiên dịch viên (tập trung vào ngôn ngữ nói) và dịch giả (tập trung vào ngôn ngữ viết) trong lĩnh vực công nghệ, từ năm 2017 đến năm 2020, Trần Anh Tuấn nói. Tuấn, người làm việc cho Trung tâm dự báo nhu cầu nhân lực và thông tin thị trường lao động tại thành phố Hồ Chí Minh không bao gồm các ngành khác trong dự đoán của mình, cũng đang tuyển mộ những người có cùng kỹ năng ngôn ngữ này.

Trong khi tuyên bố đó có vẻ hơi bị thổi phồng (và có giá trị hơn một chút so với sự suy đoán của Tuấn), rõ ràng là các công nghệ tiên tiến như robot, in 3D, máy bay, trí thông minh nhân tạo và thực tế ảo sẽ tạo ra những biến động lớn trong tất cả các loại thị trường lao động, không chỉ riêng công nghệ trong vài năm tới. Chỉ trong tháng vừa qua, hầu hết mọi công việc được đăng trên các trang web việc làm trên khắp Việt Nam đều sử dụng song ngữ. Mức lương cao hơn cho những người làm việc ở các vị trí công nghệ cao và có thế nói được một ngoại ngữ như tiếng Anh ngoài tiếng Việt, Trần Quang Anh thuộc Viện Công nghệ Bưu chính Viễn thông cho biết.

Thật không may, các cuộc khảo sát cho thấy hầu hết sinh viên Việt Nam tốt nghiệp chỉ có thể hiểu một vài cụm từ ngoại ngữ cơ bản, và thực tế không ai trong số họ có thế nói bất kỳ ngoại ngữ nào một cách mạch lạc. Các công việc tốt với mức lương cao và nhiều chế độ đãi ngộ chỉ có sẵn cho những biên dịch và thông dịch viên chuyên về các công việc công nghệ cao, Anh nói. Nhưng nó không chỉ là tiếng Anh - sinh viên tốt nghiệp cần thông thạo các ngôn ngữ Trung Đông như tiếng Ả Rập, Farsi và Pashto (Afghanistan) cũng như tiếng Đức, tiếng Nhật và tiếng Hàn. Tiếng Tây Ban Nha cũng có nhu cầu cao ở Việt Nam, chủ yếu là vì nó là ngôn ngữ phổ biến thứ hai ở Mỹ sau tiếng Anh.

Một cuộc triển lãm công nghệ gần đây tại Hà Nội do Vietnamworks và Tập đoàn Navigos tài trợ đã thu hút gần 4.000 sinh viên mới tốt nghiệp ngành công nghệ và nhà tuyển dụng từ 14 công ty

hàng đầu đang tìm kiếm vị trí tuyển dụng với công nhân song ngữ có tay nghề cao. Các nhà tuyển dụng cho

biết những người nộp đơn xin việc đều trẻ và siêng năng. Tuy nhiên, lại thiếu các ứng cử viên với các kỹ năng ngôn ngữ cần thiết và hầu hết thiếu các kỹ năng mềm cơ bản như khả năng giao tiếp bằng lời nói và văn bản để giao tiếp hiệu quả ngay cả trong tiếng mẹ đẻ của họ.

Đáng chủ ý, các nhà tuyển dụng cho biết họ coi khả năng ngôn ngữ và kỹ năng mềm giống như, nếu không muốn nói là quan trọng hơn cả khả năng học tập. Tuy nhiên, hầu như tất cả các nhân viên có trình độ học vấn tiềm năng đều thiếu thậm chí cơ bản nhất về khả năng giao tiếp giữa các cá nhân.

Câu 99: Đáp án D

CHỦ ĐỀ COMPETITIONS

Đâu sẽ là tiêu để thích hợp nhất cho bài viết?

A. Nhóm nhạc nổi tiếng thăm Trường B. Giáo viên dạy tiết học âm nhạc

C. Học sinh vui khi đi dã ngoại D. Trường tổ chức cuộc thi ban nhạc học sinh

Thông tin đầu tiên của đoạn 1:

“On March 15, Dunes View Middle School held a contest for school bands.”

(Vào ngày 15 tháng 3, trường Trung học Dunes View tổ chức cuộc thi cho các ban nhạc trưởng). Các đoạn sau là chi tiết về cuộc thi này.

Câu 100: Đáp án B

Điều gì có thể đúng đối với những buổi dã ngoại trong những năm gần đây?

A. Chúng được tổ chức vào tháng ba. B. Chúng không phải là rất thú vị.

C. Chúng rất vất vả để tổ chức. D. Chúng bao gồm các buổi biểu diễn âm nhạc

Thông tin ở đoạn 2:

“The picnic is one of the biggest events of the year, but it has become a hit formulaic,” said Ms. Cho. “The activities are the same every year. We thought that a performance by a student band would make the school picnic more interesting and fun."(“Cuộc picnic là một trong những sự kiện lớn nhất của năm nhưng nó đã dần trở nên tẻ nhạt”, cô Cho nói. “Những hoạt động như thế cứ năm nào cũng như năm nào. Chúng tôi đã nghĩ rằng một màn biểu diễn bởi một ban nhạc trong trường sẽ làm cuộc picnic thú vị và vui vẻ hơn”)

Câu 101: Đáp án D

Từ “Tormuioic” có ý nghĩa gần nhất với .

A. đắt B. không chắc chắn C. dài D. buồn tẻ, chán; đơn điệu

Formulaic  dull: theo công thức, đơn điệu, tẻ nhạt

Câu 102: Đáp án A

Loại ban nhạc KHÔNG được để cập trong bài viết?

A. Pop B. Jazz C. Folk D. Rock

Thông tin ở câu gần cuối đoạn 2:

“The bands varied in their musical forms: there were several rock hands, a folk band, and even a jazz band.(Những ban nhạc này có sự đa dạng về chủng loại: có một vài ban nhạc rock, nhạc dân gian và một nhóm nhạc Jazz)

=> Chỉ có Pop là không được nhắc đến

Câu 103: Đáp án B

Tại sao các giáo viên yêu cầu mỗi ban nhạc chơi hai bài hát?

  1. Để quyết định xem bài hát nào sẽ được chơi tại buổi dã ngoại.
  2. Để xem liệu các ban nhac có thể chơi nhiều bài hát.
  3. Để đảm bảo cuộc thi không diễn ra quá lâu.
  4. Làm cho buổi hòa nhạc thú vị hơn cho sinh viên.

Thông tin ở câu đầu của đoạn 3:

"To see if the bands could present a wide range of musical skills, the teachers asked them each to prepare two songs" (Để xem liệu các ban nhạc có thể trình bày một loạt các kỹ năng âm nhạc, các giáo viên yêu cầu họ chuẩn bị hai bài hát)

Câu 104: Đáp án C

Theo bài viết, điều gì là không bình thường về ban nhạc Four Square?

  1. Nó được đặt tên theo một trò chơi của trẻ em phổ biến.
  2. Các thành viên luyện tập vài lần một tuần.
  3. Là một ban nhạc rock với một nghệ sĩ vi-ô-lông.
  4. Các thành viên đầu chơi ghita.

Thông tin ở đoạn 3:

“Four Square is a rock band with an unusual twist: it includes a violin player!”

(Four Square là một ban nhạc rock với một sự thay đổi bất thường: nó bao gồm một người chơi vi-ô- lông!)

Câu 105: Đáp án B

Theo bài viết. ban nhạc Four Square mong muốn làm gì trong tương lai?

  1. Biểu diễn tại dã ngoại trường học mỗi năm.
  2. Biểu diễn ở nhiều nơi khác nhau.
  3. Mua dụng cụ mới.
  4. Thu một album.

Thông tin ở câu cuối của đoạn 1:

“And since we hope to perform someday at other local places, like restaurants and parks, this will be a great first step.” (Và vì chúng tôi hi vọng một ngày nào đó sẽ được biểu diễn ở những địa điểm khác như là ở trong những nhà hàng hay là công viên chẳng hạn nên đây sẽ là bước đệm đầu tiên.)

Dịch bài

Vào ngày 15 tháng 3, trường trung học Dunes View đã tổ chức một cuộc thi cho các ban nhạc của trường. Các ban nhạc trong trường đã thi đấu với nhau để giành được cơ hội biểu diễn trong cuộc picnic của nhà trường vào cuối của tháng 6. Đội chiến thắng của cuộc thi là ban nhạc Four Square. "Chúng tôi rất tự hào khi đã giành chiến thắng và hào hứng với việc biểu diễn trong cuộc picnic”, Peter Zandt, người chơi ghi ta trong ban nhạc cho hay. Và vì chúng tôi hi vọng một ngày nào đó sẽ được biểu diễn ở những địa điểm khác như là ở trong những nhà hàng hay là công viên chẳng hạn nên đây sẽ là bước đệm đầu tiên.

Cuộc thi này là ý tưởng của thầy Lopez, một giáo viên âm nhạc và cô Cho, một giáo viên kịch. Cả hai thầy cô đã nghĩ ra ý tưởng đó trong khi đang thảo luận về những cuộc picnic gần đây của nhà trường. “Cuộc picnic là một trong những sự kiện lớn nhất của năm nhưng nó đã dần trở nên tẻ nhạt", cộ Cho nói.

"Những hoạt động như thế cứ năm nào cũng như năm nào. Chúng tôi đã nghĩ rằng một màn biểu diễn bởi một ban nhạc trong trường sẽ làm cuộc picnic thú vị và vui vé hơn." Thầy Lopez, cô Cho và ba giáo viên khác làm giám khảo cuộc thi, cái mà được diễn ra trong phòng gym. Tám ban nhạc đăng kí thử giọng. Những ban nhạc này có sự đa dạng về chủng loại: có một vài ban nhạc Rock, nhạc dân gian và một nhóm nhạc Jazz. “Tôi đã rất thất vọng khi ban nhạc của chúng tôi không giành chiến thắng nhưng tôi nghĩ ban giám khảo đã lựa chọn đúng", Marisol Varga, một thành viên trong ban nhạc dân gian Bell Girls nói, "Four Square” thực sự xuất sắc".

Để xem liệu những ban nhạc này có thể hiện được nhiều kĩ năng âm nhạc hay không, các thầy cô đã yêu cầu mỗi ban nhạc chuẩn bị hai bài hát: một bài hát chay và một bài khác chỉ chơi nhạc cụ. Giám khảo cuối cùng đã chọn ra ban nhạc chiến thắng là Four Square. Four Square là một ban nhạc Jock với một sự thay đổi bất thường: nó gồm cả một người chơi vĩ cầm. Thành viên của Four Square tự viết bài hát cho nhóm mình và luyện tập ba lần một tuần sau các giờ học ở trường. Tất cả giáo viên và học sinh đều đồng ý rằng cuộc thi thành công rực rỡ. Tất cả họ đang ngóng chờ cho cuộc picnic của trường vào tháng 6 này.

Câu 106: Đáp án B

CHỦ ĐỀ POPULATION

Từ “havoc” trong đoạn 1 gần nghĩa nhất với .

A. vấn đề bất thường B. hủy hoại lớn và rắc rối

C. tranh chấp nghiêm trọng D. một bệnh truyền nhiễm “havoc” = great destruction and trouble: tan phá, huỷ hoại nghiêm trọng

Dựa vào ngữ cảnh của câu sau:

“This distribution of the social ecology would create havoc in the future.”

(Sự phân bố này của hệ sinh thái xă hội sẽ gây ra sự tàn phá trong tương lai.)

Câu 107: Đáp án D

Nội dung chính của đoạn 1 là gì?

  1. Chính phủ đang tạo ra các ưu đãi cho trẻ em gái.
  2. Chính phủ đang cố gắng để giải mà các hàm ý của các hồ sơ điều tra dân số.
  3. Chính phủ đang xây dựng các chính sách để làm suy yếu vị trí của con trai trong xã hội.
  4. Chinh phủ đang mở rộng ưu đãi để khuyến khích cha mẹ có con gái.

Thông tin:

“The Government has embarked on policies extending innumerable incentives to the families bearing girls. Monetary support, free education, guaranteed employment is being gifted to parents who gift the country with a girl child.” (Chính phủ đã bắt tay vào các chính sách mở rộng vô số ưu đãi cho các gia đình có con gái. Hỗ trợ tiền tệ, giáo dục miễn phí, công việc được bảo đảm đang được tặng cho các cha me có con gái.)

Câu 108: Đáp án C

“Kế hoạch chi tiết” như đã thảo luận trong đoạn 1 là gì?

  1. Đó là một danh sach các quy tắc để người dân Trung Quốc làm theo để duy tri kiểm soát dân số.
  2. Đó là một kế hoạch sơ bộ để điều chỉnh hành vi của cộng đồng.
  3. Chính sách của Chính phủ bao gồm chi tiết về các quy định và khuyến khích để cải thiện tỷ lệ trẻ em gái.
  4. Đó là hướng dẫn được in ra để thực hiện các quy tắc về tỷ lệ giới tính.

Câu 109: Đáp án A

Từ “teeming” trong đoạn 2 gần nhất có nghĩa là ?

A. đầy tràn B. dân số nữ

C. cặp vợ chồng D. dân số nói chung

am dịch

Dựa vào ngữ cảnh của câu:

“Consequent to the population explosion, the Government introduced, in the 80’s, one child policy in China. Any additional pregnancy had to be terminated. This was aimed to put a check on the teeming millions.”

(Kết quả của bùng nổ dân số, vào những năm 80, chính phủ đã giới thiệu chính sách một con ở Trung Quốc. Bất kỳ thai kỳ bổ sung nào cũng phải chấm dứt. Điều này nhằm mục đích kìm nén hàng triệu người đông đúc.)

=> "teeming" = overflowing: đông đúc, nhiều, tràn đầy

Câu 110: Đáp án B

Tầm nhìn phía sau chính sách của chính phủ được thảo luận trong đoạn 2 là?

  1. Tầm nhìn về Trung Quốc với phụ nữ ở các vị trí hàng đầu trong chính phủ.
  2. Tầm nhìn cho Trung Quốc để kiểm soát dân số đang phát triển trong tương lai gần.
  3. Tầm nhìn về một xã hội phụ nữ thống trị.
  4. Tầm nhìn mà các thanh thiếu niên nam nữ Trung Quốc sẽ tìm được bạn đời.

Câu 111: Đáp án B

Điều nào dưới đây không đúng về chính sách một con ở Trung Quốc?

  1. Mỗi gia đình ở Trung Quốc chỉ được phép có một con.
  2. Dù thai kì bổ sung có là một bé gái thì cũng phải bị loại bỏ.
  3. Chính sách một con được giới thiệu bởi chính phủ Trung Quốc vào những năm 1980.
  4. Với mục đích kiểm soát sự bùng nổ dân số, chính phủ đã áp dụng chính sách một con ở Trung Quốc.

Căn cứ vào thông tin sau:

“Any additional pregnancy had to be terminated. This was aimed to put a check on the teeming millions. The policy had no relation to extermination of girl child in the womb." (Bất kỳ thai kỳ bổ sung nào cũng phải chấm dứt. Điều này nhằm mục đích kìm nén hàng triệu người đông đúc. Chính sách này không có liên quan đến việc loại bỏ đứa con gái trong bụng mẹ.)

Câu 112: Đáp án D

Nhìn vào bốn ô vuông [■] cho biết câu sau có thể được thêm vào ở đâu trong đoạn văn:

“Một lý do khác là con gái phải rời cha mẹ sau khi kết hôn làm giàu gia đình chồng họ.” Câu nào phù hợp nhất?

Đoạn văn 3 đang nói về lý do con trai được ưa thích hơn con gái, cho nên điền vào chỗ trống D là phù hợp.

Dịch bài

Ở Trung Quốc, sự cách biệt đang tăng lên về giới tính đang tạo ra dấu hiệu báo động cho Chính phủ. Theo số liệu điều tra dân số mới nhất, 119 bé trai được sinh ra trên mỗi 100 bé gái. Sự cách biệt đáng chú ý này dự kiến sẽ tăng lên vào năm 2020 với gần 40 triệu người độc thân. Sự phân bố này của bộ sinh thái xã hội sẽ gây ra sự tàn phá trong tương lai. Các nhà lãnh đạo xã hội đang cố gắng tạo áp lực lên dân chúng để sinh thêm nhiều bé gái. Chính phủ đã bắt tay vào các chính sách mở rộng vô số ưu đãi cho các gia đình có con gái. Hỗ trợ tiền tệ, giáo dục miễn phí, công

việc được bảo đảm đang được tặng cho các cha mẹ có con gái. Chính phủ đang cố gắng thuyết phục mọi người kìm nén sở thích cá nhân của họ và điều chinh hành vi của cộng động theo kế hoạch mới để kích thích tỷ lệ nữ. Đôi khi Chính phủ cố gắng lôi kéo họ và đội khi họ sử dụng chính sách gốc để buộc họ thực hiện.

Kết quả của bùng nổ dân số, vào những năm 80, chính phủ đã giới thiệu chính sách một con ở Trung Quốc. Bất kỳ thai kỳ bổ sung nào cũng phải chấm dứt. Điều này nhằm mục đích kìm nén hàng triệu người đông đúc. Chính sách này không có liên quan đến việc loại bỏ đứa con gái trong bụng mẹ. Nhưng các nhà hoạch định chính sách không biết về ảnh hưởng lâu dài của nó. Con người, với tư duy gia trưởng, thích một đứa con trai. Ý niệm về một gia đình hạnh phúc trở thành ‘cha mẹ với một đứa con trai’.

Văn hoá Trung Hoa luôn luôn đề cao con trai hơn con gái vì xã hội đã bị thống trị bởi nam giới. Ở nhiều làng, nơi cần sự chăm chỉ để duy trì nông nghiệp, một chàng trai luôn thích hợp hơn do sức mạnh thể chất của mình cao hơn so với một cô gái. Một lý do khác là con gái phải rời cha mẹ sau khi kết hôn làm giàu gia đình chồng họ. Trong những trường hợp như vậy, mong muốn một đứa con trai có vẻ hợp lý. Nếu mọi người phải giới hạn gia đình của họ, rõ ràng họ sẽ thích một cậu bé hơn một cô gái. Vấn đề này đã được nêu bật bằng việc sử dụng siêu âm giúp xác định giới tính thai nhi. Công nghệ này đã đóng một yếu tố quyết định trong việc tạo ra sự mất cân bằng giới.

Câu 113: Đáp án A

Chủ đề POPULATION (Trích đề THPTQG 2015)

Trong đoạn văn 1, cái nào dưới đây được đưa ra định nghĩa?

A. Bùng nổ dân số B. Mật độ dân số

C. Canh tác thủ công D. Nghèo đói

Căn cứ vào câu đầu tiên của đoạn văn: “Overpopulation, the situation of having large numbers of people with too few resources and too little space, is closely associated with poverty.” ( Bùng nổ dân số, một tình huống xảy ra khi có rất nhiều người nhưng lại với quá ít tài nguyên và chỗ ở, có liên hệ mật thiết đến đói nghèo.) => Bùng nổ dân số là từ được định nghĩa => Đáp án A

Câu 114: Đáp án A

Điều gì sẽ bị ảnh hưởng khi mật độ dân số vượt quá cao?

A. Nguồn tài nguyên sẵn có B. Lao động có tay nghề

C. Phương pháp canh tác D. Diện tích đất

Căn cứ vào thông tin sau: “|Excessively high population densities put stress on available resources.”

(Mật độ dân số vượt quá cao gây áp lực đến nguồn tài nguyên sẵn có.)

=> Đáp án A

Câu 115: Đáp án A

Cụm từ “ that number” trong đoạn văn số 1 ám chỉ tới số tượng .

A. người B. mật do C. nguồn tài nguyên D. các quốc gia

Căn cứ vào câu có chứa cụm từ đó: “Only a certain number of people can be supported on a given area of land, and that number depends on how much food and other resources the land can provide.” (Chỉ một số tượng người nhất định có thể sống được trên một diện tích đất có sẵn và số người đó phụ thuộc vào khả năng vùng đất đó có thể cung cấp được bao nhiều lương thực và tài nguyên.)

Câu 116: Đáp án A

Ở một số quốc gia nhất định, những vùng đất rộng lớn chỉ có thể cho ra một lượng lương thực nhỏ bởi vì

.

A. thiếu sự cơ giới hóa B. có ít lao động

C. có nhiều tài nguyên D. thiếu lao động có tay nghề

Căn cứ vào thông tin sau trong đoạn văn: “ln countries where people live primarily by means of simple fanning, gardening, herding, hunting, and gathering, even large areas of land can support only small numbers of people because these labour intensive subsistence activities produce only small amounts of food.” (Ở những quốc gia nơi mà con người sống chủ yếu bằng phương thức canh tác thô sơ như làm vườn, săn bắt, thu lượm, thì thậm chí trên một diện tích đất đai rộng lớn cũng chỉ có thể nuôi sống được một số tượng ít người bởi vì hoạt động sản xuất này chỉ tạo ra một lượng thực phẩm nhỏ.)

Câu 117: Đáp án B

Bangladesh Id một quốc gia nơi mà mức độ nghèo tùy thuộc cơ bản vào .

  1. chỉ mật độ dân số của nó
  2. cả mật độ dân số lẫn năng suất nông nghiệp
  3. mật độ dân số ở những khu vực trung tâm
  4. năng suất nông nghiệp cao

Căn cứ vào thông tin sau trong đoạn văn: “A country's level of poverty can depend greatly on its mix of population density and agricultural productivity. Bangladesh, for example...” (Mức độ nghèo của một quốc gia phụ thuộc chủ yếu vào mật độ dân số và năng suất lao động của nó. Bangladesh là một ví dụ...)

Câu 118: Đáp án C

Từ “infertile” trong đoạn văn số 4 có thể có nghĩa là .

A. không còn dùng đến nữa B. không thể

C. không màu mỡ D. không thể tới gần được

Dựa vào ngữ cảnh của câu sau: “Many people in these countries practise manual subsistence farming; these countries also have infertile land, and lack the economic resources and technology to boost

productivity. (Nhiều người ở những quốc gia này sinh sống bằng phương thức canh tác thuần nông; những quốc gia này cũng có đất không còn màu mỡ và thiếu các nguồn lực kinh tế và công nghệ để thúc đẩy năng suất)

Câu 119: Đáp án D

Cái nào sau đây là yếu tổ dẫn tới bùng nổ dân số ở các nước đang phát triển?

A. Cơ sở vật chất công nghệ cao B. Nguồn lực kinh tế

C. Đủ hỗ trợ về tài chính D. Tỉ lệ sinh cao

Căn cứ vào câu đầu của đoạn cuối:

"High birth rates contribute to overpopulation in many developing countries.”

(Tỉ lệ sinh cao dẫn tới bùng nổ dân số ở nhiều quốc gia đang phát triển)

Câu 120: Đáp án B

Cái nào dưới đây có thể là chủ đề thích hợp nhất cho đoạn văn này?

  1. Tỉ lệ sinh cao và những hậu quả của nó
  2. Bùng nổ dân số: Nguyên nhân của đói nghèo
  3. Bùng nổ dân số: Một vấn đề toàn cầu
  4. Đói nghèo ở các quốc gia đang phát triển

Tạm dịch

Bùng nổ dân số, tình trạng có rất nhiều người trong khi có quá ít tài nguyên và không gian, gắn liền với đói nghèo. Nó có thể là kết quả của mật độ dân số cao, hoặc từ các nguồn tài nguyên ít ỏi, hoặc từ cả hai. Mật độ dân số tăng cao đặt áp lực lên các nguồn lực sẵn có. Chỉ có một số người nhất định có thể được hỗ trợ trên một diện tích đất nhất định và số đó phụ thuộc vào lượng thức ăn và các nguồn tài nguyên khác mà đất có thể cung cấp. Ở những quốc gia mà người dân sống chủ yếu bằng phương pháp canh tác đơn giản, làm vườn, chăn thả gia súc, săn bắt và thu hái, thậm chí điện tích đất rộng chỉ có thể hỗ trợ cho một số lượng nhỏ người dân bởi vì các hoạt động sản xuất cần nhiều lao động này chỉ sản xuất một lượng nhỏ lương thực.

Ở các nước phát triển như Hoa Kỳ, Nhật Bản, và các nước Tây Âu, bùng nổ dân số thường không được coi là nguyên nhân chính của đói nghèo. Các nước này sản xuất số lượng lớn thực phẩm thông qua nông nghiệp cơ giới hóa, phụ thuộc vào phân bón thương mại, thủy lợi quy mô lớn và máy móc nông nghiệp. Hình thức sản xuất này cung cấp đủ thức ăn để hỗ trợ mật độ cao của người dân ở các khu vực đô thị.

Mức nghèo đói của một quốc gia có thể phụ thuộc rất nhiều vào việc kết hợp mật độ dân số và năng suất nông nghiệp. Ví dụ, Bangladesh là một trong những mật độ dân số cao nhất thế giới, với 1.147 người trên mỗi km vuông. Đa số người Bangladesh tham gia vào sản xuất nông nghiệp thủ công năng suất thấp, khiến cho mức nghèo đói của đất nước này rất cao. Một số nước nhỏ hơn ở Tây

Âu, như Hà Lan và Bỉ, có mật độ dân số cao. Các nước này thực hiện nông nghiệp cơ giới hóa và tham gia vào các ngành công nghệ cao, tuy nhiên, và do đó có mức sống cao.

Ở đầu cuối của quang phổ, nhiều quốc gia ở châu Phi Vang ha Sahara có mật độ dân số dưới 30 người trên mỗi km vuông. Nhiều người ở những nước này thực hiện canh tác tự cung tự cấp; những nước này cũng có đất đai kém cỏi, thiếu nguồn lực kinh tế và công nghệ để tăng năng suất. Hậu quả là các quốc gia này rất nghèo. Hoa Kỳ có mật độ dân số tương đối thấp và năng suất nông nghiệp cao; nó là một trong những quốc gia giàu có nhất thế giới.

Tỷ lệ sinh cao làm tăng dân số ở nhiều nước đang phat triển. Trẻ em là tài sản của nhiều gia đình nghèo vì họ cung cấp lao động, thường là cho nông nghiệp. Các chuẩn mực văn hoá trong các xã hội truyền thống ở nông thôn thường tuân theo giá trị của các gia đình lớn. Ngoài ra, các chính phủ của các nước đang phát triển thường cung cấp ít hoặc không hỗ trợ, tài chính hay chính trị cho kế hoạch hóa gia đình; ngay cả những người muốn giữ cho gia đình họ ít con cũng gặp khó khăn trong việc làm việc đó. Vì tất cả những lý do này, các nước đang phat triển có xu hướng có tỷ lệ tăng dân số cao.

Câu 121: Đáp án A

CHỦ ĐỀ LIFE IN THE FUTURE

Câu nào trong các câu sau có thể là ý chính của đoạn văn?

A. Thế giới sẽ thay đổi như thể nào trong 500 năm tới?

B.Chúng ta sẽ làm gì trong 5 thế kỉ tới?

  1. Những vấn đề nào sẽ xảy ra trong thế kỉ 26?
  2. Công nghệ sẽ cải thiện cuộc sống trong tương lai xa như thế nào?

Căn cứ thông tin đoạn 1:

But what if we could look 500 years into the future and glimpse the Earth of the 26th century? Would the world seem as different to us as the let century would have seemed to residents of the 16th century? (Nhưng liệu chúng ta có thể nhìn thấy được những gì trong 500 năm nữa và nhìn thoáng qua Trái đất của thế kỉ 26? Liệu thế giới đó có khác với chúng ta như thế kỉ 21 khác với người dân thế kỉ thứ 16?)

Các đoạn còn lại của đoạn văn cũng nói về cuộc sống trong thế kỉ 26, nên đây sẽ là nội dung chính của đoạn văn.

Câu 122: Đáp án B

Từ “infernatty” trong đoạn 2 gần nghĩa nhất với từ nào?

A. khác B. cực kì C. lâu dài D. cùng thời Từ đồng nghĩa: infernally (cực kì) = extremely

Depending on whom you ask. the 26th century will either be a little chilly or infernally hot. (Tuỳ thuộc vào người bạn hỏi là ai, thế kỉ 26 hoặc là sẽ một chút băng giá, hoặc là sẽ cực kì nóng.)

Câu 123: Đáp án C

Theo đoạn văn, các chuyên gia dự đoán về thời tiết trong tương lai như thế nào?

  1. Nhiệt độ sẽ giảm nhiều nhất có thể bởi vì sự sử dụng nhiên liệu hoá thạch quá mức.
  2. Thời đại Kỷ Băng Hà nhỏ sẽ quay trở lại vì ảnh hưởng của năng lượng mặt trời.
  3. Trái Đất sẽ phải đối mặt với thời tiết cực đoan như băng giá hay giai đoạn nóng như thiêu đốt.
  4. Sự biến đổi khí hậu sẽ xảy ra nhanh hơn bởi vì sự chặt phá rừng.

Căn cứ vào thông tin đoạn 2:

Depending on whom you ask, the 26th century will either be a little chilly or infernally hot. Some solar output models suggest that by the 25005, Earth's climate will have cooled back down to near Little Ice Age conditions. Other studies predict that ongoing climate change and fossil fuel use will render much of the planet too hot for human life by 2300. (Tùy thuộc vào người bạn hỏi là ai, thế kỉ 26 hoặc là sẽ một chút băng giá, hoặc là sẽ cực kì nóng. Một số mô hình đầu ra năng lượng mặt trời cho thấy rằng vào những năm 2500, khi hậu Trái Đất sẽ lạnh dần xuống gần điều kiện Kỷ Băng Hà Nhỏ. Các nghiên cứu khác dự đoán rằng sự biến đổi khí hậu đang diễn ra và việc sử dụng nhiên liệu hóa thạch sẽ khiến cho hành tinh này trở nên quá nóng cho cuộc sống con người vào năm 2300.)

Câu 124: Đáp án C

Từ “horness" ở đoạn 4 có thể được thay thể bởi từ nào?

A. làm mới B. điều chỉnh C. khai thác D. khám phá Từ đồng nghĩa: harness (khai thác) = exploit

In other words, we'll become a species that can harness the entire sum of a planet's energy. (Nói cách khác, chúng ta sẽ trở thành một loài mà có thể khai thác toàn bộ năng lượng của một hành tinh.) Câu125: Đáp án D

Từ “they” trong đoạn 4 để cập đến từ nào?

A. phản ứng tổng hợp và năng lượng mặt trời B. công nghệ năng lượng sạch

C. những chuyên gia D. những người ở thể kỉ 26 Từ “they” thay thế cho danh từ những người ở thế kỉ 26.

Wielding such power, 26th-century humans will be masters of clean energy technologies such as fusion and solar power. Furthermore, they‘ll be able to manipulate planetary energy in order to control global climate. (Nắm giữ sức mạnh như vậy, những người thể kỉ 26 sẽ trở thành những chuyên gia về công nghệ năng lượng sạch như phản ứng tổng hợp và năng lượng mặt trời. Hơn nữa, họ sẽ có thể thao túng năng lượng hành tinh để kiểm soát khí hậu toàn cầu.)

Câu 126: Đáp án B

Câu nào trong những câu sau đúng về những dự đoán trong tương lai?

  1. Michio Kaku cho rằng sự tiến bộ từ nền văn minh loại 0 lên loại 1 sẽ mất khoảng 2 thế kỉ.
  2. Con người trong những năm 2500 có thể kiểm soát nguồn năng lượng của Trái Đất để làm giảm khí hậu toàn cầu.
  3. Tốc độ cải thiện công nghệ sẽ tiếp tục để thay đổi trong tương lai xa.
  4. Năng lượng mặt trời sẽ là nguồn năng lượng chính cho công dân thế kỷ 26.

Căn cứ thông tin đoạn 4:

Theoretical physicist and futurist Michio Kaku predicts that in a mere 100 years, humanity will make the leap from a type zero civilization to a type I civilization on the Kardashev Scale. In other words, we'll become a species that can harness the entire sum of a planet's energy. Wielding such power, 26th- century humans will be masters of clean energy technologies such as fusion and solar power. Furthermore, they'll be able to manipulate planetary energy in order to control global climate. Physicist Freeman Dyson, on the other hand, estimates the leap to a type I civilization would occur within roughly 200 years.

(Nhà vật lý lý thuyết và tương lai học Michio Kaku dự đoán rằng chỉ trong vòng 100 năm nữa, nhân loại sẽ tạo ra một bước nhảy vọt từ nền văn minh loại 0 lên nền văn minh loại 1 trên thang Kardashev. Nói cách khác, chúng ta sẽ trở thành một loài mà có thể khai thác toàn bộ năng lượng của một hành tinh. Nắm giữ sức mạnh như vậy, những người thế kỉ 26 sẽ trở thành những chuyên gia về công nghệ năng lượng sạch như phản ứng tổng hợp và năng lượng một trời. Hơn nữa, họ sẽ có thể thao túng năng lượng hành tinh để kiểm soát khí hậu toàn cầu. Mặt khác, nhà vật lý Freeman Dyson ước tính bước nhảy vọt lên nền văn minh loại một sẽ xảy ra trong vòng 200 năm)

Câu 127: Đáp án D

Theo Adrian Berry, những điều sau là việc mà những người trong tương lai có thể làm, ngoại trừ

.

A. trồng trọt dưới đại dương B. du lịch giữa các vì sao

C. khám phá vũ trụ bằng robot D. sống bao lâu cũng được

Căn cứ thông tin đoạn cuối:

Futurist and author Adrian Berry believes the average human life span will reach 140 years and that the digital storage of human personalities will enable a kind of computerized immortality. Humans will farm the oceans, travel in starships and reside in both lunar and Martian colonies while robots explore the outer cosmos.

(Nhà tương lai học và tác giả Adrian Berry tin rằng tuổi thọ trung hình của con người sẽ đạt đến 140 tuổi và việc lưu trữ kĩ thuật số của tính cách con người sẽ cho phép một loại bất tử trên máy tính. Con người

có thể trồng trọt dưới đại dương, du hành giữa các vì sao và cư trú ở cả sao Hoả và mặt trăng trong khi robot khám phá vũ trụ bên ngoài.)

Câu 128: Đáp án A

Có thể suy ra từ đoạn văn rằng .

  1. Không ai có thể chắc chắn cuộc sống vào những năm 2500 sẽ trông như thế nào.
  2. Những gì chúng ta tưởng tượng về cuộc sống trong thế kỉ 26 cũng giống như những người ở thế kỉ 16 nghĩ về cuộc sống hiện tại.
  3. Những dự đoán của các nhà khoa học khác nhau có thể vẽ ra cùng 1 thế giới trong 500 năm tới.
  4. Công nghệ sẽ ảnh hưởng hầu hết các lĩnh vực của cuộc sống trong tương lai.

Căn cứ vào thông tin cả bài:

Mỗi nhà khoa học lại dự đoán cuộc sống trong tương lai khác nhau nên không ai có thể chắc chắn về cuộc sống vào những năm 2500 như thế nào.

Bài dịch

Nếu bạn có thể quay trở lại trong thời gian năm thế kỷ, bạn sẽ gặp phải một để chế Aztec phát triển mạnh ở Trung tâm Mexico, một “Mona Lisa” được sơn mới ở Châu Âu Phục Hưng và nhiệt độ lạnh hơn ở Bắc bán cầu. Đây là một thế giới ở giữa Kỷ Băng Hà nhỏ (từ năm 1300 đến năm 1850) và một thời kỳ thăm dò rộng lớn ở châu Âu hiện nay được gọi là Age of Discovery (thời kì khám phá). Nhưng nếu chúng ta có thể nhìn 500 năm trong tương lai và nhìn thoáng qua Trái đất của thế kỷ 26? Liệu thế giới có về khác với chúng ta khi thế kỷ 21 có vẻ như là cư dân của thể kỷ 16? Để bắt đầu, thời tiết sẽ như thế nào?

Tuỳ thuộc vào người bạn hỏi là ai, thế kỉ 26 hoặc là sẽ một chút băng giá, hoặc là sẽ cực kì nóng. Một số mô hình đầu ra năng lượng mặt trời cho thấy rằng vào những năm 2500, khí hậu Trái Đất sẽ lạnh dần xuống gần điều kiện Kỷ Băng Hà Nhỏ. Các nghiên cứu khác dự đoán rằng sự biến đổi khí hậu đang diễn ra và việc sử dụng nhiên liệu hoá thạch sẽ khiến cho hành tinh này trở nên quá nóng cho cuộc sống con người vào năm 2300.

Một số chuyên gia xác định sự khởi đầu của biến đổi khí hậu của con người từ cuộc cách mạng công nghiệp vào những năm 1800, những người khác xác định do việc chặt và đốt rừng làm nông nghiệp trong thời tiền sử. Dù bằng cách nào, những con người nắm giữ công cụ làm thay đổi môi trường của họ - và các công cụ thể kỷ 26 của chúng ta có thể khá ấn tượng.

Nhà vật lý lý thuyết và tương lai học Michio Kaku dự đoán rằng chỉ trong vòng 100 năm nữa, nhân loại sẽ tạo ra một bước nhảy vọt từ nền văn minh loại 0 lên nền văn minh loại một trên thang Kardạshev. Nói cách khác, chúng ta sẽ trở thành 1 loài mà có thể khai thác toàn bộ năng lượng của một hành tinh. Nắm giữ sức mạnh như vậy, những người thế kỉ 26 sẽ trờ thành những chuyên gia về công nghệ năng lượng sạch như phản ứng tổng hợp và năng lượng mặt trời. Hơn

nữa, họ sẽ có thể thao túng năng lượng hành tinh để kiểm soát khí hậu toàn cầu. Mặt khác, nhà vật lý Freeman Dyson ước tính bước nhảy vọt lên nền văn minh loại một sẽ xảy ra trong vòng 200 năm.

Công nghệ đã được cải thiện theo cấp số nhân từ những năm 1500, và tốc độ này có thể sẽ tiếp tục trong nhiều thế kỷ tới. Nhà vật lý Stephen Hawking đề xuất rằng vào năm 2600, sự tăng trưởng này sẽ chứng kiến 10 giấy tờ vật lý lý thuyết mới được xuất bản cứ 10 giây một lần. Nếu Định luật Moore vẫn đúng và tốc độ và độ phức tạp của máy tính tăng gấp đôi sau mỗi 18 tháng, thì một số việc trong các nghiên cứu này có thể là công việc của những cỗ máy thông minh.

Những công nghệ nào khác sẽ định hình thế giới của thể kỷ 26? Nhà tương lai học và tác giả Adrian Berry tin rằng tuổi thọ trung bình của con người sẽ đạt đến 140 tuổi và việc lưu trữ kĩ thuật số của tính cách con người sẽ cho phép một loại bất tử trên máy tính. Con người có thể trồng trọt dưới đại dương, du hành giữa các vì sao và cư trú ở cả sao Hoà và Mặt trăng trong khi robot khám phá vũ trụ bên ngoài.

Câu 129: Đáp án A

CHỦ ĐỀ CELEBRATIONS

Đoạn văn này chủ yếu thảo luận về điều gì?

  1. Những hoạt động kỉ niệm dành cho một đứa trẻ mới ở Châu Phi.
  2. Nghi lễ sinh nhật cho trẻ em ở Châu Phi.
  3. Truyền thông của người Masai khi sinh con.
  4. Các hoạt động trong lễ kỉ niệm một đứa trẻ chào đời.

Căn cứ vào thông tin toàn bài:

Đoạn văn đang nhắc đến các hoạt động kỉ niệm được tổ chức cho một đứa trẻ ở châu Phi từ khi mới sinh ra cho đến khi sẵn sàng lập gia đình mới.

Câu 130: Đáp án B

Người cha và những người lớn tuổi trong làng ở Kenya thường đặt tên cho đứa trẻ ở đâu?

A. ở trong làng B. tại nơi chăn nuôi gia súc

C. ở nhà nọ D. tại hàng ròa của chuồng gia súc

Căn cứ vào thông tin đoạn 1:

In Kenya, the mother takes the baby strapped to her back into the thorn enclosure where the cattle is kept. There, her husband and the village elders wait to give the child his or her name.

(Ở Kenya, người mẹ đưa em bé được địu vào lưng mình vào trong vùng đất có rào gai vây quanh, nơi gia súc được nuôi. Ở đó, chồng bà và những người lớn tuổi trong làng chờ đợi đểđặt tên cho đứa tré.) Câu 131: Đáp án C

Câu nào không được đề cập đến trong đoạn 2?

  1. Một đứa trẻ tám ngày tuổi sẽ được đỡ bước đi bộ đầu tiên bởi mẹ của nó.
  2. Trẻ em phải học về văn hoá và truyền thống của bộ tộc mình khi chúng đủ lớn.
  3. Trẻ em được chuẩn bị cho cuộc sống trường thành khi được viếng thăm bởi bạn bè và người thân.
  4. Trẻ em có thể phải vượt qua các cuộc kiểm tra để bước vào thế giới trưởng thành.

Căn cứ thông tin đoạn 2:

In West Africa, after the baby is eight days old. the mother takes the baby for it's first walk in the big, wide world, and friends and family are invited to meet the new baby. In various African nations, they hold initiation ceremonies for groups of children instead of birthdays. When children reach a certain designated age, they learn the laws, beliefs, customs, songs and dances of their tribes.

Some African tribes consider that children from nine to twelve are ready to be initiated into the grown up world. They may have to carry out several tests.

(Ở Tây Phi, sau khi em bé được tám ngày tuổi, người mẹ đưa em bé đi bộ lần đầu tiên trong một thế giới rộng lớn, và bạn bè và gia đình được mời đến gặp em bé mới. Ở các quốc gia châu Phi khác nhau, họ tổ chức các buổi lễ vỡ lòng cho các nhóm trẻ em thay vì sinh nhật. Khi trẻ em đạt đến một độ tuổi nhất định, chúng học các luật, tín ngưỡng, phong tục, bài hát và điệu múa của các bộ lạc của chúng. Một số bộ lạc châu Phi cho rằng trẻ em từ 9 đến 12 tuổi đã sẵn sàng để được bắt đầu vào thế giới trưởng thành. Họ có thể phải thực hiện một số bài kiểm tra)

Câu 132: Đáp án B

Từ “undergo" trong đoạn 3 gần nghĩa nhất với từ .

A. bắt đầu B. trải qua C. tham gia D. khám phá Từ đồng nghĩa: undergo (trải qua) = experience.

Masai boys around thirteen years old to seventeen undergo a two stage initiation. (Bé trai Masai khoảng mười ba tuổi đến mười bảy trải qua thời kì vỡ lòng gồm hai giai đoạn)

Câu 133: Đáp án A

Từ “shaved” trong đoạn 3 gần có thểcó nghĩa là .

A. cắt tóc bằng dao cạo B. buộc tóc với một cái kẹp

C. nhuộm tóc với một màu khác D. che tóc bằng một cái mũ Định nghĩa: Shaved (cạo đầu) = cut the hair off with a razor

At the end of this stage, they have their heads shaved and they are also circumcised.

(Vào cuối giai đoạn này, họ cạo đầu và họ cũng cắt bao quy đầu.)

Câu 134: Đáp án C

Từ “they” trong đoạn 3 đề cập đến từ nào?

A. đàn gia súc của Masai B. động vật hoang dã

C. những chiến binh trẻ D. những cái đầu của họ

Căn cứ thông tin đoạn 3:

Từ “they” đề cập đến các chiến binh trẻ.

The second stage, the young warriors grow their hair long and live in a camp called a manyatta where they practice hunting the wild animals that might attack the Masia herds. This stage may last a few years. When they are ready, they will marry and become owners of large cattle herds like their fathers. (Giai đoạn thứ hai, các chiến binh trẻ để tóc dài và sống trong một trại gọi là một cộng đồng người Masai nơi họ thực hành săn bắn những con vật hoang dã có thể tấn công đàn vật nuôi của Masai. Giai đoạn này có thể kéo dài vài năm. Khi họ đã sẵn sàng, họ sẽ kết hôn và trở thành chủ sở hữu của đàn gia súc lớn như cha của họ.)

Câu 135: Đáp án B

Các cô gái học điều gì trong sự khởi đầu của họ?

  1. Cách chăm sóc chồng và gia đình trong tương lai.
  2. Trách nhiệm trong hôn nhân và cách chăm sóc con cái.
  3. Chăm soc gia đình và sinh con.
  4. Cách sống sau hôn nhân và sinh con.

Căn cứ vào thông tin đoạn cuối:

The girls are initiated when they are fourteen or fifteen. They are taught by the older women about the duties of marriage and how to care for babies. Soon after that they are married and lead a life similar to that of their mothers. (Các cô gái được vỡ lòng khi họ mười bốn hoặc mười lăm tuổi. Họ được dạy bởi những phụ nữ lớn tuổi về nhiệm vụ hôn nhân và cách chăm sóc cho trẻ sơ sinh. Ngay sau đó họ kết hôn và sống một cuộc sống tương tự như của các bà mẹ của họ.)

Bài dịch

Ở châu Phi, mọi người ăn mừng với niềm vui vì sự ra đời của một em bé mới. Người Pygmy sẽ hát bài hát chúc mừng sự chào đời cho đứa trẻ. Ở Kenya, người mẹ đưa em bé được địu vào lưng mình vào trong vùng đất có rào gai vây quanh, nơi gia súc được nuôi. Ở đó, chồng bà và những người lớn tuổi trong làng chờ đợi để đặt tên cho đứa tré.

Ở Tây Phi, sau khi em bé được tám ngày tuổi, người mẹ đưa em bé đi bộ lần đầu tiên trong một thế giới rộng lớn, và bạn bè và gia đình được mời đến gặp em bé mới. Ở các quốc gia châu Phikhác nhau, họ tổ chức các buổi lễ vỡ lòng cho các nhóm trẻ em thay vì sinh nhật. Khi trẻ em đạt đến một độ tuổi nhất. định, chúng học các luật, tín ngưỡng, phong tục. bài hát và điệu múa của các bộ lạc của chúng. Một số bộ lạc châu Phi cho rằng trẻ em từ 9 đến 12 tuổi đã sẵn sàng để được bắt đầu vào thế giới trưởng thành. Họ có thể phải thực hiện một số bài kiểm tra.

Bé trai Masai khoảng mười ba tuổi đến mười bảy trải qua thời kì vỡ lòng gồm hai giai đoạn. Giai đoạn đầu tiên kéo dài khoảng ba tháng. Các chàng trai rời khỏi nhà cha mẹ, sơn cơ thể của họ màu trắng, và được dạy làm thế nào để trờ thành chiến binh trẻ. Vào cuối giai đoạn này, họ cạo đầu và họ cũng cắt bao quy đầu. Giai đoạn thứ hai, các chiến binh trẻ để tóc dài và sống trong một trại gọi là một công đồng người Masai nơi họ thực hành săn bắn những con vật hoang dã có thể tấn công đàn vật nuôi của Masai. Giai đoạn này có thể kéo dài vài năm. Khi họ đã sẵn sàng, họ sẽ kết hôn và trở thành chủ sở hữu của đàn gia súc lớn như cha của họ.

Các cô gái được vỡ lòng khi họ mười bốn hoặc mười lăm tuổi. Họ được dạy bởi những phụ nữ lớn tuổi về nhiệm vụ hôn nhân và cách chăm sóc cho trẻ sơ sinh. Ngay sau đó họ kết hôn và sống một cuộc sống tương tự như của các bà mẹ của họ.

Câu 136: Đáp án A

Chủ đề LIFE IN THE FUTURE

Chủ đề của đoạn văn là gì?

  1. Ảnh hưởng của công nghệ thông tin đối với giáo dục.
  2. Học sinh không phải tới trường nữa.
  3. Phần mềm máy tính sẽ đảm bảo cho việc học sinh học ở nhà.
  4. Học sinh có thể biết những mảng mình còn yếu để cải thiện.

Căn cứ vào câu đầu tiên của đoạn văn: "Education is another area of social life in which information technology is changing the way we communicate.” (Giáo dục là một lĩnh vực trong đời sống xã hội mà công nghệ thông tin đang dần thay đổi nó.)

Câu 137: Đáp án B

Học sinh mà trên 10 tuổi phải đi học boo nhiều lần trên tuần?

A. Không lần nào B. Một hoặc hai lần C. Ba lần D. Bốn lần

Căn cứ vào thông tin sau: "For children over the age of 10, daily attendance at schools is not compulsory. Some of the older children attend school only once or twice weekly to get tutorial support or instruction from a teacher." (Đối với những đứa trẻ trên 10 tuổi thì việc đi học hàng ngày là không bắt buộc. Một vài học sinh trong số này chỉ phải đến trường từ một đến hai buổi để nhận sự hỗ trợ và chỉ dẫn từ giáo viên)

Câu 138: Đáp án B

Từ nào dưới đây có thể dùng để thay thế cho từ " encouraged"

A. nản chí, nản lòng B. khích lệ

C. cho phép D. cấm đoán

Đáp án là B (encouraged = emnoldened)

For the most part, pupils are encouraged to work online from home. (Đa số học sinh được khuyến khích học trực tuyến tại nhà.)

Câu 139: Đáp án A

Từ “They” ở trong đọan văn số 2 ám chỉ:

A. Những học sinh B. Những giờ học

C. Những đứa trẻ D. Những đứa trẻ lớn hơn

Dựa vào ngữ cảnh của câu sau: "Students must complete a minimum number of study hours per year; however, they may make up these hours by studying at home at times that suit their family schedule. " (Mỗi năm, học sinh phải hoàn thành một số lượng rất nhỏ giờ học; tuy nhiên chúng phải bù lại số giờ đó bằng cách học tại nhà vào bất cứ khoảng thời gian nào phù hợp với lịch sinh hoạt của gia đình.)

Câu 140: Đáp án D

Ai/cái gì đếm được số giờ mà học sinh dành để học?

A. Thực tế ảo B. Giáo viên

C. Cha mẹ D. Phần mềm máy tính

Dựa vào thông tin sau: “In order to ensure that each student is learning adequately, computer software will automatically monitor the number of hours a week each student studies on-line as well as that students' learning materials and assessment activities.(Để mà đảm bảo mỗi học sinh đang học một cách hiệu quả, phần mềm máy tính sẽ tự động kiểm tra số giờ học của mỗi học sinh học online cũng như là kiểm tra các tư liệu học và các bài tập về nhà của học sinh đó.)

Câu 141: Đáp án B

Phần mềm không thể làm được điều gì dưới đây?

  1. Kiểm tra thời gian học của học sinh
  2. Thiết kế tư liệu học cho học sinh
  3. Tìm ra những hoạt động tốt nhất cho học sinh
  4. Chỉ ra điểm yếu cho học sinh
  • In order to ensure that each student is learning adequately, computer software will automatically monitor the number of hours a week each student studies. (Để mà đảm bảo mỗi học sinh đang học một

cách hiện quả, phần mềm máy tính sẽ tự động kiểm tra số giờ học của mỗi học sinh học online) 

án không phải là A.

Đáp

  • The software can then identify the best learning activities (Phần mềm có thể nhận ra được hoạt động học tập nào là tốt nhất)  Đáp án không phải là C.
  • It can also identify areas of weak achievement and produce special programs adjusted to the students’ needs. (Phần mềm cũng có thế xác định được điểm yếu của học sinh và tạo ra những chương trình đặc biệt được điều chỉnh để phù hợp với những học sinh đó)  Đáp án không phải là D.

Câu 142: Đáp án C

Cái nào dưới đây không được đề cập như là một lợi ích của công nghệ thông tin cho học sinh?

  1. Học sinh có thể học ở nhà.
  2. Học sinh có thể học bất cứ thời gian nào phù hợp với mình.
  3. Thời gian học của học sinh sẽ không được kiểm soát.
  4. Điểm yếu của học sinh có thể được nhận ra.

In order to ensure that each student is learning adequately, computer software will automatically monitor the number of hours a week each student studies on-line as well as that students’ learning materials and assessment activities. (Để mà đảm bdo mỗi học sinh đang học một cách hiện quả, phần mềm máy tính sẽ tự động kiểm tra số giờ học của mỗi học sinh học online cũng như là kiểm tra các tư liệu học và các bài tập về nhà của học sinh đó.)

Câu 143: Đáp án C

Chủ đề LIFE IN THE FUTURE

Phát minh của Briggo tốt hơn con người ở điểm nào?

A. nó có nhiều kiến thức về làm cà phê hơn. B. nó tốt hơn trong cuộc trò chuyện.

C. nó không bao giờ phải nghỉ. D. nó có thể vận hành máy móc.

Thông tin ở câu: "Unlike human baristas, it can serve multiple drinks at once and work all day and night without a break." (Không giống như những nhân viên pha chế cà phê, nó có thể phục vụ nhiều loại đồ uống cùng một lúc và làm việc cả ngày lẫn đêm không ngừng nghỉ)

Câu 144: Đáp án A

Điều gi sẽ xảy ra nếu bạn nhấn nút tạm nghỉ?

  1. Bạn sẽ phải thức dậy ngay lập tức
  2. Đồng hồ báo thức sẽ tắt mãi
  3. Báo thức sẽ dừng, nhưng sẽ sớm rung lại
  4. Không có gì xảy ra cả

Thông tin ở câu: "Once you finally catch it, you’re probably too awake to hit the snooze button.” (Một khi bạn bắt được nó, có lẽ bạn đã quá tỉnh các để nhấn nút hoãn báo thức.)

Câu 145: Đáp án D

Điều nào sau đây KHÔNG đúng về mối lo ngại về robot?

  1. Phải mất một thời gian để mọi người chấp nhận robot.
  2. Robot có thể gây hại cho con người.
  3. Ý tưởng về người máy có vẻ như khá xa vời.
  4. Robot có thể sẽ thống trị tại nơi làm việc.

Thông tin ở đoạn: "Some fear that humanity will start to decline if machines do everything for us. Others have even warned about the robot rebellion, in which robots become so smart that they may decide to turn on their masters.” (Một số người lo ngại rằng lòng nhân đạo của con người sẽ đi xuống nếu máy móc làm hết mọi thứ cho chúng ta. Một số người khác thậm chi còn cảnh báo về cuộc nổi loạn của robot khi mà robot trở nên thông minh đến mức chúng quyết định chống lại chủ nhân của mình) => Robot có thể sẽ gây hại cho con người.

=> Đáp án B đúng

“These ideas may seem a bit far—fetched, but there are certainly lots of questions that need to be answered before everyone opens up to the idea of a robotic future.”

(Những ý tưởng này có vẻ còn hơi xa vời, nhưng chắc chắn còn có nhiều câu hỏi cần được giải đáp trước

khi mọi người đón nhận ý tưởng về một tương lai robot)

=> Những ý tưởng về robot còn xa vời và còn rất nhiều người chưa chấp nhận về một tương lai với robot.

=> Đáp án A và C đúng.

Chỉ có đáp án B không được nhắc đến

Câu 146: Đáp án A

Từ "tedious" trong đoạn cuối có nghĩa gần nhất với .

A. boring (ad): nhàm chán, tẻ nhạt B. difficult (adj): khó khăn

C. intelligent (adj): thông minh D. expensive (adj): đắt tedious (adj) = boring (adj): tẻ nhạt

Câu 147: Đáp án A

Nội dung chủ yếu của đoạn cuối là về?

  1. Một số lý do khiến mọi người không chấp nhận robot
  2. Một câu chuyện khoa học viễn tưởng rất thú vị về robot.
  3. Người dân trong các vùng hỗ trợ robot như thế nào.
  4. Một vài robot mới nhất trên thị trường hiện nay.

Ở đoạn 5, tác giả có viết "many people are concerned about a future filled with robots" (nhiều người đang lo ngại đến một tương lai đầy robot) và sau đó liệt kê một số mối lo ngại của con người với robot. => Nội dung đoạn cuối là về những lí do nhiều người vẫn chưa chấp nhận robot.

Câu 148: Đáp án B

Theo đoạn văn đầu tiên, cuộc sống thay đổi như thế nào từ thời cha me chúng ta?

  1. Hệ thống giáo dục đã tệ hơn nhiều.
  2. Mọi việc không còn khó khăn như trước đây.
  3. Trẻ em phải đi bộ xa hơn để đến trường.
  4. Không có gì thay đổi nhiều.

Thông tin ở câu: “Life has changed greatly since then, and it seems to get easier year by year.” (Cuộc sống đã thay đổi rất nhiều kế từ đó, và có vẻ dễ dàng hơn theo từng năm)

Câu 149: Đáp án C

Tất cả những điều dưới đây là đúng đối với robot ngoại trừ .

  1. Chúng có thể hoạt động cả ngày lẫn đêm không ngừng nghỉ
  2. Chúng có những năng lực tuyệt vời hơn
  3. Chúng có thể lập kế hoạch để khởi tạo một câu chuyện
  4. Chúng có thể làm những công việc nhàm chán cho con người

Thông tin ở câu: "Unlike human baristas, it can serve multiple drinks at once and work all day and night without a break.” (Không giống như những nhân viên pha chế cà phê, nó có thể phục vụ nhiều loại đồ

uống cùng một lúc và làm việc cả ngày lẫn đêm không ngừng nghỉ)  Đáp án A đúng.

“Although these early home robots are somewhat basic, they will likely become more capable as times goes on.” (Mặc dù các người máy gia đình ban đầu rất cơ bản, chúng tôi sẽ có thể trở nên tài giỏi hơn theo thời gian) => Đáp án B đúng.

Although robots certainly help us to eliminate tedious tasks, many people are concerned about a future filled with robots. (Mặc dù robot chắc chắn giúp chúng ta không phải làm những công việc buồn chán, nhiều người lo lắng về một tương lai toàn là người máy) => Đáp án D đúng.

Chỉ có đáp án C là không được nhắc tới trong đoạn văn.

Câu 150: Đáp án D

Chúng ta có thể suy luận từ đoạn văn rằng .

  1. Robot trong gia đình hiện nay có thể hoạt động mà không cần bất kỳ chương trình nào
  2. Giao tiếp sẽ giảm cùng với việc sử dụng robot
  3. Cuộc nổi dậy của robot có thể xảy ra hàng ngày
  4. Robot sẽ trở thành một ngành kinh doanh chính vào năm 2030

Thông tin ở câu: "By 2030, it's estimated that robotics will be a $10 billion business worldwide.” (Trước năm 2030, ước tính rằng ngành công nghiệp robot sẽ thu được 10 tỉ đô la trên toàn thế giới) => Đến năm 2030, robot sẽ trở thành một nền công nghiệp lớn. => Đáp án D đúng.

Câu 151: Đáp án A

CHỦ ĐỀ ENDANGERED SPECIES

Mục đích của tác giả trong đoạn văn là gì?

  1. Nhằm cung cấp những thông tin cơ bản về cá voi xanh và những mối đe doạ chính của chúng.
  2. Để chứng minh rằng cá voi xanh là loài động vật có vú lớn nhất trên Trái Đất.
  3. Để cung cấp bằng chứng về việc cá voi xanh thay đổi thói quen và khu vực kiếm ăn như thế nào.
  4. Nhằm nâng cao nhận thức của mọi người về sự tuyệt chủng của cá voi xanh.

Căn cứ vào thông tin đoạn 1:

The blue whale is the largest animal ever known to have existed. During the 20th century, the species was almost exterminated due to commercial whaling. The species has slowly recovered following the global whaling ban but it remains endangered and faces a number of serious threats including ship strikes and the impact of climate change. (Cá voi xanh là loài động vật lớn nhất từng được biết đến. Trong thế kỉ 20, loài này đã gần như bị tuyệt chủng do nạn săn bắt cá voi phục vụ thương mại. Loài này đã dần hồi phục sau lệnh cấm đánh bắt cá voi toàn cầu nhưng nó vẫn đang gặp nguy hiểm và đối mặt với một số mối đe doạ nghiêm trọng bao gồm các cuộc tấn công tàu và tác động của biến đổi khí hậu.)

Bên cạnh do, các đoạn còn lại cung cấp thêm các thông tin cơ bản về loài cá voi như đặc điểm bên ngoài (đoạn 2), tập quán sinh sống (đoạn 3), sinh con (đoạn 4) và các mối đe doạ đến cá voi (đoạn cuối) Câu 152: Đáp án B

Câu nào trong các cân sau là không đúng về đặc điểm vật lý (mô tả hình dáng) của cá voi xanh?

  1. Rất dễ phát hiện sự có mặt của cá voi xanh thông qua nhịp tim của nó.
  2. Cá voi xanh đực là con dài nhất trong loài.
  3. Tiếng kêu của cá voi xanh lớn hơn tiếng kêu của bất kì loài nào trên Trái Đất.
  4. Một cái vây nhỏ ở lưng cũng là đặc điểm nhận dạng của cá voi xanh.

Căn cứ vào thông tin đoạn 2:

Blue whales are simply enormous with most ranging in length from 24-30 m. The largest ever recorded was a gargantuan 33.5 m long. Females are up to 10 m longer than males. (Cá voi xanh là loài rất to lớn với chiều dài từ 24 - 30 mét. Con lớn nhất từng được ghi nhận là 1 con cá khổng lồ dài 33,5 mét. Con cái dài hơn 10 mét so với con đực)

Câu 153: Đáp án B

Theo đoạn văn, tại sao một con cá voi xanh lại không đơn độc khi di chuyển một mình?

  1. Bởi vì hầu hết các con cá voi xanh khác cũng di chuyển 1 mình.
  2. Bởi vì nó có thể liên lạc với những con cá voi xanh khác thông qua tiếng kêu rất lớn của chúng.
  3. Bởi vì chúng cùng chia sẻ khu vực kiếm ăn với những con cá voi xanh khác.
  4. Bởi vì những con cá voi xanh khác sẽ xuất hiện khi chúng đến được đích đến.

Căn cứ thông tin đoạn 3:

Blue whales mostly travel alone or in groups of 2-3. Larger groups of up to 60 whales have been reported and are probably associated with feeding grounds. However. the blue whale has the most powerful voice in the animal kingdom and its low-frequency sounds can travel in deep water over hundreds, or even thousands. Of miles. Under these circumstances, animals which may appear to us to be traveling alone may actually be in constant contact with one another.

(Cá voi xanh chủ yếu di chuyển một mình hoặc theo nhóm từ 2 - 3 con. Những nhóm lớn hơn đến 50 con cũng đã từng được báo cáo và có thể có liên quan đến khu vực kiếm ăn. Tuy nhiên, cá voi xanh là loài có tiếng kêu to nhất trong vương quốc các loài động vật và âm thanh tần số thấp của nó có thể di chuyển sâu trong nước qua hàng tram hay thậm chí hàng ngàn dặm. Trong những hoàn cảnh này, những loài động vật mà chúng ta tưởng đang đi một mình lại thật sự có thể liên lạc thường xuyên với nhau.) Câu 154: Đáp án A

Tử “wean" trong đoạn 4 có nghĩa là gì?

A. ngừng cho cá voi con bú sữa mẹ B. ngừng tăng trưởng nhanh như trước

C. bắt đầu sống độc lập D. bắt đầu tự tìm thức ăn Nghĩa của từ: wean = cai sữa mẹ

Câu 155: Đáp án C

Từ “entangled” trong đoạn 5 gần nghĩa nhất với từ nào?

A. gặp nguy hiểm B. biến mất

C. vướng vào, mắc vào bẫy (đánh cá) D. thu hút Từ đồng nghĩa: entangled (vướng vào, mắc vào) = ensnared

Like other large whales, blue whales are threatened by chemical and sound pollution, habitat loss, overfishing of krill, ship strikes and becoming entangled in fishing gear. (Giống như các con cá voi lớn khác, cá voi xanh đang bị đe doạ bởi ô nhiễm hoá chất và âm thanh, mất môi trường sống, đánh bắt quá mức các loài nhuyễn thể tấn công tàu và mắc vào dụng cụ đánh bắt cá.)

Câu 156: Đáp án B

Tử “ít” trong đoạn cuối đề cập đến danh từ nào?

A. độ sau B. nước

C. khối nước D. sự biến đổi khí hậu

Căn cứ vào thông tin đoạn cuối:

Từ "it” thay thế cho danh từ water.

Frontal zones are boundaries between different water masses, where water can rise from the depths, bringing with it large amounts of nutrients that stimulate the growth of phytoplankton and support substantial populations of prey species for whales. (Các khu vực phía trước là ranh giới giữa các khối nước khác nhau, nơi nước có thể dâng lên từ dưới đáy, mang theo nó một lượng lớn các chất dinh dưỡng kích thích sự phát triển của thực vật phù du và hỗ trợ đáng kể các quần thể loài thức ăn của cá voi xanh.) Câu 157: Đáp án C

Theo đoạn văn cuối, sự biến đổi khí hậu có ảnh hưởng như thế nào đến cá voi xanh?

  1. Nó làm tăng lượng axit trong đại dương, làm cho nguồn nước bị ô nhiễm.
  2. Nó tác động đến thuỷ triều chứa các chất dinh dưỡng để nuôi dưỡng các loài thức ăn của cá voi xanh.
  3. Nó khiến cá voi xanh phải di chuyển xa hơn xuống phía nam để tìm khu vực kiếm ăn.
  4. Nó tăng cường các vùng phía trước để mà cá voi xanh không thể di chuyển xung quanh được.

Căn cứ vào thông tin đoạn cuối:

Climate change could also have a major impact on its food supply, since global warming and associated ocean acidification may impact krill populations. In addition, frontal zones — critical whale habitats — are projected to move further south due to climate change. Frontal zones are boundaries between different water masses, where water can rise from the depths, bringing with it large amounts of nutrients that stimulate the growth of phytoplankton and support substantial populations of prey species for whales. Blue whales would have to migrate further [perhaps 200-500 km more] to reach and feed at these food-rich areas where they build up reserves to sustain themselves for the rest of the year. These longer migration paths could increase the energy costs of migration and reduce the duration of the main feeding season.

(Biến đổi khí hậu cũng có tác động lớn đến nguồn cung cấp thực phẩm của nó, vì sự nóng lên toàn cầu và sự axit hoá đại dương có liên quan có thể ảnh huởng đến quần thể các loài nhuyễn thể. Thêm vào đó, các vùng phía trước, môi trường sống quan trọng của cá voi, được dự kiến sẽ di chuyển xa hơn về phía nam do biến đổi khí hậu. Các khu vực phía trước là ranh giới giữa các khối nước khác nhau, nơi nước có thể dâng lên từ dưới đáy, mang theo nó một lượng lớn các chất dinh dưỡng kích thích sự phát triển của thực vật phù du và hỗ trợ đáng kể các quần thể loàí thức ăn của cá voi xanh. Cá voi xanh sẽ phải di chuyển xa hơn (có lẽ khoảng 200 - 500 km nữa) để tiếp cận và kiếm ăn ở các vùng giàu thức ăn này, nơi chúng tích luỹ dự trữ để duy trì bản thân cho đến cuối năm. Những con đường di cư dài hơn này có thể làm tăng năng lượng di cư và giảm thời gian mùa ăn chính.)

Câu 158: Đáp án A

Có thể suy ra từ đoạn văn rằng .

  1. Các hoạt động của con người là một trong những nguyên nhân hàng đầu cho sự suy giảm của quần thể loài cá voi xanh.
  2. Cá voi xanh là loài động vật có tuổi thọ lâu nhất.
  3. Mặc dù là loài động vật lớn nhất, cá voi xanh cũng là loài dễ bị tổn thương nhất trên Trái Đất.
  4. Cá voi xanh sống an toàn trong đại dương nhờ vào sự đoàn kết của chúng.

Căn cứ vào thông tin đoạn đầu và đoạn cuối:

During the 20th century, the species was almost exterminated due to commercial whaling. (Trong thế kỉ 20, loài này đã gần như bị tuyệt chủng do nạn săn bắt cá voi phục vụ thương mại.)

Like other large whales, blue whales are threatened by chemical and sound pollution, habitat loss, overflshing of krill, ship strikes and becoming entangled in fishing gear. (Giống như các con cá voi lớn

khác, cá voi xanh đang bị đe doạ bởi ô nhiễm hoá chất và âm thanh, mất môi trường sống, đánh bắt quá mức các loài nhuyễn thể tấn công tàu và mắc vào dụng cụ đánh bắt cá.)

Dịch bài

Cá voi xanh là loài động vật lớn nhất từng được biết đến. Trong thế kỉ 20, loài này đã gần như bị tuyệt chủng do nạn săn bắt cá voi phục vụ thương mại. Loài này đã dần hồi phục sau lệnh cấm đánh bắt cá voi toàn cầu nhưng nó vẫn đang gặp nguy hiểm và đối mặt với một sổ mối đe doạ nghiêm trọng bao gồm các cuộc tấn công tàu và tác động của biến đổi khi hậu.

Cá voi xanh là loài rất to lớn với chiều dài từ 24 - 30 mét. Con lớn nhất từng được ghi nhận là một con cá khổng lồ dài 33.5 mét. Con cái dài hơn 10 mét so với con đực. Và chúng có thể nặng tới 200 tấn. Chỉ cần suy nghĩ rằng: một con voi châu Phi trưởng thành chỉ nặng 6 tấn. Tim cá voi xanh là kích thước bằng một chiếc xe hơi nhỏ và nhịp của nó có thể được phát hiện cách hai dặm. Nhưng đó vẫn không là gì so với tiếng kêu của chủng. Cá voi xanh là loài động vật to nhất trên trái đất và tiếng kêu của chúng to hơn một động cơ phản lực: đạt tới 188 đecibel, trong khi động cơ của phản lực ‘chỉ’ 140 decibel. Ngoài kích thước khổng lồ của chúng, cá voi xanh có thể được xác định bởi vây lưng tương đối nhỏ của chúng, một bướu khá tròn (phần phía trước của hộp sọ), và khoảng 90 rãnh thông, đạt đến rốn. Chúng cũng có khoảng 300 - 400 tấm sừng hàm trên mỗi bên của miệng, có màu đen và có chiều dài từ 50 cm ở phía trước đến 100 cm ở phía sau.

Cá voi xanh chủ yếu di chuyển một mình hoặc theo nhóm từ 2 - 3 con. Những nhóm lớn hơn đến 60 con cũng đã từng được báo các và có thể có liên quan đến khu vực kiếm ăn. Tuy nhiên, cá voi xanh là loài có tiếng kêu to nhất trong vương quốc các loài động vật và âm thanh tầm số thấp của nó có thể di chuyển sâu trong nước qua hàng trăm hay thậm chi hang ngàn dặm. Trong những hoàn cảnh này, những loài động vật mà chúng ta tưởng đang đi một mình lại thật sự có thể liên lạc thường xuyên với nhau.

Khi mới sinh, một con cá voi xanh con cũng là đứa con lớn nhất trên trái đất: dài khoảng 8m và nặng khoảng 4 tấn. Chúng phát triển với tốc độ 90 kg mỗi ngày và cai sữa sau 7-8 tháng, một khi chúng đã đạt đến khoảng 15 m chiều dài. và có thể đi theo con đường di cư hình thường một mình. Chúng đạt đến sự trường thành về tinh dục từ 5-10 năm. Tốc độ tăng trưởng này là đáng kinh ngạc và có lẽ là nhanh nhất trong vương quốc động vật. Từ lúc được thụ thai đến cai sữa, nó đại điện cho sự gia tăng vài tỷ lần trong mô chỉ trong hơn một năm rưỡi. Giống như những con cá voi có tấm sừng khác, cá voi xanh không có răng nên rất khó để biết tuổi của nó nhưng các nhà khoa học tin rằng chúng sống đến ít nhất 50 tuổi.

Giống như các con cá voi lớn khác, cá voi xanh đang bị đe doạ bởi ô nhiễm hoá chất và âm thanh, mất môi trường sống, đánh bắt quá mức các loài nhuyễn thể, tấn công tàu và mắc vào dụng cụ đánh bắt cá. Biến đổi khí hậu cũng có tác động lớn đến nguồn cung cấp thực phẩm của nó, vì sự

nóng lên toàn cầu và sự axit hoá đại dương có liên quan có thể ảnh hưởng đến quần thể các loài nhuyễn thể. Thêm vào đó, các vùng phía trước, môi trường sống quan trọng của cá voi, được dự kiến sẽ di chuyển xa hơn về phía nam do biến đổi khí hậu. Các khu vực phía trước là ranh giới giữa các khối nước khác nhau, nơi nước có thể dâng lên từ dưới đáy, mang theo nó một lượng lớn các chất dinh dưỡng kích thích sự phát triển của thực vật phù du và hỗ trợ đáng kể các quần thể loài thức ăn của cá voi xanh. Cá voi xanh sẽ phải di chuyển xa hơn (có lẽ khoảng 200 - 500 km nữa) để tiếp cận và kiếm ăn ở các vùng giàu thức ăn này, nơi chúng tích luỹ dự trữ để duy trì bản thân cho đến cuối năm. Những con đường di cư dài hơn này có thể làm tăng năng lượng di cư và giảm thời gian mùa ăn chính. Khi các khu vực phía trước di chuyển về phía nam, chúng cũng di chuyển gần nhau hơn, làm giảm tổng diện tích của khu vực kiếm ăn.

Câu 159: Đáp án B

CHỦ ĐỀ UNDERSEA WORLD

Câu nào trong các câu sau là nội dung chính mà đoạn văn thảo luận?

  1. Mực nước biển sẽ trở thành tác nhân chính của sự tẩy trắng.
  2. Các rạn san hô sẽ bị tràn ngập bởi các đại dương đang dâng nước lên.
  3. Các rạn san hô có thể thoát khỏi tuyệt chủng nhờ vào sự tăng lên của mực nước biển.
  4. Sự nóng lên toàn cầu sẽ làm mực nước biển tăng lên.

Căn cứ vào thông tin đoạn 1:

The study suggests that reefs - which are already suffering serious degradation because the world’s seas are warming and becoming more acidic - could also become overwhelmed by rising oceans.

Nghiên cứu cho thấy rằng các rạn san hô - đã bị suy thoái nghiêm trong bởi vì biển trên thế giới đang ấm lên và trở nên chua hơn - cũng có thể bị tràn ngập bởi những đại dương đang dâng nước lên.)

Câu 160: Đáp án A

Từ “compensate” trong đoạn 1 có thể có nghĩa là .

A. đền bù, bồi thường B. theo kịp

C. phát triển D. có lợi ích

Từ đồng nghĩa: compensate (đền bù, bu đắp) = recompense

They have found that most are incapable of growing quickly enough to compensate for rising sea levels triggered by global warming. (Họ đã phát hiện ra rằng hầu hết san hô không có khả năng phát triển đủ nhanh để bù đắp cho mực nước biển dâng cao do sự nóng lên toàn cầu.)

Câu 161: Đáp án B

Các nhà khoa học tại Đại học Ehreter đã tìm thấy điều gì trong nghiên cứu của họ?

  1. Các rạn san hô nhiệt đới đang tăng lên nhanh hơn các rạn san hô ở Thái Bình Dương.
  2. Đa số các rạn san hô nhiệt đới không thể bắt kịp tốc độ tăng lên của mực nước biển.
  3. Nhiều rạn san hô vẫn đang phát triển mặc dù sự thoái hoá của chúng.
  4. Sự tăng nhanh của mực nước biển không ảnh hưởng đến mật độ các rạn san hô.

Căn cứ vào thông tin đoạn 2:

The research - led by scientists at Exeter University and published in Nature this week - involved studying growth rates for more than 200 tropical western Atlantic and Indian Ocean reefs. It was found only 9% of these reefs had the ability to keep up with even the most optimistic rates of sea-level rises forecast by the Intergovernmental Panel on Climate Change. "For many reefs across the Caribbean and Indian Ocean regions, where the study focused, rates of growth are slowing due to coral reef degradation,” said Professor Chris Perry, of Exeter University. "Meanwhile, rates of sea-level rise are increasing - and our results suggest reefs will be unable to keep up. As a result, water depths above most reefs will increase rapidly through this century."

(Nghiên cứu - được dẫn dắt bởi các nhà khoa học tại Trường Đại học Exeter và được công bố trên tờ Nature tuần này - liên quan đến tốc độ tăng trưởng của hơn 200 rạn san hô vùng nhiệt đới Tây Đại Tây Dương và Ấn Độ Dương. Chỉ có 9% trong số các rạn san hô này có khả năng theo kịp ngay cả những mức độ lạc quan nhất của mực nước biển dâng do dự báo của Ủy ban liên chính phủ về biến đối khí hậu. “Đối với nhiều rạn san hô trên vùng Caribe" và Ấn Độ Dương, nơi nghiên cứu tập trung, tốc độ tăng trưởng chậm lại do suy thoái rạn san hô”, Giáo sư Chris Perry, thuộc Trường Đại học Exeter cho biết. “Trong khi đó, tỷ lệ mực nước biển dâng đang gia tăng - và kết quả của chúng tôi cho thấy rạn san hô sẽ không thể theo kịp. Kết quả là, độ sâu của nước trên hầu hết các rạn san hô sẽ tăng nhanh qua thế kỷ này.)

Câu 162: Đáp án D

Theo đoạn văn, 2 nhân tốgây ra sự tăng lên của mực nước biển là .

  1. sự nóng lên toàn cầu và sự băng giá
  2. thời tiết cực đoan và biến đổi khí hậu
  3. biến đổi khí hậu và sự mở rộng của băng
  4. biến đổi khí hậu và băng tan

Căn cứ thông tin đoạn 3:

Two key factors are involved: climate change is making ocean water warmer and so it expands. And as ice sheets and glaciers melt, they increase amounts of water in the oceans.

(Hai yếu tố chính có liên quan là biến đổi khí hậu đang làm cho nước biển ấm hơn và do đó nó mở rộng. Và khi những tảng băng và sông băng tan chảy, chúng làm tăng lượng nước trong đại dương.) Câu 163: Đáp án A

Cụm từ “these effects” trong đoạn 4 đề cập đến .

  1. sự nóng lên của đại dương và axit đại dương hoá
  2. sự yếu đi của các rạn san hô và sự nóng lên của đại dương
  3. sự yếu đi của các rạn san hô và axit đại dương hoá
  4. sự nóng lên của đại dương và sự hấp thụ CO2

Căn cứ thông tin đoạn 4:

Từ “these effects" đang nhắc đến sự nóng lên của đại dương và axit đại dương hóa.

At the same time, reefs are being weakened by ocean warming and also by ocean acidification, triggered as the seas absorb more and more carbon dioxide. These effects lead to bleaching events that kill off vast stretches of coral and limits their ability to grow.

(Đồng thời, các rạn san hô đang bị suy yếu bởi sự nóng lên của đại dương và cũng bởi sự axit hóa đại dương, được kích hoạt khi các vùng biển hấp thụ hàng càng nhiều khí CO2. Những tác động này dẫn đến các sự kiện tẩy trắng diệt hết các dải san hô rộng lớn và hạn chế năng khả năng phát triển của chúng.)

Câu164: Đáp án C

Từ “inundofion”gần nghĩa nhất với từ .

A. hạn hán B. sự tuyệt chủng C. lũ lụt D. sóng thần Từ đồng nghĩa: inundation (sự ngập lụt) = flood

“Our predictions, even under the best case scenarios, suggest that by 2100, the inundation of reefs will expose coastal communities to significant threats of shoreline change,” said co-author Prof Peter Mumhy of Queensland University. (“Dự đoán của chúng tôi, ngay cả trong các kịch bản tốt nhất, cho thấy rằng vào năm 2100, sự ngập lụt các rạn san hô sẽ đặt các cộng đồng ven biển vào các mối đe dọa đáng kể về sự thay đổi bờ biển”, Giáo sư Peter Mumby thuộc trường Đại học Queensland cho biết.) Câu 165: Đáp án C

Tác giả hàm ý trong đoạn cuối rằng .

  1. Ngay cả trong dự đoán lạc quan nhất, các rạn san hô vẫn sẽ bị tuyệt chủng.
  2. Kết quả của cuộc nghiên cứu thì nghiêm trọng hơn những gì các nhà khoa học đã dự đoán.
  3. Các hoạt động của con người không chỉ ảnh hưởng đến thế giới dưới biển mà còn đặt chính họ vào nguy hiểm.
  4. Con người thường khai thác tài nguyên thiên nhiên ở các quốc đảo và vùng lãnh thổ.

Căn cứ thông tin đoạn cuối:

“Our predictions, even under the best case scenarios, suggest that by 2100, the inundation of reefs will expose coastal communities to significant threats of shoreline change,” said co-author Prof Peter

Mumby of Queensland University. This point was backed by US marine scientist Ilsa Kuffner writing in a separate comment piece for Nature. “The implications of the study are dire. Many island nations and territories are set to quickly lose crucial natural resources.”

(“Dự đoán của chúng tôi, ngay cả trong các kịch bản tốt nhất, cho thấy rằng vào năm 2100, sự ngập lụt các rạn san hô sẽ đặt các cộng động ven biển vào các mối đe dọa đáng kể về sự thay đổi bờ biển”, giáo sư Peter Mumby thuộc trường Đại học Queensland cho biết. Điểm này được ủng hộ bởi nhà khoa học biển người Mỹ Ilse Hefner viết trong một phần bình luận riêng cho tờ Nature.” Hàm ý của nghiên cứu rất nghiêm trọng. Nhiều quốc đảo và vùng lãnh thổ được thiết lập đã nhanh chóng làm mất đi các nguồn tài nguyên thiên nhiên quan trọng".)

Như vậy, tác giả đã hàm ý rằng, các hoạt động của con người như thành lập các quốc đảo và vùng lãnh thổ đã làm mất đi các nguồn tài nguyên thiên nhiên quan trọng và việc các rạn san hô bị ảnh hưởng của biến đổi khí hậu và nóng lên toàn cầu sẽ khiến các vùng duyên hải chịu nhiều mối đe doạ.

Bài dịch

Các nhà khoa học đã phát hiện một mối đe dọa mới đối với các rạn san hô đang có nguy cơ tuyệt chủng của thế giới. Họ đã phát hiện ra rằng hầu hết san hô không có khả năng phát triển đủ nhanh để bù đắp cho mực nước biển dâng cao do sự nóng lên toàn cầu. Nghiên cứu cho thấy rằng các rạn san hộ - đã bị suy thoái nghiêm trọng bởi vì biển trên thế giới đang ấm lên và trở nên chua hơn - cũng có thể bị tràn ngập bởi những đại dương đang dâng nước lên.

Nghiên cứu - được dẫn dắt bởi các nhà khoa học tại Đại học Exeter và được công bố trên tờ Nature tuần này - liên quan đến tốc độ tăng trưởng của hơn 200 rạn san hô vùng nhiệt đới Tây Đại Tây Dương và Ấn Độ Dương. Chỉ có 9% trong số các rạn san hô này có khả năng theo kịp ngay cả những mức độ lạc quan nhất của mực nước biển dâng do dự báo của Ủy ban liên chính phủ về biến đổi khí hậu. “Đối với nhiều rạn san hô trên vùng Caribê và Ấn Độ Dương, nơi nghiên cứu tập trung, tốc độ tăng trưởng chậm lại do suy thoái rạn san hô”, Giáo sư Chris Perry, thuộc trường Đại học Exeter cho biết. “Trong khi đó, tỷ lệ mực nước biển dâng đang gia tăng - và kết quả của chúng tôi cho thấy rạn san hô sẽ không thể theo kip. Kết quả là, độ sâu của nước trên hầu hết các rạn san hô sẽ tăng nhanh qua thế kỷ này. Mực nước hiện đã dâng cao vài inch trong thế kỷ qua và các phép đo cho thấy tốc độ tăng này hiện đang tăng1ên đáng kể. Hai yếu tố chính có liên quan là biến đổi khí hậu đang làm cho nước hiện ấm hơn và do đó nó mở rộng. Và khi những tảng băng và sông băng tan chảy, chúng làm tăng lượng nước trong đại dương.

Đồng thời, các rạn san hô đang bị suy yếu bởi sự nóng lên của đại dương và cũng bởi sự axit hóa đại dương, được kích hoạt khi các vùng biển hấp thụ ngày càng nhiều khí CO2. Những tác động này dẫn đến các sự kiện tẩy trắng diệt hết các dải san hô rộng lớn và hạn chế khả năng phát triển của chúng.

“Dự đoán của chúng tôi, ngay cả trong các kịch bản tốt nhất, cho thấy rằng vào năm 2100, sự ngập lụt các rạn san hô sẽ đặt các cộng động ven biển vào các mối đe dọa đáng kể về sự thay đổi bờ biến”, giáo sư Peter Mumhy thuộc Đại học Queensland cho biết. Điểm này được ủng hộ bởi nhà khoa học biển người Mỹ Ilsa Kuffner viết trong một phần bình luận riêng cho tờ Nature. “Hàm ý của nghiên cứu rất nghiêm trọng. Nhiều quốc đảo và vùng lãnh thổ được thiết lập đã nhanh chóng làm mất đi các nguồn tài nguyên thiên nhiên quan trọng."

Câu 166: Đáp án A

CHỦ ĐỀ VỀ NATURE IN DANGER

Câu nào trong các câu sau có thể là tiêu đề phù hợp nhất cho đoạn văn?

  1. Ô nhiễm nhựa – Vấn đề và giải pháp.
  2. Những tác hại của túi nhựa lên môi trường.
  3. Ô nhiễm nhựa - Chúng ta nên làm gì?
  4. Túi nhựa - mối đe doạ mới cho môi trường.

Căn cứ vào ý chính của từng đoạn:

Tác giả cung cấp cho chúng ta thực trạng về vấn đề ô nhiễm nhựa, hậu quả và các giải pháp để làm giảm mức độ ô nhiễm. Vì vậy, đáp án A sẽ là tiêu đề phù hợp nhất.

Câu 167: Đáp án D

Từ đồng nghĩa của từ "repercussions” trong đoạn 1 là từ nào?

A. tình huống B. sự can thiệp

C. sự ô nhiễm D. hậu quả

Từ đồng nghĩa: repercussions (hậu quả) = consequence

But, most of us are blissfully unaware of the repercussions that are occurring and will take place in the future because of the plastic bags. (Tuy nhiên, hầu hết chúng ta không biết gì về hậu quả đang xảy ra và sẽ diễn ra trong tương lai vì các túi nhựa này.)

Câu 168: Đáp án D

Theo đoạn 2, câu nào là không đúng về thực trạng của túi nhựa?

  1. Đất nước càng đông dân thì ô nhiễm nhựa càng nghiêm trọng.
  2. Túi nhựa thường được dùng vì sự tiện lợi của chúng.
  3. Việc nhập khẩu rác thải nhựa từ các nước phát triển làm cho vấn đề nghiêm trọng hơn.
  4. Chính phủ không nghiêm cấm sử dụng túi nhựa ở các cửa hàng.

Căn cứ thông tin đoạn 2:

Every once in a while, the government passes out an order banning store owners from providing plastic bags to customers for carrying their purchases, with little lasting effect. Plastic bags are very

popular with both retailers as well as consumers because they are cheap, strong, lightweight, functional, as well as a hygienic means of carrying food as well as other goods. About a hundred billion plastic bags are used every year in the U.S. alone. And then, when one considers the huge economies and populations of India, China, Europe, and other parts of the world, the numbers can be staggering. The problem is further exacerbated by the developed countries shipping off their plastic waste to developing countries like India.

(Thỉnh thoảng, Chính phủ ra lệnh cấm các chủ cửa hàng cung cấp túi nhựa cho khách hàng để thực hiện việc chứa đựng hàng hoá của họ, với ít hiệu quả lâu dài. Túi nhựa rất phổ biến với cả các nhà bán lẻ cũng như với người tiêu dùng bởi vì chúng rẻ, chắc, nhẹ, chức năng, cũng như các phương tiện vệ sinh để mang thực phẩm cũng như các hàng hóa khác. Khoảng một trăm tỷ túi ni-lông được sử dụng hàng năm chỉ riêng tại Hoa Kỳ. Và rồi, khi ta xem xét các nền kinh tế và dân số khổng lồ của Ấn Độ, Trung Quốc, châu Âu và các nước khác trên thế giới, những con số có thể đáng kinh ngạc. Vấn đề càng trở nên trầm trọng hơn bởi các nước phát triển vận chuyển chất thải nhựa của họ đến các nước đang phát triển như Ấn Độ.)

Câu 169: Đáp án D

Những câu sau đây là tác hại của túi nhựa lên môi trường, ngoại trừ .

  1. Chúng làm cho đất, nước và không khí bị ô nhiễm.
  2. Các động vật dưới nước có thể chết vì ăn nhầm túi nhựa.
  3. Phải mất rất nhiều thời gian thì các túi nhựa mới bị phân huỷ.
  4. Con người không sống đủ lâu để kiểm tra liệu túi nhựa có bị phân huỷ hay không.

Căn cứ thông tin đoạn 3:

Once they are used, most bags go into landfills. Each year, more and more bags are ending up littering the environment. Once they become litter, plastic bags find their way into our waterways, parks, beaches, and streets. And, if they are burned, they infuse the air with toxic fumes. About 100,000 animals, such as dolphins, turtles, whales, penguins are killed every year due to these bags. Many animals ingest plastic bags, mistaking them for food, and therefore, die. And worse, the ingested plastic bag remains intact even after the death and decomposition of the animal. Thus, it lies around in the landscape where another victim may ingest it. One of the worst environmental effects is that they are non-biodegradable. The decomposition takes about 400 years. No one will live so long to witness the decomposition of plastic!

(Một khi chúng được sử dụng, hầu hết các túi sẽ đi vào bãi rác. Mỗi năm, ngày càng có nhiều túi được kết thúc bằng việc xả rác môi trường. Một khi chúng trở thành rác rưởi, túi ny-lon tìm đường vào

các tuyến đường thủy, công viên, bãi biển và đường phố của chúng ta. Và, nếu chúng bị đốt cháy, chúng sẽ truyền vào không khí với khói độc. Khoảng 100.000 động vật, chẳng hạn như cá heo, rùa, cá voi, chim cánh cut bị giết mỗi năm do các túi này. Nhiều loài động vật ăn phải túi nhựa, nhầm chúng với thức ăn, và do đó, bị chết. Và tệ hơn, túi nhựa ăn vào vẫn còn nguyên vẹn ngay cả sau cái chết và sự phân hủy của con vật. Vì vậy, nó nằm xung quanh trong cảnh quan nơi một nạn nhân khác có thể ăn nó. Một trong những tác động môi trường tồi tệ nhất là chúng không thể phân hủy sinh học. Quá trình phân hủy mất khoảng 400 năm. Không ai sống quá lâu như vậy để chứng kiến sự phân hủy của nhựa.)

Như vậy, việc con người không sống đủ lâu để chứng kiến túi nhựa bị phân huỷ không phải là tác hại của chúng lên môi trường.

Câu 170: Đáp án A

Từ “intact” trong đoạn 3 gần nghĩa nhất với từ nào?

A. nguyên vẹn, không bị phá huỷ B. phá vỡ

C. gây hại D. nguy hiểm

Từ đồng nghĩa: intact (nguyên vẹn) = undamaged

And worse, the ingested plastic bag remains intact even after the death and decomposition of the animal. (Và tệ hơn, túi nhựa ăn vào vẫn còn nguyên vẹn ngay cả sau cái chết và sự phân hủy của con vật) Câu 171: Đáp án C

Theo như đoạn văn, dầu mỏ được mô tả là 1 nguồn tài nguyên mà .

  1. làm tăng giá của sản phẩm.
  2. bị lãng phí vào các nhu cầu năng lượng của cuộc sống.
  3. không thể thiếu trong hầu hết các hoạt động của thế giới hiện đại.
  4. được thay thế bởi các nguồn năng lượng thay thế khác trong sản xuất túi nhựa.

Căn cứ thông tin đoạn 4:

Petroleum products are diminishing and getting more expensive by the day, since we have used this non-renewable resource increasingly. And to make plastic, about 60-100 million barrels of oil are needed every year around the world. Surely, this precious resource should not be wasted on producing plastic bags. should it? Petroleum is vital for our modern way of life. It is necessary for our energy requirements - for our factories, transportation, heating, lighting, and so on. Without viable alternative sources of energy yet on the horizon, if the supply of petroleum were to be turned off, it would lead to practically the entire world grinding to a halt.

(Các sản phẩm dầu mỏ đang giảm dần và ngày càng đẳt đỏ hơn vì chúng ta đã sử dụng nguồn tài nguyên không tái tạo này ngày càng tăng. Và để sản xuất nhựa, cần khoảng 60 -100 triệu thùng dầu mỗi năm trên khắp thế giới. Chắc chắn, nguồn tài nguyên quý giá này không nên lãng phí vào sản xuất túi nhựa, phải không? Dầu khí rất quan trọng cho lối sống hiện đại của chúng ta. Nó là cần thiết cho các yêu

cầu năng lượng - cho các nhà máy, giao thông vận tải, sưởi ấm, ánh sáng, và những thứ như thế. Nếu không có nguồn năng lượng thay thế khả thi nào trên thế giới, nếu việc cung cấp xăng dầu bị cạn kiệt, nó sẽ dẫn đến thực tế toàn bộ thế giới sẽ dừng lại.)

Câu 172: Đáp án B

Từ “it” trong đoạn cuối đề cập đến danh từ nào?

A. người thu tiền B. cái túi C. việc mua sắm D. một vật thay thế tốt

Căn cứ thông tin đoạn cuối:

Từ “it" thay thế cho danh từ the bag.

A tote bag can make a good substitute for carrying groceries and the shopping. You can keep the bag with the cashier, and then put your purchases into it instead of the usual plastic bag. (Một cái túi đi chợ có thể là một thay thể tốt để mang hàng hóa và đồ mua sắm. Bạn có thể để cái túi chỗ người tính tiền, và sau đó đặt hàng hoá của bạn vào nó thay vì túi nhựa thông thường)

Câu 173: Đáp án A

Tác giả để xuất gì trong đoạn cuối?

  1. Túi nhựa nên bị hạn chế nhiều nhất có thể để bảo vệ Trái Đất khỏi những tác hại của chúng.
  2. Mỗi cá nhân nên tái sử dụng túi nhựa của họ để tiết kiệm tiền mua hàng hoá.
  3. Chính phủ nên hợp tác với mọi công dân trong việc dọn sạch túi nhựa trong các đại dương bị ô nhiễm.
  4. Giữ cho môi trường trong sạch ngay lúc đầu dễ hơn là bảo vệ nó sau khi đã bị ô nhiễm.

Căn cứ thông tin đoạn cuối:

A tote bag can make a good substitute for carrying groceries and the shopping. You can keep the bag with the cashier, and then put your purchases into it instead of the usual plastic bag. Recycling the bags you already have is another good idea. These can come into use for various purposes, like holding your garbage, instead of purchasing new ones. While governments may be working out ways to lessen the impact of plastic bags on the environment: however, each of us should shoulder some of the responsibility for this problem, which ultimately harms us. Plastics are not only non-biodegradable, but are one of the major pollutants of the sea. For a clean and green environment, try to use alternatives to plastic whenever and wherever possible. Cut down your use of plastic, and do your bit to save our planet.

(Một cái túi đi chợ có thể là một thay thế tốt để mang hàng hoá và đồ mua Sắm. Bạn có thể để cái túi chỗ người tính tiền, và sau đó đặt hàng hóa của bạn vào nó thay vì túi nhựa thông thường. Tái chế chiếc túi bạn đã có là một ý tưởng hay. Những chiếc túi này có thể được sử dụng cho các mục đích khác nhau, như đựng rác thải, thay vì mua những cái mới. Trong khi các chính phủ có thể đang tìm cách giảm thiểu tác động của túi nilon lên môi trường; tuy nhiên, mỗi người chúng ta phải gánh vác trách nhiệm đối với vấn đề này, cái mà cuối cùng cũng sẽ gây hại cho chúng ta. Nhựa không chỉ không phân hủy sinh học

mà còn là một trong những chất ô nhiễm chính của biển. Để môi trường xanh và sạch, hãy thử sử dụng các giải pháp thay thế cho nhựa bất cứ khi nào và bất cứ nơi nào có thể. Cắt giảm việc sử dụng nhựa của bạn, và đóng góp một chút công sức để cứu hành tinh của chúng ta.)

Tạm dịch

Túi nhựa được sử dụng bởi tất cả mọi người. Từ một nhà cung cấp rau đến một cửa hàng thiết kế, mọi người dường như đều sử dụng chúng. Mặc dù chúng là một trong những tiện nghi hiện đại mà chúng ta dường như không thể làm mà không có, nhưng chúng lại gây ô nhiễm, giết chết động vật hoang dã, và sử dụng các nguồn tài nguyên quý giá của Trái Đất. Tuy nhiên, hầu hết chúng ta đều không biết gì về hậu quả đang xảy ra và sẽ diễn ra trong tương lai bởi vì các túi nhựa.

Thỉnh thoảng, chính phủ ra lệnh cấm các chủ cửa hàng cung cấp túi nhựa cho khách hàng để thực hiện việc chứa đựng hàng hoá của họ, với ít hiệu quả lâu dài. Túi nhựa rất phổ biến với cả các nhà bán lẽ cũng như với người tiêu dùng bởi vì chúng rẻ, chắc, nhẹ, chức năng, cũng như các phương tiện vệ sinh để mang thực phẩm cũng như các hàng hóa khác. Khoảng một trăm tỷ túi ni- lông được sử dụng hàng năm chỉ riêng tại Hoa Kỳ. Và rồi, khi ta xem xét các nền kinh tế và dân số khổng lồ của Ấn Độ, Trung Quốc, châu Âu và các nơi khác trên thế giới, những con số co thể đáng kinh ngạc. Vấn đề càng trở nên trầm trong hơn bởi các nước phát triển vận chuyển chất thải nhựa của họ đến các nước đang phát triển như Ấn Độ.

Một khi chúng được sử dụng, hầu hết các túi sẽ đi vào bãi rác. Mỗi năm, ngày càng có nhiều túi được kết thúc bằng việc xả rác môi trường. Một khi chúng trở thành rác rưởi, túi ny-lon tìm đường vào các tuyến đường thủy, công viên, bãi biển và đường phố của chúng ta. Và, nếu chúng bị đốt cháy, chúng sẽ truyền vào không khí với khói độc. Khoảng 100.000 động vật, chẳng hạn như cá heo, rùa, cá voi, chim cánh cụt bi giết mỗi năm do ăn túi này. Nhiều loài động vật ăn phải túi nhựa, nhầm chúng với thức ăn, và do đó bị chết. Và tệ hơn, túi nhựa ăn vào vẫn còn nguyên vẹn ngay cả sau cái chết và sự phân hủy của con vật. Vì vậy, nó nằm xung quanh trong cảnh quan nơi một nạn nhân khác có thể ăn nó. Một trong những tác động môi trường tồi tệ nhất là chúng không thể phân hủy sinh học. Quá trình phân hủy mất khoảng 400 năm. Không ai sống quá lâu như vậy để chứng kiến sự phân hủy của nhựa. Vì vậy, hãy bảo vệ môi trường cho các thế hệ trong tương lai cũng như cho các loài động vật.

Các sản phẩm dầu mỏ đang giảm dần và ngày càng đắt đỏ hơn vì chúng ta đã sử dụng nguồn tài nguyên không tái tạo này ngày càng tăng. Và để sản xuất nhựa, cần khoảng 60-100 triệu thùng dầu mỗi năm trên khắp thế giới. Chắc chắn, nguồn tài nguyên quý giá này không nên lãng phí vào sản xuất túi nhựa, phải không? Dầu khí rất quan trọng cho lối sống hiện đại của chúng ta. Nó là cần thiết cho các yêu cầu năng lượng - cho các nhà máy, giao thông vận tải, sưởi ấm, ánh

sáng, và những thứ như thế. Nếu không có nguồn năng lượng thay thế khả thi nào trên thế giới, nếu việc cung cấp xăng dầu bị cạn kiệt, nó

sẽ dẫn đến thực tế toàn bộ thế giới sẽ dừng lại.

Vậy, giải pháp nào có thể được thực hiện? Một cái túi đi chợ có thế là một thay thể tốt để mang hàng hoá và đồ mua sắm. Bạn có thể để cái túi chỗ người tính tiền, và sau đó đặt hàng hoá của bạn vào nó thay vì túi nhựa thông thường. Tái chế chiếc túi bạn đã có là một ý tưởng hay. Những chiếc túi này có thể được sử dụng cho các mục đích khác nhau, như đựng rác thải, thay vì mua những cái mới. Trong khi các chính phủ có thể đang tìm cách giảm thiểu tác động của túi nilon lên môi trường; tuy nhiên, mọi người chúng ta phải gánh vác trách nhiệm đối với vấn đề này, cái mà cuối cùng cũng sẽ gây hại cho chúng ta. Nhựa không chỉ không phân hủy sinh học mà còn là một trong những chất ô nhiễm chính của biển. Đế môi trường xanh và sạch, hãy thử sử dụng các giải pháp thay thế cho nhựa bất cứ khi nào và bất cứ nơi nào có thế. Cắt giảm việc sử dụng nhựa của bạn, và đóng góp một chút công sức để cứu hành tinh của chúng ta.

Câu 174: Đáp án C CHỦ ĐỀ ANIMALS

Đoạn văn chủ yếu thảo luận gì ?

  1. Thông tin chứa trong pheromones
  2. Sự di cư rất đông của kiến
  3. Làm thế nào kiến đánh dấu và đi theo một dấu vết hóa học
  4. Các loài kiến khác nhau trên thế giới

Câu 175: Đáp án D

Từ “in termittenfly” ở đoạn 1 có ý nghĩa gần nhất với .

A. roughly (adv): thô B. incorrectly (adv): không đúng

C. rapidly (adv): nhanh D. periodically (adv): định kỳ

Căn cứ vào ngữ cảnh của câu:

“As a worker ant returns home after finding a source of food, it marks the route by intermittently touching its stinger to the ground and depositing a tiny amount of trail pheromone - a mixture of chemicals that delivers diverse messages as the context changes." (Khi một con kiến thợ trở về nhà sau khi tìm ra nguồn thức ăn, nó đánh dấu đường đi bằng chạm ngòi của nó ngắt quãng trên mặt đất và để một lượng pheromone nhỏ - một hỗn hợp các chất hóa học cung ứng các thông điệp đa dạng khi bối cảnh thay đổi)

=> “intermittently” ~ periodically: ngắt quãng, định kỳ

Câu 176: Đáp án A

Cụm từ “the one” để cập đến một .

A. message: thông điệp B. food trail: throng kiếm ăn

C. dead ant: con kiến chết D. species: loài

Căn cứ vào thông tin trong câu sau:

“Unlike some other messages, such as the one arising from a dead ant, a food trail has to be kept secret from members of other species.(Không giống như một số thông điệp khác, chẳng hạn như một thông điệp phát sinh từ một con kiến chết, một con đường kiếm ăn phải được giữ bí mật khỏi các loài khác)

=> “the one" nhắc đến “message”: Unlike some other messages, such as the one arising from a dead ant

Câu 177: Đáp án D

Theo đoạn văn này, tại sao kiến lại sử dụng các hợp chất khác nhau như dấu vết pheromones?

  1. Để giảm độ nhạy cảm của chúng đối với một số hóa chất
  2. Để chỉ ra cách thức ăn bao xa
  3. Thu hút các loại kiến khác nhau
  4. Để bảo vệ dấu vết của chúng khỏi các loài khác

Thông tin: a food trail has to be kept secret from members of other species.

Câu 178: Đáp án B

Tác giả đề cập đến dấu vết pheromone của kiến cắn lá ở đoạn 2 để chỉ ra .

  1. Một loại kiến phổ biến ở nhiều nơi trên thế giới
  2. Cần rất ít Pheromone để đánh dấu một dấu vết
  3. Các loại pheromones khác nhau ma kiến có thể sản sinh
  4. Một số kiến có thể sản sinh đến một miligarn pheromone

Căn cứ vào thông tin trong câu sau:

“Investigators working with the trail pheromone of the leafcutter ant Atta texana calculated that one milligram of this substance would suffice to lead a column of ants three times around Earth.” (Các nhà điều tra làm việc với dấu vết pheromone của loài kiến cắn lá Atto texono tính toán rằng một miiigarn của chất này sẽ đủ để dẫn một dãy kiến đi ba lần vòng quanh Trái đất.)

Câu 179: Đáp án A

Theo đoạn văn, kiến được hướng dẫn bằng pheromone như thế nào?

  1. Chúng có thể cảm nhận hơi qua ăngten của chúng.
  2. Chúng tránh không gian hơi bằng cách di chuyển theo một đường thẳng.
  3. Chúng tập trung vào mùi thức ăn.
  4. Chúng đi theo một con kiến quen thuộc với dấu vết.

Căn cứ vào thông tin trong câu sau:

“The vapor of the evaporating pheromone over the trail guides an ant along the way, and the ant detects this signal with receptors in its antennae.” (Hơi của pheromone bay hơi trên dấu vết sẽ đến một con kiến dọc theo con đường, và con kiến phát hiện ra tín hiệu này với các thụ thể trong ăngten của nó.) Câu 180: Đáp án D

Theo đoạn văn, lượng pheromone cao nhất được tìm thấy .

A. trong nguồn thức ăn B. trong các thụ thể của kiến

C. dưới đất dọc theo dấu vết D. ngay phía trên dấu vết

Căn cứ vào thông tin trong câu sau:

“A trail pheromone will evaporate to furnish the highest concentration of vapor right over the trail,” (Một dấu vết pheromone sẽ bay hơi để cung ứng nồng đó hơi cao nhất trên dấu vết.)

Bài dịch

Nhiều con kiến đi kiếm thức ăn với số lượng lớn và di cư nhiều; những hoạt động này diễn ra bởi một con kiến tạo ra một dấu vết trên mặt đất để những con khác đi theo. Khi một con kiến thợ trở về nhà sau khi tìm ra nguồn thức ăn, nó đánh dấu đường đi bằng chạm ngòi của nó ngắt quãng trên mặt đất và để một lượng pheromone nhỏ - một hỗn hợp các chất hóa học cung ứng các thông điệp da dạng khi bối cảnh thay đổi. Những dấu vết này không kết hợp thông tin phương hướng và có thể được đi theo sau bởi các con kiến khác ở cả hai hướng.

Không giống như một số thông điệp khác, chẳng hạn như một thông điệp phát sinh từ một con kiến chết, một con đường kiếm ăn phải được giữ bí mật khỏi các loài khác. Không có gì đáng ngạc nhiên khi kiến sử dụng nhiều loại hợp chất như dấu vết pheromones. Kiến có thể rất nhạy cảm với những tín hiệu này. Các nhà điều tra làm việc với dấu vết pheromone của loài kiến cắn lá Atta texana tính toán rằng một miligam của chất này sẽ đủ để dẫn một dãy kiến đi ba lần vòng quanh Trái Đất.

Hơi của pheromone bay hơi trên dấu vết sẽ dẫn một con kiến dọc theo con đường, và con kiến phát hiện ra tín hiệu này với các thụ thể trong ăngten của nó. Một dấu vết pheromone sẽ bay hơi để cung ứng nồng độ hơi cao nhất trên dấu vết, trong cái gọi là không gian hơi. Khi đi theo dấu vết, kiến di chuyển sang phải và trái, dao động từ bên này sang bên kia đường thẳng của dấu vết, đưa ăngten đầu tiên và sau đó là ăngten khác vào không gian hơi. Khi con kiến di chuyển sang phải, ăng ten bên trái của nó sẽ đến không gian hơi. Các tín hiệu nhận được làm cho nó ngoặt sang trái, và con kiến phía sau đó theo cách này cho đến khi các ăng-ten bên phải của nó đến không gian hơi. Sau đó nó quay về phía bên phải, và tới lui theo dấu vết.

Câu 181: Đáp án B

CHỦ ĐỀ CONSERVATION

Đoạn văn này chủ yếu thảo luận về điều gì?

  1. Định nghĩa về nạn phá rừng.
  2. Những tác động tiêu cực của nạn phá rừng.
  3. Những nguyên nhân vì sao nạn phá rừng lại xảy ra thường xuyên.
  4. Những giải pháp hiệu quả để xử lý nạn phá rừng trên khắp thế giới.

Căn cứ thông tin đoạn 1:

The loss of trees and other vegetation can cause climate change, desertification, soil erosion, fewer crops, flooding, increased greenhouse gases in the atmosphere, and a host of problems for indigenous people. (Tình trạng mất cây và thảm thực vật khác có thể gây ra biến đổi khí hậu, sa mạc hóa, xói mòn đất, ít vụ mùa, lũ lụt, tăng khí nhà kính trong bầu khí quyển, và một loạt các vấn để cho người dân bản địa.)

Như vậy, đoạn văn đang thảo luận về các tác động tiêu cực của nạn phá rừng lên môi trường và con người.

Câu 182: Đáp án C

Từ "indigenous” trong đoạn 1 có thể có nghĩa là .

A. nước ngoài B. nông dân C. bản địa D. vùng núi Từ đồng nghĩa: indigenous (ban địa) = native

The loss of trees and other vegetation can cause climate change, desertification, soil erosion, fewer crops, flooding, increased greenhouse gases in the atmosphere, and a host of problems for indigenous people. (Tình trạng mất cây và thảm thực vật khác có thể gây ra biến đổi khí hậu, sa mac hóa, xói mòn đất, ít vụ mùa, lũ lụt, tăng khí nhà kính trong bầu khí qnyển, và một loạt các vấn đề cho người dân bản địa.)

Câu 183: Đáp án D

Câu nào trong các câu sau không được nhắc đến như là nguyên nhân của nạn phá rừng trong đoạn 2?

A. chặt cây lấy gỗ B. chăn nuôi gia súc

C. trồng trọt D. săn bắt lấy thức ăn

Căn cứ thông tin đoạn 2:

Deforestation occurs for a number of reasons, including: farming, mostly cattle due to its quick turn around; and logging for materials and development. It has been happening for thousands of years, arguably since man began converting from hunter/gatherer to agricultural based societies, and required larger, unobstructed tracks of land to accommodate cattle, crops, and housing. It was only after the onset of the modern era that it became an epidemic.

(Nạn phá rừng xảy ra vì một số lý do, bao gồm: chăn nuôi, chủ yếu là gia súc do sự quay vòng nhanh; và khai thác gỗ làm vật liệu và phát triển. Nó đã xảy ra hàng ngàn năm, được cho là kể từ khi con người bắt đầu chuyển đổi từ săn bắt/ hái lượm sang xã hội dựa trên nông nghiệp, và yêu cầu những thửa đất rộng lớn, không bị cản trở để chứa gia súc, cây trồng và nhà ở. Chỉ sau khi bắt đầu thời hiện đại, nó đã trở thành một đại dịch.)

Câu 184: Đáp án B

Từ “others“ trong đoạn 3 đề cập đến từ nào?

A. vòm cây B. các loài C. rừng mưa nhiệt đới D. nhiều cây Từ “others" thay thế cho danh từ các loài ở câu trước.

The trees of the rainforest that provide shelter for some species also provide the canopy that regulates the temperature, a necessity for many others. (Những cây rừng nhiệt đới cung cấp nơi trú ẩn cho một số loài thì cũng cung cấp tán cây điều chỉnh nhiệt độ, một điều cần thiết cho nhiều loài khác.)

Câu 185: Đáp án D

Những câu sau đây là tác hại của nạn phá rừng, ngoại trừ .

  1. Nhiều loài thực vật và động vật đã bị tuyệt chủng trước khi chúng ta tìm ra chúng.
  2. Sự biến đổi nhiệt độ trong một ngày hoàn chỉnh đang trở nên khắc nghiệt hơn vì sự mất rừng.
  3. Đất không còn màu mỡ để canh tác vụ mùa bởi vì thiếu mưa và xói mòn.
  4. Ngày càng nhiều độngvật phải tìm nơi trú ẩn dưới vòm lá cây.

Căn cứ thông tin đoạn 3, 4, 5:

One of the most dangerous and unsettling effects of deforestation is the loss of animal and plant species due to their loss of habitat; not only do we lose those known to us, but also those unknown, potentially an even greater loss. (Một trong những tác động nguy hiểm nhất và bất ổn nhất của nạn phá rừng là sự mất mát của các loài động vật và thực vật do mất môi trường sống; chúng ta không chỉ mất những loải đã được biết đến, mà con cả những loài vẫn chưa biết, có khả năng sự mất mát còn lớn hơn nữa.)

Its removal through deforestation would allow a more drastic temperature variation from day to night, much like a desert, which could prove fatal for current inhabitants. (Việc loại bỏ vòm cây thông qua phá rừng sẽ khiến cho sự biến đối nhiệt độ mạnh hơn từ ngày sang đêm, giống như một sa mạc, có thể gây tử vong cho cư dân hiện tại.)

With fewer trees left, due to deforestation, there is less water in the air to be returned to the soil. In turn, this causes dryer soil and the inability to grow crops, an ironic twist when considered against the fact that 80% of deforestation comes from small-scale agriculture and cattle ranching. (Với ít cây còn lại, do mất rừng, sẽ có ít nước trong không khí được trả lại đất. Đổi lại, điều này khiến cho đất khô và không có

khả năng trồng trọt, một điều mỉa mai khi trong thực tế rằng 80% nạn phá rừng đến từ nông nghiệp quy mô nhỏ và chăn nuôi gia súc.)

Further effects of deforestation include soil erosion and coastal flooding. (Những ảnh hưởng xa hơn của nạn phá rừng bao gồm xói mòn đất và lũ lụt ở vùng duyên hải.)

Câu 186: Đáp án A

Từ “perpetuate" trong đoạn cuối gần nghĩa nhất với từ .

A. duy trì B. dừng lại C. ngăn cản D. bắt đầu Từ đồng nghĩa: perpetuate (duy trì) = maintain

In addition to their previously mentioned roles, trees also function to retain water and topsoil, which provides the rich nutrients to sustain additional forest life. Without them, the soil erodes and washes away, causing farmers to move on and perpetuate the cycle. (Ngoài các vai trò đã để cập trên đây của chúng, cây cũng có chức năng giữ nước và lớp đất trên bề mặt, cung cấp các chất dinh dưỡng phong phú để duy trì cuộc sống của rừng bổ sung. Nếu không có chúng, đất sẽ bị xói mòn và rửa trôi, khiến nông dân phải tiếp tục di chuyển và duy trì chu trình này)

Câu 187: Đáp án A

Vùng duyên hải được để cập trong đoạn cuối như là 1 ví dụ về các vùng mà .

  1. dễ dàng bị tổn thương trong 1 cơn bão.
  2. bị bỏ lại bởi vì sự suy giảm của rừng.
  3. không bị ảnh hưởng nhiều bởi lũ lụt.
  4. thường chứng kiến cảnh hạn hán hay xói mòn.

Căn cứ vào thông tin đoạn cuối:

The barren land which is left behind in the wake of these unsustainable agricultural practices is then more susceptible to flooding, specifically in coastal regions. Coastal vegetation lessens the impact of waves and winds associated with a storm surge. Without this vegetation, coastal villages are susceptible to damaging floods.

(Vùng đất cằn cỗi bị bỏ lại sau sự trỗi dậy của những hoạt động nông nghiệp không bền vững này thi dễ bị ngập lụt hơn, đặc biệt là ở các vùng ven biển. Thảm thực vật ven biển làm giảm tác động của sóng và gió từ một cơn bão. Nếu không có thảm thực vật này, các làng ven biển dễ bị lũ lụt gây hại.)

Câu 188: Đáp án A

Đoạn văn sau đoạn cuối cùng trong bài văn này có thể thảo luận về .

  1. Một loạt các bất lợi mà nạn phá rừng gây ra cho người dân bản địa
  2. Một vài biện pháp quyết liệt nên được thực hiện để giai quyết nạn phá rừng.
  3. Thực trạng của nạn phá rừng ở một vài đất nước cụ thể.
  4. Quan điểm của các nhà khoa học về nạn phá rừng.

Căn từ vào thông tin đoạn 1 và các đoạn khác trong bài:

Trong đoạn 1, nội dung chính của bài đọc là các tác động tiêu cực của nạn phá rừng như: biến đổi khí hậu, sa mạc hóa, xói mòn đất, ít vụ mùa, lũ lụt, tăng khí nhà kính trong bầu khí quyển, và một loạt các vấn đề cho người dân bản địa. Trong các đoạn văn tiếp theo, tác giả đã nói về các tác động trên ngoại trừ những vấn đề mà nạn phá rừng gây ra cho người dân bản địa. Vì vậy, đoạn sau đoạn cuối trong bài sẽ nói về các vấn đề đó.

Dịch bài

Phá rừng là việc làm sạch, phá hủy hoặc loại bỏ cây cối thông qua các hoạt động có chủ ý, tự nhiên hoặc ngẫu nhiên. Nó có thể xảy ra ở bất kỳ khu vực đông cây cối và các loài thực vật khác, nhưng phần lớn nó hiện đang diễn ra trong rừng mưa nhiệt đới Amazon. Tình trạng mất cây và thảm thực vật khác có thể gây ra biến đổi khí hậu, sa mạc hóa, xói mòn đất, ít vụ mùa, lũ lụt, tăng khí nhà kính trong bầu khí quyển, và một loạt các vấn đề cho người dân bản địa.

Nạn phá rừng xảy ra vi một số lý do, baa gồm: chăn nuôi, chủ yếu là gia súc do sự quay vimg nhanh; và khai thác gỗ làm vật liệu và phát triển. Nó đã xảy ra hàng ngàn năm, được cho là kể từ khi con người bắt đầu chuyển đổi từ săn bắt/ hái lượm sang xă hội dựa trên nông nghiệp, và yêu cầu những thửa đất rộng lớn, không bị cản trở để chứa gia súc, cây trồng và nhà ở. Chỉ sau khi bắt đầu thời hiện đại, nó đã trở thành một đại dịch.

Một trong những tác động nguy hiểm nhất và bất ổn nhất của nạn phá rừng là sự mất mát của các loài động vật và thực vật do mất môi trường sống; chúng ta không chỉ mất những loài đã được biết đến, mà còn cả những loài vẫn chưa biết, có khả năng sự mất mát còn lớn hơn nữa. Bảy mươi phần trăm động vật và thực vật trên cạn của Trái đất sống trong rừng, và nhiều loài không thể sống sót sau nạn phá rừng phá hủy nhà của chúng. Những cây rừng nhiệt đới cung cấp nơi trú ẩn cho một số loài thì cũng cung cấp tán cây điều chinh nhiệt độ, một điều cần thiết cho nhiều loài khác. Việc loại bộ vòm cây thông qua phá

rừng sẽ khiến cho sự biến đổi nhiệt độ mạnh hơn từ ngày sang đêm, giống như một sa mạc, có thể gây tử vong cho cư dân hiện tại.

Ngoài việc mất môi trường sống, việc thiếu cây cũng khiến một lượng lớn khí nhà kính được thải vào khí quyển. Hiện tại, các khu rừng mưa nhiệt đới ở Nam Mỹ chịu trách nhiệm cho 20% oxy của Trái Đất và chúng đang biến mất với tốc độ 4 ha/ một thập kỷ. Nếu những tỷ lệ này không bị dừng lại và đảo ngược, hậu quả sẽ trở nên nghiêm trọng hơn.

Cây cũng giúp kiểm soát mức nước trong khí quyền bằng cách giúp điều chỉnh chu kỳ nước. Với ít cây còn lại, do mất rừng, sẽ có ít nước trong không khí được trả lại đất. Đổi lại, điều này khiến cho đất khô và không có khả năng trồng trọt, một điều mỉa mai khi trong thực tế rằng 80% nạn phá rừng đến từ nông nghiệp quy mô nhỏ và chăn nuôi gia súc.

Các tác động khác của nạn phá rừng bao gồm xói mòn đất và ngập lụt ven biển. Ngoài các vai trò đã đề cập trên đây của chúng, cây cũng có chức năng giữ nước và lớp đất trên bề mặt, cung cấp các chất dinh dưỡng phong phú để duy trì cuộc sống của rừng bổ sung. Nếu không có chúng, đất sẽ bị xói mòn và rửa trôi, khiến nông dân phải tiếp tục di chuyển và duy trì chu trình này. Vùng đất cằn cỗi bị bỏ lại sau sự trỗi dậy của những hoạt động nông nghiệp không bền vững này thì dễ bị ngập lụt hơn, đặc biệt là ở các vùng ven biển. Thảm thực vật ven biển làm giảm tác động của sóng và gió từ một cơn bão. Nếu không có thảm thực vật này, các làng ven biển dễ bị lũ lụt gây hại.

Câu 189: Đáp án B

CHỦ ĐỀ SOURCE OF ENERGY

Theo đoạn văn, điều quan trọng của nến là gì?

  1. Chúng ta cần chúng phòng khi cúp điện.
  2. Chúng ta có được cảm giác bình lặng khi chúng được thắp sáng trong một căn phòng.
  3. Chúng ta thắp sáng chúng để tạo ra một bầu không khí1ãng mạn hơn.
  4. Chúng ta không thể làm gì nếu không có chúng trong cuộc sống hàng ngày.

Căn cứ vào thông tin sau:

“Thanks to electricity, candles are no longer a necessity in our lives, but they are still a pleasure. The warm flame of candlelight can quickly alter the mood and atmosphere of a room, often creating a peaceful scene that electric light just cannot match.” (Nhờ có điện, nến không còn là một vật cần thiết trong cuộc sống của chúng ta, nhưng chúng vẫn là một niềm vui. Ngọn lửa ấm áp của ánh nến có thể nhanh chóng thay đổi tâm trạng và bầu không khí của căn phòng, thường tạo ra một cảnh yên bình mà ánh sáng điện không thể sánh được)

Câu 190: Đáp án B

Tại sao tác giả lại đề cập đến người Roma và Ai Cập cổ đại trong đoạn văn?

  1. để chỉ ra rằng trước khi làm nến, chúng ta nên học về lịch sử của chúng
  2. để chỉ ra một trong những cách mà nến được tạo ra trong quá khứ
  3. để chỉ ra nến được phát minh như thể nào
  4. để chỉ ra rằng nến luôn luôn được sử dụng

Căn cứ vào thông tin sau:

“The ancient Romans and Egyptians made candles from a type of fiber coated with wax. However, up until the nineteenth were made from a substance called tallow, obtained from beef fat.” (Người La Mã cổ đại và người Ai Cập đã làm nến từ một loại sợi phủ sáp. Tuy nhiên, cho đến thế kỷ XIX, hầu hết nến được làm từ một chất gọi là mỡ, thu được từ mỡ bò.)

Câu 191: Đáp án D

Tại sao stearin được sử dụng trong nến?

  1. Nó tạo ra ngọn lửa nhiều màu sắc hơn so với tallow.
  2. Nó Che được mùi khó chịu trong không khí.
  3. Nó được sử dụng để loại bỏ chất béo trong nến.
  4. Nó giúp nến đốt cháy lâu hơn.

Căn cứ vào thông tin sau:

"Candles that contained stearin would burn longer than previous ones and had a better smell."

(Nến có chứa stearin sẽ cháy lâu hơn những loại trước và có mùi thơm hơn.)

Câu 192: Đáp án A

Theo đoạn văn, điều gì làm tăng mức doanh thu của nến?

  1. Mi ọngười tận hướng hiệu ứng làm dịu mà nến tạo ra.
  2. Sự đa dạng trong việc lựa chọn nến làm cho chúng hấp dẫn hơn.
  3. Mọi người mua nến làm quà tặng thường xuyên hơn so với trước đây.
  4. Nến bây giờ rẻ hơn để mua so với trước đây.

Căn cứ vào thông tin sau:

"Sales Of candles have increased greatly over the last few years, Showing that they have become part of our lives again, not through necessity, but because of the magical atmosphere they create. In our increasingly stressful lives, the calming quality of candlelight has a relaxing effect that many enjoy.” (Việc bán nến đã tăng lên đáng kể trong vài năm qua, cho thấy rằng họ đã trở thành một phần của cuộc sống của chúng ta một lần nữa, không phải thông qua sự cần thiết, mà là vì bầu không khí kỳ diệu mà chúng tạo ra. Trong cuộc sống ngày càng căng thẳng của chúng ta, sự êm dịu của ánh nến có các dụng thư giãn mà nhiều người thích.)

Câu 193: Đáp án C

Theo đoạn văn, cần điều gì để làm nến thành công?

A. một vài năm luyện tập B. rất nhiều thời gian và tiền bạc

C. sẵn lòng chấp nhận rủi ro D. nhiều vật liệu đắt tiền

Căn cứ vào thông tin sau:

"For those would like to learn to make candies, finding and buying candle- making kits is easy. Candle-making is definitely enhanced by the exciting possibilities of experimentation with various materials. Be brave and try out different effects — some of the most wonderful creations can happen by accident. With a bit of practice, you will be amazed at the very professional finish that can be achieved.” (Đối với những người muốn học cách làm nến, việc tìm kiếm và mua bộ dụng cụ làm nến thật dễ dàng. Làm nến chắc chắn được tăng cường bởi những khả năng thú vị của thử nghiệm với các vật liệu khác nhau. Hãy dũng cảm và thử các hiệu ứng khác nhau - một số trong những sáng tạo tuyệt vời nhất có thể xảy ra một cách tình cờ. Với một chút luyện tập, bạn sẽ ngạc nhiên trước kết thúc rất chuyên nghiệp có

thể đạt được.)

Câu 194: Đáp án A

Đoạn văn cảnh báo người đọc không làm gì?

  1. Đốt nến mà không có sự giám sát của người lớn
  2. Rời khỏi một căn phòng nơi một ngọn nến đang cháy
  3. Sử dụng các thành phần không được chấp thuận trong nến
  4. Đốt nến chỉ trong vài phút

Căn cứ vào thông tin sau:

"Despite their delicate beauty, candles can, of course, be highly dangerous. One should never leave lit candles unattended, even for a few moments. Always make sure candles are securely placed within candleholders."

(Mặc dù vẻ đẹp tinh tế của chúng, nến có thể, tất nhiên, rất nguy hiểm. Người ta không bao giờ để nến cháy mà không có sự giám sát, ngay cả trong một vài phút. Luôn đảm bảo rằng nến được đặt an toàn trong cốc nến.)

Câu 195: Đáp án C

Điều nào sau đây thể hiện ý tưởng chính của đoạn văn này?

  1. Làm nến có thể là một ngành kinh doanh thành công.
  2. Nến được làm từ nhiều loại vật liệu khác nhau.
  3. Nến tiếp tục là một phần của cuộc sống của mọi người.
  4. Làm nến đã thay đổi rất ít trong những năm qua.

Giải thích: Nội dung bài bao gồm các công dụng của nến, nguyên liệu để làm nến, việc học làm nến, sự nguy hiểm của nến. Do do, ý chính của bài phải là “Nến tiếp tục là một phần của cuộc sống của mọi người.”

Bài dịch

Nhờ có điện, nến không còn là một vật cần thiết trong cuộc sống của chúng ta, nhưng chúng vẫn là một niềm vui. Ngọn lửa ấm áp của ánh nến có thể nhanh chóng thay đổi tâm trạng và bầu không khí của căn phòng, thường tạo ra một cảnh yên bình mà ánh sáng điện không thể sánh được.

Nến là một phần quan trọng của nhiều lễ hội văn hóa và tôn giáo và đã bị đốt cháy dưới nhiều hình thức trong nhiều thể kỷ. Người La Mã cổ đại và người Ai Cập đã làm nến từ một loại sợi phủ sáp. Tuy nhiên, cho đến thế kỷ XIX, hầu hết nến được làm từ một chất gọi là mỡ, thu được từ mỡ bò. Những cây nến trát rất khói và dễ hiểu, có mùi khó chịu.

Vào thế kỷ 19, các nhà sản xuất nến đã học cách tách stearin, chất béo dạng rắn được sử dụng như một loại keo, từ mỡ động vật và sử dụng nó để làm cứng các chất béo khác. Nến có chứa stearin sẽ chảy lâu hơn những loại trước và có mùi thơm hơn. Stearin vẫn là một trong những

thành phần chính của việc chế tạo nến hiện đại, và các kỹ thuật được sử dụng để tạo ra nến ngày nay cũng giống như chúng đã tồn tại trong nhiều năm. Nhưng ngày này, phạm vì gia tăng của thuốc nhuộm sáp, nước hoa và các chất phụ gia khác hiện đang có sẵn làm cho việc làm nếu là một sở thích rất thú vị và bổ ích.

Việc bán nến đã tăng lên đáng kể trong vài năm qua, cho thấy rằng nến đã trở thành một phần của cuộc sống của chúng ta một lần nữa, không phải thông qua sự cần thiết, mà là vì bầu không khí kỳ diệu mà chúng tạo ra. Trong cuộc sống ngày càng căng thẳng của chúng ta, sự êm dịu của ánh nến có tác dụng thư giãn mà nhiều người thích.

Đối với những người muốn học cách làm nến, việc tìm kiếm và mua bộ dụng cụ làm nến thật dễ dàng. Làm nến chắc chắn được tăng cường bởi những khả năng thú vị của thử nghiệm với các vật liệu khác nhau. Hãy dũng cảm và thử các hiệu ứng khác nhau - một số trong những sáng tạo tuyệt vời nhất có thể xảy ra một cách tình cờ. Với một chút luyện tập, bạn sẽ ngạc nhiên trước kết thúc rất chuyên nghiệp có thể đạt được.

Mặc dù có vẻ đẹp tinh tế, nến có thế, tất nhiên, rất nguy hiểm. Không bao giờ nên để nến cháy mà không có sự giám sát, ngay cả trong một vài phút. Luôn đảm bảo rằng nến được đặt an toàn trong cốc nến.

Cần dạy cho trẻ em ý thức về một ngọn nến đang cháy, và tất nhiên để ngọn nến đang cháy cách xa các vật liệu dễ cháy. Giữ nhà của bạn an toàn bằng cách nhớ rằng một cây nến là huyền diệu, nhưng lửa có thế rất phá hoại. Hãy cẩn thận, và tận hưởng vẻ đẹp của nến!

Câu 196: Đáp án D

CHỦ ĐỀ CONSERVATION

Câu nào sau đây là tiêu đề tốt nhất cho đoạn văn này?

  1. Sự kỳ diệu của tái chế: Đem lại những gì đã từng bị mất.
  2. Tái chế, chôn lấp hay ủ phân: cái nào tốt nhất cho bạn?
  3. Làm những gì bạn có thể: Làm thế nào để cứu trái đất bằng cách tái chế và ủ phân.
  4. Các phương pháp quản lý chất thải: Những ưu và nhược điểm.

Thông tin: "There are four methods to managing waste: recycling, landfilling, composting, and incinerating. Each method has its strengths and weaknesses. Let's take a quick look at each." (Có bốn phương pháp để quản lý chất thải: tái chê, chôn lấp, ủ và đốt. Mỗi phương pháp đều có điểm mạnh và điểm yếu. Chúng ta hãy cùng xem xét từng cái)

Câu 197: Đáp án D

Theo đoạn văn, tất cả những điều dưới đây được đề cập đến như là một vấn đề với bãi chôn lấp TRỪ ?

  1. bãi chôn lấp có mùi
  2. bãi chôn lấp có thể làm ô nhiễm nguồn cung cấp nước
  3. khó tìm được vị trí bãi chôn lấp
  4. vật liệu có thế sử dụng được bị lãng phí trong các bãi chôn lấp

Thông tin:

  • Not to mention that all of that garbage stinks. Nobody wants to live next to a landfill.
  • Landfills may pollute the local water supply.
  • This makes it hard to find new locations for landfills. Chỉ có đáp án D không được nhắc đến.

Câu 198: Đáp án A

Từ “stinks” ở đoạn 3 nghĩa là gì?

    1. mùi khó chịu B. có vẻ bẩn C. trông hấp dẫn D. cảm thấy mềm

Thông tin:

"Not to mention that all of that garbage stinks. Nobody wants to live next to a landfill."(Chưa kể đến tất cả rác đó bốc mùi hôi thối. Không ai muốn sống bên cạnh bãi rác) => “stinks” = smells unpleasant

Câu 199: Đáp án B

Câu nào giải thích đúng nhất tại sao việc ủ phân là không khả thi trên quy mô lớn?

  1. Người ta không muốn chạm vào tất cả những thức ăn thừa ghê bẩn.
  2. Chất dẻo sẽ rơi vào đống ủ phân và biến nó thành chất gây ô nhiễm.
  3. Nó có thể bốc mùi quá nặng ở các thành phố đông dân.
  4. Nó sẽ thu hút các loài gặm nhấm lây lan bệnh.

Thông tin:

“This is because plastic and other inorganic materials must be removed from the compost pile or they will pollute the soil. There's a lot of plastic in garbage, which makes it hard to compost on a large scale." (Điều này là do các chất dẻo và các chất vô cơ khác phải được loại bỏ khỏi đống ủ hoặc chúng sẽ làm ô nhiễm đất. Có rất nhiều chất dẻo trong rác thải, làm cho khó ủ phản ở quy mô lớn)

Câu 200: Đáp án D

Theo đoạn văn câu nào sau định nghĩa đúng nhất từ “incineration”?

  1. mua vật liệu phế thải trong một hố lớ
  2. cho phép các chất thải phân hủy và trở thành phân bón
  3. biến vật liệu phế thải thành các sản phẩm như bìa sách
  4. đốt vật liệu phế thải và thu năng lượng

Thông tin:

“The second is to burn the waste directly. The heat from the incineration process can boil water, which can power steam generators." (Thứ hai là đốt cháy trực tiếp chất thải. Nhiệt từ quá trình đốt có thế đun sôi nước, có thể cấp điện máy phát điện hơi nước.)

Câu 201: Đáp án B

Từ "it” ở đoạn cuối đề cập đến

    1. công đồng B. rác thải C. chất thải D. sở hữu

Từ “it” đề cập đến “garbage": Usually, the community in which you live manages waste. Once you put your garbage in that can, what happens to it is beyond your control. (Thông thường, cộng đồng mà bạn sống quản lý chất thải. Một khi bạn bỏ rác vào thùng rác, điều gì sẽ xảy ra với nó nằm ngoài tầm kiểm soát của bạn.)

Câu 202: Đáp án C

Theo đoạn văn có bao nhiêu cách chính để đốt chất thải?

A. một B. ba C. hai D. bốn

Thông tin:

“One thing that is easier to do is burning garbage. There are two main ways to incinerate waste." (Một điều dễ làm hơn chính là đốt. Có hai cách chính để đốt các chất thải.)

Câu203: Đáp án B

Kết luận nào có thể được rút ra từ đoạn văn?

  1. Tái chế không nghi ngờ gì là cách tốt nhất để xử lý chất thải.
  2. Mỗi phương pháp quản lý chất thải đều có những hạn chế.
  3. Đốt là cách tốt nhất để xử lý chất thải.
  4. Tất cả các thành phố lớn nên tạo ra các đống ủ phân lớn.

Thông tin:

'There are four methods to managing waste: recycling, landfilling, composting, and incinerating. Each method has its strengths and weaknesses.” (Có bốn phương pháp để quản lý chất thải: tái chế chôn lấp, ủ và đốt. Mỗi phương pháp đều có điểm mạnh và điểm yếu.)

Dịch bài:

Thùng rác không phải là những chiếc cổng huyền diệu. Rác không biến mất khi bạn ném nó vào một chiếc thùng rác. Tuy nhiên, một người Mỹ trung bình ném ra khoảng 1.600 cân phế thải mỗi năm. Nếu không có thùng rác, tất cả số rác đó đi đâu? Có bốn phương pháp để quản lý chất thải: tái chế, chôn lấp, ủ và đốt. Mỗi phương pháp đều có điểm mạnh và điểm yếu. Chúng ta hãy cùng xem xét từng cái. Tái chế là quá trình biến chất thải thành vật liệu mới. Ví dụ, giấy đã qua sử dụng có thể được biến thành bìa giấy, có thể được sử dụng để làm bìa sách. Tái chế có thể làm giảm ô nhiễm, tiết kiệm vật liệu và

giảm sử dụng năng lượng. Tuy nhiên, một số người cho rằng tái chế lãng phí năng lượng. Họ tin rằng việc

thu gom, xử lý và chuyển đổi chất thải sẽ sử dụng nhiều năng lượng hơn mức tiết kiệm. Tuy nhiên hầu hết mọi người đồng ý rằng tái chế tốt hơn cho hành tinh so với việc chôn lấp.

Chôn lấp là phương pháp lâu đời nhất để quản lý chất thải. Trong hình thức đơn giản nhất, chôn lấp rác là khi người ta chôn rác trong một hố. Theo thời gian việc thực hiện của chôn lấp đã có tiến bộ. Rác được kết chặt lại trước khi nó được ném vao hố. Theo cách này nhiều rác có thể vừa khít trong mỗi bãi chôn lấp. Các lớp lót lớn được đặt dưới đáy bãi chôn lấp để nước thải độc hại không ngấm vào lòng đất. Đáng buồn thay, những lớp lót này không phải lúc nào cũng hiện quả. Các bãi chôn lấp có thể làm ô nhiễm nguồn nước địa phương. Chưa kể đến tất cả rác đó bốc mùi hôi thối. Không ai muốn sống bên cạnh bãi rác. Điều này làm cho khó tìm được vị trí mới cho bãi chôn lấp.

Ủ là khi người ta để chất hữu cơ, như chất thải thực phẩm, và để nó phân hủy. Sản phẩm của phân hủy này là phân. Phân có thể được bổ sung vào đất để làm cho đất màu mở hơn và tốt hơn cho cây trồng. Trong khi ủ phân rất dễ dàng ở một nơi nào đó, như ở nhà hoặc ở trường, thật khó để làm sau khi rác thải được trộn lẫn. Điều này là do các chất dẻo và các chất vô cơ khác phải được loại

bỏ khỏi đống ủ hoặc chúng sẽ làm ô nhiễm đất. Có rất nhiều chất dẻo trong rác thải, làm cho khó ủ phản ở quy mô lớn.

Một điều dễ làm hơn chính là đốt. Có hai cách chính để đốt các chất thải. Đầu tiên là tạo ra hoặc thu hoạch một nhiên liệu từ chất thải, như khí mê-tan, và đốt nhiên liệu. Thứ hai là đốt cháy trực tiếp chất thải. Nhiệt từ quá trình đốt có thể đun sôi nước, có thể cấp điện máy phát điện hơi nước. Thật không may, đốt rác thải gây ô nhiễm không khí. Ngoài ra, một số nhà phê bình lo ngại lò đốt sẽ tiêu hủy các nguồn tài nguyên quý giá có thể được tái chế.

Thông thường, cộng động mà bạn sống quản lý chất thải. Một khi bạn bỏ rác vào thùng rác. điều gì sẽ xảy ra với nó nằm ngoài tầm kiểm soát của bạn. Nhưng bạn có thể lựa chọn trong khi nó vẫn còn thuộc sở hữu của bạn. Bạn có thể chọn để tái chế, bạn có thể chọn để ủ phân, hoặc bạn có thể chọn để cho người khác xử lý nó. Sự lựa chọn là của bạn.

Câu 204: Đáp án A

CHỦ ĐỀ WATER SPORTS

Câu nào trong các câu sau có thể là tiêu đề của đoạn văn?

  1. Sự hấp dẫn mạnh mẽ của môn lặn có bình khi.
  2. Sở thích mới trên Trái Đất.
  3. Những yêu cầu cho bộ môn lặn có bình khí.
  4. Nơi để tập luyện môn lặn có bình khí.

Căn cứ vào thông tin đoạn 1:

One of the most interesting parts of the earth for many people was and continues to be the beautiful world of the sea. People were always attracted to the mysteries of the deep waters when they

saw a little part of the under water world on television or in photographs. Due to this intense attraction caused by the mystery of the waters, more and more people choose to practice scuba diving, which becam in time one of the most popular sports around the world.

(Một trong những phần thú vị nhất của Trái Đất đối với nhiều người đã và sẽ tiếp tục là thế giới tuyệt đẹp của biển. Mọi người luôn bị thu hút bởi những bí ẩn của vùng nước sâu khi họ nhìn thấy một phần nhỏ của thế giới dưới nước trên truyền hình hoặc trong các bức ảnh. Do sự hấp dẫn dữ dội này được gây ra bởi sự bí ẩn của vùng biển, ngày càng có nhiều người chọn thực hành lặn có bình khi, cái đã trở thành một trong những môn thể thao phổ biến nhất trên thế giới)

Câu 205: Đáp án C

Từ “intense” trong đoạn 1 gần nghĩa nhất với từ

    1. khiêm tốn B. đặc biệt C. mạnh mẽ D. âm ỉ Từ đồng nghĩa: intense (mạnh mẽ, dữ dội) = strong

Due to this intense attraction caused by the mystery of the waters, more and more people choose to practice scuba diving, which became in time one of the most popular sports around the world. (Do sự hấp dẫn dữ dội này được gây ra bởi sự bí ẩn của vùng biển, ngày càng có nhiều người chọn thực hành lặn có bình khí, cái đã trở thành một trong những môn thể thao phổ biến nhất trên thế giới.)

Câu 206: Đáp án C

Từ “it” trong đoạn 1 đề cập đến từ nào?

A. giá trị thực của nó B. nghiên cứu C. môn lặn có bình khí D. thế giới

Căn cứ thông tin đoạn 1: Từ “it" thay thế cho danh từ scuba diving.

Scuba diving is a complex sport that requires some research before being understood at its real value. In order for someone to practice it, it is required for them to take some courses so accidents can be avoided. (Lặn có bình khí là một môn thể thao phức tạp đòi hỏi một số nghiên cứu trước khi hiểu được giá trị thực của nó. Để ai đó thực hành nó, họ cần phải tham gia một số khóa học để có thể tránh được các tai nan.) Câu 207: Đáp án D

Theo đoạn văn, lĩnh vực nào của môn lặn có binh khí không được đề cập đến?

A. giải trí B. thương mại

C. các hoạt động của quân đội D. cuộc phiêu lưu

Căn cứ thông tin đoạn 2:

At the moment, scuba diving has four main areas of interest: recreation purposes, commercial purposes, scientifically research and military activity as well. (Tại thời điểm này, lặn có bình khí có bốn lĩnh vực quan tâm chính: mục đích giải trí, mục đích thương mại, nghiên cứu khoa học và hoạt động quân sự.) Câu 208: Đáp án A

Từ "attached” trong đoạn 3 có thể có nghĩa là

A. tham gia, kết nối B. đính kèm C. phù hợp D. tấn công Từ đồng nghĩa: attached (gắn bó, tham gia vào) = joined

For the beginning, you can only rent scuba diving equipment, but if you get attached to this sport, then you will have to purchase one of your own at some moment. (Ban đầu, bạn chỉ cần thuê thiết bị lặn có bình khí, nhưng nếu bạn gắn liền với môn thể thao này, bạn sẽ phải mua một bộ của riêng bạn tại một thời điểm nào đó.)

Câu 209: Đáp án C

Câu nào là đúng về môn lặn có bình khí?

  1. Mặc dù bạn rất thích môn lặn có bình khi, ban không thế có một công việc thành công với môn này.
  2. Nhiều người muốn thử môn lặn có bình khí để khám phá thế giới mới.
  3. Để thực hành môn lặn có bình khí, người ta cần rất nhiều thiết bị quan trọng.
  4. Mua thiết bị lặn có bình khí phù hợp là một ý hay cho những người chơi môn thể thao này.

Căn cứ vào các thông tin sau:

This sport can be transformed into a successful career by those who are truly passionate about it. (Môn thể thao này có thể được biến thành một công việc thành công bởi những người thực sự đam mê nó.)

The most frequent reason why people go for scuba diving is because they want to relax in a special way, by having contact with a less known world. (Lý do thường xuyên nhất khiến mọi người đi lặn có bình khí là vì họ muốn thư giãn theo một cách đặc biệt, bằng cách tiếp xúc với một thế giới ít được biết đến hơn.)

Some of the most important parts of the scuba gear are the mask, the snorkel, the diving suit, the gloves, the boots and the diving regulator. (Một số bộ phận quan trọng nhất của thiết bị lặn là mặt nạ, ổng thở, bộ đồ lặn, găng tay, ủng và bộ điều chỉnh lặn.)

For the beginning, you can only rent scuba diving equipment, but if you get attached to this sport, then you will have to purchase one of your own at some moment. (Ban đầu, bạn chỉ cần thuê thiết bị lặn có bình khí nhưng nếu bạn gắn liền với môn thể thao này, bạn sẽ phải mua một bộ của riêng bạn tại một thời điểm nào đó.)

Câu 210: Đáp án A

Lợi ích nào của môn lặn có bình khí mà tác giả đề cập trong đoạn cuối?

  1. Khám phá nhiều điểm đến tuyệt vời hơn trên thế giới.
  2. Quan sát nhiều địa danh để hoàn thành đam mê của mình.
  3. Tham dự các khoá học lặn để hiểu về môn thể thao này.
  4. Lựa chọn một địa danh để viếng thăm vùng biển đẹp nhất.

Căn cứ thông tin đoạn cuối:

If you finally decided to try scuba diving and you already took some scuba courses then you must be thinking about what location to choose. This is one of the most difficult decisions to make as our world is filled with numerous destinations, equally beautiful and interesting. If this passion catches you then you will surely get to visit more and more amazing locations each year. There is no such thing as the most beautiful waters to scuba diving in as each part has something to offer and shelters special creatures that should be observed.

(Nếu bạn cuối cùng đã quyết định thử lặn có bình khí và bạn đã tham gia một số khóa học lặn thì bạn phải suy nghĩ về địa điểm để chọn. Đây là một trong những quyết định khó khăn nhất để thực hiện khi thế giới của chúng ta tràn ngập nhiều điểm đến, đẹp và thú vị như nhau. Nếu niềm đam mê này gắn với bạn, sau đó bạn chắc chắn sẽ đến thăm nhiều địa điểm tuyệt vời hơn nữa mỗi năm. Không có nơi nào là vùng biển đẹp nhất để lặn biển vì mỗi nơi đều có thứ gì đó để khám phá và là nơi trú ẩn của những sinh vật đặc biệt cần được quan sát.)

Dịch bài:

Một trong những phần thú vị nhất của trái đất đối với nhiều người đã và sẽ tiếp tục là thế giới tuyệt đẹp của biển. Mọi người luôn bị thu hút bởi những bí ẩn của vùng nước sâu khi họ nhìn thấy một phần nhỏ của thế giới dưới nước trên truyền hình hoặc trong các bức ảnh. Do sự hấp dẫn này được gây ra bởi sự bí ẩn của vùng biển, ngày càng có nhiều người chọn thực hành lặn có bình khí, cái đã trở thành một trong những môn thể thao phổ biến nhất trên thế giới. Lặn có bình khí là một môn thể thao phức tạp đòi hỏi một số nghiên cứu trước khi hiểu được giá trị thực của nó. Để ai đó thực hành nó. họ cần phải tham gia một số khóa học để có thể tránh được các tai nạn.

Lặn có hình khí không chỉ là lặn và quan sát môi trường tuyệt vời tồn tại dưới nước. Môn thể thao này có thể được biến thành một công việc thành công bởi những người thực sự đam mê nó. Tại thời điểm này, lặn có hình khi có bốn lĩnh vực quan tâm chính: mục đích giải trí, mục đích thương mại, nghiên cứu khoa học và hoạt động quân sự. Lý do thường xuyên nhất khiến mọi người đi

lặn có bình khí là vì họ muốn thư giãn theo một cách đặc biệt, bằng cách tiếp xúc với một thế giới ít được biết đến hơn.

Một điều tuyệt vời về môn thể thao này là nó có thế được thực hành bất cứ lúc nào quanh năm; tuy nhiên, bạn nên chọn mùa hè nếu bạn gặp vấn đề với nhiệt độ thấp. Một khi bạn tham gia các bài học, bạn sẽ có được kiến thức cơ bản trước khi lặn, bạn nên nói chuyện với một chuyên gia và yêu cầu thiết bị chất lượng cho hoạt động này. Ban đầu, bạn chỉ cần thuê thiết bị lặn có bình khí, nhưng nếu bạn gắn liền với môn thể thao này, bạn sẽ phải mua một bộ của riêng bạn tại một thời điểm nào đó. Một số bộ phận quan trọng nhất của thiết bị lặn là mặt nạ, ống thở, bộ đồ lặn, găng tay, ủng và bộ điều chỉnh lặn.

Nếu bạn cuối cùng đã quyết định thử lặn có bình khí và bạn đã tham gia một số khóa học lặn thì bạn phải suy nghĩ về địa điểm để chọn. Đây là một trong những quyết định khó khăn nhất để thực hiện

khi thế giới của chúng ta tràn ngập nhiều điểm đến, đẹp và thú vị như nhau. Nếu niềm đam mê này gắn với bạn, sau đó bạn chắc chắn sẽ đến thăm nhiều địa điểm tuyệt vời hơn nữa mỗi năm. Không có nơi nào là vùng biển đẹp nhất để lặn biển vì mỗi nơi đều có thứ gì đó để khám phá và là nơi trú ẩn của những sinh vật đặc biệt cần được quan sát.

Câu 211: Đáp án A

Chủ đề về BOOKS (Trích đề thi THPTQG 2017)

Đoạn văn nói về cái gì?

  1. The need for digital comic books: nhu cầu về truyện tranh kỹ thuật số
  2. Children’s most favourite type of books: thể loại sách yêu thích nhất của trẻ em
  3. Problems in the book industry: những vấn đề trong ngành công nghiệp sách
  4. The manga industry in japan: Công nghiệp manga ở Nhật Bản

Căn cứ vào thông tin: "Children all around the world love to read comic books. Maybe they like to read the kind of comic books that come oat once a month, like Spiderman or Batman, or maybe they like manga-style comic books. That means that publishers who make comic books need to keep in touch with the things young people like to do. One thing that many young people today are interested in is technology. so of course kids want to read digital comic books on electronic devices.” (Trẻ con trên toàn thế giới đều thích truyện tranh. Có lẽ, chúng thích đọc

những loại truyện tranh mà nó xuất bản một tháng một lần, giống như là Spiderman hoặc là Batman hoặc là chúng có thể thích đọc các loại chuyện tranh Nhật Bản. Điều đó có nghĩa là những nhà xuất bản người mà sản xuất ra truyện tranh cần phải bắt kịp với những thứ mà giới trẻ thích. Một thứ mà nhiều

bạn trẻ bây giờ quan tâm đó là công nghệ, vì vậy tất nhiên trẻ con cũng muốn đọc truyện tranh kĩ thuật số trên các thiết bị điện tử.)

=> nhu cầu về truyện tranh kỹ thuật số

Câu 212: Đáp án A

The word "they” in paragraph 1 refers to

    1. children: trẻ con B. comic books: truyện tranh

C. publishers: nhà xuất bản D. things: những thứ Dựa vào phần dịch của câu 36 => They = children

Câu 213: Đáp án B

Theo như đoạn văn 1, Spiderman là tên của

A. a TV programme: 1 chương trình TV B. a comic book: 1 cuốn truyện tranh

C. an electronic device: 1 thiết bị điện tử D. a child: 1 đứa trẻ

Dẫn chứng: "Maybe they like to read the kind of comic books that come out once a month, like Spidennan or Batman” (Có lẽ chúng thích đọc thể loại truyện tranh- cái mà xuất bản một lần mỗi tháng, giống như “Người nhện” hay “Người dơi”)

Câu 214 Đáp án B

Theo như đoạn văn số 2, việc "scanlation" liên quan chủ yếu tới

  1. bán những cuốn truyện tranh Nhật đã được dịch
  2. quét và dịch truyện tranh Nhật
  3. quét những cuốn truyện tranh Nhật mà đã được tải lên
  4. dịch những cuốn truyện tranh Nhật mà đã được tải lên

Dẫn chứng: "By combining the words scan and translation they call it "scanlation"

(Bằng việc kết hợp giữa những từ ‘quét’ và ‘dịch’ họ gọi nó là 'scanlation’.)

Câu 215: Đáp án C

Theo như đoạn văn này, ngày xưa một số người hâm mộ truyện tranh Nhật người mà biết tiếng Nhật đã dịch truyện tranh này bởi vì .

  1. Nó là thư thái để dịch những quyển sách tiếng Nhật
  2. Họ muốn tạo lên tên tuổi cho chính mình
  3. Nó là rất khó để mua được những quyển truyện tranh Nhật đã được dịch
  4. Họ muốn đọc truyện tranh Nhật miễn phí

Dẫn chứng: "in the past, it was difficult to find translated versions of manga in comic bookstores."

(Trong quá khứ, rất khó để tìm thấy một bản truyện tranh đã dịch ở cửa hàng sách.)

Câu 216: Đáp án A

Theo như đoạn văn số 2, ai có thể được lợi từ scanlation?

    1. độc giả B. Nhà xuất bản

C. nhà văn D. nghệ sĩ

Dẫn chứng: 'The people who do scanlation usually share their manga for free. So readers do not buy manga books.” (Người làm công việc dịch truyện thường chia sẻ truyện của họ miễn phí nên người đọc không mua sách bản giấy.)

=> Người đọc vừa được đọc truyện đã dịch, vừa không mất tiền.

Câu 217: Đáp án C

Từ " piracy” trong đoạn văn số 2 ám chỉ hành động .

“piracy”: Sự in lậu (tác phẩm đã được bảo hộ bản quyền)

  1. controlling the production ofa book: kiểm soát việc sản xuất một quyển sách
  2. hurting the sale of illegal comic books: làm giảm doanh thu của những cuốn truyện tranh bất hợp pháp
  3. copying and using a book illegally: sao chép và sử dụng cuốn sách 1 cách trái pháp luật
  4. advertising a book without permission: quảng cáo 1 cuốn sách mà không có sự cho phép

Câu 218: Đáp án B Chủ đề SPORTS

Các trọng tài làm gì trong thời gian trận bóng diễn ra?

    1. làm tăng sự phấn khích giữa các fan B. giám sát trận đầu

C. giữ vé D. người xem trận đầu

Dẫn chứng: Also, there are officials carrying whistles and flags to make certain that the rules of the game are followed during the game. (Cũng như vậy, có những trọng tài chính thức người mà mang còi và cờ để đảm bảo rằng luật chơi được tuân thủ trong suốt trận đầu.)

Câu 219: Đáp án A

Như đã để cập trong văn bản, những ai hoạt động tích cực nhất khi đội của họ có bóng?

A. các tiền đạo B. các hậu vệ

C. các trọng tài D. những người hâm mộ

Dẫn chứng: Each team has offensive players who play when the team has possession of the ball. (Mỗi đội có các tiền đạo người à chơi khi đội mình sở hữu bóng.)

Câu 220: Đáp án A

Chơi bóng đá Mỹ là giống như chơi nhất.

A. Bóng bầu dục B. Bóng rổ C. Bóng chày D. Bóng chuyền Bóng bầu dục còn được gọi là bóng đá kiểu Mỹ. Rugby football: American football.

Câu 221: Đáp án A

Miếng đệm giúp người chơi bảo vệ chỗ nào trên cơ thể?

A. Chân và cánh tay B. Đầu của chúng C. toàn thân D. khuôn mặt

Dẫn chứng: whereas pads are worn to protect the shoulders, arms, and legs.

(Trái lại miếng đệm được dán để bảo vệ vai, cánh tay và chân)

Câu 222: Đáp án D

Môn thể thao nào phố biến nhất ở Mỹ?

A. Bóng bầu dục B. Bóng đá C. Bóng đá Mỹ D. Không đề cập đến Dẫn chứng: Football in American is one of the most popular sports besides basketball and baseball. -> không đề cập môn thể thao nào được chơi nhiều nhất.

Câu 223: Đáp án D

Trò chơi nổi tiếng nhất được tổ chức hàng năm vào khoảng thời gian nào?

A. vào mùa thu B. tháng Giêng

C. tháng hai D. tháng Giêng hoặc tháng Hai

Dẫn chứng: The most famous game of the year is Super Bowl that is played in January or February. (Trò chơi nổi tiếng nhất của năm là Super Bowl mà nó được chơi vào tháng Giêng hoặc tháng Hai.)

Câu 224: Đáp án D

Tại sao có những vòng màu trắng xếp liền?

A. để đánh đầu khoảng cách đến vùng cuối B. để giúp người chơi chạy

C. giúp người chơi ghi được điểm D. giúp người chơi thấy bóng

Dẫn chứng: These rings help the players see the ball when it is thrown or someone is running with it. (Những vòng này giúp cho người chơi thấy bóng khi bóng được ném đi hoặc khi ai đó đang chơi nó.) Câu225: Đáp án C

Từ “grip” trong đoạn văn có nghĩa là .

A. tranh cái gì B. lấy đi một thứ gì đó

C. giữ một cái gi đó chặt chẽ D. phát hiện một cái gì đó Grip: nắm chặt

Câu 226: Đáp án C

CHỦ ĐỀ RECREATIONS

Có thể suy luận từ đoạn 1 rằng tác giả của đoạn văn nghĩ

  1. xem hoặc đọc tin tức là rất nhàm chán
  2. rằng hầu hết các câu chuyện tin tức là sai
  3. hầu hết mọi người không nhân ra những tin tức khác nhau từ thực tế như thế nào
  4. rằng hầu hết mọi người không chú ý đến tin tức

Căn cứ vào thông tin sau:

“How is the news different from entertainment? Most people would answer that news is real but entertainment is fiction. However, if we think more carefully about the news, it becomes clear that the news is not always real." (Tin tức khác với giải trí như thế nào? Hầu hết mọi người sẽ trả lời rằng tin tức là có thật nhưng giải trí là hư cấu. Tuy nhiên, nếu chúng ta suy nghĩ cẩn thận hơn về tin tức, rõ ràng là tin tức không phải lúc nào cũng thực.)

Câu 227: Đáp án D

Theo đoạn 2, điều nào sau đây là đúng?

  1. Một ảnh hướng của chủ nghĩa thương mại là những câu chuyện tin tức có nội dung phức tạp hơn.
  2. Mạng ABC sở hữu Disney Studios.
  3. Một số chương trình phát sóng tin tức được chiếu mà không có quảng cáo.
  4. Nhiều thời gian dành cho tin tức trên truyền hình hơn 50 năm trước đây.

Căn cứ vào thông tin sau:

"The amount of time that the average TV station spends on news broadcasts has grown steadily over the last fifty years - largely because news is relatively cheap to produce, yet sells plenty of advertising.” (Lượng thời gian mà đài truyền hình trung bình dành cho việc phát sóng tin tức đã tăng đều trong năm mươi năm qua - phần lớn bởi vì tin tức là tương đối rẻ để sản xuất, nhưng lại bán nhiều quảng cáo.) Câu 228: Đáp án B

Tại sao tác giả đề cập đến Mickey Mouse trong đoạn 2?

  1. Để chỉ ra rằng ABC hiển thị các câu chuyện tin tức giải trí.
  2. Để đưa ra một ví dụ về các câu chuyện tin tức cũng là quảng cáo.
  3. So sánh phong cách ABC với phong cách của CBS.
  4. Để đưa ra một ví dụ về nội dung tin tức không nghiêm trọng.

Căn cứ vào thông tin sau:

“Some news broadcasts are themselves becoming advertisements. For example, during one week in 1996 when the American CBS network was airing a movie about the sinking of the Titanic, CBS news ran nine stories about that event (which had happened 84 years before). The ABC network is owned by Disney Studios, and frequently runs news stories about Mickey Mouse." (Một số chương trình phát sóng tin tức đã trở thành quảng cáo. Chẳng hạn, trong một tuần vào năm 1996 khi mạng CBS của Mỹ phát sóng một bộ phim về vụ chìm tàu Titanic, tin tức của CBS đã đưa ra 9 câu chuyện về sự kiện đó (đã xảy ra 84 năm trước). Mạng ABC thuộc sở hữu của Disney Studios, và thường xuyên chạy các tin tức về Mickey Mouse.) Câu 229: Đáp án A

Theo đoạn 3, lợi thế của công thức kim tự tháp ngược cho các nhà báo là .

  1. nếu một câu chuyện được cắt bởi biên tập viên, chỉ có những thông tin ít quan trọng hơn sẽ bị mất
  2. nó làm cho một câu chuyện có nhiều khả năng bị cắt bởi biên tập viên
  3. làm cho một câu chuyện để thu hút sự chủ ý của khán giả
  4. nó làm cho một câu chuyện đơn giản hơn và dễ hiểu hơn

Căn cứ vào thông tin sau:

“Instead, they depend upon certain story formulas, which they can reuse again and again. One example is known as the inverted pyramid. In this formula, the journalist puts the most important information at the beginning of the story, than adds the next most important, and so on.”

(Thay vào đó, họ phụ thuộc vào các công thức câu chuyện nhất định mà chúng có thể sử dụng lại. Một ví dụ được gọi là kim tự tháp đảo ngược. Trong công thức này, nhà báo đưa ra những thông tin quan trọng nhất ở phần đầu của câu chuyện, hơn là thêm vào phần quan trọng tiếp theo, …)

Câu 230: Đáp án D

Từ “relayed” ở đoạn 3 gần nghĩa nhất với?

    1. chọn B. biết C. tập hợp D. gửi

Căn cứ vào ngữ cảnh của câu sau:

“The inverted pyramid originates from the age of the telegraph, the idea being that if the line went dead halfway through the story, the journalist would know that the most crucial information had at least been relayed.

(Kim Tự tháp đảo ngược bắt nguồn từ thời đại của điện tín, ý tưởng là nếu đường dây đi chết nửa chừng qua câu chuyện, phóng viên sẽ biết rằng thông tin quan trọng nhất ít nhất đã được chuyển tiếp.)

Câu 231: Đáp án B

Theo đoạn văn, điều nào sau đây có xu hướng dẫn đến sự bao phủ đồng nhất?

  1. Các nhà báo sử dụng các chuyên gia làm nguồn.
  2. Các nhà báo trở thành bạn với nguồn của chúng.
  3. Các nhà báo tìm kiếm các quan điểm thay thế.
  4. Các nhà báo sử dụng các quan chức chính phủ làm nguồn.

Căn cứ vào thông tin sau:

"Over time, the journalists may even become close friends with their sources, and they stop searching for alternative points of View. The result tends to be narrow, homogenized coverage of the same kind. " (Theo thời gian, các nhà báo thậm chí có thể trở thành bạn thân với nguồn của họ, và họ ngừng tìm kiếm các điểm thay thế xem. Kết quả có xu hướng thu hẹp, đồng nhất hóa phạm vi bao quát của cùng một loại.) Câu 232: Đáp án D

Từ “them” trong đoạn 4 để cập tới?

    1. các nhà báo B. các tổ chức C. các chuyên gia D. các nguồn

Căn cứ vào thông tin sau:

“All the major news organizations use some of the same sources (many of them anonymous), so the same types of stories always receive attention (Tất cả các tổ chức tin tức lớn sử dụng một số nguồn giống nhau (nhiều nguồn vô danh), vì vậy cùng một loại câu chuyện luôn được chú ý.)

Câu 233: Đáp án A

Câu nào sau đây thể hiện tốt nhất thông tin cần thiết trong các câu được đánh dấu “Giải quyết triệt để các vấn đề … hơn là về các mục tiêu chiến dịch của các chính trị gia" trong đoạn văn?

  1. Các nhà báo tập trung vào các số liệu thăm dò thay vì các vấn đề chiến dịch bởi vì nó dễ dàng hơn.
  2. Các nhà báo quan tâm nhiều hơn đến các vấn đề và quan điểm của ứng cử viên, nhưng người xem quan tâm nhiều hơn đến ai là người chiến thắng.
  3. Trong chiến dịch bầu cử, các nhà báo tập trung chủ yếu vào phạm vi “đua ngựa".
  4. Quan điểm của ứng cử viên và cách giải thích của nhà báo có thể có ảnh hưởng lớn đến số liệu thăm dò.

Dịch bài

Tin tức khác với giải trí như thế nào? Hầu hết mọi người sẽ trả lời rằng tin tức là có thật nhưng giải trí là hư cấu. Tuy nhiên, nếu chúng ta suy nghĩ cẩn thận hơn về tin tức, rõ ràng là tin tức không phải lúc nào cũng thực. Tin tức không cho chúng ta biết tất cả các sự kiện trong ngày, nhưng những câu chuyện từ một số sự kiện đã chọn. Việc tạo ra các câu chuyện tin tức có những hạn chế cụ thể, giống như việc tạo ra các tác phẩm hư cấu. Có nhiều khó khăn, nhưng ba trong số những điều quan trọng nhất là: thương mại, công thức câu chuyện, và các nguồn.

Báo chí, đài phát thanh và đài truyền hình là các doanh nghiệp, tất cả đều là đối thủ cho khán giả và doanh thu quảng cao. Lượng thời gian mà đài truyền hình trung bình dành cho việc phát sóng tin tức đã tăng đều trong năm mươi năm qua - phần lớn bởi vì tin tức là tương đối rẻ để sản xuất, nhưng lại

bán nhiều quảng cáo. Một số chương trình phát sóng tin tức đã trở thành quảng cáo. Chẳng hạn, trong một tuần vào năm 1996 khi mạng CBS của Mỹ phát sóng một bộ phim về vụ chìm tàu Titanic, tin tức của

CBS đã đưa ra 9 câu chuyện về sự kiện đó (đã xảy ra 84 năm trước). Mạng ABC thuộc sở hữu của Disney Studios, và thường xuyên chạy các tin tức về Mickey Mouse. Hơn nữa, động cơ lợi nhuận làm cho các tổ chức tin tức chú ý nhiều hơn tới những câu chuyện có khả năng tạo ra một lượng khán giả lớn và tránh xa những câu chuyện có thể quan trọng nhưng ngớ ngẩn. Áp lực này mang tính giải trí đã tạo ra những câu chuyện ngắn gọn hơn: tập trung vào người nổi tiếng hơn người khác, tập trung nhiều hơn vào tin đồn hơn là tin tức, và tập trung nhiều hơn vào các sự kiện kịch tính hơn là về những vấn đề đa dạng.

Khi những người bận rộn dưới áp lực không ngừng để sản xuất, các nhà báo không thể trải qua hàng ngày khổ sở qua cách tốt nhất để trình bày những câu chuyện. Thay vào đó, họ phụ thuộc vào các công thức câu chuyện nhất định mà chúng có thể sử dụng lại. Một ví dụ được gọi là kim tự tháp đảo ngược.

Trong công thức này, nhà báo đưa ra những thông tin quan trọng nhất ở phần đầu của câu chuyện, hơn là thêm vào phần quan trọng tiếp theo, vân vân. Kim Tự tháp đảo ngược bắt nguồn từ thời đại của điện tín, ý tưởng là nếu đường dây đi chết nửa chừng qua câu chuyện, phóng viên sẽ biết rằng thông tin quan trọng nhất ít nhất đã được chuyển tiếp. Các nhà báo hiện đại vẫn đánh giá công thức vì một lý do tương tự. Các biên tập viên của họ sẽ cắt các câu chuyện nếu quá dài. Công thức khác liên quan đến việc giảm một câu chuyện phức tạp thành một cuộc xung đột đơn giản. Ví dụ tốt nhất là bảo hiểm bầu cử “đua ngựa". Sự giải thích triệt để các vấn đề và quan điểm của ứng cử viên là rất phức tạp. Các nhà báo vì thế tập trung nhiều hơn vào ai là người giành chiến thắng trong các cuộc thăm dò ý kiến, và liệu người lười biếng có thể theo kịp các con số hơn là về các mục tiêu chiến dịch của các chính trị gia.

Các nguồn là một sự hạn chế đối với các nhà báo và cách thức họ che giấu nó. Các nguồn tin nổi bật là các nhân viên thông tin công cộng trong các doanh nghiệp và văn phòng chính phủ. Phần lớn các nhân viên này cố gắng tự khẳng định mình là những chuyên gia đủ điều kiện để cung cấp thông tin cho các nhà báo. Làm sao các nhà báo biết ai là chuyên gia? Nói chung, họ không. Họ sử dụng các nguồn

không dựa trên kiến thức chuyên môn thực tế, nhưng về sự xuất hiện của chuyên môn và sự sẵn lòng chia sé nó. Tất cả các tổ chức tin tức lớn sử dụng một số nguồn giống nhau (nhiều nguồn vô danh), vì vậy cũng một loại câu chuyện luôn được chủ ý. Theo thời gian, các nhà báo thậm chí có thể trở thành

bạn thân với nguồn của họ, và họ ngừng tìm kiếm các điểm thay thế xem. Kết quả có xu hướng thu hẹp, động nhất hóa phạm vi bao quát của cùng một loại.

Câu 234: Đáp án D CHỦ ĐỀ BOOKS

Ý chính của tác giả trong đoạn văn là gì?

  1. Internet làm mọi người xao nhãng khỏi những đam mê khác.
  2. Sách đã được cải thiện cả về hình thức lẫn nội dung.
  3. Internet và sách sẽ thay thể lẫn nhau.
  4. Sách sẽ không bị thay thế bởi Internet.

Căn cứ vào thông tin đoạn đầu và cả bài:

Tác giả khẳng định, internet sẽ không thể nào thay thế được sách và đưa ra các lý do để chứng minh điều đó.

The Internet is very much like television in which it takes time away from other pursuits, provides entertainment and information, but in no way can compare with the warm, personal experience‘of reading a good book. This is not the only reason why the Internet will never replace books, for books provide the in-depth knowledge of a subject that sitting in front of a computer monitor cannot provide. (Internet giống như truyền hình, cái mà chiếm mất nhiều thời gian dùng để theo đuổi các mục đích khác, cung cấp giải

trí và thông tin, nhưng không cách nào có thể so sánh với kinh nghiệm cá nhân ấm áp của việc đọc một cuốn sách hay. Đây không phải là lý do duy nhất tại sao Internet sẽ không bao giờ thay thế được sách, vì sách cung cấp kiến thức chuyên sâu về một chủ đề mà việc ngồi trước màn hình máy tính không thể cung cấp được)

Câu 235: Đáp án C

Từ “this” trong đoạn 1 thay thế cho từ nào?

  1. Một cuốn sách hay
  2. Internet
  3. kinh nghiệm cá nhân ấm áp của việc đọc 1 cuốn sách hay
  4. giải trí và thông tin

Căn cứ vào thông tin đoạn 1:

Từ “this" thay thế cho kinh nghiệm cá nhân ấm áp của việc đọc 1 cuốn sách hay.

The Internet is very much like television in which it takes time away from other pursuits, provides entertainment and information, but in no way can compare with the warm, personal experience of reading

a good book. This is not the only reason why the Internet will never replace books, for books provide the in-depth knowledge of a subject that sitting in front of a computer monitor cannot provide. (Internet giống như truyền hình, cái mà chiếm mất nhiều thời gian dùng để theo đuổi các mục đích khác, cung cấp giải

trí và thông tin, nhưng không cách nào có thể so sánh với kinh nghiệm cá nhân ấm áp của việc đọc một cuốn sách hay. Đây không phải là lí do duy nhất tại sao Internet sẽ không bao giờ thay thế được sách, vì sách cung cấp kiến thức chuyên sâu về một chủ đề mà việc ngồi trước màn hình máy tính không thể cung cấp được.)

Câu 236: Đáp án A

Từ "aesthetic” gần nghĩa nhất với từ

    1. thẩm mĩ, đẹp B. bình thường

C. bản in D. không thể nhìn thấy được

Từ đồng nghĩa: aesthetic (thẩm mĩ) = artistic

We can download text from an Internet source, but the aesthetic quality of sheets of downloaded text leave much to be desired. A well-designed book enhances the reading experience.

(Chúng ta có thế tải một văn bản từ một nguồn Internet, nhưng chất lượng thẩm mỹ của các tờ văn bản tải xuống không như mong muốn. Một cuốn sách được thiết kế tốt sẽ nâng cao trải nghiệm đọc sách.) Câu 237: Đáp án B

Câu nào trong các câu sau đây được đề cập đến như là thuận lợi của sách trong đoạn 2?

A. đắt đỏ, khiêm tốn và dễ mang theo B. nhỏ gọn, rẻ và thuận tiện

C. xa hoa, to lớn và linh động D. không phổ biến, rẻ và giàu kiến thức

Căn cứ thông tin đoạn 2:

The book is still the most compact and inexpensive means of conveying a dense amount of knowledge in a convenient package. The easy portability of the book is what makes it the most user-friendly format for knowledge ever invented. (Sách vẫn là phương tiện nhỏ gọn và rẻ tiền nhất để truyền tải một lượng kiến thức dày đặc trong một kích thước thuận tiện. Tính di động dễ dàng của cuốn sách là điều làm cho nó trở thành định dạng thân thiện với người dùng nhất cho kiến thức từng được phát minh.)

Câu 238: Đáp án A

Từ "astounding” trong đoạn 3 có thể được thay thế tốt nhất bởi từ .

A. ngạc nghiên, kinh ngạc B. không ngạc nhiên

C. chặt chẽ D. tăng lên

Từ đồng nghĩa: astounding (ngạc nhiên, kinh ngạc) = astonishing

Not only has the art and craft of printing and book manufacturing been greatly improved over the centuries, but the great variety of subject matter now available in books is astounding, to say the least.

(Không chỉ có nghệ thuật và thủ công in ấn và sản xuất sách được cải thiện đáng kể qua nhiều thế kỷ, mà sự đa dạng lớn của chủ đề bây giờ có sẵn trong sách cũng là đáng kinh ngạc, ít nhất là như vậy.)

Câu 239: Đáp án C

Tác giả đề cập đến Internet trong đoạn cuối như là một công cụ mà .

  1. sẽ thay thế sách nếu tác giả không muốn xuất bản tác phẩm của họ.
  2. yêu cầu nhiều thiết bị để sử dụng hơn là sách.
  3. cung cấp cho tác giả những thông tin về chủ đề họ đang tìm kiếm cho cuốn sách của họ.
  4. không thể bị cầm, nhìn thấy hay cảm nhận được trong tay chúng ta.

Căn cứ thông tin đoạn cuối:

Another important reason why the Internet will never replace books is because those who wish to become writers want to see their works permanently published as books - something you can hold, see, feel, skim through, and read at one's leisure without the need for an electric current apart from a lamp. The writer may use a word processor instead of a typewriter or a pen and pad, but the finished product must eventually end up as a book if it is to have value to the reading public. The writer may use the Internet in the course of researching a subject just as he may use a library for that purpose, but the end product will still be a book.

(Một lý do quan trọng khác tại sao Internet sẽ không bao giờ thay thế được sách là bởi vì những người muốn trở thành nhà văn muốn xem tác phẩm của họ được xuất bản vĩnh viễn dưới dạng sách - một thứ bạn có thể năm giữ, xem, cảm nhận, đọc lướt qua và đọc giải trí mà không cần dòng điện ngoài một chiếc đèn. Người viết có thể sử dụng bộ xử lý văn bản thay vì máy đánh chữ hoặc bút và giấy, nhưng sản phẩm hoàn chỉnh cuối cùng phải kết thúc dưới dạng sách nếu nó có giá trị cho cộng đồng đọc sách. Nhà văn có thể sử dụng Internet trong quá trình nghiên cứu một chủ đề giống như ông có thể sử dụng một thư viện cho mục đích đó, nhưng sản phẩm cuối cùng vẫn sẽ là một cuốn sách.)

Câu 240: Đáp án C

Theo đoạn văn, câu nào là không đúng về sách?

  1. Bất kì khi nào sách còn hữu ích cho xã hội chúng ta, chúng không thể bị thay thế.
  2. Một cuốn sách được thiết kế tốt thì sẽ giúp việc đọc sách có hiệu quả hơn là văn bản tải xuống từ Internet.
  3. Không có một cuộc tiến hoá sách nào bởi vì sự thuận tiện không phổ biến của nó.
  4. Qua nhiều thế kỉ, bề ngoài của sách đã được nâng cấp.

Căn cứ vào các thông tin sau trong bài đọc:

This is not the only reason why the Internet will never replace books, for books provide the in-depth knowledge of a subject that sitting in front of a computer monitor cannot provide. (Đây không phải là lí

do duy nhất tại sao Internet sẽ không bao giờ thay thế được sách, vì sách cung cấp kiến thức chuyên sâu về một chủ đề mà việc ngồi trước màn hình máy tính không thể cung cấp được.)

A well-designed book enhances the reading experience. (Một cuốn sách được thiết kế tốt sẽ nâng cao trải nghiệm đọc sách.)

We take such uncommon convenience for granted, not realizing that the book itself has undergone quite an evolution since the production of the Gutenberg Bible in 1455 and Shakespeare's First Folio in 1623, just three years after the Pilgrims landed at Plymouth to colonize the New World. (Chúng ta đã coi sự thuận tiện không phổ biến đó là đương nhiên, không nhận ra rằng bản thân cuốn sách đã trải qua một

quá trình tiến hóa kể từ khi sản xuất Kinh Thánh Gutenberg năm 1455 Vd Folio đầu tiên của Shakespeare năm 1623, chỉ ba năm sau khi những người hành hương đến Plymouth để chiếm đóng thế giới mới.)

Not only has the art and craft of printing and book manufacturing been greatly improved over the centuries, but the great variety of subject matter now available in books is astounding, to say the least. (Không chỉ có nghệ thuật và thủ công in ấn và sản xuất sách được cải thiện đáng kể qua nhiều thế kỷ, mà sự đa dạng lớn của chủ đề bấy giờ có sẵn trong sách cũng là đáng kinh ngạc, ít nhất là như vậy.)

Tạm dịch

Internet giống như truyền hình, cái mà chiếm mất nhiều thời gian dùng để theo đuổi các mục đích khác, cung cấp giải trí và thông tin, nhưng thế nào có thể so sánh với trải nghiệm cá nhân ấm áp của việc đọc một cuốn sách hay. Đây không phải là lý do duy nhất tại sao Internet sẽ không bao giờ thay thế được sách, vì sách cung cấp kiến thức chuyên sâu về một chủ đề mà việc ngồi trước màn hình máy tính không thể cung cấp được. Chúng ta có thể tải một văn bản từ một nguồn Internet, nhưng chất lượng thẩm mỹ của các tờ văn bản tải xuống không như mong muốn. Một cuốn sách được thiết kế tốt sẽ nâng cao trải

nghiệm đọc sách.

Sách vẫn là phương tiện nhỏ gọn và rẻ tiền nhất để truyền tải một lượng kiến thức dày đặc trong một kích thước thuận tiện. Tính di động dễ dàng của cuốn sách là điều làm cho nó trở thành định dạng thân thiện với người dùng nhất cho kiến thức từng được phát minh. Ý tưởng rằng người ta có thể mang trong túi của mình một vở kịch của Shakespeare, một cuốn tiểu thuyết của Charles Dickens hoặc Tom Clancy, Đối Thoại của Plato, hay Kinh Thánh trong một ấn bản bìa mềm nhỏ làm tâm trí lưỡng lự. Chúng ta đã coi sự thuận tiện không phố biển đó là đương nhiên, không nhận ra rằng bản thân cuốn sách đã trải qua một quá trình tiến hóa kể từ khi sản xuất Kinh Thánh Gutenberg năm 1455 và F olio đầu tiên của Shakespeare năm 1623, chỉ ba năm sau khi những người hành hương đến Plymouth để chiếm đóng thế giới mới.

Không chỉ có nghệ thuật và thủ công in ấn và sản xuất sách được cải thiện đáng kể qua nhiều thế kỷ, mà sự đa dạng lớn của chủ đề bây giờ có sẵn trong sách cũng là đáng kinh ngạc, ít nhất là như vậy.

Trên thực tế, Internet đòi hỏi đầu vào liên tục của tác giả và sách của họ để cung cấp cho nó thông tin làm cho nó trở thành một công cụ hữu ích để khám phá và học tập.

Một lý do quan trọng khác tại sao Internet sẽ không bao giờ thay thế được sách là bởi vì những người muốn trở thành nhà văn muốn xem tác phẩm của họ được xuất bản vĩnh viễn dưới dạng sách - một thứ bạn có thể nắm giữ, xem, cảm nhận, đọc lướt qua và đọc giải trí mà không cần dòng điện ngoài một chiếc đèn.

Người viết có thể sử dụng bộ xử lý văn bản thay vì máy đánh chữ hoặc bút và giấy, những sản phẩm hoàn chỉnh cuối cùng phải kết thúc dưới dạng sách nếu nó có giá trị cho cộng đồng đọc sách. Nhà văn có thể sử dụng Internet trong quá trình nghiên cứu một chủ đề giống như ông có thể sử dụng một thư viện cho mục đích đó, nhưng sản phẩm cuối cùng vẫn sẽ là một cuốn sách.

Câu 241: Đáp án B

CHỦ ĐỀ INTERNATIONAL ORGANIZATIONS

Đoạn văn này chủ yếu thảo luận về cái gì?

A. Lịch sử của WWF B. Những việc mà WWF đã làm để bảo tồn thiên nhiên

C. Sự thành lập WWF D. Sứ mệnh của WWF

Cả bài văn nói về những việc mà tổ chức WWF đã làm để bảo tồn thiên nhiên.

Câu 242: Đáp án B

Tổ chức WWF được thành lập vào năm nào?

A. Năm 1960 B. Năm 1961 C. Năm 1989 D. Năm 1990

Căn cứ thông tin đoạn 1:

In 1960. a group of British naturalists—most notably biologist Sir Julian Huxley. artist and conservationist Peter Scott, and ornithologists Guy Mountfort and Max Nicholson—led an effort to establish an organization that protected endangered species and their habitats. The following year the World Wildlife Fund was founded.

(Năm 1960, 1 nhóm các nhà tự nhiên học người Anh – đáng kể nhất là nhà sinh vật học ông Juiion Huxley nhà hoạ sĩ và bảo tồn học Peter Scott và những nhà điểu cầm học Guy Mountfort và Max Nicholson - đã cùng nỗ lực thành lập một tổ chức để bảo vệ các loài động vật đang có nguy cơ tuyệt chủng và môi trường sống của chúng. Năm sau đó, tổ chức Quỹ Động vật hoang dã Thế giới được thành lập.)

Câu 243: Đáp án C

Từ “ensuing” trong đoạn 1 gần nghĩa nhất với từ nào?

A. quá khứ B. hiện tại C. tiếp theo D. hiện tại Từ đồng nghĩa: ensuing (tiếp theo, sau đó) = following.

Câu 244: Đáp án D

Theo đoạn văn, sứ mệnh hiện tại nào của WWF không được để cập đến trong bài?

  1. để bảo vệ sự đa dạng sinh học của thể giới.
  2. để làm giảm ô nhiễm và sự tiêu thụ lãng phí.
  3. để đảm bảo rằng nguồn tài nguyên thiên nhiên có thể tái tạo được sử dụng một cách bền vững.
  4. để bảo vệ các loài động vật có nguy cơ tuyệt chủng và môi trường sống của chúng.

Căn cứ thông tin đoạn 1:

Today its mission statement is threefold: to conserve the world’s biological diversity, to ensure that the use of renewable natural resources is sustainable, and to promote the reduction of pollution and of wasteful consumption. (Ngày nay, sứ mệnh của tổ chức này gồm 3 phần: để bảo tồn sự đa dạng sinh học của thế giớí, đế đảm bảo rằng việc sử dụng các nguồn tài nguyên thiên nhiên có thể tái tạo là bền vững và để thúc đấy sự gíảm thiểu ô nhiễm và sự tiêu thụ lãng phí.)

Câu 245: Đáp án A

Từ “it” trong đoạn 1 đề cập đến ?

A. Tổ chức B. Sự chú ý

C. Dữ liệu khoa học chắc chắn D. Hoạt động được tổ chức tốt

Căn cứ thông tin đoạn 1:

The organization has long included both conservationists and businesspeople with the intention of combining solid scientific data with well-managed action. It also seeks cooperation between nongovernmental organizations, local governments, and local populations. (Tổ chức này từ lâu đã bao gồm cả những nhà bảo tồn học và doanh nhân với dự định kết hợp dữ liệu khoa học vững chắc với hoạt động được quản lý chặt chẽ. Nó cũng tìm kiếm sự hợp tác giữa các tổ chức phi chính phủ, chính quyền địa phương và người dân địa phương.)

Câu 246: Đáp án C

Có thể suy ra từ đoạn văn này là .

  1. WWF được thành lập bởi một nhóm các nhà sinh vật học người Mỹ.
  2. WWF đã hợp tác với nhiều tổ chức để đổi tên của nó.
  3. WWF đã nỗ lực để cung cấp cho con người một thế giới bền vững.
  4. WWF sử dụng dịch vụ hoán đổi nợ vì tự nhiên để cho các nước ngoài mượn tiền.

Căn cứ thông tin đoạn 2:

The group has also been involved in efforts to provide a safe and sustainabie habitat for the world’s peoples, both urban and rural, including clean water, clean air, healthful food, and rewarding recreation areas. (Tổ chức này cũng đã nỗ lực để cung cấp 1 môi trường sống bền vững và an toàn cho

các dân tộc trên thế giới, cả vùng đô thị và nông thôn, bao gồm nước sạch, không khí trong lành, thức ăn lành mạnh và các khu vực giải trí bổ ích.)

  • Câu A sai vì theo thông tin đoạn 1: In 1960, a group of British naturalists - most notably biologist Sir Julian Huxley, artist and conservationist Peter Scott, and ornithologists Guy Mountfort and Max Nicholson - led an effort to establish an organization that protected endangered species and their habitats. (Năm 1960, một nhóm các nhà tự nhiên học người Anh - đáng kể nhất là nhà sinh vật học ông Julian Huxley, nhà hoạ sĩ và bảo tồn học Peter Scott và những nhà điểu cầm học Guy Mountfort và Max Nicholson - đã cùng nỗ lực thành lập một tổ chức để bảo vệ các loài động vật đang có nguy cơ tuyệt chủng và môi trường sống của chúng.)
  • Câu B sai vì không có thông tin nào trong bài cho biết tổ chức này hợp tác với các tổ chức khác để đổi tên.
  • Câu D sai vì theo thông tin đoạn 2: Among the WWF’s notable achievements is its use of debt-for- nature swaps, in which an organization buys some of a country’ 5 foreign debt at a discount, converts the money to local currency, and then uses it to fiance conservation efforts. (Trong các thành tựu đáng kế của WWF là dịch vụ hoán đổi nợ vì tự nhiên, trong đó, một tổ chức sẽ mua một số khoản nợ của một quốc gia khác với mức chiết khấu, chuyển đổi thành nội tệ và sử dụng nó để cấp tài chính cho các hoạt động bảo tồn.)

Câu 247 Đáp án A

Cụm từ “ocused on” trong đoạn 2 có thể được thay thế bởi .

A. tập trung vào B. phụ thuộc vào C. dựa vào D. dựa vào Từ đồng nghĩa: focused on (tập trung vào) = concentrated on

Câu 248: Đáp án D

Câu nào là đúng về tổ chửc WWF?

  1. Biểu trưng con gấu trúc của tổ chức này được tạo ra bởi hai người đàn ông, một hoạ sĩ và một nhà bảo tồn học.
  2. Tổ chức này có hàng triệu người ủng hộ ở gần 100 quốc gia.
  3. Trụ sở của tổ chức này ở Thuỵ Điển.
  4. Các chương trình của tổ chức này bao gồm các vấn đề của cả khu vực trên cạn lẫn dưới nước.

Căn cứ thông tin đoạn 2:

These include programs focused on individual species, forests, and freshwater and marine issues as well as climate change and responsible international trade. (Các sáng kiến này gồm các chương trình tập trung vào các cá thể loài, rừng, các vấn đề nước ngọt và biến cũng như thay đổi khí hậu và trách nhiệm trong thương mại quốc tế)

  • Câu A sai vì theo thông tin đoạn 1: The organization’s distinctive panda logo was created by Scott (Biểu trưng con gấu trúc đặc biệt của tổ chức này được tạo ra bởi Scott) và thông tin "artist and conservationist Peter Scott” (Peter Scott vừa là một hoạ sĩ vừa là một nhà bảo tồn học.)
  • Câu B sai vi theo thông tin đoạn 3: At the beginning of the let century, the WWF was active in more than 100 countries and had more than five million supporters. (Đầu thế kỉ 21, tổ chức WWF đã hoạt động ở trên 100 quốc gia và có hơn 5 triệu người ủng hộ.)
  • Các C sai vì theo thông tin đoạn 3: The organization’s international headquarters are in Gland, Switz.

(Tổ chức này có trụ sở quốc tế ở Gland, Thuỵ Sĩ.)

Dịch bài

WWF. tên đầy đủ là Quỹ Toàn cầu về Thiên nhiên, là tổ chức quốc tế cam kết bảo tồn môi trường. Ở Bắc Mỹ, nó được gọi là Quỹ động vật hoang dã thế giới. Năm 1960, một nhóm các nhà tự nhiên học người Anh, nhà sinh vật học nổi tiếng Sir Julian Huxley, hoạ sĩ và nhà bảo tồn Peter Scott, và các nhà điểu cầm học Guy Mountfort và Max Nicholson đã nỗ lực thiết lập một tổ chức bảo vệ các loài có nguy cơ tuyệt chủng và môi trường sống của chúng. Năm sau, Quỹ Động vật hoang dã Thế giới được thành lập; tên quốc tế sau đó đã được đổi thành World Wide Fund for Nature vào năm 1989, mặc dù ở Hoa Kỳ và Canada, tên này vẫn giữ nguyên như tên sáng lập. Biểu tượng gấu trúc đặc biệt của tổ chức được tạo ra bởi Scott. Khi đối mặt với các mối đe dọa môi trường ngày càng tăng trong những năm tiếp theo, các hoạt động của WWF mở rộng trong phạm

  1. Ngày nay, sứ mệnh của tổ chức này gồm 3 phần: để bảo tồn sự đa dạng sinh học của thế giới, để đảm bảo rằng việc sử dụng tài nguyên thiên nhiên tái tạo là bền vững, và thúc đẩy giảm ô nhiễm và tiêu thụ lãng phí. Tổ chức từ lâu đã bao gồm cả các nhà bảo tồn và doanh nhân với mục đích kết hợp dữ liệu khoa học vững chắc với hành động được quản lý tốt. Nó cũng tìm kiếm sự hợp tác giữa các tổ chức phi chính phủ, chính quyền địa phương và người dân địa phương. WWF làm việc chặt chẽ với Liên minh Bảo tồn Thế giới và đã hình thành quan hệ đối tác với Liên Hợp Quốc, Ngân hàng Thế giới và Liên minh Châu Âu.

WWF cung cấp tiền cho các sáng kiến bảo tồn trên toàn thế giới. Chúng bao gồm các chương trình tập trung vào các loài cá thế, rừng, và các vấn đề nước ngọt và biến cũng như biến đối khí hậu và thương mại quốc tế có trách nhiệm. Nhóm cũng đã tham gia vào các nỗ lực để cung cấp môi trường sống an toàn và bền vững cho các dân tộc trên thế giới, cả thành thị và nông thôn, bao gồm nước sạch, không khí trong lành, thực phẩm lành mạnh và các khu vực giải tri bổ ích. Trong số những thành tựu đáng chú ý của WWF là việc sử dụng các giao dịch hoán đổi nợ, trong đó một tổ chức mua một số khoản nợ nước ngoài của một quốc gia với mức chiết khấu, chuyển đổi tiền thành nội tệ và sử dụng nó để tài trợ cho các hoạt động bảo tồn. Cuộc hoán đổi nợ thành công đầu tiên của WWF diễn ra vào năm 1987 tại Ecuador.

Vào đầu thế kỷ 21, WWF đã hoạt động tại hơn 100 quốc gia và có hơn 5 triệu người ủng hộ. Trụ sở quốc tế của tổ chức này nằm ở Gland, Thug Si và có hơn 90 văn phòng trên toàn thế giới. Câu 249: Đáp án C

Đoạn băn chủ yếu là về .

    1. Cách The Beatles thành công hơn các nhóm khác
    2. Tại sao The Beatles chia tay sau 7 năm
    3. Sự nổi tiếng và thành công của The Beatles
    4. Nhiều người có khả năng hát một bài hát của The Beatles

Câu250: Đáp án B

Bốn chàng trai aria The Beatles .

A. Xuất thân từ cùng một gia đình B. Đến từ một thị trấn ở phía Bắc nước Anh

C. Ở cùng độ tuổi D. Nhận được sự huấn luyện về âm nhạc Thông tin: "They were four boys from the north of England and none of them had any training in music.” Câu 251: Đáp án D

Từ “sensational” có ý nghĩa gần nhất với .

A. Khét tiếng B. Xấu C. Sốc D. Phổ biến

Thông tin: “Since then. there have been a great many groups that have achieved enormous fame, so it is perhaps difficult now to imagine how sensational. The Beatles were at that time.”(Kể từ đó, đã có rất nhiều nhóm nhạc đã đạt được danh vọng rất lớn, vì vậy có lẽ bây giờ khó tưởng tượng Beatles gây ấn tượng lớn thế nào vào thời điểm đó.)

=> "sensational" ~ popular: gây ấn tượng lớn, phổ biến.

Câu 252: Đáp án B

Điền gì không đúng về The Beatles?

  1. Các thành viên không được đào tạo về âm nhạc.
  2. Họ đã có một sự nghiệp ổn định lâu dài.
  3. Họ trở nên nổi tiếng khi họ viết những bài hát của mình.
  4. Họ sợ bị làm tổn thương bởi người hâm mộ.

Thông tin: "The Beatles did not have a long career. Their first hit record was in 1963 and they split up in 1970.” (The Beatles đã không có một sự nghiệp lâu dài. Kỷ lục lớn đầu tiên của họ là vào năm 1963 và họ giải tán năm 1970.)

Câu 253: Đáp án D

The Beatles ngừng biểu diễn trực tiếp bởi vì .

A. Họ đã kiếm được đủ tiền B. Họ không muốn làm việc với nhau

C. Họ dành nhiều thời gian viết ca khúc D. Họ sợ bị người hâm mộ làm tổn thương.

Thông tin: “They stopped doing live performances in 1966 because it had become too dangerous for them - their fans were so excited that they surrounded them and tried to take their clothes as souvenirs!” (Họ dừng biểu diễn trực tiếp vào năm 1966 bởi vì nó trở nên quá nguy hiểm đối với họ - người hâm mộ của họ kích động đến nỗi vây quanh họ và cố gắng lấy quần áo của họ làm quà lưu niệm!)

Câu 254: Đáp án B

Từ “they” có nghĩa gần nhất là .

A. The Beatles B. người hâm mộ C. các bài hát D. các buổi trình diễn Thông tin: “They stopped doing live performances in 1966 because it had become too dangerous for them — their fans were so excited that they surrounded them and tried to take their clothes as souvenirs!" (Họ dừng biểu diễn trực tiếp vào năm 1966 bởi vì nó trở nên quá nguy hiểm đối với họ - người hâm mộ của họ kích động đến nỗi vây quanh họ và cố gắng lấy quần áo của họ làm quà lưu niệm!)

=> "they" đề cập đến “fans" (người hâm mộ)

Câu 255: Đáp án A

Từ nào có thể mô tả thái độ của người viết?

A. ngưỡng mộ C. trung lập B. chỉ trích D. mỉa mai

Dịch bài

Trong những năm 1960, The Beatles có lẽ là nhóm nhạc pop nổi tiếng nhất trên thế giới. Kể từ đó, đã có rất nhiều nhóm nhạc đã đạt được danh vọng rất lớn, vì vậy có lẽ bây giờ khó tưởng tượng Beatles gây ấn tượng lớn thế nào vào thời điểm đó. Họ là bốn chàng trai từ phía bắc nước Anh và không ai trong số họ đã được đào tạo về âm nhạc. Họ bắt đầu bằng cách biểu diễn và thu âm các bài hát của người Mỹ da đen và họ đã có một số thành công với những bài hát này. Sau đó, họ bắt đầu viết bài hát của mình và đó là khi họ trở nên nổi tiếng. The Beatles thay đổi nhạc pop. Họ là nhóm nhạc pop đầu tiên đạt được thành công lớn từ những bài hát họ tự viết. Sau đó các nhóm và ca sĩ viết các bài hát của mình trở nên phổ biến. The Beatles đã không có một sự nghiệp lâu dài. Kỷ lục hit đầu tiên của họ là vào năm 1963 và họ giải tán năm 1970. Họ dừng biểu diễn trực tiếp vào năm 1966 bởi vì nó trở nên quá nguy hiểm đối với họ - người hâm mộ của họ kích động đến nỗi vây quanh họ và cố gắng lấy quần áo của họ làm quà lưu niệm! Tuy nhiên, ngày hôm nay một số ca khúc của họ vẫn nổi tiếng như khi chúng phát hành. Trên khắp thế giới, nhiều người có thể hát một phần bài hát Beatles nếu bạn hỏi họ.

Câu 256: Đáp án A

CHỦ ĐỀ WOMEN IN SOCIETY

Tiêu đề phù hợp nhất cho bài đọc này là gì?

  1. Phụ nữ có nên đi làm sau khi kết hôn?
  2. Vai trò của phụ nữ trong xã hội.
  3. Những công việc nào là phù hợp cho phụ nữ sau khi kết hôn?
  4. Các đặc điểm cần thiết cho phụ nữ đi làm sau khi kết hôn.

Câu 257: Đáp án C

Thông tin nào về phụ nữ trong đoạn 1 sau đây là không chính xác?

  1. Vai trò chính của phụ nữ trong quá khứ là sinh con và chăm sóc gia đình.
  2. Phụ nữ sẽ có cơ hội làm việc bình đẳng nếu họ có đủ khả năng như nam giới.
  3. Việc ra ngoài đi làm của phụ nữ không thay đổi bất kì điều gì trong cuộc sống của gia đình họ.
  4. Phụ nữ đóng vai trò quan trọng trong gia đình của họ.

Căn cứ vào thông tin đoạn 1:

Therefore, women began to make the scene and, eventually, there was a steady flow of women leaving their homes. A certain vacuum or emptiness was thus created in the households. No more could the husbands turn to their wives after a hard day's work. (Vì vậy, phụ nữ bắt đầu thay đổi và cuối cùng, có một dòng chảy ổn định nữ giới rời nhà đi làm. Một sự trống vắng và trống trải vì vậy đã được tạo ra trong các hộ gia đình. Những người chồng không còn hướng về phía vợ mình sau một ngày làm việc vất vả nữa)

Câu 258: Đáp án A

Từ "marveled" ở đoạn 1 có thể được thay thế bởi

    1. ngac nhiên B. hao hứng

C. mê hoặc D. tức giận

Từ đồng nghĩa: marveled (ngac nhiên, kinh ngac) = was amazed (ngac nhiên)

Câu 259: Đáp án C

Từ “this” trong đoạn 2 đề cập đến .

  1. đứa con của người phụ nữ.
  2. sự ảnh hưởng của người phụ nữ ở nhà cô ấy.
  3. sự chăm sóc của người mẹ là điều cần thiết nhất cho con cô ấy.
  4. nghề nghiêp mà người phụ nữ muốn làm.

Căn cứ thông tin đoạn 2:

What a child needs most is his mother's care because how the child is molded depends greatly on her. It is a real pity that women who leave their homes solely in search of a career seldom give a thought to this. (Cái mà một đứa trẻ cần nhất là sự chăm sóc của người mẹ bởi vì một đứa trẻ được nuôi nấng như thế nào phụ thuộc rất lớn vào người mẹ đó. Có một sự thật đáng tiếc là những người phụ nữ bỏ lại gia đình để ra ngoài đi làm lại ít khi nghĩ về điều này.)

Câu 260: Đáp án D

Từ "matrimony" trong đoạn 3 gần nghĩa nhất với từ .

    1. li hôn B. tái hôn C. độc thân D. hôn nhân Từ đồng nghĩa: matrimony (hôn nhân) = marriage.

Câu 261: Đáp án D

Theo đoạn văn, công việc nào là không phù hợp với phụ nữ sau khi kết hôn?

A. y tá B. thư ký

C. nhân viên đánh máy D. nhân viên quan hệ công chúng

Căn cứ thông tin đoạn 4:

Nevertheless, there are many professions which would not be advisable for women to indulge in after marriage. A public relations officer spends almost three-quarters of her time in her career. She has heavy tasks to shoulder which might require her to entertain others till odd hours at night. (Tuy nhiên, có nhiều nghề nghiệp không khuyến khích phụ nữ say mê vào sau khi kết hôn. Một nhân viên quan hệ công chúng sử dụng gần 3/4 thời gian của cô ấy cho công việc. Cô ấy có những nhiệm vụ nặng nề phải gánh vác, cái mà có thể yêu cầu cô ấy phải làm vui lòng những người khác đến tận tối muộn.)

Câu 262: Đáp án B

Tại sao một nữ doanh nhân nên suy nghĩ kĩ trước khi bắt đầu sự nghiệp của mình?

  1. Bởi vì cô ấy có những nhiệm vụ nặng nề phải gánh vác đến tận tối muộn.
  2. Vì sẽ rất khó cho cô ấy hoàn thành cả hai vai trò cùng một lúc.
  3. Vì cô ấy phải sử dụng 3/4 thời gian của mình vào kinh doanh.
  4. Vì cô ấy phải đâm đầu vào công việc này.

Căn cứ thông tin đoạn 4:

Women who venture into the business world should think twice before plunging into it. It would be beyond their means to fulfil two demanding roles at the same time.

(Những người phụ nữ muốn thử sức với kinh doanh nên suy nghĩ kĩ trước khi lao đầu vào nghề này. Điều này có nghĩa là họ sẽ vượt ngoài khả năng hoàn thành cả hai vai trò được yêu cầu cùng một lúc.)

Câu 263: Đáp án D

Có thể suy ra điều gì từ đoạn văn cuối?

  1. Phụ nữ nên ở nhà để hoàn thành trách nhiệm của họ.
  2. Phụ nữ nên nghĩ về hậu quả trong nhà họ.
  3. Phụ nữ không nên có sự nghiệp riêng.
  4. Phụ nữ nên hoàn thành vai trò cơ bản của mình ở nhà trước khi bắt đầu sự nghiệp riêng của mình.

Căn cứ thông tin đoạn cuối:

Unless and until she can fulfill the basic role of a housewife and mother, she should not make a career her sole responsibility. (Trừ khi và cho đến khi nào phụ nữ có thể hoàn thành vai trò cơ bản của một người

vợ và người mẹ, cô ấy mới nên có sự nghiệp riêng cho mình.)

Dịch bài

Đó là một thực tế không thể phủ nhận rằng vị trí của người phụ nữ đã từng là ở trong nhà. Trong quá khứ, phụ nữ chỉ được yêu cầu hoàn thành vai trò của mẹ và bà nội trợ. Hiện nay, tình trạng này đã thay đổi rất nhiều. Nếu một người phụ nữ sở hữu các thuộc tính và phẩm chất như nam giới, cô ấy chắc chắn sẽ có cơ hội bình đẳng trong thị trường việc làm mà không có nhiều thiên vị. Do đó, phụ nữ bắt đầu thay đổi và cuối cùng, có 1 dòng chảy ổn định nữ giới rời nhà đi làm. 1 sự trống vắng và trống trải vì vậy đã được tạo ra trong các hộ gia đình. Những người chồng không còn hướng về phía vợ mình sau 1 ngày làm việc vất vả nửa. Xă hội ngạc nhiên trước khả năng của phụ nữ, nhưng nó cũng phải chịu đựng khi nhận ra vai trò quan trọng của phụ nữ trong nhà của họ. Nếu phụ nữ được phép làm việc sau khi kết hôn thì sao? Câu trả lời chắc chắn là tích cực mặc dù vấn đề này là rất đáng tranh cãi về bản chất của các ngành nghề liên quan. Nếu một người phụ nữ theo đuổi sự nghiệp của mình nhưng đồng thời có thể chăm sóc cho gia đình và con cái của mình, thì không ai có thể tìm thấy bất kỳ lý do tại sao cô ấy không nên được phép làm như vậy.

Ảnh hưởng của một người phụ nữ là rất cần thiết trong nhà, với trẻ em. Cái mà 1 đứa trẻ cần nhất là sự chăm sóc của người mẹ bởi vì 1 đứa trẻ được nuôi nấng như thế nào phụ thuộc rất lớn vào người mẹ đó. Có 1 sự thật đáng tiếc là những người phụ nữ bỏ lại gia đình để ra ngoài đi làm lại ít khi nghĩ về điều này. Những đứa trẻ, là những sinh vật bất lực và phụ thuộc, có thể không có ai để tâm sự ở nhà, ngoại trừ người giúp việc hoặc người thân. Với sự trở về của người mẹ chỉ sau một ngày làm việc vất và, những đứa trẻ chắc chắn không nhận được nhiều sự chú ý.

Việc một người phụ nữ nên tiếp tục làm việc sau khi kết hôn hay không sẽ phụ thuộc vào bản chất của nghề nghiệp của mình. Đó là một sự lãng phí tài nguyên nếu phụ nữ, sau khi học đại học, ngay lập tức từ bỏ sự nghiệp của họ sau khi hôn nhân. Nỗ lực của một người phụ nữ cũng có thể góp phần vào sự thịnh vượng và phát triển của xã hội. Trong bối cảnh của Malaysia, một giáo viên chỉ được yêu cầu làm việc một tuần năm ngày với những kì nghỉ. Hơn nữa, cô ấy chỉ đến trường nửa ngày. Nửa ngày còn lại có thể được dành cho gia đình của cô. Một giáo viên, ngoài việc giáo dục xã hội, có thể hoàn thành vai trò của

cả mẹ và bà nội trợ. Có nhiều nghề nghiệp khác như những y tá, nhân viên và người đánh máy nơi phụ nữ có thể hoàn thành vai trò kép.

Tuy nhiên, có rất nhiều nghề mà sẽ không được khuyến khích cho phụ nữ để say mê sau khi kết hôn. Một nhân viên quan hệ công chúng sử dụng gần 3/4 thời gian của cô ấy cho công việc. Cô ấy có những nhiệm vụ nặng nề phải gánh vác, cái mà có thể yêu cầu cô ấy phải làm vui lòng những người khác

đến tận tối muộn. Những người phụ nữ muốn thử sức vào thể giới kinh doanh nên suy nghĩ kỹ trước khi lao vào nó. Nó sẽ vượt quá khả năng của họ để hoàn thành hai vai trò được yêu cầu cũng một lúc.

Vì vậy, trách nhiệm quan trọng nhất của người phụ nữ vẫn nằm ở gia đình cô ấy. Không có cô ấy ở trong nhà, người ta không dám nghĩ về hậu quả. Trừ khi và cho đến khi cô ấy có thể hoàn thành vai trò cơ bản của một bà nội trợ và mẹ, cô ấy mới nên có sự nghiệp cho riêng mình.

Câu 264: Đáp án C

CHỦ ĐỀ SPACE CONQUEST

Có thể suy luận từ đoạn văn .

  1. một ngày trên trái đất ngắn hơn một ngày trên sao Mộc.
  2. có các cấu trúc khác trên sao Mộc kích thước bằng với Điểm Đỏ.
  3. có những khi không thể nhìn thấy Điểm Đỏ ở mặt đất.
  4. Điểm Đỏ lớn là cấu trúc duy nhất trên sao Mộc.

Dẫn chứng: “The most famous mark on Jupiter is the Great Red Spot. It has shown variations in both intensity and color, and at times it has been invisible, but it always returns after a few years." (Đặc điểm nổi bật nhất trên sao Mộc là Điểm Đỏ. Nó đã cho thấy các biến thể về cường độ và màu sắc, và đôi khi nó vô hình, nhưng nó luôn luôn trở lại sau một vài năm.)

Câu 265: Đáp án C

Theo đoạn văn, hành tinh nào thường tỏa sáng nhất?

    1. Trái Đất B. sao Mộc C. Venus D. sao Hỏa

Dẫn chứng: “It is well placed for observation for several months in every year and on average is the brightest of the planets apart from Venus.” (Nó được quan sát trong vài tháng mỗi năm và trung bình là hành tính sáng nhất nếu không tính Venus.)

Câu 266: Đáp án A

Theo đoạn văn, Điểm Đỏ .

A. có màu sắc khác nhau B. lớn như trái đất

C. là một cấu trúc rắn nổi trong không khí D. tăng kích thước qua nhiều năm

Dẫn chứng: “The most famous mark on Jupiter is the Great Red Spot. It has shown variations in both intensity and color, and at times it has been invisible, but it always returns after a few years” (Đặc điểm nổi bật nhất trên sao Mộc là Điểm Đỏ. Nó đã cho thấy các biến thể về cường độ và màu sắc, và đôi khi nó vô hình, nhưng nó luôn luôn trở lại sau một vài năm.)

Câu 267: Đáp án B

Từ “it” trong đoạn 2 đề cập đến .

A. Sao Mộc B. Điểm Đỏ C. Cường độ D. Màu

Dẫn chứng: "The most famous mark on Jupiter is the Great Red Spot. It has shown variations in both intensity and color, and at times it has been invisible, but it always returns after a few years."

Câu 268: Đáp án D

Từ “exceptional” ở đoạn 2 chủ yếu có nghĩa là .

A. cực đoan B. bền vững C. tạm thời D. không bình thường Dẫn chứng: “Its longevity may well due to its exceptional size, but there are signs that it is decreasing in size, and it may not be permanent.” (Tuổi thơ của nó cũng có thể do kích thước đặc biệt của nó, nhưng có những dấu hiệu cho thấy nó đang giảm về kích thước, và nó có thể không vĩnh viễn.)

=> Exceptional (a): hiếm có, không bình thường

=> exceptional = infrequent

Câu 269: Đáp án D

Theo đoạn văn, điều nào sau đây KHÔNG đúng?

  1. Mộc tinh lớn hơn tất cả các hành tinh khác trong hệ mặt trời.
  2. Một ngày ở sao Mộc kéo dài gần 10 giờ.
  3. Điểm Đỏ di chuyển nhiều theo chiều dọc hơn chiều ngang.
  4. Các nhà khoa học có bằng chứng cho thấy các đốm đỏ nhỏ hơn đang gia tăng kích thước của chúng để trở thành các Điểm Đỏ khác.

Dẫn chứng: “Several smaller red spots have been seen occasionally but have not 1asted." (Một số vết đỏ nhỏ hơn thi thoảng được nhìn thấy nhưng không kéo dài.)

Câu 270: Đáp án B

Đoạn văn có thể được lấy từ .

    1. một tạp chí nghệ thuật B. một tạp chí địa chất

C. một sách giáo khoa trung học D. một cuốn khảo cổ học.

Dịch bài

Sao Mộc là hành tinh lớn nhất và đứng thứ năm theo thứ tự khoảng cách từ mặt trời. Nó được quan sát trong vài tháng mỗi năm và trung bình là hành tinh sáng nhất nếu không tính Venus, mặc dù trong một thời gian tương đối ngắn Sao Hỏa sáng hơn nó. Thời gian quay vòng chưa tới 10 giờ của sao Mộc làm cho nó trở thành hành tinh có ngày ngắn nhất trong hệ mặt trời so với các hành tinh chính. Không có mùa thật sự trên sao Mộc vì độ nghiêng dọc theo trục vuông góc của mặt phẳng quỹ đạo chỉ hơn 3 độ - ít hơn so với bất kỳ hành tinh nào khác.

Đặc điểm nổi bật nhất trên sao Mộc là Điểm Đỏ. Nó đã cho thấy các biến thể về cường độ và màu sắc, và đôi khi nó vô hình, nhưng nó luôn luôn trở lại sau một vài năm. Ở mức độ lớn nhất, nó có thể dài

40.000 km và rộng 14.000 km, vì vậy diện tích bề mặt của nó lớn hơn Trái Đất. Mặc dù vĩ độ của Điểm Đỏ rất khác nhau, nhưng nó có thể về dưới kinh độ. Trong thế kỷ vừa qua, nó trôi đạt theo chiều dọc tổng

cộng đã lên đến khoảng 1200 °. Vĩ độ nói chung rất gần -22 °. Đã có lần người ta nghĩ rằng Điểm Đó có thể là một vật thể rắn chắc hoặc nửa cứng trôi nổi bên ngoài của Mộc tinh. Tuy nhién, những kết quả của Pioneer và Voyager đã bác bỏ ý tưởng đó và đã chứng minh được Điểm đó là một hiện tượng khí tượng học Jovian. Tuổi thọ của nó cũng có thể do kích thước đặc biệt của nó, nhưng có những đầu hiện cho thấy nó đang giảm về kích thước, và nó có thể không vĩnh viễn. Một số vết đó nhỏ hơn thi thoảng được nhìn thấy nhưng không kéo dài.

Câu 271: Đáp án B

CHỦ ĐỀ GENERATION GAP

Mục đích của tác giả trong đoạn văn này là gì?

  1. Để miêu tả tình trạng khoảng cách thế hệ trong lực lượng lao động.
  2. Để đề xuất các giải pháp để vượt qua khoảng cách thế hệ giữa các công nhân lớn tuổi và trẻ
  3. Để đánh giá thế hệ nào sẽ có ảnh hưởng nhiều hơn trong lực lượng lao động.
  4. Để chỉ ra sự khác nhau giữa người lớn tuổi và người trẻ trong công việc.

Căn cứ vào thông tin đoạn 1:

However, members of each generation can close the gap between them if they're willing to meet one another halfway. (Tuy nhiên, các thành viên trong mỗi thế hệ có thể thu hẹp khoảng cách giữa họ nếu họ sẵn sàng gặp nhau nửa chừng)

Bên cạnh đó, trong mỗi đoạn 2, 3, 4 và 5, tác giá đều đưa ra 1 giải pháp để vượt qua khoảng cách thế hệ nơi làm việc.

Câu 272: Đáp án C

Mỗi thế hệ nên kính trọng thế hệ còn lại trong công việc để .

  1. khẳng định khả năng hoặc kinh nghiệm cá nhân mà họ đóng góp vào công việc.
  2. thể hiện sự công nhận vị trí của thế hệ còn lại trong nơi làm việc.
  3. khiến người khác cảm thấy rằng họ còn cần thiết hay có ích ở nơi làm việc.
  4. đối xử bình đẳng với thế hệ kia trong công việc.

Căn cứ thông tin đoạn 2:

Older workers can show respect to the youngerset by asking for their opinions and recognizing their contributions to the workplace as valid, or complimenting them on a job well done. Younger workers can show their elders respect by asking for advice on how to manage a situation with work, based on the older worker's many years of experience. It's important for both 'entry- and senior-level workers to see each other as equals, regardless of the type of position in which they work. No one wants to feel inferior or irrelevant just because of their age. (Các công nhân lớn tuổi có thể thể hiện sự kính trọng với người trẻ bằng cách hỏi ý kiến họ và công nhận những đóng góp hợp lý ở nơi làm việc hay khen ngợi họ về một công việc được thực hiện tốt. Những công nhân trẻ hơn có thể tôn trọng người lớn hơn bằng cách hỏi sự

tư vấn cách xử lý một tình huống ở công việc dựa trên kinh nghiệm nhiều năm của những người công nhân lớn tuổi. Điều quan trọng là cả công nhân mới vào nghề hay người có số năm lao động lâu hơn đều được đối xử công bằng, bất kể vị trí làm việc của họ là gì. Không ai muốn cảm thấy thua kém hay không phù hợp với công việc chỉ vì tuổi tác của họ.)

Câu 273: Đáp án D

Tác giả có ý gì khi nói “put themselves- in their colleagues'shoes” trong đoạn 3?

  1. Các công nhân nên thử đi giày của đồng nghiệp để hiểu sự khác biệt của thế hệ họ.
  2. Các công nhân nên mua giày của đồng nghiệp để không làm phiền họ về sự khác biệt về tuổi tác thế hệ.
  3. Các công nhân nên xác định những sự khác nhau giữa tuổi tác thế hệ để mà họ có thể hiểu hoàn cảnh của đồng nghiệp. ,
  4. Các công nhân nên đặt mình vào hoàn cảnh của đồng nghiệp để hiểu và thông cảm cho những khó khăn mà họ gặp phải.

Thành ngữ put themselves in their colleagues‘ shoes được hiểu là “đặt mình vào vị trí của người khác” thông thường là khi những người đó đang gặp những tình huống khó khăn để hiểu họ hơn.

Câu 274: Đáp án A

Từ “him" trong đoạn 3 đề cập đến .

    1. Một người công nhân lớn tuổi B. Một người công nhân trẻ tuổi

C. Một đồng nghiệp D. Một người nào đó Từ "him" thay thế cho danh từ an older worker trong câu trước.

If a person is much older than another, perhaps it is bitterness about fewer job opportunities, or fear that a younger worker might seem more relevant and edge him out of his job. (Nếu một người lớn tuổi hơn một người khác, có lẽ đó là sự cay đắng về ít cơ hội làm việc hơn, hay lo sợ rằng một người công nhân trẻ tuổi dường như phù hợp hơn trong công việc và khiến anh ta bị mất việc)

Câu 275: Đáp án C

Từ nào là từ đồng nghĩa của từ “stereotypes” trong đoạn 4?

A. thành tựu B. thất bại C. thành kiến D. sự ghen tị Từ đồng nghĩa: stereotypes (thành kiến) = prejudices

If age seems to be a problem for someone at the workplace, it can be helpful to do the very opposite of what a co-worker might'expect from someone of a different age set due to stereotypes. ( Nếu tuổi tác có vẻ là một vấn đề với ai đó ở nơi làm việc, thì việc làm ngược lại với những gì mà đồng nghiệp có thể mong đợi từ một người ở độ tuổi khác do các định kiến có thể sẽ có ích.)

Câu 276: Đáp án D

Theo đoạn văn, những điều sau đây là giải pháp để vượt qua khoảng cách thế hệ, ngoại trừ .

A. Giữ 1 tâm trí cởi mở B. Làm điều trái ngược lại

C. Yêu cầu phản hồi D. Đánh giá cao cơ hội

Căn cứ vào thông tin đoạn 3, 4 và 5: (ý chính của mỗi đoạn)

Đoạn 3: Giữ một tâm trí cởi mở để hiểu cho hoàn cảnh của đồng nghiệp.

Đoạn 4: Làm điều trái ngược với những gì mà người ta thường định kiến sẵn về người kia. Đoạn 5: Yêu cầu phản hồi về công việc bằng cách trao đổi hay hỏi về lời khuyên về công việc. Câu 277: Đáp án A

Từ “acknowledge” gần nghĩa nhất với từ .

A. công nhận, thừa nhận B. nâng cao

C. nhận thức D. phân biệt

Từ đồng nghĩa: acknowledge (công nhận, thừa nhận) = recognise

For example, older workers who are unfamiliar with new software that younger colleagues understand might acknowledge to them that they did the same tasks differently in years past but show interest in learning the program to keep up with modern technology. (Ví dụ, những công nhân lớn tuổi không quen thuộc với các phần mềm mới mà các đồng nghiệp trẻ hiểu thì có thể thừa nhận với họ rằng trong quá khứ họ đã làm những công việc này theo một cách khác, nhưng thể hiện sự quan tâm đến việc học các chương trình này để theo kịp công nghệ hiện đại.)

Câu 278: Đáp án B

Có thể suy ra từ đoạn văn là .

  1. Những công nhân trẻ và công nhân lớn tuổi nên gặp nhau trên đường để làm giảm khoảng cách thế hệ ở nơi làm việc.
  2. Nguyên tắc chính để vượt qua khoảng cách thế hệ là cân bằng giữa kinh nghiệm và công nghệ.
  3. Thái độ tích cực sẽ giúp giảm mâu thuẫn giữa người lớn tuổi và người trẻ tuổi.
  4. Môi trường cạnh tranh ở nơi làm việc có thể khiến khoảng cách thế hệ rộng hơn.

Căn cứ vào nội dung trong bài:

Trong các giải pháp tác giả đưa ra để vượt qua khoảng cách thế hệ thì nguyên tắc xuyên suốt trong đó là dung hoà giữa kinh nghiệm của những người lớn tuổi với công nghệ mà giới trẻ lĩnh hội được để đem vào công việc.

Bài dịch:

Một khoảng cách thế hệ tại nơi làm việc có thể làm cho cả công nhân trẻ và công nhân lớn tuổi cảm thấy thua kém, cũng như cản trở năng suất và làm việc theo nhóm. Sự khác biệt giữa các thế hệ có thể được nhìn thấy trong nguyên tắc công việc, thói quen và phong cách giao tiếp. Các công nhân trẻ tuổi hơn có thể

lo sợ không được các đồng nghiệp lớn tuổi của họ coi trọng, trong khi các công nhân lớn tuổi có thể lại lo sợ rằng kinh nghiệm của họ không có giá trị mà bị thay thế bởi công nhân có kiến thức về công nghệ hiện tại hơn. Tuy nhiên, các thành viên của mỗi thế hệ có thể thu hẹp khoảng cách giữa họ nếu họ sẵn sàng

gặp nhau nửa chừng.

Các công nhân lớn tuổi có thể thể hiện sự kính trọng với người trẻ bằng cách hỏi ý kiến họ và công nhận những đóng góp hợp lý ở nơi làm việc hay khen ngợi họ về 1 công việc được thực hiện tốt. Những công nhân trẻ hơn có thể tôn trọng người lớn hơn bằng cách hỏi sự tư vấn cách xử lý 1 tình huống ở công

việc dựa trên kinh nghiệm nhiều năm của những người công nhân lớn tuổi. Điều quan trọng là cả công nhân mới vào nghề hay người có số năm lao động lâu hơn đều được đối xử công bằng, bất kế vị trí làm việc của họ là gì. Không ai muốn cảm thấy thua kém hay không phù hợp với công việc chỉ vì tuổi tác của họ. Thay vào đó, một khoảng cách thế hệ tại nơi làm việc có thể là một cơ hội học tập.

Người lao động cũng có thể tự đặt mình vào vị trí của đồng nghiệp để xác định điều gì có thể làm phiền họ về sự khác biệt tuổi tác thế hệ của họ. Nếu một người già hơn người khác nhiều, có lẽ đó là sự cay đắng về ít cơ hội việc làm hơn, hoặc lo sợ rằng một người lao động trẻ tuổi dường như phù hợp hơn và

khiến anh ta mất việc. Nếu người lao động mở tâm trí của họ để hiểu vị trí đồng nghiệp đang đứng, nó có thể giúp giảm bớt căng thẳng giữa họ và đánh giá cao những đóng góp công việc của nhau.

Nếu tuổi tác có vẻ là một vấn đề đối với một ai đó tại nơi làm việc, có thể hữu ích khi làm điều ngược lại với những gì một đồng nghiệp có thể mong đợi từ một người ở độ tuổi khác do các định kiến. Ví dụ, nếu một người lao động khá trẻ, chẳng hạn như vừa tốt nghiệp đại học, thì cô có thể chia sẻ thông tin

nghiên cứu để cho biết rằng cô ấy biết những gì cô ấy đang làm, hoặc thể hiện sự tò mò thay vì buồn bã để bộc lộ sự trường thành về mặt tình cảm nếu 1 ai đó buông lời gièm pha về tuổi tác của mình. Công nhân lớn tuổi có thể duy trì một thái độ nhiệt tình về công việc thay vì thể hiện sự nhàm chán hoặc cay đắng từ

những kinh nghiệm trong quá khứ.

Những công nhân có thể, hơn nữa, trực tiếp giải quyết mối quan tâm của sự khác biệt tuổi tác tại nơi làm việc với các đồng nghiệp bất hoà bằng cách yêu cầu người đó cho lời khuyên xây dựng về cách giải quyết vấn đề. Ví dụ, những công nhân lớn tuổi không quen thuộc với các phần mềm mới mà các đồng

nghiệp trẻ hiểu thì có thể thừa nhận với họ rằng trong quá khứ họ đã làm những công việc này theo 1 cách khác, những thể hiện sự quan tâm đến việc học các chương trình này để theo kịp công nghệ hiện đại. Học cách sử dụng ngôn ngữ công nghệ có thể làm cho họ cảm thấy kết nối hơn. Tương tự như vậy, một công

nhân trẻ tuổi có thể thừa nhận là còn non trong môi trường làm việc, nhưng mong muốn có được kinh nghiệm bằng cách học hỏi từ các đồng nghiệp đi trước.

Câu 279: Đáp án D

CHỦ ĐỀ SEVEN WONDERS

Tiêu đề phù hợp nhất cho đoạn văn là gì?

    1. Các vấn đề với việc xây dựng Kim Tự tháp. B. Khám phá lăng mộ vua Cheops.

C. Biểu tượng của Kim Tự tháp. D. Kỳ quan của Kim Tự tháp Giza.

Căn cứ vào thông tin:

“One of the seven wonders of the ancient world, the Great Pyramid of Giza was a monument of wisdom and prophecy built as a tomb for Pharaoh Cheops in 2720 B.C.” (Một trong bảy kỳ quan của thể giới cổ đại, Kim Tự tháp Giza là tượng đài trí tuệ và tiên tri được xây dụng như một ngôi mộ cho Pharaoh Cheops vào năm 2720 trước Công nguyên.)

Câu 280: Đáp án B

Tại sao Kim tự tháp Giza là một trong bảy kỳ quan của thế giới?

  1. Nó được xây dựng bởi lực lượng siêu nhiên.
  2. Nó hoàn toàn phù hợp với bốn hướng của la bàn và chứa nhiều lời tiên tri.
  3. Nó được chọn làm ngôi mộ của Pharaoh Cheops.
  4. Nó rất cố.

Căn cứ vào thông tin:

“Despite its antiquity, certain aspects of its construction makes it one of the truly wonders of the world. The thirteen- acre structure near the Nile river is a solid mass of stone blocks covered with limestone. Inside are the number of hidden passageways and the burial chamber of the Pharaoh. It is the largest single structure in the world." (Mặc dù ở thời cổ đại, một số khía cạnh về xây dụng khiến nó trở thành một trong những kỳ quan thật sự của thế giới. Cấu trúc mười ba mẫu gần sông Nile là một khối đã phủ đầy đá vôi. Bên trong là nhiều lối đi ẩn và buồng chôn cất của Pharaoh. đây là cấu trúc đơn lẻ lớn nhất trên thế giới.)

Câu 281: Đáp án C

Lý do nào có thể xảy ra nhất để giải thích cho việc có nhiều lối đi bí mật?

  1. Để cho cân nặng của kim tự tháp cân bằng.
  2. Để cho phép các linh mục cao cả cầu nguyện vào ban đêm.
  3. Làm cho những tên cướp mộ không tìm thấy ngôi mộ và kho báu được chôn với Pharaoh.
  4. Cho phép gia đình Pharaoh mang thực phẩm theo hành trình của mình đến thế giới bên kia.

Căn cứ vào thông tin:

“Inside are the number of hidden passageways and the burial chamber of the Pharaoh.” (Bên trong là nhiều lối đi ẩn và buồng chôn cất của Pharaoh.)

Câu 282: Đáp án A

Từ "feat” trong đoạn văn số 1 gần nghĩa nhất với ?

  1. accomplishment /ə'kɔmpli∫mənt/ (n): thành tựu, thành tích, thành quả.
  2. festivity /fe’stiviti/(n): sự vui mừng; sự hân hoan.
  3. appendage /ə’pendidʒ/ (n) Vật thêm vào, phần thêm vào.
  4. structure /'strʌkt∫ə/ (n): cấu trúc, kết cấu.

Căn cứ vào ngữ cảnh của câu:

"The four sides of the pyramid are aligned almost exactly on true north, south, east and west-an incredible engineering feat” (Bốn cạnh của kim tự tháp được sắp xếp gần như chính xác về phía bắc, nam, đông và tây - một thành tựu kĩ thuật đáng kinh ngạc)

=> feat (n): kỳ công, thành tựu Đáp án A (accomplishment = feat) Câu 283 Đáp án D

Những người Ai Cập cổ đại đã tính toán dựa trên cơ sở nào?

    1. Các công cụ đo lường tiên tiến B. Kiến thức về bề mặt trái đất

C. Công nghệ tiên tiến D. Quan sát của các thiên thể

Căn cứ vào thông tin:

“The ancient Egyptians were sun worshippers and great astronomers, so computations for the Great Pyramid were based on astronomical observations.”

(Người Ai Cập cổ đại là những người tôn thờ mặt trời và là các nhà thiên Văn học vĩ đại, vì vậy các tính toán cho Kim tự tháp được dựa trên những quan sát thiên văn.)

Câu 284: Đáp án D

Nghiên cứu nền móng tiết lộ điều gì?

  1. Có những vết nứt trong nền móng.
  2. Đạo mộ đã đánh cắp xác của Pharaoh.
  3. Một chủng tộc vượt trội của loài người đã xây dựng.
  4. Các đường biểu diễn các sự kiện quan trọng.

Căn cứ vào thông tin:

"Further scientific study indicates that these represent a type of timeline of events - past, present and future.” (Nghiên cứu khoa học sau đó chỉ ra rằng đây là một dòng chảy của sự kiện - quá khứ, hiện tại và tương lai.)

Câu 285: Đáp án A

Trong đoạn văn 2, từ ”prophesied” gần nghĩa nhất với ?

    1. foretell /fɔ:'tel/ (v): béo trước B. terminate /'tə:mineit/ (v): chấm dứt

C. precipitate /pri'sipi,teit/ (v): kết tủa D. affiliate /ə'filieit/ (v): liên kết

Căn cứ vào ngữ cảnh của câu: “Many of the events have been interpreted and found to coincide with known facts of the past. Others are prophesied for future generations and are currently under investigation.” (Nhiều sự kiện đã được diễn giải và tìm thấy trùng khớp với những sự kiện đã biết của quá khứ. Những điều thì được tiên đoán cho các thế hệ tương lai và hiện đang được điều tra)

=> prophesy /'prɔfi,sai/ (v): tiên tri = foretell

Tạm dịch

Một trong bay kỳ quan của thế giới cổ đại, Kim Tự tháp Giza là tượng đài trí tuệ và tiên tri được xây dựng như một ngôi mộ cho Pharaoh Cheops vào năm 2720 trước Công nguyên. Mặc dù ở thời cổ đại, một số khía cạnh về xây dựng khiến nó một trong những kỳ quan thật sự của thế giới. Cấu trúc mười ba mẫu gần sông Nile là một khối đã phủ đầy đá vôi. Bên trong là nhiều lối đi ẩn và buồng chôn cất của Pharaoh. Đây là cấu trúc đơn lẻ lớn nhất trên thế giới. Bốn cạnh của kim tự tháp được sắp xếp gần như chính xác về phía bắc, nam, động và tây - một thành tựu kỹ thuật đáng kinh ngạc. Người Ai Cập cổ đại là những người tôn thờ mặt trời và là các nhà thiên văn học vĩ đại, vì vậy các tính toán cho Kim tự tháp được dựa trên những quan sát thiên văn.

Những khám phá và kiểm tra chi tiết của cấu trúc cho thấy nhiều đường giao nhau. Nghiên cứu khoa học sau đó chỉ ra rằng đây là một dòng chảy của sự kiện - quá khứ, hiện tại và tương lai. Nhiều sự kiện đã được diễn giải và tìm thấy trùng khớp với những sự kiện đã biết của quá khứ. Những điều thì được tiên đoán cho các thế hệ tương lai và hiện đang được điều tra. Nhiều người tin rằng kim tự tháp có sức mạnh siêu nhiên và điều này cũng không ngoại lệ. Một số nhà nghiên cứu thậm chí còn liên kết nó với các sinh vật ngoài trái đất trong quá khứ cổ đại.

Liệu cấu trúc thượng tầng này do những con người bình thường tạo ra hay được tạo ra bởi một giống loài vượt trội so với bất kỳ loài nào ngày nay?

Câu 286: Đáp án B

Chủ đề WATER SPORTS

Chủ đề của đoạn văn này là gì?

  1. Những lí do giải thích tại sao mọi người thích môn lặn
  2. Lịch sử của môn lặn
  3. Thiết bị cho môn lặn
  4. Những lí do giải thích tại sao không có nhiều người luyện tập môn lặn

Xuyên suốt cả bài là nói về lịch sử của môn lặn với rất nhiều sự kiện và mốc thời gian

=> chủ đề của bài là lịch sử của môn lặn.

Câu 287: Đáp án B

Trò lặn giải trí ______ _.

  1. Đòi hỏi thiết bị nhiều hơn là kĩ thuật lặn
  2. Được rất nhiều người tham gia như một hoạt động giải trí và thư giãn
  3. Không cần thiết bị gì
  4. Cần nhiều kinh nghiệm hơn là kĩ thuật lặn

Thông tin ở phần đầu tiên của đoạn văn:

Recreational diving or sport diving is a type of diving that uses scuba equipment for the purpose of leisure and enjoyment in some diving circles, the term “recreational diving” is used in contradistinction to “technical diving", 3 more demanding aspect of the sport which requires greater levels of training, experience and equipment.

(Môn giải trí lặn hay là môn thế thao lặn là một loại hình lặn mà nó sử dụng những thiết bị lặn cho mục đích là để thư giãn và giải trí. Ở một số phương diện của môn này, thuật ngữ “recreational diving” được dùng khác với thuật ngữ “technical diving” một lĩnh vực thể thao nhiều thách thức hơn, lĩnh vực này đòi hỏi mức độ cao hơn về sự luyện tập, kinh nghiệm và thiết bị hỗ trợ.)

Câu 288: Đáp án C

Những chuyến du ngoạn dưới nước trước đây từng bị hạn chế _______ _.

  1. Vì việc săn bắt dưới nước bị cấm
  2. Bởi vì những thứ cần thiết đảm bảo cho việc thờ được ở dưới nước là quá đắt để chi trả.
  3. Bởi vì những người lặn không thể mang đủ oxi để thở dưới nước
  4. Bởi vì những thứ cần thiết cho việc thở ở dưới nước là quá nặng để mang.

Theo thông tin ở đoan văn số 2:

For a long time, recreational underwater excursions were limited by the amount of breath that could be held. (Trong một thời gian dài, những chuyến du ngoạn ở dưới nước đã bị bạn chế bởi số lượng oxi có thế mang theo.)

Câu 289: Đáp án B

Từ “subsequent” có thể thay thế bằng từ nào dưới đây:

A. successive (a): liên tục, liên tiếp B. following (a): theo sau, nối tiếp

C. ancient (a): cổ xưa D. succeeding (a); tiếp theo, thừa kế

Câu 290: Đáp án D

Theo như đoạn văn số 2, trong suốt những năm 1950 và 1960, môn lặn giải trí là một thể thao chỉ dành cho một số người nhất định bởi_______ _.

  1. Những người lặn không thích tham gia vao các khóa huấn luyện chuyên sâu
  2. Không có bất cứ một khóa huấn luyện chuyên sâu nào cho người lặn
  3. Không có đủ đồ cho nhiều người lặn
  4. Đồ lặn và các khóa huấn luyện chuyên sâu là quá đắt cho nhiều người có thể trả.

Theo thông tin của đoạn văn số 2:

However, for much of the 19505 and early19605, recreational scuba diving was a sport limited to those who were able to afford or make their own kit, and prepared to undergo intensive training to use it. (Tuy nhiên, trong những năm 1950 và 1960, môn lặn giải trí là một môn thể thao chỉ dành cho những người người mà có thể đủ sức trả cho những đồ lặn và những khóa huấn luyện chuyên sâu để thực hành nó.)

Câu 291: Đáp án A

Tất cả những câu dưới đây đều đúng ngoại trừ ______ _.

  1. Trong những năm 1950 bất cứ ai muốn lặn đều có thể được đào tạo chuyên nghiệp.
  2. Trong những năm 1950 không có khóa đào tạo nào có sẵn cho những người người đã mua những thiết bị lặn đời đầu.
  3. Khi hoạt động lặn giải trí dần trở lên phổ biến hơn, các nhà sản xuất đã tạo ra nhiều thiết bị lặn hơn.
  4. Những tiến bộ trong công nghệ lặn đã khuyến khích ngày càng nhiều người tập luyện và tham gia vào môn thể thao này.

Theo thông tin của đoạn văn số 3:

Until the early 19505, navies and other organizations performing professional diving were the only providers of diver training, but only for their own personnel and only using their own types of equipment. There were no training courses available to civilians who bought the first scuba equipment. (Cho tới đầu những năm 1950. Quân đội và những tổ chức huấn luyện lặn là những đơn vị duy nhất cung cấp chương trình đào tạo này, nhưng chỉ dành cho nhân viên của họ và chỉ sử dụng thiết bị chuyên dụng của họ. Không có bất cứ khóa huấn luyện nào cho dân thường người mà đã mua những thiết bị lặn đầu tiên.)

Câu 292: Đáp án C Holiday divers _______ _.

  1. Không thích lặn ở những vùng nhiệt đới và cận nhiệt đới
  2. Có thể lặn sâu như họ thích bởi vì có sự an toàn
  3. Là những người mà đi xa nhà để lặn
  4. Chỉ tập trung ở những vùng nhiệt đới và cận nhiệt đới.

Theo thông tin ở đoạn văn cuối:

In tropical and sub-tropical parts of the world, there is a large market in holiday divers, who train and dive while on holiday, but rarely dive close to home. (Ở những vùng nhiệt đới và cận nhiệt đới có một thị trường cho những holiday divers, là những người tập luyện và lặn trong kì nghỉ nhưng hiếm khi nào lại lặn ở gần nhà.)

Câu 293: Đáp án C

Chủ đề NATURE IN DANGER

Chủ đề của đoạn văn là gì?

A. Các nguồn gây tổn hại môi trường B. Sự ô nhiễm từ thành phố

C. Ảnh hưởng xấu của chất thải công nghiệp D. Chất lượng môi trường

Căn cứ vào nhân thông tin đầu tiên của đoạn văn: “Pollution emitted in industrial areas represents a threat to human health and the surrounding natural resources. We have a tendency to believe that the production processes are the only source of environmental damage, and often forget about the possible long-term effects of harmful production practices." (Ô nhiễm trong lĩnh vực công nghiệp là mối đe dọa đối với sức khoẻ con người và các nguồn tài nguyên thiên nhiên xung quanh. Chúng ta có khuynh hướng tin tưởng rằng quá trình sản xuất là nguồn gây ra thiệt hại về môi trường duy nhất và thường quên đi những ảnh hưởng lâu dài có thể xảy ra đối với các hoạt động sản xuất có hại.)

Câu 294: Đáp án B

Theo đoạn văn, ngành công nghiệp có thể sẽ được nghĩ là _____ _.

  1. một mối nguy hiểm cho môi trường
  2. nguồn ô nhiễm duy nhất
  3. hoạt động gây hại tối đa
  4. một mối đe dọa đối với sức khoẻ con người

Thông tin: We have a tendency to believe that the production processes are the only source of environmental damage. (Chúng ta có khuynh hướng tin tưởng rằng quá trình sản xuất là nguồn gây ra thiệt hại về môi trường duy nhất.)

Câu 295: Đáp án A

Từ “nó” trong đoạn đầu đề cập đến _____ _.

A. chất thái hiện có B. nguy hiểm

C. môi trường D. sự đe dọa của chất thải hiện có

Căn cứ vào thông tin sau: "Unfortunately, this ignores the threat of the remaining waste, which is abandoned and poorly stored. It represents an even bigger danger because it stands neglected as it degrades and leaks into the earth without any control at all." (Thật không may, điều này đã bỏ qua mối đe dọa của chất thải hiện có, chúng bị đào thải và kém trong việc lưu trữ. Nó đại diện cho một

nguy hiểm lớn hơn bởi vì nó bị lãng quên khi nó giảm và rò rỉ vào đất mà không có bất kỳ kiểm soát nào cả.)

Câu 296: Đáp án A

Vấn đề nào sau đây ảnh hưởng đến toàn bộ hệ sinh thái?

A. Ô nhiễm nước bề mặt B. Sự ô nhiễm đất

C. Ô nhiễm nước ngầm D. Ô nhiễm không khí

Thông tin: "Changes in the water chemistry due to surface water contamination can affect all levels of an ecosystem." (Những thay đổi trong hóa học nước do ô nhiễm nước bề mặt có thể ảnh hưởng đến tất cả các cấp của một hệ sinh thái.)

Câu 297: Đáp án D

Theo đoạn văn, cái gì hỗ trợ hệ sinh thái lành mạnh?

A. Các sinh vật chuỗi dưới thức ăn B. Động vật

C. Sinh vật sống dưới nước D. Đất ngập nước

Thông tin: “It can damage the health of wetlands and damage their ability to support healthy ecosystems, control flooding, and filter pollutants from storm vlwater runoff.”

(Nó có thể làm hại vùng đất ngập nước và làm hư hại khả năng hỗ trợ các hệ sinh thái lành mạnh, kiểm soát lũ lụt, và lọc các chất gây ô nhiễm khi bão lũ.)

Câu 298: Đáp án C

Cái gì không bị ảnh hưởng xấu bởi nước ngầm bị ô nhiễm?

A. con người B. cây cối C. hòn đá

Câu 299: Đáp án D

Cái nào là dòng chảy của nước từ mặt đất đến bề mặt?

A. dòng B. ao C. sông D. suối

Thông tin: "Depending on the study of rocks of the area, groundwater may rise to the surface through Springs or seeps, flow sideways into nearby rivers, streams, or ponds, or sink deeper into the earth.” (Tùy thuộc vào các loại đá của vùng, nước ngầm có thể trào lên bề mặt thông qua suối hoặc bể nước, chảy theo dòng vào sông, suối, hoặc ao nuôi, hoặc chìm sâu vào lòng đất.)

Câu 300: Đáp án B

Câu nào sau đây có ý nghĩa gần nhất với từ "hấp thụ" ở đoạn cuối?

A. tiêu thụ B. hấp thu vào C. nuốt D. chất đống